Download as pdf or txt
Download as pdf or txt
You are on page 1of 292

>­ ·C:

·-·- u , ca
-
1-1
+-'
ca · ·- a,
o
a u
' .. ■ ■
Ul
·C:-
'0
ca .S· ::I
C
0
+' a:: ',::( 2
I U- I I ;, I
LO ·a, � N C'l
Cl Cl) Lil .LU JJ,1
PAST PAPER
AL CE DSE
BY-TOPIC
DSE Physics - Section D : M.C. PD-EMl-M/01 DSE Physics - Section D : M.C. PD-EMl-M/02
EMl : Electrostatics EMl : Electrostatics

TI
The following list of formulae may be found useful : 3. < HKCE 1984 Paper II - 24 >

Coulomb's law

+
X

Electric field strength due to a point charge


Two uncharged metal spheres in contact are suspended by dry cotton threads. When a positively charged rod Xis brought
near them as shown above, the distnOution of charges on the spheres will be

00
A. B.o:J C. D.
Electric field between parallel plates (numerically)

Use the following data wherever necessary :


4. < HKCE 1985 Paper Il- 33 >
Acceleration due to gravity g = 9.81 m s-2 (close to the Earth)
Charge of electron e = 1.6 x 10- 19 C
+
X
Electron rest mass mo = 9.11 x 10- 31 kg

Permittivity of free space Two uncharged metal spheres are suspended by insulating threads as shown in the diagram. A plastic sheet is put between
them. When a positively charged rod Xis brought near them as shown, which of the following diagrams shows the resulting
charge distnbution of the spheres ?

nm
A B
c. Ch
vµ �
Part A : HKCE examination questions .� .rh
Ct---]
1. < HKCE 1981 Paper n- 26 >

ti
s. < HKCE 1986 Paper II - 28 >

Two light conducting spheres are suspended from two silk threads as shown. If they are found to attract one another' which
one of the following conclusions is/are correct?
(I) They carry smri1ar charges. The diagram shows two conducting spheres X and Y mounted on insulating stands. Sphere X carries positive charges and
(2) They carry opposite charges. sphere Y carries negative charges. As Yis moved slowly towards X(but without touclringXJ the total charges onX
(3) Only one of them is charged. A. increase and are positive.
A. (2)only B. remain unchanged.
B. (3) only C. become negative.
C. (1) & (3) only D. disappear.
D. (2) & (3) only

tttttt
6. < BKCE 1987 Paper II- 37 >
2. < BKCE 1982 Paper II - 35 > In the figure shown, an electron travels in a horizontal direction and enters a
uniform electric field. The direction ofthe electric field is as shown. What
is the direction of the force due to the electric field acting on the electron?
A. upwards
B. downwards
C. into the page
D. out ofthepage
Three metal spheres X, Y and Z areplaced in contact as shown. A negatively charged rod is brought near z. The sphere yis
then earthed while the charged rod is still nearby. 'Which of the following statements is/are true?
(1) Xis negativelycharged. 7. < HKCE 1988 Paper II- 28 >
(2) Y is neutral. A positively charged metal sphere Xis brought near an uncharged metal sphere Y as
(3) Z is positively charged. shown. Y is then touched momentarily with a finger. Xis then taken away. Y is now
A. (3) only A. positively charged.
B. (I) & (2) only B. negatively charged.
C. (2) & (3) only C. negatively charged on the left side and positively charged on the right side.
D. (!), (2) & (3) D. positively charged on the left side and negatively charged on the right side.
DSE Physics - Section D : M.C. PD-EMl-M/03 DSE Physics - Section D : M.C. PD-EMl-M/04
EMl: Electrostatics EMl : Electrostatics

:ti:
8. < HKCE 1990 Paper Il 30 > w
14. < HKCE 1995 Paper Il • 29 >
X y
A positively-charged metal sphere is mounted on an insulating stand. When the sphere is earthed with a finger,
A. protons will move from the sphere to the earth.
B. protons will move from the earth to the sphere.
C electrons will move from the sphere to the earth.
D. electrons will move from the earth to the sphere.

9. < BK.CE 1991 Paper 11 � 29 > Two insulated uncharged metal spheres X and Y are in contact with each other. A positively charged rod is brought nearXas

w
In the figure shown, X and Y are two insulated metal spheres in contact with each shown above. Which of the following diagrams correctly shows the distribution of charges on the spheres?
A.

CD
other. A negatively charged object C is brought near X Xis touched momentarily B.
with a finger, and thenX and Y are separated. The charges onXand Y are
SphereX Sphere Y
A. positive positive D.

CD
B. positive C.
negative
C. negative negative
D. positive "'°

10. < HKCE 1992Paper Il 30 > w 15. < HKCE 1996 Paper II - 29 >
A positively charged rod is brought near an insulated uncharged metal A small conducting sphere is suspended by an insulated thread. When a positively
sphere as shown in the figure. If1he sphere is earthed with a finger, charged rod is brought near the sphere, the sphere is attracted as shown. Which of
A. positive charges flow from the earth to the sphere. the following is/are reasonable dednction(s) ?
B. electrons flow from the earth to the sphere. (1) The sphere may carry a positive charge.
C. electrons flow from the sphere to the earth.
D. the sphere is completely discharged. (2) The sphere may carry a negative charge.
(3) The sphere.maybe neutral.
11. < HK.CE 1993 Paper Il 31 >w
A. (2)only
B. (1) & (3) only
An uncharged light conducting sphere is suspended by an insulating thread. The metal dome of an operating Van de Graaff C. (2) & (3) only
generator is brought near the sphere. Which of the following statements best descnl>es the motion of the sphere ? D. (1), (2) & (3)
A. The sphere remains stationazy.
B. The sphere moves away from the dome.
C. The sphere moves towards the dome, touches it and remains there. 16. <HKCE 1997Paperll- 27 >
D. The sphere moves towards the dome, touches it and then moves away.
The following are the steps to charge an isolated metal sphere by induction, but they are in the wrong order.
(1) Removing the finger away from the metal sphere.
12. < BKCE 1994 Paper n- 25>
(2) Touching the metal sphere with a finger.
Which of the following involve(s) the application of electrostatics ? (3) Bringing a positively charged rod near the metal sphere.
(I) A photocopier
(4) Removing the positively charged rod away from the metal sphere.
(2) A precipitator in the chimney ofa coal-fired power station
(3) A VandeGraaffgenerator Which of the following shows the correct order?
A. (3) only A. (2), (3), (4), (1)
B. (1) & (2) only B. (3), (2), (1), (4)
C. (1) & (3) only C. (3). (2), (4), (1)
D. (1), (2) & (3) D. (3), (4), (2), (1)

13. < BKCE 1994 Paper Il- 24 > 17. <HKCE1998Paperil-29>


An Ulllulated charged metal rod is brought near an insulated uncharged metal Three insulated uncharged metal spheres X, Y and Z are placed in
sphere as shown in the figure. Which of the following can charge the sphere?
(I) Touching the sphere momentarily with a finger. W2/Jc__+_+�+�/ contact as shown. A negatively charged rod is brought near X and
sphere Y is then earthed momentarily. If the charged rod is still near X y z
(2) Touching the sphere momentarily with the rod. X, which ofthe following descnDes the charges onX, Y and Z?
(3) Touching the rod momentarily with a finger and then touching the
sphere momentarily with the same finger. SphereX Sphere Y SphereZ
A. (1) only A. positive zero
""
W-0
B. (1) & (2) omy B. positive negative
C. (2) & (3) only C. positive positive positive
D. (1), (2) & (3) D. "'° ze,o ze,o
DSE Physics - Section D : M.C. PD-EMl-M/05 ! DSE Physics - Section D : M.C. PD - EM1 - M / 06
EMl : Electrostatics / EM l Electro stati s
lh====:::;:;;��;::::::: ���;:,:::;::;.c;::============aa::.l

.
18. < HKCE 2000Paper 11- 28 > 21. < HK.CE 2003Paper II - 31 >
+ t"'"
' . -----------•' - .
''
•' '
': •
C
'l
''' '''
- L --------- >. -l +
Four point charges of equal magnitude are placed at the four vertices of a square. The signs of the charges are as shown. A
Two insulated uncharged metal spheres X and Y are p laced in contact. A positively-charged rod is brought near X as shown pciint charge C is placed at the centre of the square. What will be the direction of the resultant electrostatic force, if any,
above. Xis then earthed momentarily. The charged rod is removed and the two spheres are then separated. Which of the acting on C?
following dc:scnbes the charges onXand Y?
A. ➔
SphereX SphereY
B. t
A. negative negative
B. negative uncharged C. The resultant force acting on C is zero.
C. positive positive D. It cannot be determined sincethe sign of C is not given.
D. uncharged uncharged

22. < HKCE2004Paper ll - 27 >


19. <HKCE 2001Paper1I - 28 >
Three conducting spheres are suspended by separate nylon threads. When any two of them are brought near each other, they
attract each other. Which of the following deductions is correct?
A. All three spheres are charged.
B. Only one sphere is charged and the other two are uncharged.
C. One sphere is uncharged and the other two carry like charges.
B
D. One sphere is uncharged andthe other two carry ualike charges.
A
Two charged spheres _A and B hanging at a point on a ceiling by two identical nylon threads. They remain at rest as shown
above. Which of the following statements must be correct? 23. < HK.CE 2005 Paper II - 17 >
(1) Both spheres carry positive charges. Two conducting spheres are hanging freely, in air by insulating threads. In which of the following will the two spheres attract
(2) The force acting on B byA is larger thanthe force acting onA by B. each other? Note: 'N' denotes that the sphere is uncharged.
(3) The mass ofA is largerthanB. (!) (2) (3)
A. (1) only
B. (3) only
C. (1) & (2) only
D. (2) & (3) only

+ + N N
20. <HKCE2002Paperll-31>
A. (1) on ly
B. (2) only
C. (3) on ly
D. (!), (2) & (3)

24. < HKCE2006 Paper II- 22 >


A rubbed plastic ruler can attract paper scraps before actually touching them. Which of the following descriptions about the
A plastic ruler is placed close to a stream of running water coming from an insulated tap. The stream of water is attracted charges on the rubbed ruler and the original charges on the paper scraps are possible?
towards the ruler as shown above. Which of the follcrwing statements are correct ?
(1) The ruler and the paper scraps carry opposite charges.
(1) The ruler carries an electric charge.
(2) The ruler and the running water are attracting each other with forces of equal magnitude. (2) Both the ruler and the paper scraps are uncharged.
(3) Both positive and negative charges are induced on the running water (3) The ruler carries charges and the paper scraps are uncharged.
A. (1) & (2) only A. (1) & (2) only
B. (1) & (3) only B. (1) & (3) only
C. (2) & (3) only C. (2) & (3) only
D. (!), (2) & (3) D. (!), (2) & (3)
PD-EMl-M/08
1
DSE Physics - Section D : M.C. PD-EMl-M/07 DSE Physics - Section D : M.C.
EMl : Electrostatics EMl : Electrostatics

2S. < HKCE 2007 Paper ll - 19 > 29. < HK.AL 1985 Paper I - 45 >
t

p Q
a.,.,:::;
highvo::l
J
-C
In the above figures, P, Q, Rand Sare identical light conducting spheres and they are hanging freely by insulating threads of
the same length. Which of the following deductions is/are correct? A light conducting ball is placed between two metal plates connected through an �eter to a de. s� e of high volta? e.
(1) P and Q must carry like charges. The ball shuttles back and forth between the plates, making alternate contacts with each plate. Which of the followmg
statements are correct ?
(2) R and S must carry un1ilre charges.
(3) P and Q must carry the same amount of net charges. (I) The ball carries charges, sometimes positive, and sometimes negative.
A. (1) only (2) Ifthe separation ofthe metal plates increases, the :frequency of oscillation of the ball decreases.
B. (2) only (3) The galvanometer shows a current flowing always in the same direction.
C. (I)&(3)only
D. (2) & (3) only A. (1) & (2) only
B. (1) & (3) o,ly
C. (2) & (3) only
D. (!), (2) & (3)

Part B: HKAL examination questions


{);HI{AL 1987 Paper I- 30 >


I , +lC -lC
26. <BKAL1982Paperl-27>
The diagram shows a negative charged particle inside a uniform electric field E ' '''
pointing in downward direction. At the instant shown, the particle is tzavelling ''' '
in the direction of the arrow v. Which of the anows 1 to 4 gives the direction of '' '
the acceleration of the particle at this instant? '' ''
A. 1
B. 2 '' y
C. 3
D. 4 Et +'s Q -lC
Three charge + I C, - 1 C and - 1 C are fixed at the comers W, Xand Yrespectively of a square as shown. A fourth charge,
Q, is fixed atZ. Ifthe net electrostatic force acting on the charge atXis indicated by the mow, what is the magnitude of Q?
27. < BKAL 1983 Paper I- 20 >
A. -.Jzc
2
Two parallel metal plates are placed horizontally with a separation of 0.05 m. A voltage of2000 Vis connected across the B. + ✓ C
plates. An oil drop carrying a charge of - 1.6 x 10-19 C is found to be at rest between the plates. Find the mass of the drop. C. -2./zC
(Take the acceleration due to gravity to be 10 m s-2.) D. +2$C
A 1.6 X 10 -18 kg.
B. 1.6 X 10-15 kg.
C. 6.4 X 10-14 kg. 1 31. < HKAL 1990 Paper I - 27 >
D. 6.4 x 10-16 kg.

28. < BKAL 1983 Paper I - 40 >


Which ofthe following statements about the coulomb is/are cornet?
(1) When one coulomb of positive charges flows across a potential difference of one volt from higher to lower
potential, one joule of electrical energy is given out. A light conducting sphere is hanged from a long insulating thread It is placed between two oppositely charged metal plates
(2) The force exerted on a charge of 1 coulomb in an electric field of 1 volt per metre is I newton. connected to a high voltage supply. If the sphere is given a negative charge, it will
(3) One coulomb is the total charge of one mole of electrons. A. move to the positive plate and stick to it
A. (1) only B. move to the negative plate and stick to it.
B. (3) only C. oscillate, touching each plate in tum, beginning with the positive plate.
C. (1) & (2) only
D. (2) & (3) only D. oscillate, touching each plate in tum, beginning v;ith the negative plate.
DSE Physics - Section D : M.C. PD-EMl-M/09 DSE Physics - Section D : M.C. PD-EMl-M/10
EMl : Electrostatics EMl : Electrostatics

32. <BKAL 1993 Paperl-35> 36. <BKAL 2005 PaperllA-13 >

[,:!"-'
=ml +
pl,to

r
A negatively charged oil drop is kept stationary between two horizontal metal plates connected to a d.c. supply as shown.
almninium The oil drop then acquires an additional negative charge. Which of the following changes will be able to hold the oil drop
'°""'t �
�--�EHT---
stationary?
(1) Disconnecting the plates from the supply and moving the plates closer
(2) Keeping the separation between the plates unchanged and increasing the p.d. between the plates
Two parallel metal plates are connected to the terminal of an EHT. When a charged aluminium foil strip is placed between (3) Keeping the p.d. between plates unchanged and moving the plates further apart.
the plates, the deflection of the foil is shown in the above :figure. Which of the following statements is/are correct ? A. (l)only
(1) The charge on the foil is negative. B. (3) only
(2) Ifthe separation between the two plates decreases, the deflection of the :full increases. C. (1)& (2) o oly
(3) When moving the foil towards the positive plate, the deflection ofthe foil increases. D. (2) & (3) ool y
A. (l)only

m
B. (3) only
C. (1) & (2) only 37. <BKAL2007PaperIIA-14>
D. (2) & (3)ooly

33. <HKAL 2000 Paper IIA- 26 >


Two parallel plates are connected to anE.H.T. supply giving a d.c. voltage of 4.5 kV.· If the separation ofthe two plates is
set at 1.5 mm, calculate the acceleration ofan electron placed inside the plates.
A LOxl09ms-2
+Q 360
B. l.2x lQUm s-z
C. 1.6 x 101s m s-z The above :figµre gives the electric field strength (in arbitrary units) at various points near an isolated point charge +Q.
D. 5.3 X 1Ql7mg-l 'Which of the following correctly gives the electric field strength (in the same arbitrary units) atX and at Y?
electric field strength at X electric field strength at Y
34. <BKAL2003 Paper IIA-23 > A. 72 30
B. 72 36
The magnitudes of the charges on two identical small metal spheres are in the ratio 5 ; 1. The electrostatic force between
th.em is F1. If they are brought into contact and then separated to their respective original positions, the electrostatic force c. 90 30
between them becomes F2. Which of the following m.a:y be the magnitude ofthe ratio Ft : A? D. 90 36
(1) 5: 9
(2) 5,4
38. < BKAL 2007 Paper IIA- 13 >
(3) s,2
A {1) only O:
B. (3) only '¢
C. (1) & (2) only 0:
D. (2)& (3) only '
'
y
35. < HKAL 2005 Paper IlA- 12 > X
l; Yand z.are small identical metal spheres. X and Y are fixed at a certain separation in air and they carry charges of the The figure shows two small charged spheres X and Y suspended by identical nylon threads from a fixed point 0. At
same magmtude. The attractive force between them is F. Sphere Z is initially uncharged. It first touches X and then it equilibrium, the threads OX and Ofmake angles 0 and ¢ ( 0 < ¢)with the vertical. Which ofthe following conclusions must
touches Y. What is the electrostatic force betweenXand Y after Z is taken away? be correct?
(1) Both X and Y carry positive charges.
A. ¼F
(2) The charge onX is greater than that on Yin magnitude.
B. tF (3) The mass ofXis greater than that of Y.
A. (I) only
C. ¾F
B. (3) only
D. 'fF C. (l)and(2)only
D. (2) and (3) only
PD-EMl-M/11 PD-EMl-M/12
DSE Physics - Section D : M.C. 1
DSE Physics - Section D : M.C. 1
EMl : Electrostatics EMl : Electrostatics
39. < HKAL 2008 Paper IlA-14 > 43.
X y

+Q -2Q

A point charge +Q is fixed at pointXwhile another point charge-2Q is fixed at point Yas shown. Ex andEy denote the
magnitude of the electric field due to the point charge atX and Y respectively. For the points on the line passing throughX
and Y, how many ofthem haslhave Ex = Ev and how many has/have zero resultant electric field ?
(No need to consider the points at infinity.) A small neutral conducting sphere is suspended by an insulated thread. When a positively charged metal rod is brought near
A. There are 3 points with Ex = Ev and the resultant field is zero at 2 ofthem. the sphere, which of the following consequences are correct ?
B. There are 2 points with Ex =Ev and the resultant field is zero at both of them. (1) The sphere is attracted by the charged rod due to the induced charge on the sphere.
C. There are 2 points with Ex =Ey and the resultant field is zero at 1 of them. (2) The sphere is repelled from the charged rod after touching the charged rod
D. There is 1 point with Ex= EY and the resultant field is zero at that point.
(3) The sphere finally carries positive charge.
A. (I) & (2) oc ly
40. < BKAL 2009 Paper llA - 33 > B. (1) & (3) only
C. (2) & (3) only
A potential difference of 3 kV is applied across two horizontal D. (!), (2) & (3)
metal plates of separation 1.5 cm. A charged oil drop ofmass +3 kV negatively
9.6 x 10- 15 kg is kept stationary between the plates as shown. charged oil
Find the quantity of charge acquired by the oil drop? d,op 44.
(Tal<:e the acceleration due to gravity as 10 m s-2.)
A. 1.6 X 10-19 C
B. 3.2 X 10-19 C
c. 4.s x w--- 19 c
D. 6.4 X 10-19 C

Part C : Supplemental exercise When a plastic ruler is placed close to pieces of small paper, some of them are found to be attracted towards the ruler.
Which of the following statements is/are correct ?

'.Yrr
41. Two insulated uncharged � spheres X and Y are placed in contact. X y (I) The attraction is due to the electric force between the ruler and the pieces of paper.
A positively-charged rod is brought near X as shown. Xis then earthed
momentarily. The charged rod is then removed. Which of the following (2) Those pieces of paper attracted by the ruler remain neutral.
descnDes the charges onXand Y? (3) The attraction acting on each piece of paper is greater than the attraction acting on the ruler by that piece ofpaper.
SphereX Sphere Y A. (1) only
A. negative negative B. (3) only
B. negative uncharged C. (1) & (2) only
C. positive positive D. (2) & (3) o cly
D. uncharged uncharged
4S.
42.
(±)-----1----L---!-----G) 2 �-
''
---------:f
''
' '
3 ''
: 4 :
'
Two identical spheres carrying equal amount of positive charges are placed as shown in the figure above. Which of the ' ''
following statements are correct? t----------i
(1) At the mid-p:iint Ybetween the two spheres, it is a neutral point +Q -Q
(2) Ifa small negative charge is placed at polntX, it experiences a net electric force towards the left.
Point charges, each of magnitude Q, are placed at three corners ofa square as shown in the diagram. What is the direction of
(3) If a small positive charge is placed at point Z, it experiences a net electric force towards the right the resultant electric field at the fourth comer?
A. (!) & (2) ocly A Directionl
B. (!) & (3) only B. Direction2
C. (2) & (3) only C. Direction 3
D. (I). (2) & (3) D. Direction4
DSE Physics - Section D : M.C. PD-EMl-M/13 DSE Physics - Section D : M.C. PD-EMl-M/14
EMl : Electrostatics EMl : Electrostatics

46.
----+----+----+----+----
D
so.
A B C
A, B, C., D � four points on a strai � line as shown in the di agram. A point charge+ Q is fixed at A. When another point
_
charge -Q 1S mov:ed fromB to C, which of the following statements is/are correct?

y
(1) The electrostatic force between the two charges increases.
(2) The magnitude of the electric field strength at the pointD in creases.
(3) PointB becomes the neutral point.
A. (1) only
The above figure shows the electric field pattern around two point charges P and Q. Xis the mid-point between P and Q.
Y
B. (2) only
is a neutral point. Which ofthe following deductions is/are correct 1
C. (1) & (3) only
D. (2) & (3) only (1) Both charges P and Qare positive.
(2) The magnitude of chargePis greater than that of Q.
(3) The electrostatic force acting on Q byPis greater than that onPby Q.
47. The charge on theuraniui:nnucleus is 1.5 x 10-17 Cand the charge on the a.-particle is 3.2 x 10-19 C. What is the electrostatic
force between a uranium nucleus and an a-particle separated by 1.0 x 10-13 m? A. (1) only
A 4.32 x l0--33 N B. (1) & (2) only
C. (2) & (3) only
B. 4.32 x 10-2° N D. (1), (2) &(3)
C. 43Zx10-13 N
D. 4.32N supply. Neglect
51. A charged particle is accelerated across the gap between two parallel plates connected to a constant voltage
the effect of gravity, the kinetic energy gained by the particle in crossing the gap depends on
48. What is the magnitude of the electric field strength at a distance r from an isolated stationary nucleus of proton number (1) the mass of the charged particle
(atomic number) Z?
A z, (2) the separation between the two parallel plates
411:&0r (3) the voltage across the two parallel plates

B. (Z e)1 A. (1) only


41ts 0r2 B. (3 ) only
C. (1) & (2) only
c. Ze
D. (2) & (3) only
41ts0 r2
D. Zez
41tc0 r

Part D : BKDSE examination questions


49.
52. < HKDSE Sample Paper IA- 24 >
Two conducting spheres are hanging freely in air by insulating threads. In which of the following will the two spheres attract
each other?
Note: 'N' denotes that the sphere is uncharged.

(1) • (2) • (3) '

Thediagram. shows a pattern of electric :field lines in whichx; YandZ are points marked on one of the field lines. It would
be correct to say that
z.
(1) the electric field at Xis weaker than that at
+ + N N
z
(2) a negative charge placed at Z would experience an electric force to the left along the tangent of the field line at
(3) the force exerted on a charge at Ywould be greater than ifthe charge were placed atX
A. (1) & (2) owy A. (1) only
B. (1) & (3) only B. (2) only
C. (2) & (3) only C. (3) only
D. (1).(2)&(3) D. (1), (2) & (3 )
DSE Physics - Section D : M.C. PD-EMl-M/15 DSE Physics - Section Ji : M.C. PD-EMl-M/16
1 1
EMl : Electrostatics EMl : Electrostatics
53. < HKDSE Sample Paper IA - 32 > 58. < HKDSE 2013 Paper IA- 24 >
The figure shows the location of an isolated charge of size +Q. The magnitude Xand Y are two small identical metal spheres carrying charges -2Q and +6Q respectively. WhenXand Y are separated by a
of the electric field strength is marked at certain points. What is the magnitude certain distance, the magnitude ofthe electrostatic force between them is F.
of the electric field strength atXand Y?
electric field strength atX X X y

@--r �
electric field strength at Y
A.
B.
c.
72
72
90
30
36
30
72 y
\
D. 90 36 40 Toe spheres are �ught to touch each other and then placed back to their original positions. The electrostatic force between
+Q 360
them becomes

54. < HKDSE Practice Paper IA- 24 > A ¼ F, attractive B. ¼ F, repulsive


X-- y-

�---�p:.:.----:----�---�:----�
r--- T- ---, --,----T-- -.,- - --,---r---, ---,----, C. ½ F, attractive D. ½ F, repulsive

q 59. < HKDSE 2014 Paper IA- 20 >


Three identical point charges q (represented by dots) are situated in the space as shown. 'Which of the following descriptions
about the direction and Jllagnitude of the electric field£ atXand at Yis correct?
Direction Magnitude
-A.... Same Ex> Ey
--,,__· Same Ex< Ey
C. Opposite Ex> By
D. Opposite Two insulated uncharged metal spheres X and Y are placed in contact A positively-charged rod is brought near Xas shown.
Ex< Ey Xis then touched by a finger momentarily and the two spheres are then separated by removing Y. The charged rod is
removed afterwards. Which of the following describes the charges onX and Y?
55. < .HKDSE 2012 Paper IA - 25 >
sphereX sphere Y
W X Y Z
------------1------ ---@----!----+-- -i----- O··· --- ---- ;------- A. uncharged uncharged
+2Q M -Q B. uncharged positive
Two point charges +2Q and -Q are situated at fixed positions as shown. Mis the mid-point bet\veen the charges. W,
X, Y
c. negative uncharged
and Z are points marked on the line joining these two charges. At which point could the resultant electric field due to the two D. negative negative
charges be zero ?
A. w
B. X 60. < HKDSE 2014 Paper IA- 22 >
c. y Two parallel metal plates P and Qare maintained at a certain p.d. by a battery
D. Z (not shown in the figure). An electron placed between the p lates would
experience an electrostatic force of 8.0 x 10-18 N towards P. Which of the

7.,"'.
56. < HKDSE 2012 Paper IA- 24 > following descriptions about the electric field E between the plates is correct ?
A. E=0.02NC-1 fromQtoP . p� ,:;:
P, Q,R, Sare charged objects. When two of them are brought close to each other, P and Q repel, Rand S also repel while B. E=0.02NC-1 fromP toQ
QandR attract each other. Which ofthe following descriptions about their charges .is/are correct?
C. E=S0NC- 1 fromQtoP
(1) P and R are negatively charged.
D. E=S0NC-1 fromPtoQ
(2) Q ands are positively charged.
(3) Pis positively chaiged and Sis negatively charged.
A. (l)only
61. <BKDSE2014PaperIA-21>
B. (3) only
c. (1)& (2) only Q, Q, Q,
D. (2) & (3) only

Three point charges Qi, Qi and 0 are :fixed on a straight line with 0 at the mid-point of Qi and 0. The resultant
57. < HKDSE 2013 Paper IA- 25 > electrostatic force on each charge is zero. Which oftbe following can be the sign and the magnitude (in the same arbitrary
Llghtning flash may occur when the strength of the �lectric field (assumed uniform) between a thundercloud and the ground units) of Q1, Qz andQ3?
reaches �C:.1• A lightning flash on average discharges about 20 C of charge. If a thunderdoud is at a height
of Q, Q, Q,
. ..2
.o:!>_:a;i�ovethe ground, estimate the order ofmagnitude of the energy released in a lightning flash.
A. 106 J A. +2 +1 +2
B. 10s J B. +2 -1 +2
C. 1010 J C. -4 +1 +4
D. 1012 J D. -4 +1 -4
DSE Physics - Section D : M.C. PD-EMl-M/17 DSE Physics - Section D : M.C. PD-EMl-M/18
EMl : Electrostatics EMl : Electrostatics

62. < HKDSE 2015 Paper IA- 21 > 66. <HKDSE2017PaperlA-23>


Three conducting balls are suspended by insulating threads. Any two of them are found to attract each other ifbrou� close A point charge +Q is fixed at a distanced away from the origin Oas shown.
to each other. Which conclusion below is correct ?
A. Only one ball is uncharged while the other two carry like charges, +Q 0
___,,,______;:__________-,.x
B. Only one ball is uncharged while the other two carry unlike charges.
C. Only one ball is charged.
Which of the following graphs best represents the variation of the electric field strength E
along the x-axis ? (Take the
D. All three balls are charged.
electric field pointing to the right as positive.)
A. B.
E E
63. < HKDSE 2015 Paper IA- 22 >
-4Q +Q

w X y z 0 X
0 X
-d -d
Two point charges -4Q and +Q are fixed as shown. At which point indicated in the figw-e is the resultant electric field due to : :
these two charges zero ?
A. w
&. X C. D.
C. y
D. Z

__J_J.:::====+
64. < BKDSE 2016 Paper IA - 24 >

-----+-----➔
+4Q +Q------► x X
-d: -d: 0
0 15= 0

Point charges +4Q and +Q are fixed on the x-axis with +4Q at the origin O and +Q at x = 15 cm as shown. The respective
electric fields due to the two charges are equal at 67. <HKDSE 2019 Paper IA-23>
A. x = lOcm.
B. x = 12cm.
C. x=20 cm.
D. x=30cm.

65. < HKDSE 2017 Paper IA- 22 >


In the figure, two charged conducting spheres of the same mass m are put in a vertical plastic cylinder. The inner wall of the
cylinder is smooth. The spheres are separated by a distance d and remain in equ.ilibrium.

Which of the following statements MUST BE correct?


(1) Both spheres caa:ypositive charges.
(2) The amount of charges on the two spheres arc the same.
(3) The separation d depends on m.
A (1) only
B. (3) only
C. (1) & (2) only
D. (2) & (3) only
DSE Physics - Section D : MC. Solution PD-EMl-MS/01 DSE Physics - Sectiori1) : MC. Solution PD-EMl-MS/02
EMl : Electrostatics EMl: Electrostatics
HKEAA's Marking Scheme is prepared for the markers' refc:rence. It should not be regarded as a set of model answexs.
Students and t:eacbel1 who are not involved in the marlcing process are advised to intczpret the Marking Scheme with care. 5. B
Sphere Xis not in contact with. other bodies,
M.C. Answers
thus the total charges on sphere Xmust remain unchanged.
I. D 11. D 21. C 31. C 41. A 51. B 61. D
2. C 12. D 22. D 32. C 42. A 52. D 62. B 6. A
3. C 13. B 23. D 33. D 43. D 53. B 63. D Electron carries negative charge, thus the electric force on an electron is opposite to the direction of the EMfield.
4. A 14. C 24. B 34. C 44. C 54. D 64. D
Therefore, the electron experiences an upwards electric force inside the parallel plates.
5. B 15. C 25. A 35. B 45. A 55. D 65. B
6. A 16. B 26. D 36. B 46. B 56. B 66. D 7. B
7. B 17. A 27. D 37. B 47. D 57. C Bringing (+) charged sphere X towards Y and then earthing, negative charges are induced on Y.
67. A
8. D 18. A 28. C 38. B 48. C 58. D WhenX is removed, f carries negative charges that would distnbute uniformly on the whole surface.
9. D 19. B 29. D 39. C 49. D 59. C
10. B 20. D 30. D 40. C 50. B 60. D
8. D
The(+) charge sphere is earthed
M.C. Solution ⇒ the sphere becomes neutral

1. D ⇒ electrons flow from the earth to the sp here(as proton does not move)

" (1) Repulsion exists between similar charges.


✓ 9. D
(2) Attraction exists between opposite charges.
✓ When the negatively charged object C is brought nearX,
(3) Attraction exists between a charged body and an uncharged body.
(+) charges are induced on the left side ofX and(-) charges are induced on the right side ofY.

2. C When Xis touched momentarily with a finger,


the(-) charges on Yflows to the earth but the(+) charges on Xis still attracted by C.
Bringing(-) charge towards Z ⇒ {+) charge induced on 2 and(-) charge induced onX
WhenX and Y are separated, X carries (+} charges and Y is neutral.
Y is earthed ⇒ (-) charge onXrnoves to the earth but(+) charge on Z is still attracted by the charged rod
Finally, X and Y are neutral and Z is positively charged
10. B

3. Bringing(+) charged rod ⇒ (-) charge induced at left.side ofthe sphere and(+) charge at ri�t side of the sphere
C
Bringing(+) charged rod towards right ball Earthed with a finger ⇒ electrons flow from the Earth to the sphere to neutralize the(+) charge at the right side

⇒ (-) charge induced on right side of the right ball


11. D
⇒ (+) charge induced on the left side of the left ball
When the sphere is brought near, attraction e_xist5 between the charged dome and the neutral sphere.
4. A After touching, the sphere shares some like charges and thus repulsion exists to move it away.
Bringing (+) charge towards right ball
⇒ (-) charge induced on right side of the right ball 12. D

⇒ (+) charge induced on the left side of the right ball (as plastic sheet does not allow flow of charge) ✓ (1) Photocopier makes use of electrostatic charge to give photocopying

⇒ (-) charge induced on right side of the left ball ✓ (2) Precipitator makes use of electrostatic charge to attract the coal dust

⇒ (+) charge induced on left side oftbe left ball ✓ (3) A Van de Graaff generator gives large amount of electrostatic charge for demonstrating
DSE Physics - Section D : M.C. Solution PD-EMl-MS/03 DSE Physics - Section D : M.C. Solution PD-EMl-MS/04
EMl : Electrostatics EMl : Electrostatics

13. B 19. B
✓ (1) Since the two spheres repel, the two spheres may both carry(+) charge or both carry(-) charge
(1) Bringing positively charged rod near the sphere
⇒ (-) charge induced at the left side and(+) charge induced at the right side of the sphere (2) Since the force acting on B by A and the force acting on A by B are action and reaction pruI,
⇒ upon earthing, electrons flow to the sphere to neutralize the(+) charge at the right side they must be equal in magnitude but opposite in direction
.·. the sphere finally carries negative charges. ✓ (3) Assume the electric force F acting on the sphere is approximately horizontal
✓ Touching the sphere with the rod would make the sphere share some positive charges. Resolving the tension into two components
(2)
Positive charges of the rod are discharged upon earthing by the finger Tcos0=mg and Tsin0=F tanB=_!_
(3) mg
but the human body always remains neutral without carrying net charge mg
For sphere A, Bis smaller, tan 0is smaller, thus mis greater.
:. no charge is shared with the sphere by the finger, thus the sphere remains neutral

20. D
14. C
✓ (1) Water is neutral, as the ruler attracts the water, the ruler must carry an electric charge.
Due to attraction between unlike charges, negative charge is induced at the left side ofX
✓ (2) The attractive forces between the running water and the ruler are action and reaction pruI,
Due to repulsion between like charges, positive charge is induced at the right side ofY. so they are of equal magnitude.
✓ (3) The side of the running water near the ruler would induce the opposite type of electric charge
15. C while the side of the rw:ming water further away from the water would induce the same type of charge

(1) If the sphere carries (+ ) charges, repulsion occurs between the sphere and the rod
✓ (2) Attraction exists between unlike charges. 21. C

✓ (3) Attraction exists between a charged body and a neutral body. At the centre, the forces due to the two(+) charges on the point charge C balance each other
while the forces due to the two(-) charges on the point charge C also balance each other, thus resultant force on C is zero.

16. B

(3)' (-) charges are induced on the sphere at the side near the rod while(+) charges appear at the far side 22.
Suppose the three spheres are A, Band C such that A is uncharged, Bis positively charged and C is negatively charged.
(2)' electrons flow from the earth to the sphere to neutralize the(+) charges

(l) : the finger must be removed first before the removing ofthe charged rod
a, A and B attract each other since a positively charged body would attract a neutral body.

A and C attract each other since a negatively charged body would attract a neutral body.
(4) : after the rod is removed, the sphere becomes (-) charged.
® Band C attract each other since a positively charged body attracts a negatively charged body.
Thus, one sphere is uncharged and the other two caey unlike charges.
17. A
When the charged rod is brought near .X, (+) charges are induced onX and (-) charges are induced on z.
23. D
After Yis earthed, only the induced(+} charge remains on the left side ofX due to the attraction by the(-) charged rod.
✓ (1) Two balls carrying unlike charges would attract each other.
Z would become neutral as the negative induced charges would flow to earth during earthing.
✓ (2) A ball carrying positive charge would attract an uncharged ball by induced charge.

✓ (3) A ball carrying negative charge would attract an uncharged ball by induced charge.
18. A

a, presence of positively charged rod ⇒ (-) charge is induced on X while (+) charge is induced on Y

"'
24. B
Xis earthed momentarily ⇒ (-) charge on Xleft but (+) charge on Y is neutralized by electrons from the earth ✓ Attraction force exists between two opposite charges.
(1)
<l) The charged rod is removed ⇒ the (-) charge onX is then shared between Xand Y No electric force exists between two uncharged objects.
(2)
© Two spheres are separated ⇒ bothX and Ybecomes negatively charged ✓ A ruler carrying charge would attract uncharged paper scraps by induced charges on the paper scraps.
(3)
DSE Physics - Section D : M.C. Solution PD-EMl -MS/05 DSE Physics - Section D : M.C. Solution PD-EMl -MS/06
EMl : Electrostatics EMl : Electrostatics

25. A C
✓ ( !) Since repulsion exists between P and Q, they must carry like charges. The sphere carries (-)-charge ⇒ moves towards and touches (+) plate first
X (2) Since attraction exists between R and S, It then shares some (+) charge and moves towards and touches the opposite (-) plate
they may carry unlike charges OR one of them carries charge but the other is neutral. :. The sphere oscillates.
X (3) Even P and Q carry different amount of charges, same force would exist between them.

32. C
26. D
✓ (!) Since the strip is deflected towards the positive plates, the strip should cany negative charges.
As the .&field is in downward direction
The negatively charged particle experiences an upward electric force (F = - q E) ✓ (2) ByE=� :. dt ⇒Et :. deflectioni
d
Thus, the particle accelerates in upward direction.
X
(3) E-field is constant within 2 charged plates, thus there is no change in deflection of the strip.

27. D
33. D
Downward weight =Upward electric force from electric field

mg= qE = q!
E= f. =
d
. 03
(4 5x1 )
(15xl0"3)
= 3xl06 V
m (10) = (1.6 X 10-19) X (2xIO ) l
:. m = 6.4 x 10-16 kg F = q E = (1.60xl0-19)(3x10 ) = 4.8x10-1J N
6

(0.05)
.Sxl0-13 - '1
F 4 S.3 x10 ms-2
a =;; = 9 .llxl0-31 -
28. C
✓ (!) By U=qV :.U=(lC)x(IV)=lJ 34. C
✓ (2) By F= qE :. F= (lC)x(lVm-1) = lN Case I: Two charges areofthe same sign, i.e. (+5Q) and(+lQ).

X (3) Charge on 1 mole of electrons = 6.02 x 1023 x 1.6 x 10-19 C = 96320 C ;,; I C After sharing, they become (+3Q) and (+3Q).
= (SQ)Oq) = (3Q)(3�)
,; F,
4m: 0 r 4,rs 0 r
29. D
:. Fi:A = S:9
✓ (!) After touching (+)-plate, it carries(+) charge. After touching {-)-plate, it carries (-) charge.
Case 2: Two charges are ofthe opposite sign, i.e. (+sQ) and (-lQ).
✓ (2) d1' ⇒ E.J, (by E =!:::,) ⇒ pJ, (by F=q E) ⇒ a J. (by a=!..) ⇒ frequency of oscillation .J,
d m After sharing, they become(+2Q) and (+2Q).
✓ Carrying (+)-charge to (-)-plate ⇒ current flows in clockwise direction Q Q (2Q)(2QJ
(3) F. = (S )O z ) F. =
' 4m:.r 2 4m.r z
Carrying (-)-charge to (+)-plate � current flows in clockwise direction
:. Fi:F.:i.=5:4
30. D
35. B
Let the length of the square be r.
Since the electrostatic force is attractive, X and Y carry unlike charges.
Distance betweenXand Zis ✓2 r.
Let the charge carried byXbe +Q and the charge carried by Ybe -Q.
For sharing of charges, Qi + Q2 = Q + Q where Q is the final charge at each of the two spheres.

Since the net force onXis to the left, thus the upward force by Y is balanced by the downward component of force by z AfterZtouches.X,Xcarries +½ Q andZcarries +½ Q.
2
Fz cos 45 ° = Fy .-. Q(l) ° I
( )
I .-. Q=2✓ C AfterZtouchesY,Ycarries -¼Q andZcarries -¼Q.
4n:sJ/2'rf 7z = 41ts0 ?

Since the electric between% and 2 is attractive :. Z is(+)


Electrostatic force : p = Qi Qi
4m: 0 r2
DSE Physics - Section D : M.C. Solution PD-EMl-MS/07 DSE Physics - Section D : MC. Solution PD-EM1-MS I 08
EMl : Electrostatics EMl : Electrostatics
36. B
40. C
(1) After the plates are disconnected from the supply, charges on the plates and thusE-field are unchanged (
E = K_ = 3000) = 2xllY vm-1
Th.us the electric force is increased by F = qE as q is increased. The oil drcp then bas a net force. d (0.015)
(2) Increasing the p.d. between the plates would increase the electric field by E = VI d. Bymg=qE
15
:. (9.6 x 10- ) (10) = q (2 x 10 )
5
q = 4.&x10-19 c
Thus the electric force is increased by F = q E as E is increased. The oil drop then bas a net force. To balance the downward weight, the electric force F must be upwards.
✓ (J) Moving the plates further apart decreases the E-field by E = VI d. Since the Wl"ection ofE-:field is vertically downwards, the charge is negative.
The electric force may then remain unchanged by F = q E as q is increased but E is decreased. Thus, the charge carried by the oil drop is - 4.8 x 10- 19 C.
The unchanged electric force then balances the weight of the oil drop by q E = mg.

41. A
37. B The positively-charged rod is brought near X X: negative Y: positive
Since poinJ:Xhas the same distance as the point of 72, the electric field at Xis 72. Xis earthed momentarily X: negative Y; uncharged

Distance of point Yfrom +Q is .j(3) 2+


(1) 2 =
-J1o. The charged rod is removed X: negative Y: negative

Electric field; E = _Q_


4:rcs0 r2 42. A
✓ (1) The mid-point Yis a neutral point, where the electric :field due to the two charges balance each other.
Thus, electric field obeys inverse-square law, i.e. E eel_ ✓
r' (2) Since the left sphere is closer to X, the negative charge is attracted by the left sphere
and thus the net electric force is towards the left
Elccttic field at Y = 360 x _l_ = 36. Since the right sphere is closer to Z, the positive charge is repelled by the right sphere
(3)
(✓IO )'
and thus the net electric force is towards the left

38. B
43. D
(1) They may both carry positive charges OR both carry negative charges. ✓ Some induced negative charges appear at the left side
(1)
(2) The charge on X may be greater OR smaller than that on Y. and induced positive charges appear at the right side of the sphere.
Attraction force then exists between the positive charged rod and the negative induced charges.
Ho wever, the forces between them are equal and opposite since they are action and reaction pair.
✓ ✓ (2) After touching the charged rod, the sphere shares some positive charges from the metal rod
(3) Since the angle Bis smaller, thus the weight ofXis greater.
and is repelled away.

39. C ✓ (3) The sphere is :finally positively charged by sharing.

A X B y
44. C
+Q -2Q
✓ (1) Electric force exists between the induced charges in the papers and the electric charges in the ruler.
By E=-Q- , ✓
4:n:a,, r1 (2) Since both positive and negative charges are induced in the paper, the paper remains neutral.

E-field depends on the charge Q and distance r.


• (J) Since the two forces are action and reaction pair, they should be equal in magnitude.

At point A, charge at Xis smaller and distance is closer, charge at Y is greater but distance is longer, thus Ex= F}y.
45. A
At pointB, charge at Xis smaller and distance is closer, charge at Yis greater but distance is longer, thus Ex= By.
The electric field due to the upper charge is towards the left.
There are 2 points with magnitude : Ex= By. The electric field due to the lower left charge is upwards.
At point A, Ex is towards the left butEv is towards the right, thus the resultant field is zero. The resultant of these two fields points towards direction 1.
At pointB, Ex is towards the right and Ex is also towards the right, thus the resultant field is not zero. The electric field due to the lower right charge is along direction 4,
There is 1 point with zero resultant field. however, this field is weaker than the resultant of the other two, thus the overall resultant field is along direction 1.
DSE Physics · Section D : MC. Solution PD-EMl-MS/09 DSE Physics · Section D : M.C. Solution PD-EMl-MS/10
EMl : Electrostatics EMl : Electrostatics
46. B 52. D
(I) rt ⇒ p-!, ✓ (I) Two balls canyingwilike charges would attract each other.

✓ (2) A ball carrying positive chaige would attract an .mcharged ball by induced charge.
✓ (2) AtpointD, E-field due to the positive charge points to the right ✓ (3) A ball carrying negative charge would attract an uncharged ball by induced cbarge.
and that due to negative charge points to the left.

As the negative charge is nearer to D, the resultantE-fieldpoints to the left. 53. B


As the - Q is moved nearer toD, E-field due to -Q further increases, and the resultantE increases. Since pointXhas the same distance as the point of 72, the electric field atXis 72.

(3) At the mid pointB, the direction ofE-field due to both +Q and-Qare towards the right, Distance ofpoint Yfrom +Q is ✓(Jf +(1)
2
= ✓JO.
thus, it cannot be a neutral point
Since electric field obeys inverse"square law,
[Note that there is no neutral point in this situation.]

47. D

F = Q1 Q2 "" (l.5xl0-l7)(32xl0-l9)
2 4.32N
4,i:e r 41t(8.85xl0-12)(1.0xI0-13 )2

Electric field at Y = 360 x _I_ = 36.


48. C (Ml'

E a __Q_2
=�
2
41te0 r 41t&0 r 54, D
Assume that the three charges are allpositive (it is arbitrary, same result obtained if assumed negative).

. .
49. D The tfuection ofE,.field by a positive charge is away from the charge.


q,
✓ (!) The density of electric field lines represents the strength of the electric field. X qz
q1 E2 E
As the field lines atZis closer, E-field strength atZ is greater. �
E,
✓ (2) Negative charged particle experiences an electric force opposite to theE-field
AtpointX, the electric field due to q1 and (ft are in opposite directions and caned each other,
Thus, electric foroe on the negative charge points towards the left along the tangent of field line.
the net £.field is E3 and directed towards the left
✓ (3) Electric field lines at Yis closer than thatat2; thus the electric field strength at Yis greater,
At point Y, the electric field due to the three charges are all towards the right,
therefore, a charge would experience a greater force at Y, by F= q E.
thus the resultant E-field is rightwards, therefore, the direction of£..field atX and Yare in opposite directions.

so. B The magnitude of E3 are the same atXand Y, but there areE1 andEz in addition.,
thus the resultant E-:field at Y is greater than that atX
✓ (I) Since the field lines are directed away from the charges, the two charges are positive.
✓ (2) As the neutral po.int is further away from P, the charge Pis greater than that of Q.
55. D
' (3) The two forces are action and reaction pair, they must be equal in magnitude. W X Y E2 Z E1
----------1-----------©---r----- --1-----0 ·---+-'+-•
M
+2Q -Q
51. B
' (!) Gain of KE does not depend on the mass. The electric :field due to a (+) point charge is away from the charge.
The larger the mass, the smaller the speed, but the same KE. The electric field due to a(-) point charge is towards the charge.
' (2) For cQilStant voltage between the two parallel plates, At Z, the field due to +2Q is E1 while that due to -Q is Ei,
the gain of KE depends on voltage only but not affects by the separation between the two plates.
and they are in opposite direction, thus they may be balanced to give zero resultant field, that is, the neutral point.
✓ (3) Gain of KE = loss of electric PE = q V.
[ Note that W cannot be a neutral point as theE-field due to the greater charge 2Q must be greater since it is closer.]
The greater the voltage V, the greater the gain of KE.
DSE Physics - Section D : M.C. Solution PD-EMl-MS/ 11 DSE Physics - Section D : M.C. Solution PD-EMl-MS/12
EMl : Electrostatics EMl : Electrostatics

B 6i. D
Since P and Q repel, they must catt}' like charges. Electrostatic force: F = Qi Qz
41t&., r2
Since R and S repel, they must cacy like charges.
A. To give zero resultant force on 0, the sign of Q1 and Q,. should be opposite.
Since Q and R attract, they IllUSt caey 'Unlike charges.

(l) Pand R must cany unlike charges, tlms they cannot be both negatively charged. B. Distance of Q1 from Q:; is two times that of Qz from 0,
thus the magnitude of the charge of Qi should be 4 times as that of Qz, not 2 times.
(2) Q and S must cany unlike charges, thus they cannot be both positively charged.
✓ C. To give zero resultant force on Q1, the sign of Qz and (23 should be opposite.
(3) P and S must cany unlike charges, thus P may be positively charged and Smay be negatively cllarged.
✓ D. Forces between Qi and Q:; is tepulsive with magnitude : F1 (4)(4)
41t&. (2r)2
57. C
Potential difference between the thundercloud and the ground (assume that theyform.2parallel plates) Forces between Qz and Q3 is attractive with magnitude: F1 = �
41tf: 0 (7)2
9
V =Ed"" (3 x 106)x(500) = 1.Sx 10 V As Fi = Fi, the resultant force on Q; is zero.
Energy released :
1
U= QV= (20)(15x1G9) = 3xl0QJ 62. B
Order ofmagnitude ofthe energy released = 10 10 J Due to the induced charges, a neutral ball and a positive charged ball would attract each otbct.
Due to induced charges, a neutral ball and a negative charged ball would attract each other.
58. D The deduction should be :
Before touching, electrostatic force between the two spheres : (D one ball carries positive charge

F • (2Q)(6Q) • l2_g__ <J) one ball carries negative charges


41te0r2 4ite0r 2 <l) one ball is uncharged (neutral).
After touching, the net charges are: (6Q) + (-2Q) = +4Q
63. D
This charge is then shared betweenXand Y, each has +2Q.
After touching, electrostatic force between the two spheres :

F' • (2Q)(2Q) • 4_g__ • �F


41t&0,-2 4,u.-072 3
X
The electrostatic forces betweenXand Y are repulsive as they carry like charges. Assum:e the neutral point Pis at a distance ofx at the right side of Q2.
Since Q 1 is negative, the electric field Bi due to Qi is towards the left.
59. C
Since Qi is positive, the electric field E2 due to Qz is towards the right.
When the positively charged rod is brought near X,Xis negatively charged and Y is positively charged.
To be the neutral point, the two electric fields must be equal and opposite.
WhenXis touched by the finger,Xis still negatively charged but Ybecomes uncharged.
:. X = 3
When Y is removed, Xremains negatively charged and Yremafus uncharged.

When the charged rod is removed, Xre.ma.ins negatively charged and Y remains uncharged. The neutral point is atZ.

60. D 64. D
Since the electton carries negative charge, the dttection of electric :field should be from Pto Q. There are two points that the magnitude ofthe electric field due to the two charges are equal.
For the same electric field (both magnitude and direction), the point must be at the right side of +Q.
By F = qE
Let the point from O be x.
:. (8.0 x 10-18) = (1.6 x 10-� E
Q ••• X = 30cm
:. E = S0NC-1 41tE:., -x2
DSE Physics - Section D : M.C. Solution
EMl : Electrostatics
PD-EMI-MS/13

I DSE Physics - Section D : Question


EMl : Electrostatics
PD-EM! -QI 01 II
65. B
Thefollowinglistofformulae maybe found useful:
(!) Since the electrostatic forces between the two spheres are repulsive, both spheres cany like charges.
-
F- Q,Q,
41te0 r'
However, both spheres may carry positive crulrges OR both may carry negative charges. Coulomb's law

(2) The arttOUllt of charges on the two spheres may NOT be the same.
Electric field strength due to a point charge
✓ (3) The electrostatic force F acting on the higher sphere is upwards that balance its downwards weight mg.
:, mg=_g&_
2
4ne0 d Electric field between par.ill.el plates (numerically)
Thus, the separation d depends on m.

Use the following data wherever necessary :


66. D
Acceleration due to gravity g = 9.81ms~2 (close to the Earth)
By E =--Q-
4xe0 r2 Charge of electron e = l.6x10-19 c
Elec'.ron rest mass m. = 9.11 x 10-31 kg
Thus, E must be positive and tends to zero as rtends to infinity, therefore, option A and B must be incorrect.
Permittivity offree space
Since the charge Q is placed at- d, the electric field must exist starting from - d, thus option Dis correct.

Part A � HK.CE examination questions

1 1. < HKCE 1985 Paper I- 9 >


(a) A girl, after combing her hair on a dry day, holds the comb near small pieces of paper. What will be observed if the
comb is made of

(i) plastic, and


(ii) aluminium ?
Explain briefly in each case. (4=b)

I\
(b) Two similar charged metal-coated balls, A and B, are suspended from two i.nsulating threads as shown in the figure.

(i) Draw on the diagram all the forces acting on the two balls.

(n) Ifthe ball A is earthed by touching. what would happen to the two balls ? Explain briefly.
DSE Physics - Section D : Question PD-EMl-Q/02 DSE Physics - Section D : Question PD-EMl- Q/03
EMl : Electrostatics EMl : Electrostatics

,�
2. <BKCE 1987 Paper I- 9 > 4. < HKAL 2011 Paper I - 7 >
E.H. T. via an ammeter as shown
The below figure shows two aluminium. foils held by plastic clips and a negatively charged plastic block. They are used for A small conducting ball is placed midway between two parallel metal plates connected to an
charging the aluminium foils by induction. in the Figure.

p1'stiobln,kQi
aluminium foil

(,) Descnbe the steps taken in charging the aluminium foils. (3 marks)

E.H.T.
(b) What type of charge is induced on the aluminium foils ? (1 mark)
(a) State how to make the ball acquire positive charges. (1 mark)

(,) Explain briefly why the clips should be an insulator but not a conductor in this experiment. (2 marks)

Part B : RKAL examination questions

3. < BKAL 2006 Paper I - 4 > (b) After the ball acquires positive charges, explain why it can shuttle continuously between the two plates.
In a vacuum, a beam of electrons with an initial horizontal velocity enters midway into a region of electric field between
two horizontal square metal plates as shown in the figure below. A p.d. of 320 V is applied across the plates and the
separation between them is 1.6 cm.

,_L 116=

r
320 V clootron •._
T plates
(c) State and explain how the average current registered by the ammeter is affected if the separation of the two metal
is decreased. (3 marks)

{a) Find the electric field strength between the plates. (2marks)

(b) The electron beam reaches one of the plates. Sketch in the above figure the path of the electron beam between the two
plates. (Neglect the weight of the electron..) (2nwks)
DSE Physics - Section D : Question PD-EMl -Q/04 DSE Physics - Section D : Question PD-EMl-Q/05
EMl : Electrostatics EMl : Electrostatics

Part C : HKDSE examination questions 6. < UKDSE 2013 Paper IB-11 >
Figure (a) shows two identical small metal spheresX and Y suspended by insulating -9threads of the sam� length. Each sph�
( C). The separation d of the spheres is
s. < BKDSE Practice Paper IB � 8 > has amass of LO x 10-s kg and each carries a positive charge of3.l nC 1 nC = 10
p
As shown in the Figure, two lar ge vertical parallel metal plates, each in a slotted base, are placed on a polystyrene tile. The 10 cm. The size of spheres is negligible compMed with their separation, therefore they can be treated as oint charges.
plat.es are connected to the positive and negative tenninals of an EHT supply respectively. The plates' separation d= IO cm. Truce_!_,,, 9xt0 Nm C .
9 2 -2
41te0

m"'1
plate

Diagram NOT drawn to scale


Figure(a)

X cf Y
EHT supply
<:-------->
d=l0cm
polystyrene
til•
(a) Find the angle between the threads.
A small charged ball is suspended by a n ylon thread and is placed midway between the plates. The thread makes an angle 0
to the vertical when the ball is in equilibrium.
(a) By using a :free--body diagram, draw and name all the forces acting on the charged ball. Also indicate in your diagram
the direction of the electric field between the plates. (3 llllll'ks )

(b) Point P is vertically below the fixed point O and it is 10 cm from each sphere as shown in Figure (b).

(i) Indicate the direction of the resultant electric field at P due to these two charged spheres. (1 mark)

(b) (i) Express tan 0 in terms of the electric force F acting on the hall and the wei ght W of the ball. (1 mark)
Diagram.NOT drawn to scale
Figure(b)

(ri) Given that the mass of the ball is 0.07 g. When the voltage between the plates is 4000 V, 0 = 2°. Estimate
the magnitude ofthe charge carried by the ball. Assume that the electric field between the plates is unifurm.
( 3 """")

(c) Using the setup in the Figure, suggest a simple method to test whether the electric field between the plates is uniform.
(3 marks)
(ii) A neutral metal sphere of finite size is now placed at P. State whether the separation dwould increase, decrease or
remain unchanged due to the presence of this sphere. (1 mark)

Separation d ....... ______


DSE Physics • Section D : Question Solution PD-EM1 -QS / 01 DSE Physics - Section D : Question Solution PD-EM1 -QS / 02
EMl : Electrostatics EMl : Electrostatics
HKEAA's Mar1cin&: Scheme is prepared for 1he markers' :mfercnce. It should not be regarded as a set of model answers.

1
Students and teachers who me not involved in the marking process are advised to inteipret the Markixlg Scheme with care. 3. (b)

Question Solution
320 V ol,ctrnn
:_:::::::> 11.6=
1. (a) (i) Small pieces of paper are attracted by the comb and are found on the comb

l-
[!] T
Since the plastic comb is an insulator, charges produced by robbing remain on the comb [!]

(ii) Small pieces of paper are not affected and stay at the original places [!] [!]
< The path bends upwards >
Since aluminium is a conductor, charges produced by rubbing will be conducted a:way. [!] [!]
< The path is a curve >

(b) (i)
T T

[!]
4. (a) Let the ball touch the left plate to share some positive charges.

F'+---\ [!]
j--+p (b) The ball is repelled by left plate and attracted to the right plate.
The ball then aequires negative charges when touching the right plate and the process repeats. [!]
Hence, the ball shuttles between the two plates.
w w
(c) As the plate separation d decreases, the electric field between the plate increases ( E = VI d). [!]
< weight of the two balls marked correctly> [!]
Therefore, electric force on the ball increases. [!]
< tension of the strings marked correctly> [!]
The acceleration of the ball increases and thus the average CU1Tent increases. [!]
< electrostatic force between the two balls marked correctly > [!]

(u) Both ball.A and ball B fall, [!]


touch and then separate again [!] 5. (a) T (tension)

�"--�--
Since charges in ball A is conducted away, it is attracted by ball B [!]
When it makes contact with ball B, it shares similar charges and repels away. [!]

direction of
electric field
2. (a) Place the aluminium foils near the plastic block. [!] w(weight)
Touch the foil momentarily with a finger. [!] [!]
< Weight and tension correctly drawn with correct name >
Then remove the block. [!] [!]
< The electrical force correctly drawn with correct name >
< Direction of electric field correct > [!]
(b) positive charge [!]

[!]
(c) Iftbe clip is a conductor, the induced charge would escape (b ) (i) tanB • f_
through the clip and the band.
[!]
w
[!]
(ii) For parallel plates: E = !:. = 4ooo = 40000 V m-1 [!]
d 0.1

3. (a)E=� tan0 = !_ == qE
d W mg

320 q•(40000)
[l] [!]
0.016 tan2" ==
(0.07 xio-')(9.81)
= 2xl04 Vm-1 (OR 20000NC-1 ) [!] [!]
:. q = 6.0ox10-10c
DSE Physics - Section D : Question Solution PD-EMl-QS/03
• Hong Kong Diploma of Secondary Education Examination
EMl: Electrostatics : Physics - Compulsory part (;1,if<Jl,l-)
, Section A-Heat and Gases (�'f"liUll:)
, 1. Temperature,Heat and Internal energy (m.&, ;;,,;i;�R�)
5. (c} Fix the plates separation and the output voltage of the EHT Supply. [!] 2. Transfer Processes (1¥.#:fJ:i&.fl.)
Move the polystyrene tile so that the ball is placed in different positions in the space between the plates. 3. Change of State (%.@:�iltyt)
[!]
4. General Gas Law (-ti&�J'llt;t.#)
The angleBshould remain the same if the electric field between the plates is unifonn. [!] 5. Kinetic Theory ( '.S,-"7'.i{�rt,i)
SectionB -Force and Motion (:h:ft!Jl10)
1. Positionand Movement (1ir.1f.'f":11*-i)
6. (a) 2. Newton's1aws(4,<ll1i::t.#)
T 3. Moment ofForce (iJ#!)

(
4. Work,Energy and Power (,fj:J:/J � j�:1,:;fcr.;&*)
5. Momentum(Z'l:ft)
6. Projectile Motion (&:lti'E:ti'J)
7. CircularMotion(l!)J;ll�!»)
F+- 8. Gravitation ('iii])
Section C- Wave Motion (ii:.111)
w 1. Wave Propagation (/&'.{t,i;J!til!)
2. WavePhenomena(JBtj).lJl.$.)
.--9 2
3. Reflection and Refraction ofLight ( 16ft:l �MA:brM)
F = �2 = (9 x 109) x (3.Ixl0 ) = 8.65 x lO�N [I] 4. Lenses (iS.�)
4ite0 r (O.lo)2 5. Wave Nature ofLlght (:J'e.�ii.:ti'J�·!t.)
"" 5 6. Sound(tt)
W =mg= (LO x 10 ) (9,81) = 9.81 x I<r-5N
Section D - Electricity and Magnetism ( 11!::fl'.fit)
ResolvethetensionT: TsinB=F and TcosB= W 1. Electrostatics (ff-1!!:�J!,)
6 2. Electric Cimlits ( 'l!t.'13-)
:. tanB = !_ "" 8.65xI0-""5 [!] 3. DomesticE!ectricity(Ull!'ilt)
W 9.81xl0
4. MagneticField(:qt.¼,Jt)
°
:. 0 = 5.04 5. Electromagnetic Induction ('it.i;;lt� .(Ji)
Angle between the threads = 2B = 10,l 0 6. Alternating Cmrent ( 5c,,m.'1ft)
[I] Section E- Radioactivity and Nuclear Energy (:tk¼tJJ!.�;f";ft1!5)
(b) (i ) 1. Radiation and Radioactivity ( .tiMt'f�<l'l:.M JJL ,t..)
2. Atomic Model (,IJT.fU:D
3. Nuclear Energy(#.!:�)

Physics -Elective part (;!if'Jl�')


Elective 1 -Astronomy and Space Science (�X.-$:Jiu,tt;i<;..f'I-,$)
1
1. The universe as seen indifferent scales (;;i;!IIJ'!E. M-iUtrl!ll�'iliilbtt.)
2. Astronomythroughhistoly (;t:si:.,:!j!,l!ll-§--1..t.)
3. Orbital motions under gravity ( Jii1Tft:l.$1t,i!ifitb)
4. Stars and the universe (•/Y.£.'fu�ii)
Elective 2 - Atomic World CW. T-fr.ff..)
L Rutherford's atomic model (11£�-f.T-lJt�)
2. Photoeleetric effect (/6 '!ltl!tli)
3. Bohr's atomic model ofhydrogen (Jk.ril 18i.�T-ilt�)
4. Particles or waves ($.T�it)
5. Probinginto nano scale Cm�#i'*�.J})
Elective 3 - Energy and Use of Energy (tl'§it'f":ll.i.!f.�{j!_lfi)
1. Electricityathome(�.lk.lli1!!:)
< direction ofE: vertically downwards> [!] 2. Energy efficiency in building ( J:l!A� � �Si.!Mt 41)
3. Energy efficiency in transportation c�.ni-�Mi �Si!F.�!fl:)
(ii) Sopa,,tinn d , d0<,,e,,., 4. Non-renewable energy sources (;;i;-r�i.�i.'l'i)
[!]
5. Renewable energy sources (".f .,il}i.�6�)
Elective 4- Medical Physics ( f} ,!f:4v,i lf.!J/,)
1. Making sense ofthe eye (!l!HS@li. if)
2. Making sense ofthe ear (.lfft:lti. 'is)
3. Medical imaging using non-ionizing radiation ( -'If 11:Nt.tiM-I' *i:51l*)
4. Medical imaging usingionizingradiation ('lll:Mll#rM-&<$-JMt�)
I DSE Physics - Section D : M.C.
EM2 : Electric Circuits
PD-EM2-M/Ol II DSE Physics - Section D : M.C.
EM2 : Electric Circuits
PD-EM2-M/02

The following list of formulae may be found useful : 2. < HKCE 1980 Paper Il- 38 >

Resistance and resistivity R


,---C'.'.:::J---{ A

Resistors in series R""R1+R1 R K

Resistors in parallel 1 1 1 Two identical resistors R, an ammeter, a switchK and a cell are connected as shown. When K is open, the ammeter reading
-=-+-
R R, R , is 1 A. When K is closed, the reading will
A not change.
Power in a circuit B. increase to 2 A
C. increase to 4 A.
D. decrease to A ½
USe the following data wherever necessary: 3. <HKCE 1980 Paper ll- 44 >

Acceleration due to gravity g "- 9.81 ms-1 (close to the Earth)

Charge of an electron e = 1.60 x 10- 19 C

Electron rest mass m,, = 9.11 X 10-31 kg


100n
Permittivity offree space
In the circuit shown, Vis a vol1meter of high intemal resistance and A is an ammeter oflow internal resistance. What is the
voltmeterreading when (a) switch K is open, and (b) switch K is closed ?
Kopen Kclosed
Part A: HKCE examination questions
A OV ov
1. < HK.CE 1980 Paper 11- 32 >
B. ov 2V
C. 1V IV
In the circuit shown, the battery has an e.m.f. of 6V with negligible internal resistance. Four resistors are connected to the D. 2V ov
battecy as shown :in the figure.

40 p "' 4. < HKCE 1981 Paper n- 25 >


Suppose that it takes� to boil a kettle of water. If the beating coil of the kettle is shortened to half its original length
and the supply voltage remains unchanged, then to boil the same amount of water will take
A. 16min.
B. Smin.
40 C. 4min.
Q D. 2min.

5. < HKCE 1981 Paper n- 29 >

6V

What is the potential difference between points P and Q 1


A 1V In the circuit shown, what is the power dissipated in the 4 .Q resistor ?
B. 2V A. o.sw
B. LOW
C. 3V
C. I.SW
D. 4V D. 2.0W
DSE Physics - Section D : M.C.
EM2 : Electric Circuits
PD-EM2-M/03
11
I DSE Physics - Section D : M.C.
EM2 : Electric Circuits
PD -EM2 -MI 04 II
6. < HKCE 1981 Paper n - 28 > 10. < BKCE 1982 Paper ll- 27 >
R,

R,
A

A,
sooon

�---1 B In the circuit shown, the cell has negligiOle intemal resistance. Ifthe lamp LJ bums out, then
A. L 1 becomes brighter.
In the circuit shown, ammeter A1 reads 2 mA and ammeter Ai reads 8 mA What is the voltage across AB? B. k becomes brighter.
A lOV C. L 1 becomes dimmer.
B. 40V D. L1 retains the same brightness.
C, 50V
D. lOOV
11. < HKCE 1982 Paper Il- 30 >

7. <BKCE1981PaperII-27> Suppose you were given four similar lamps and a battety. Which of the follo wing ways of connecting the lamps to the battezy
would give the maximum brightness overall ?
C rn rn D A. a
2Q -
A rn rn B

The diagram shows a network of resistors, where the resistance of each resistor is 2 n. What are the equivalent resistances
across AB and AD respectively ?
AB AD
C. D.
A. I.67 n l.OOQ
B. 1.67Q 1.67 n
C. 1.670 3.670
D. 2.son 1.oon

8. < BKCE 1982 Paper II- 31 >


In the circuit shown, the cell has negligible internal resistance. R
Toe two resistors Rare identical. If the power dissipated in the
circuit is P when K is open, find the power dissipated when K is
closed.
A. ¾P R K 12. < HKCE 1982 Paper Il- 33 >
B. ½P
C. 2P
D. 4P

9. < BKCE 1982 Paper 11-1 >


Which of the following are vectors?
{I) weight
(2) charge In the given circuit diagram, the battery has a _voltage__Q.f -� .Y. and negligible internal resistance. The three resistors are
(3) voltage identical and the voltmeter has the same resistan� as each resistor. What is the reading on the voltmeter?
A (1} only A l�V
B. (1) & (3) only B. 1.2 V
C. (2) & (3) only C. 1.5V
D. (1). (2) & (3) D. 2.0V
I DSE Physics - Section D : M.C.
EM2 : Electric Circuits
PD-EM2-M/05
11
I DSE Physics - Section D : M.C.
EM2 : Electric Circuits
PD-EM2-M/ 06 II
13. < HKCE 1983Paper II - 27 > 17. <HKCE1983Paperll-30>
6V

III
,-------, I I I I-----, 3A lA

2A
A

In the circuit shown, what is the current passing through the ammeter A ?
In the circuit shown. the resistances ofR1,& and & are all equal to 2 n. The power dissipated by& is A. 0A
A 2W B. IA
B. 4W C. 2A
C. 6W D. 4A
D. SW

18. < HKCE 1984 Paper 11-12 >


14. < HK.CE 1983Paper 11- 32 >
An immersion heater of resistance SO n raises the temperature of 1 kg of water by 20°C in 3 minutes, the voltage supply
The resistance of a given conducting wire may be increased by being 200 V. \Vhat would be the energy wasted?
(1) · decreasing the :radius of the wire.
(Specific heat capacity of water= 4.2 kJ kg"1 0 C-1 )
(2) increasing the length ofthe wire.
(3) winding the vvire in the form of a coil A 0.44kJ
A. (1) only B. 2.32kJ
B. (1) & (2) only c. 44kJ
C. (1) & (3) only
D. 60kJ
D. (1), (2) & (3)

19. < BKCE 1984 Paper II - 32 >

·l
� HKCE 1983Paper 11- 28 >
The diagrams below show three possible arrangements of four identical resistorsR.
JWhen connected in series man electric CII'CUlt, the power diss1patcd m two resistors R1 and R:.z lS in the ratio of 1 : 4. What
will be the ratio of the power dissipated inR1 and& when they are connected in parallel? (a) R
A. 1 :2
B. 2 :1 X�Y
c. 1 :4
D. 4:1
( b) R R

16. <BKCE1983PaperII-31> X�Y

(o) R
s
X y

In the above circuit, A1, A2 and AJ are ammeters of negligible internal resistance. What will happen to the readings of the
anuneters if the switch Sis closed?
List the arrangements in order of�equivalent resistance, as measured betweenX and Y.
Reading of At Reading ofAi Reading of .43 A. (a), (b), (o)
A decreases increases becomes zero B. (b), (o), (a)
13. increases decreases decreases
C. (o), (a), (b)
C. unchanged increases becomes zero
'"D. unchanged decreases increases D. (o), (b), (a)
l ti
=
_ ==
==D::: EM2S:::E:::Phy=::: sics=-:::S:::ee1l
: Electric :::":::on:::D=: M c
Crrcuits . .
2
= P D - E M =- M / O 7.
::: ::: :::=== = ::: :::::: ::: ::: :::::: ::;J ..
I I DSE Physics - Section D : M.C.
EM2 : Electric Crrcuits
PD-EM - /08
2 M
11

20/ 7< HK.CE 1984 Paper n - 33 > 24. < HKCE 1985Paper Il - 44 >
I I
V Which of the following will increase the resistance ofa metal wire ?
(1) increasing the length of the wire
(2) increasing the cross-sectional area of the wire
(3) increasing the temperature ofthe wire
p R A. (I) only
B. (1) & (3) only
C. (2) & (3) only
D. (1), (2) & (3)

Four identical bulbsP, Q, Rand Sare connected as shown in the two circuit diagrams above. What happens to the brightness
of P andR if the switches of both circuits are closed?
Brightness of P Brightness of R
A. decreases remains constant
B. remains constant decreases
C. increases remains constant
D. decreases decreases
In the circuit, whenK1 is closed the voltmeter reads 12 V. What would the reading be ifbothKi and Ki are closed?
A. 8V
21. < HKCE 1985 Paper ll- 36 > B. 12V
C. 16V
D. 18V

26.

In the circuit shown, all bulbs Bare identical. If ammeter A1 reads 1.8 A, what does ammeter A2 read?
A 1.2A
B. 0.9A
C. 0.6A
D. 0.3A
In the above circuit A 1, /4i ancl'A3 are ammeters connected to a constant voltage source. What will happen to the readings of
the ammeters if the switch Sis closed ?
22. < HK.CE 1985 Paper Il- 34 > Ai A2 Al
An immer.rion heater A. talces 10 minutes to boil a kettle of water while another h�ter B takes 40 minutes to boil the same A. decreases increases becomes zero
kettle of water under the same vOitage Sllppiy. If the resistance ofheatet: A is 100 .n, "what is-the resistance of heater _B � B. increases decreases decreases
C. no change increases becomes zero
A 500.
D. no change decreases increases
B. 100.Q
C. 2000
27. <HKCE1986Paperil-30>
D. 4000
Ammge the following circuits in ascending order of equivalent resistances :
(1) (2)
23. < HK.CE 1985 Paper II- 35 >
A student, who wishes to find the resistance ofa light bulb when �
connected to at:� !J�, incorrectly connects a�cal
v.o!Eueter and a ;practii;_al ammeter as shown. What woii1:o'bethe (3)
most probable outcome -Of his error?
>------< A ,_____,
A. The voltmeter would indicate zero voltages. o---+--{�J--+---.
B, The ammeter would bum out A. (3), (2). (1)
B. (1), (3). (2)
C, The light bulb would bum out.
C. (2), (3), (1)
D. Both the ammeter and the voltmeter would bum oul D. (2). (1). �)
I DSE Physics - Secti.on D : MC.
EM2 : Electric Circuits
PD-EM2-M/09 I I DSE Physics - Section D : M.C.
EM2 : Electric Circuits
PD-EM2-M/ 1011

28. < HKCE 1987 Paper II - 28 > 32. < HKCE 1988 Paper n- 27 >
What should be the potential difference between P and Qin the
cixcu.it shown 7
p A. 3V
B. 6V
C. 9V
Q D. 12V

33. < HKCE 1988 Paper n- 30 >


Four identi.callampsP, Q, R andS are connected to a battery as shown above. Ifiamp Pis blown, which of the followlng
would happen ? In the circuit shown. which ofthe switches should be closed in order
A. Lamp R becomes brighter. to get the maximum brightness in lamp L?
B. Lamp Q becomes brighter. A. Xonly
C. Lamp S becomes brighter. B. Yonly
D. Lamps Q and S :remain at the same degree ofbrightness.
C. Z only
D. XandYonly
29. <HKCE 1987 PaperII-32 >

34. < HKCE 1989 Paper ll- 34 >


Two copper wires, A and B, of the s� have the ratio of� ofj,,1 Assuming the cross-sections are unifonn, what
is the ratio ofthe resistance ofA to that ofB?
A. 2:3
p Q R s B. 3:2
C. 4:9
The above circuit shows two resisrors R1 and R;. connected in se_ries'to a battery. The resistance of R1 is greater than that of D. 9:4
,&. The ends of the resistors are markedP, Q, Rand S. Which of the graphs best shows how the potential V varies along PS?

� 6-rr b_ �
A. B. C. D.
35. < HKCE 1989 Paper II- 31 >
In the circuit shown. resistors A, Band Care identical. WhenK is open,
the power dissipated by A is P1. When K is closed, the power dissipated
by A becomes P2. The ratio Pi : P2 is equal to
A. 3:2
P Q R S P Q R S P Q R S
B. 2: l
P Q R S
C. 9:4
30. < HKCE 1987 hper 11- 30 > D. 4: 1
In the circuit shown, all resistors are the same. If the current
passing through point X is 1 A. what will the current delivered
from the battery be ? R 36. < HKCE 1989 Paper II- 38 >
lA
A. lA
X
B. 2A
C. 2.SA A
D. 3A R
V }-+=:::i-....J
31. < HKCE 1988 Paper II- 26 > K
In the circuit shown, the energy required by an electron to travel p 2Q
fromQ to P through Xis Et and that through Yis £2. vVhich of Q In the circ-ujt above, the two resistors are identical When the switch K is closed, what happens to the readings of the
X
the following is true 'I ammeter A and the voltmeter V?
4Q Reading of ammeter A Reading of voltmeter V
A. E1 = 4E2
y
B. Et = 2£2 A. increases decreases to zero
C. E1 = Ez B. decreases decreases to zero
C. decreases decreases
D. 2E1 = Bi D. increases decreases
1
37.
DSE Physics - Section D : M.C.
EM2 : Electric Circuits

< BKCE 1989 Paper II- 33 >


PD-EM2-M/ll

I I 41.
DSE Physics - Section D : M.C.
EM2 : Electric Circuits

< HK.CE 1991 Paper Il- 30 >


PD-EM2 -M /1211
A B C D

'---------<,1�-...,,...,,_--1 In the above circuit, P, Q and R are identical resiston. Which of the following is true when switch Sis closed ?
A. The power output from the battery increases.
B. The voltage across P remains unchanged.
In the circuit shown, a 4 n resistor is tobe connected to two of the terminals A, B, C andD to give the greatestbrightness in C. The cime:nt tbroughQ remains unchanged.
the lampbulb. Which connection should be made? D. The currentthroughP decreases.
A. acrossA.B
B. acrossBC
C. across CD 42. < HKCE 1991 Paper II - 33 >
D. acrossAD In the circuit shown, what are ·the readings of voltmeters Vi and V2
if switch Sis closed ?

38. <HKCE1990Paperll-35> Voltmeter Vt Voltmeter J'i

An electric heater takes a time Ttoboil a kettle of water when connected to a 200 V source. '\Vb.at willbe the time required A. ov 2V 100
to boil the s.une kettle ofwater when the heater is connected to a 00 V source? B. ov 3V
(You may assume that the resistance of the heater remains unchanged.)
1
C. IV 2V
A. T/4 D. 3V ov
B. T/2
C. 2T
43. < BK.CE 1991 Paper II- 31 >
D. 4T
The following diagrams show three networks consisting of different resistors.

�----"
39. <HKCE1990Paperll-32> (1
) ~--+�---!
II: !--+--�

� �
(2) 10 !Mn

6V
( 3) !Mn
In the circuit shown, the potential differencebetweenXand Yis
A. ""'·
B. LS V.
C. 3.0V
D. 4.SV. Arrange the above networks in descending order of equivalent resistances :
A. (1), (2), (3)
B. (2), (!), (3)
40. <HKCE 1990 Paper Il - 34 > C. (2), (3), (1)
D. (3). (!). (2)
In the circuit shown, Li ,Li andL3 are identical light bulbs.
Which of the following statements is/are correct when the
switch Sis closed ? 44. <HKCE 1991 Paper II-1 >
(1) The brightness ofL1 decreases. Which of the following is/are vectors ?
(2) L,. and Ls are of $ame brightness. (1) momentum
(3) Ldsbrighterthan.Lz. (2) power
(3) voltage
A. (2) only A. !) only
B. (3) only B. (�) only
C. (!) & (2) only C. (1) & (2) on ly
D. (2) & (3) only D. (2) & (3) only
I DSE Physics - Section D : M.C.
m.12 : Electric Circuits
PD-
EM M
2-
/13
11
DSE Physics - Section D : M.C.
m.12 : Elec1ric Circuits
PD-EM2-M/14
1

Ji------.
45. <HKCE 1992 Paper II - 29 > 49. < HKCE 1993 Paper II- 29 >
The potential difference between t:wo pointsX and Yin a circuit is 10 V. Which of the following must be true ?
A. 1 J of electrical potential energy is transferred in passing 10 C of charges fromXto Y. �---11
B. SJ of electrlcal potential energy is transferred in passing 2 C of charges fromXto Y.
C. 10 J ofelectrical potential energy is transferred in passing l C of charges fromXto Y.
D. The resistance between%and Yis 10 n.
A,
46. < HKCE 1992 Paper II- 31 >
A,)---C=}-� L--j=:::J---{A,}------'
In the circuit shown above, all resistors are identical and the ammeters are of negligible resistance. If the readings of
A, }---..----J==}--r------+ ammeter A1 is 2 A, find the readings of ammeters Ai and AJ .
s A, A,

A. 2A 2A
A, }----- -C==:J---- .J B. 2A 4A
C. 4A 2A
D. 4A 6A
In the circuit above, what happens to the readings of the three ammeters if switch Sis closed?
Ai Ai A:i

""""""
A. increases increases increases 50. < HKCE 1993 Paper II- 30 >
B. remains unchanged becomes zero increases
C. increases becomes zero Which of the following circuits can be used to measure the resiS!mC� of the light bulb?
A B.
�-----1111---�
D. decreases increases becomes zero

47. <HKCE1992Paperll-32>
A

�----< V I---�

Jn the circuit shown, what is the power dissipated in the 10 O resistor ?


A. 0.8W C. D.
B. 1.6W
c. 2.4 W
D. 3.6W

48. < HKCE 1993 Paper II- 32 >

L,

L,
51. <HKCE 1994 Paper II- 26 >
A student uses an ammeter and a voltmeter to find the resistance
In the circuit, L1, M. andL3 are identical light bulbs. Which of the following statements is/are true? of a light bulb. He incorrectly connects the circuit as shown.
(1) Li andL3 are ofthe same brightness. Which of the following is the most probable outcome ?
(2) Li is brighter thank. A The ammeter bums out V
(3) The power dissipated in L1 is equal to the sum ofpowers dissipated in Li and £3•
B. The light bulb burns oul
A. {2) only
B. (3) only C. The reading of the voltmeter is zero.
C. (1) & (2) only
D. (1) & (3) only
D. The reading of the ammeter is zero.
52.
DSE Physics - Section D : M.C.
EM2 : Electric Circuits

< HKCE 1994 Paper II - 28 >


PD-EM2-M/15

I
56.
DSE Physics - Section D : MC.
EM2 : Electric Circuits

< HKCE 1995 Paper II- 33 >


PD-EM2-M/16
11

A,
A,·'----<

L-----/ V l---_J
In the above circuit, all the light bulbs are identical. What happens to the brightness of the bulbs L1 , k and LJ if switch S is
closed?
In the circuit above, the ammeters have negligi'ble resistance. Which of the following statements are true if switch S is
L, L, L,
closed?
A. decreases increases remains unchanged (I) The reading ofammeter A1 decreases.
B. increases remains unchange d increases (2) Thereading ofammeterkincreases. v'
C. decreases remains unchanged remains unchanged (3) The reading of the voltmeter remains unchanged.
D. remains unchanged decreases increases A. (1) & (2) only
B. (1) & (3) only

�T
C. (2) & (3) only
< HKCE 1994 Paper II- 27 >
D. (1), (2) & (3)
57, <HKCE 1995 Paperll-1 >

4<l
x4" Which ofthe following pairs of physical quantities has the same units ?
A. Charge and current
4!2 B. Workandvoltage
y C. Kinetic energy and heat
D. Force and momentum
Find the p.d. betweenXand Yin the above circuit
A. ov
B. 1 V 58. <HKCE 1996 Paper II- 30 >
C. 2V It is knovm that the resistance ofa resistor R is about 5 kn. Which of the following circuits is most suitable for measuring the
D. 3V resistance ofR? The ammeter and voltmeter used are common moving coil meters.
B.
54. < HK.CE 1994 Paper II- I>
A, ---"II,___
--- I I 1---
Wf!ich of the following is a correct unit for the corresponding physical quantity? R
Physical quantity Unit .--r----,_-1 A
A. Work watt
B. Electromotive force newton '---< V '-----' '----" A l--___,
C, Momentum newton second
D. Heat capacity joule per kilogram
C. D.

I
55. < HKCE 1995 Paper II- 27 >
10n
p I 1----C::::J---, Q

sn
,------[=}--j A,

ron
L--CJ----{ A, 59. <HKCE 1996 Paper II-1 >
In the above circuit, the reading of ammeter A1 is 0.6 A. Find the p.d. between pointsP and Q. Which of the following expressions does not represent energy?
A, 3V A. Force x displacement
B. 6V B. ± x mass x (speed)2
C. 9V C. (Cwrent)2 x resistance
D. 12V D. Cwrent x voltage x time
I
60.
DSE Physics - Section D : M.C.
EM2 : Electric Circuits

< HKCE 1996 Paper II- 27 >


PD-EM2-M/17
11

64.
DSE Physics - Section D : M.C.
EM2 : Electric Circuits

<HKCE 1997 Paper 11-31 >


PD-EM2-M/18

y
X
6Q

y\----

In the circuit shown, the reading ofammeter A2 is 0.3 A. Find the reading ofammeter A1. Jn the circuit shown, X and Y are identical beaters. The ammeter has negligible resistance while the voltmeter has veiy high
A. 0.75A resistance. When switch Sis closed, the ammeter records a reading but the voltmeter reading is zero. Which ofthe following
B. 0.6A provides a possible explanation?
C. 0.SA A. The heater Xburns out.
D. 0.45A B. The heater Ybums out.
C. The heater Xis short.circuited..
D. The heater Yis short-circuited.
61. < HK.CE 1996 Paper II -32 >
65. < HKCE 1998 Paper Il - 30 >
�1[� 6V
---'I[>-----
� 6Q

R
A.H=::i-.
Figure(a) Figure(b)
12"
In Figure (a), the ammeter reading and the total power dissipated in the two identical resistors are I and Prespectively. The
resistors are rearranged as shown in Figure (b). Find the ammeter reading and the total power dissipated ii:!. the two resistors.
Ammeter reading Total power dissipated In the above circuit, the reading ofthe ammeter is 0.4 A. Find the resistance of the resistor R.
A. 3Q
A. 2l 2P B. sn
B. 2l 4P C. 6Q
C. 41 2P D. 9Q
D. 41 4P

66. < HKCE 1999 Paper II- 27 >


62. < HK.CE 1997 Paper II - 1 >
Which ofthe following expressions represents a physical quantity which is different from the others ?
A. Work / Time
B. (Voltagef-/ Resistance
C. Force xVelocity
D. Mass x Specific latent heat of fusion

63. < HK.CE 1997 Paper II- 29 >


All the resistors in the below networks are identical.
(1) (2) (3) A student uses the above circuit to :find the resistance of a resistor R
voltmeterreading
(i.e. resistance ).
ammeter reading
Which ofthe following statements is/are correct?
{1} The am.meter reading records the actual current passing through R.
(2) The voltmeter reading records the actual voltage across R.
Arrange the networks in ascending order of equivalent resistances : {3) The value ofthe resistance ofR obtained is smaller than its actual value.
A. (1), (2), (3) A. (1) only
B. (1), (3), (2) B. (3) only
c. (2), (1), (3) C. (1) & (2) only
D. (3), {!), (2) D. (2) & (3) only
I
67.
DSE Physics - Section D : M.C.
EM2 : Electric Circuits
< HKCE 1999 Paper II- 28 >
I
PD-EM2-M/1911

70.
DSE Physics - Section D : M.C.
EM2 : Electric Circuits

< BKCE 2000 Paper II - 34 >


9V
PD-EM2-M/201

I •
I -
�--<ljt---�

60'2 A
-
60Q

- 30Q
A resistor and a bulb are connected in parallel to a 9 V battery as shown above. The reading of the ammeter is 5 A. H the
power dissipated by the resistor is 18 W, find the power dissipated by the bulb.
In the above circuit, the current delivered by the cell is I. What is the current passing through the 30 n resistor?
A. 9W
A. ½I B. 18W
B. t1 C. 22.SW
D. 27W
C. ¼1
D. t1
71. < HK.CE 2000 Paper II- 36 >

I
'
68. <HKCE 2000 Paper Il - 30 >
I +
I
R1=6.Q
4!l p Q

_fu= 12.Q
Three resistors are connected to a battery as shown above. Which of the following statements is/are correct?
(1) The current passing through R1 1s equal to that passing through .R;;.
(2) The voltage across R1 is equal to that across k
(3) The energy dissipated by one coulomb of charge passing through the 4 !2-resistor is equal to that dissipated by one
coulomb of charge passing through PQ. Q R
A. (3)only
1l, (1) & (2) only
C. (2) & (3) only Bwb
.0.. (1), (2) & (3)
Voltmeter
69. < HKCE 2000 Paper II- 31 >
s The above figure shows an experimemal set-up for measuring the resistance of a bulb. To which of the temrinals P, Q andR
ofthe ammeter and voltmeter should each of the wires be connected 1
R
1--_.._-c':::i--{ A A, Wire (1) Wire (2) Wire (3)
II
R, A. p Q R
j-----C_j-----, A A, B. p R Q
C. Q p R
R, D. R p Q
'-----{C::J------{ A A,
If switch Sin the above circuit is closed, which of the following statements is/are correct? 72. < lll(CE 2001 Paper II - 27 >
(1) The readings of ammeters A1 andA2 are both increased.
(2) The ratio of the readings of ammeters Ai and A2 is increased. Which of the following relations is incorrect 1
(3) The reading of ammeter A3 remains unchanged. A. 1 ohm (0) 1 volt per ampere (V K1)
A. (I) only B. 1 watt (W) 1 joule per second (J s-1)
B. (3) only
C. (1) & (2) only C. 1 coulomb (C) 1 ampere per second (A s-1)
D. (2) & (3) only D. 1 volt (V) 1 joule per coulomb (J C-1)
DSE Physics· Section D : M.C. PD-EM2-M/21 DSE Physics • Section D : M.C. PD-EM2-M/22
EM2 : Electric Circuits EM2 : Electric Circuits
73- < BKCE 2001 Paper II - 30 > 77. <HKCE 2003 Paper II- 33 >
V

+-----{ V }------.
A A,
s
xL.-{=::J--...,---__jy .�------+I
Figure(a) Figure (b)
In the above circuit, the two resistors are identical. If switch Sis closed, which ofthe folloWlllgpredictions is incorrect?
A. The :reading ofthe ammeterA 1 increases. Figure (a) shows a circuit used to investigate how the voltage Vacross a resistance wire varies with the current I through the
B. The reading ofthe ammeter Az remains unchan ged. wire. The result is shown in Figure (b). If the experiment is repeated usin g a thinner wire of the same material and of equal
C. The voltage betweenpointsXand Yin.creases. length, which of the following graphs (denoted by the dashed line) represents the expected result ?
D. The power delivered by the ba ttery increases. A. B. C �

74. < BKCE 2001 Paper II- 29 >


Ifthe ammeter in the circuit reads 3 A, find the voltage of
the battery.
A. 11 V
B. 12V
C. 13.SV
D. 15V rn A
,�,,�'�'�'

7S. <BKCE2002Paperll-33> 78. <BKCE 2003 PaperII-32 >
V A square loop P Q R S is made of uniform resistance wire. Let X, Y andZ be the equivalent resistance of the loop when
y connected as shown below:
X Method of connection Equivalent resistance

«:.---------+I
RegionQ
b:'
S R
X

:q
The figure above shows the voltage-current ( V -1) graphs of two resistors X and Y. Which of the following deductions
is/are correct?
(1) The resistance ofXis higher than that ofY.
(2) IfXand Y are connected in series, the V-I graph of the combined resistor will lie inregionP.
(3) IfX and fare connected in parallel, the V-1 graph of the combined resistor will lie in region Q. y
A. (2) only S R
B. (3) only
C. (1) & (2) only '·
D. {I) & (3) only

DQ
76-. �_HKCE 2002 Paper II -30 >
z
In the circuit shown, both ammeters At and Ai read 1 A
when the switch Sis open. Find the readings of the two R
ammeters when S is closed. ,
A, A,
A. 0.5 A I.SA Which ofthe following relatians is correct?
B. 0.67A lA A. X=Y=Z
C. lA 1.SA B. X=Z>Y
C. X=Z<Y
D. lA 3A
D. X< Y<Z
I DSE Physics - Section D : MC.
EM2 : Electric Circuits
PD-EM2-M/23
11
I DSE Physics - Section D : M.C.
EM2 : Electric Circuits
PD-EM2-M/24
11

�---; �-1
79. <HKCE2004Paperil-31> 83. <HKCE2005Paperll-19>
12V
o----�
6 Vbattery
s

�--IV
In the above circuit, what is the reading of the voltmeter when switch S is closed?
A. zero
B. 6V
C. 12V
An ammeter and a rheostat of range 0-40 n are connected to a 6 V battery. The sliding contact is located at apositionX D. It cannot be detemined since insufficient information is given.
where AX "" ¼ AB as shown above. Find the ammeter reading.
A. 0.ISA Questions 84 and 85 :
B. 0.2A
C. 0.45A
D. 0.6A then:nistor

80. < HK.CE 2004 Paper II- 28 >


The photograph shows a rechargeable cell If the cell is charged
for 16 hours using the standard charge mode, estimate the total
amount of charge flowed through the charging circuit A teacher sets up the circuit as shown in the above Figure and provides the following information to her �ts.
A. 182.4C
B. 3040 C
Standard charge 16hours at 190mA Thermistors are devices whose resistance varies with temperature. The variation of
C. 10 944C
Fast charge 5 hours at 475 mA the resistance of the thermistor used with temperature is shown in the figme below.
D. 27 360 C
Resistance / .Q

81. < HK.CE 2004Paper II - 29 >

----s•-l----�
An ammeter with negligl.Ole resistance and a high-remtance voltmeter 6V
are connected into a circuit as shown. Find the mnneter and voltmeter
readings.

Am.meter reading / A Voltmeter reading / V 20


A 0 2

'
B. I 2 84. < HKCE 2005 Paper II - 40 >
C. 3 2 Which of the following shows the variation of the ammeter reading I with the temperature ofthe thermistor{)?
D. 3

w
82. < HKCE 2084 Paper II - 32>
6V 12V

c::J 85.
'.t=..

<BKCE2005Paperil-41>
. .L. Cb. .. -�·

t=. L==.. �- �-
Which of the following shows the variation of the voltmeter reading Vwith the temperature of the thermistor fJ?
A B. Q �
In the above circuits,X, Yand Z are identical resistors. The power dissipated in Xis 20 W. Find the total power dissipated in
YandZ.
A. lOW
B. 20W
C. SOW
D. 160 W
I
86.
DSE Physics - Section D : M.C.
EM2 : Electric Circuits

< BKCE 2005 Paper II • 18 >


PD -EM2-M/ 2511

I
89.
DSE Physics - Section D : MC.
EM2 : Electric Circuits

< HKCE 2006 Paper II- 23 >


PD-EM2-M/2611

-----/-/"\+----/ A A, L......Cx
,__ �-\
Ifthe current in the circuit shown is 0.8 A, what is the number ofelectrons passing through the pointX in l minute?
A. 8.3 X 1016
B. 1.6 x 1017
ill the above circuit, the bulbs are identical. If ht e reading ofammeter A1 is 1A. find the readings ofammeters A:zandAl, C. 5.0 X 1018
Reading of.Al Reading of.43 D. 3.0 X 102°
A. 0.5A lA
B. 0.5A LSA 90. < BKCE 2006 Paper II- 24 >
C. 2A 2A
D. 2A 3A !·-·---·-·---·-·-·-·-·-·-·-·-·-· ;
87. < HK.CE 2005 Paper IT· 39 >
p
I
+ ! 1.SV -
i 1.SV
; 1.SV -
'
'
!
)1.SV--
' .
'-·-·-·-----·-----·-·-·-·-·-·-· '
i.-•-•-·-•••-•-•-•-•-•-•-·-•• I
battery box
The diagram shows the circuit of a camping light which uses four 1.5 V cells and four identical light bulbs. The current
passing through each light bulb is I A. What is the cun'ent passing through the point P and what is the power of each light
bulb?
Current passing through the pointP Power of each bulb
A. 4A 6W
B. 4A !SW
A student wants o
t measure theresistance ofaresistor Rand sets up a circuit as shown above. Which of the following C. IA 6W
descnDe(s) themistake(s) made by the student in setting up the circuit? D. lA l.SW
{1) The polarity of the ammeter is reversed.
(2) The polarity of the voltmeter is reversed.
(3) The voltmeter is connected across both R and the rheostat. 91. < HKCE 2006 Paper II- 26 >
A. (1) only
B. (2) only
c. (1) & (3) only
D. (2) & (3) only

88. <HK.CE 2006 Paperll-35 > ----- -----�


,.- -
Lithium cell : 3.6V
Capacity: 800mAh
Time (stand-bymode): about3 days
Three identicallight bulbs are connected to a cell as shown in the diagram. After some time, the filament of bulb Z breaks.
The figure shows a ab l el on a lithimn cell of a mobile phone. The capacity 800 mA h :indicates the quantity of electric What will haPPen to the brightness ofthe remirining light bulbs ?
charges that 1he cell will discharge in 3 days when the mobile phone is in stand-by mode. Estimatethe averagepower of the

"""""'
Brightness ofX Brightness of Y
cell assumlllg the vota
l ge remains constant during the discharge.
A. 16mW A increases
B. 40mW B. increases decreases
C. 120mW C. decreases increases
D. 960mW D. decreases decreases
92.
DSE Physics - Section D : MC.
EM2 : Electric Circuits

< HKCE 2007 Paper II - 20 >


"' 2Q
PD-EM2-M/27
1
1
96.
DSE Physics - Section D : M.C.
EM2 : Electric Circuits

< BK.CE 2007 Paper II- 41 >


PD-EM2-M/28
11

battery

X y

6Q 4Q
In the circuit shov;n, a voltage of 10 Vis applied acrossXY. What is the current passing through the 8 Q resistor?

amm-
A. 0.5 A
B. 0.8 A voltmeter
C. IA
D. 2A

93. < HKCE 2007 Paper n - 21 > B

A student sets up the above clrcuit to study the effect of the length of a resistance wire on its resistance. As the crocodile
lQ clip is moved fromA to B along the resistance wire, which of the following statements is/are correct?
2'l (1) The readlllg of the ammeter lllcreases.
A (2) The reading ofthe voltmeter lllcreases.
sn (3) The voltmeter reading: is directly proportional to the reading of the ammeter.
�-
A. (l)only
If the ammeter in the above circuit reads 3 A, what is the voltage of the battery? B. (2) only
A. 8.SV C. (1) & (3) only
D. (2) & (3) only
B. 9.6V
C. 21 V 97. < HK.CE 2008 Paper II- 20 >

�:-�1,
D. 24V
The figure shows two light bulbs A andB, which are connected iD series.
The voltages across A and B are 3 Vand l V respectively. Which of the
94. < HK.CE 2007 Paper 11 - 22 > following statements is/are correct ?
(1) The resistance of A is greater than that ofB.
The voltage and the capacity o�Jhe-ce� of a portable music player are 3.6 V and 70_0 mA h respectively. The continuous , (2) The current £lowing through A is greater than that through B.
playing time of the player is 1"5 hours./The capacity 700 mA h indicates the quantity of electric charges that the cell will (3) AisdimmerthanB.
discharge in 15 hours ofplayirig;• Assume that the voltage remains constant during discharge, what is the estimated power of A. (1) only
the player when it is playing? B. (3) only

A 0.047W C. (1) & (2) only
B. 0.168W D. (2) & (3) only
C. 0.194 W
98. < HKCE 2008 Paper Il - 23 >
D. 0..252W
A cell fully discharges at a constant ljUITCD.t225 mA in 10 hours. If its average voltage is 1.2 V, what is the estimated total
energy stored in the cell ?
95. < HKCE 2007 Paper II- 23 > A. 2.71
B. 67501
In the following circuit, three identical light bulbs are connected to a cell. Under what conditions will light bulb P have the C. 8100 J
maximum brightness? D. 91201

99. < HK.CE 2008 Paper II- 21 >

6'2
12'2 A.}-c:'.=f-i
6Q
SwitchX Switch Y
In the above circuit, the reading of the ammeter is 0.3 A. What is the voltage Vofthe battery?
A. open closed A. 4.8V
B. open open B. 5.4V
C. closed closed C. 9.0V
D. closed open D. 14.4 V
I DSE Physics - Section D : M.C.
EM2 : Electric Circuits
100. < HKCE 2008 Paper Il-18 >
PD-EM2- M/29

II 1 DSE Physics - Section D : M.C.


EM2 : Electric Circuits
103. < HKCE 2010 Paper II- 18 >
1.SV
PD-EM2-M/30
I
--0--
Method of connection Equivalent resistance

y- y
In the circuit above, voltmeters P and Q of very high resistance are connected across the switch and the light bulb
respectively. What are the voltm.eter readings when the switch is open ?

A.
reading of P JV
0
reading of Q/V
0

--y
B. 0 1.5
C. l .5 0
z D. LS 1.5

"'* '-l-----+------§'
104. < HKCE 2010 Paper II- 21 >
x
A circular loop is made of uniform resistance wire. Let X. Y and Z be the equivalent resistance of the loop when connected
___,_a p
R

cp,
as shown above. Which of the following is correct ?
A. X= Y=Z
B. X> Y=Z
C. X<Y=Z
D. X<Y<Z

101. <BKCE 2009 Paperll-19 > Q s


Three identical bulbs x;YandZ ate connected to a 12 V battery in the circuit above. Which of the following statements are

�-- •--1--�
Two identical resistors are connected in series in the circuit shown.
12V correct?
What are the readings of the voltmeter when S is open and when S
is closed respectively 7 (1) The voltage across PQ is greater than 6 V.
(2) The voltage across QS is zero.
Sis open Sis closed
s (3) If Ybums out and becomes open circuit, Zbecomes brighter.
""' A. (1) & (2) only
A.
B.
C.
=•
12V
6V
12V
6V '-------{v
B. (1) & (3) only
C. (2) & (3) only
D. 12V 12V D. (!), (2) & (3)

102. <IIKCE 2010 Paperll-17> 105. < HKCE 2010 Paper II- 41 > s
A car bas two doors and an indicator lamp is installed to act as j\ car door watlling signal. When either or both doors are
"'

E�
open, the lamp lights up. A student designs four different circuits for this application. A switch is installed in each door.
The switch is open when the car door opens. Which ofthe following circuits is correct ?
A. R
R

1n

Q
A
c. D. �
In the circuit shown, wires Xand Yare to be connected to two points in the resistor networkPQRS. Across which two points
should.X and Ybe connected to so that tbeammtu:r reading will be a .minimum ?
A. Pand.Q
B. QandR
C. RandS
D. SandP
I DSE Physics - Section D : MC.
EM2 : Electric Circuits
PD -EM2-M/31 I I DSE Physics - Section D : M.C.
EM2 : Electric Circuits
PD-EM2-M/32 II
106.. < HKCE 2011 Paper Il - 18> Part B : HKAL examination questions
The figure below shows the current-voltage (l-V) graphs oftwo resistors P and Q.
I 110. < HK.AL 1980 Paper I- 21 >
p The voltmeter reads 3 V in the circuit shown. The e.m.f. of the battery is
15 V. If the internal resistance of the battery is negligible. What is the
internal resistance of the voltmeter ?
Q 3kn

,1,_____
A 0.75kn lk.G
B. I.SO kn
C. 3.00kn '--------i
D. 3.75kn lSV
Which ofthe following statements about the two resistors are correct ?
(1) Both ofthem obey Ohm's law.
(2) The resistance of Pis greater than that of Q. 111. <BKAL1980Paperl-47>
(3) If they are connected in parallel to a battery, the current passing through P will be greater than that passing
throughQ. When the switch Sis closed in the circuit shown, only L1 lights up.
A. (1) & (2) only Which ofthe following possibilities would account for this ?
B. (1)&(3)only (1) There is a short circuit across 4
C (2) & (3) only (2) The :filament ofLz is burnt out.
D. (1), (2) & (3) (3) The :filaments ofbothl,z and[,,. are burnt out. 2V

I ?.
A (1) only
107. < HKCE 2011 Paper Il- 19 > B. (3) only
C. (1) & (2) only
Which of the following resistor networks gives the largest equivalent resistance across PQ if all the resistors have the same D. (2) & (3) only
resistance?
A. B.
112. < HKAL 1982 Paper I- 22 >
In the circuit shown, an electron travelling from Yto X through R1 loses
energy E1, and an electron travel.ling from Yto Xtbrough R2 loses energy
E2. What is the relation betweenE1 andE2?
c. D. A. 4E1 = Bi xr-----,==i----➔r
B. 3E1 = E2 & = 30
C. E1 = Bi
p Q D. Ei = 3E2

113. <HKAL1983PaperI-20>
In the circuit shown, X and Yare two identical cells, of e.m.f. 10 V and
108. < HKCE 2011 Paper n - 20 > 6Q internal resistance 4 n. What is the current given out by each cell?
A. zoro
Jn the circuit shown, the ammeter reading is LOA when Sis open. X SQ y
What is the ammeter readingwhenS is closed? B. 0.42A
A
A. I.SA C. 050A
60
B. 2.0A D. 0.83 A
C. 2.5 A 3!l s
D. 3.0A
114. < HK.AL 1984 Paper I- 21 >
109. < HKCE 2011 Paper Il- 40 > An ideal voltmeter is connected between points X and Yin the circuit 12V 3!l
shown. Find the readings of the voltmeter when the switch is open and
In the circuit shown, both bulbsXand Yligb� up normally. Both the closed respectively.
ammeter reading and the voltmeter reading are non-zero. Which of
the following will cause theammeter reading to drop to zero while
X
y
A
switch open switch closed
the.:voltmeterreading is stillno'i-zero? Assume meteria!'e ideal. A. l.SV 25V y
A. Bulb Xbecomes short circuit. B. 2.SV 8.0V
B. Bulb Ybecomes short circuit.
C. BulbXis burnt out and becomes open circuit. C. 12V 4.SV
'-----/ V }-----_J
D Bulb Yis burnt out and becomes open circuit. D. 12V 75V
11
115. <HKAL 1985 PaperI-44 >
DSE Physics - Section D : M.C.
EM2 : Electric Circuits
PD-E
M2-M/3
311 I
119. <HKAL1989P.aperI-28>
DSE Physics - Section D : M.C.
EM2 : Electric Circuits
PD-
EM 2-M
/ 3411
,

�;�7
The e.m.f. of a battery is equal to
(1) the electrical power it given out divided by the cunentit delivers.
(2) the elecirical energy it transfers to unit coulomb of charge passing through the battery.

CL---��
(3) its terminal voltage when the battery is on open circuit.
A. (3) only
B. (1) & (2) only
C. (2) & (3) only In the circuit shown, Vis a voltmeter nfbjgh intemal resistanc'I and A is an ammeter oflow internal resistance. What is the
D. (1), (2) & (3) voltmeterreading when (a) switchK is open, (b) switchK is closed?
Kopcn K closed
116. <RKAL 1986 Paperl-34 >
"' sn ,n
A.
B.
C.
av
av
3V
OV
2V
2V
D. 3V av

A
"' X y "' B 3n
120, < HKAL 1989 Paper I- 29 >

"'
In the circuit shown, four resistors are connected to form a network.
"' "' The equivalent resistance between A andB is
A. o.s n.
sn�sn

A�t
In the circuit shown, if a battery of e.m.f. 20 V with neglig1Dle internal resistance is connected across XY, what is the current B. 120.
given out by this battery ? c. un.
A. 1.0A. o. 2.on.
B. 2.0A. 40
c. 2.5 A.
D. 4.0A 121. <HKAL 1990 PaperI-33 >
In the circuit shown, the battery has negligible internal resistance.
The cuttentl delivered by the battery is
117. < BKAL 1987 Paper I - 31 >
A. 0.5 A.
B. 0.9 A.

A"--'r---..-c=:H
s B sn C. 1.2A.
D. 1.5 A.

·,.L ·,.� ",t= · . b


2V
122.. < BKAL 1990 Paper I- 34 >
In the above figure, AB is a resistance wire of uniform cross--section. and S is a sliding contact The 2 V battezy has'
negligr.ble intemal resistance, and the connected voltmeter is ideal 'Which of the fullow:ing graphs shows the correct Which ofthe following circuits is best used for the measurement of a high resistance R?
variation of voltage mea51Il"ed by the voltmeter when the contacts is moved from A to B? A -

+ R
A -

A B A B A B A B

118. <HKAL 1989 Paper I- 31 > C. D.


In the circuit shown. AB is a metre-wire ofresistance 12 n. V&en Xis moved �
to the mid-point ofAB, the p.d. across AX will be
R + R
A. 0.9V. A -
B. 1.2V.
C. UV.
D. 1.8 V.
I DSE Physics - Section D : M.C.
EM2 : Electric Circuits

123. < HKAL 1995 Paper 11A - 23 >


PD-E
M2-M
/35
11 Ii
127. <HKAL1999PaperIIA -17>
DSE Physics - Section D : M.C.
EM2 : Electric Circuits
PD-E 2�1
M

Two cells of negligible internal resistance are connected with two resistors Three resistors of resistanceRi,Ri andR; are connected in parallel. It is known thatR1 > R2 >RJ. The equivalent resistance
as shov..n. What is the potential difference between% and Y? of this combination isR. Which of the following statements is/are correct?
A. 1.33 V (1) Energy dissipated in moving l C ofcharge through the resistor ofresistanceR1 is greater than that throughR2.
(2) RissmallertbanR1.
B. 1.67V (3) Ifthe resistor with resistanceR; is removed, the resulting equivalent resistance is increased.
C. 2.00V A. (I) only
D. 233V B. (3)only
C. (!) & (2) only
D. (2) & (3) only
124. < HKAL 1997Paper IIA- 25 >
Two cylindrical wires, Xand Y, are made from the same metal and have the same volume. The length ofXis three times that

�----1
128. < BKAL 1999Paper IIA - 19 >
of Y. If currents of 1 A and2 A pass tbroughXand Yrespectively, the ratio of the power dissipation inXto that in Yis
A. 1: 4 In the circuit shown, the battery has negligible internal resistance.
B. 9 :2 The three resistors have same resistance. If switch S is closed, what
C. 3:4 would happen to the electric potential at pointsX and at Y?
D. 9:4 Potential at X Potential at Y
R,
A. increase in,,,.,, X R,
y
125. < HKAL 1998Paper llA - 19 > B. increase decrease
,n.,,.,,, R, ,j s
=
current/ A C. decrease
D. decrease doo,ease
5.0 -------------------
129. < RKAL 1999Paper IIA- 16 >

1.0
'-====---'------'-----�
,._.___......, V ,._______,
o voltage/ V
10 20
The variation of current with the voltage applied across a device is as shown in the figure. 'What is the change in resistance R
of the device when the voltage in.creases from IO Vto 20 V? The above circuit can be used to find the r esistance of the resistor R. The voltmeter and the ammeter are not ideal. Which of
A. It increases by 2.5 .n. the following statements is/are correct?
B. It decreases by 15 .0.. (I) The reading of the ammeter is in fact larger than the actual current passing throughR,
c. It increases by 6 n. (2) The ratio ofvoltm.eterreading to ammeter reading is in fact smaller than the resistance ofR.
D. It decreases by 6 .Q_ (3) The circuit is suitable fur measuring high resistance.
A. (!) only
'11, (3) only
126. <HKAL1998PaperlIA-39>
--.....,11 ......--
C. (!) & (2) only
TJ. (2) & (3) only

130. < HKAL 2000Paper IlA - 23 >


s

In the above circuit, the two resistors are identical. The battery has an e.m.£ of 4V and it has negligible intemaJ. resistance.
The voltmeter is ideal. What are the voltmeter readings 'Ml.en Sis open and when Sis closed ? 1Q R
The figure shows part of the circuit in a network of resistors. According to the matl<:.ed values, find the magnitude and
Sopen Sclosed
direction ofthe current passing through the resistorR.
A. 2V 4V A. 0.2 A from right to left
B. 2V 2V B. 0.2 A from left to right
C. ov 4V C. 0.4 A from right to left
D. 4V ov D. 0.4 A from left to right
I DSE Physics - Section D : M.C.
EM2 : Electric Circuits
131. < HKAL 2003 Paper IlA - 25 >
6V
PD-E
M2-M 3
/ 7
11
DSE Physics - Section D : M.C.
EM2 : Electric Circuits
135. < HKAL 2008 Paper 11A - 15>
PD-EMZ-M/ 81
3

In the circuit shoVlll, a battery of e.m.f. 6 V and negligible


internal resistance is connected to three resistors. What are
the electric potential at Xbefore and after switch S is closecj. ?
�-----a1111---� current JI mA '
100
0
Before After
0 ./
A, +6V +3V 1./
I/
40
B. +3V +3V s 0
C. +2V +2V /
D. +2v +3V voltage V/V
4
The above graph shows theJ-Vrelationship of an electric device. V/bich ofthe following statements is/are correct?
132. < HKAL 2003 Paper IIA- 27 >
20 (1) When the applied voltage is 0.5 V, the resistance of the device is mfuilie.
In the circuit shown, the battery bas constant e.m.f. and
negligible in� resistance. An ideal voltmeter connected
,----C:=f---r---C:=}---x (2) When the applied voltage is between 1 V and 5 V, the resistance ofthe device is constant.
(3) -when the applied voltage is between 1 V and 5V, the current is directly proportional to the voltage.
across terminals Xand Yreads 2 V. Ifan ideal ammeter is A. (1) only
connected across X and Y, the ammeter should read B. (3) only

'--c=:::i-----c=:::i-
"' -
2"
A. 0.3A C. (1) & (2) only
B. 0.5A D. (2) & (3) only
C. 0.6A 20
D. LOA y 136. < HKAL 2009Paper IlA- 28 >
p
133. < BKAL 2003 Paper IIA- 20 >
R X
B=---C:=J--�C p
R
9V-=-
100 R V
y
A'----C:=J----">oD Q '-----Q
In the network of resistors shown above, the resistance of Sis infinitely large and the two resistors R are identical. If the Figure(a) Figure (b)
equivalent resistance across CD is 25 n, what is the equivalent resistance across AC? Figure (a) shows a rotary-type potential divider PQR connected to an 9 V battery of negligible internal resistance. The
A. 250 internal structure of the potential divider is shown in Figure (b). XYZ is a uniform resistance wire in the form of a circuit arc
B. 35.0 with centre O and LXOY= 90°. OZ is a sliding contact and LXOZ= 120°. Find the reading of the ideal voltmeter.
c. son A. 2.5 V
D. infinitely large B. 4.0V
C. 5.0V
D. 6.8V
134. <IIKAL 2007 PaperllA- 16>
voltage VIV
137, <.HKAL 2010 PaperJIA- 23 >
In the network shown, the resistance ofeach resistor is 6 n.
4 Find the equivalent resistance·ofthe network across a and b.
I
2
0 ' A. 1.20
B. 1.5n
C. 2.00
;/ Y, D. 2.4Q
6
4
'
2 138. <HKAL2010PaperllA-25>
� current!/ A
0.2 Which of the following statements concerning an ammeter are correct ?
0.4 0.6
The above figure shows the V-1 characteristic curves ofresistance wire X and filament bulb Y. If they are connected in series
t0 An ammeter should be connected in series to a circuit.
R,] An ammeter should·have a low resistance.
to a 12 V d.c. supply ofnegligible internal resistance, what is the voltage across the resistance wireX? (3) An ammeter ofhigh resistance significantly changes the current in the circuit to which it is connected.
A- 9V A. (1) & (2 ) only
B. 8V B. (1) & (3) only
C. 7V C. (2) & (3) only
D. 6V D. (1), (2) & (3)
I DSE Physics - Section D : M.C.
EM2 : Electric Crrcuits
139. < HKAL 2013 Paper IIA- 24 >
PD-E
M2-
M/3
911 I
143.
DSE Physics - Section D : M.C.
EM2 : Electric Circuits
PD-
EMZ
-M/
4011

In the circuit shown, the battery has negligible internal resistance.


The three resistors have the same resistance. Which statement is
NOT correct when switch Sis closed ? A X
A. The potential difference acrossP and Q remains unchanged.
B. The electric potential atP increases. p
C. The electric potential at Q decreases.
D. The CUITent flowing through resistor R2 decreases. y

In the above circuit,XYis a -uniform resistance 'Mte. What is the change of the reading of the ammeter A when the sliding
}40. < BKAL 2013 Paper IlA- 25 > contact Pis moved fromXto Y?
,n A. graduallyincreases
B. gradually decreases
C. increases and then decreases
D. decreases and then increases

144, Which of the following statements concenrlng two identical resistors ' connected in parallel compared with one resistor is/are
3V correct?
I.n the above circuit, the cell has an e.m.f. of 3 V and negligible internal resistance. Ammeters Ai and A2 have zero (1) Two resistors consume more power than one resistor,
resistance. Whatare the readings ofthe two ammeters ? (2) Two resistors draw more cittrent than one resistor.
ammeter Ai ammeterAz (3) The equivalent resistance oftwo resiston; is greater than that of one resistor.
A. LOA 1.5A A (!) only
B. ISA 3.0A __B. (3) only
C. l.SA 4.SA C. (!) & (2) only
D. 2.SA 4.SA D. (2) & (3) only

145. Which of the following is a correct statement ofOhm's Law ?


A. The resistance of a conductor is always constant
Part C : Supplemental exercise B. The voltage across a conductor is always proportional to the current flowing1hrough it.
C. The resistance ofa conductor increases with the temperature.
141. Two light bulbs Pand Qare connected in parallel t.o' a power supply. The resistance of Pis greater than that of Q. Which of D. The resistance ofa conductor is constant only ifthe temperature of the conductor is constant
the following statements is/are correct?
(1) The voltage aeross Pis greater than that across Q.
146. L2A
(2) The current through Pis smaller than.that ofQ. 2.0A
(3) Pis brighter than Q.
A. (1) only
B. (2)only
C. (!)& (3) only
. D. (2) & (3) onl y SQ

142. Two light bulbsP and Qare CODD.ected in series to a power supply. The resistance ofPis greater than that ofQ. Which of
the following statements is/are correct?
(I ) The voltage across Pis greater than that across Q.
A cell has a constant e.m..t: and internal resistance. If a resistor of resistance 5 n is connected in series with. the cell, the
(�) The current tbroughP is smaller than that ofQ. current is 2.0 A. If the resistor is replaced by another resistor of resistance 10 n, the current in the circuit becomes l.2 A.
(3) Pis brighter than Q. 'What is the value'ofthe internal resistance?
A. (1) only A. 1.SQ
B. (2) only B. 2.on
C. (!) & (l) onl y c. 2sn
D. (2) & (3) only D. 3.0n
I DSE Physics - Section D : M.C.
EM2 : Electric Circuits
PD-EM2-M/41 II DSE Physics - Section D : M.C.
EM2 : Electric Circuits
PD-EM2-M/42

Part D : HKDSE examination questions 150. < HKDSE Practice Paper IA - 29 >

147. <HKDSESamplePaperlA-27>
�-�1��A k
y

1---+n.\----i A k p

In the circuit shown, three identical light bulbs are connected to a cell. Under what conditions will light bulb P have the
maximmn brigh1ness?
In the above circuit, the bulbs are identical. If the reading of ammeter A1 is 1 A, find the readings of ammeters k and/4. SwitchX SwitchY
Reading of.k Reading of -43 A closed op ,n
B. closed closed
A 2.0A 2.0A C. op,n open
B. 2.0A 3.0A D. open closed
C. O.SA LOA
D. O.SA I.SA
151. < BKDSE Practice Paper IA - 27 >

c-+--1'--------------·
' : •I
148. <HKDSE Sample Paper IA- 30 >

+
:'

The :figure shows a battery ofo.m.f. 3.0 V and internal resistance 2.0 0: is connected to a light bulb of resistance 10.0 n.. A
voltmeter of internal resistance 10 ill is connected in parallel with the light bulb. What is the reading of the voltmeter?
A. 2.4 V
B. 2.5 V
C. 2.9V
D. 3.0V
A student wants to measure the resistance ofa resistor R and sets up a circuit shown. The student made which of these
mistakes in setting up the circuit ?
(1) The polarity of the ammeter was reversed. 152. < BKDSE Practice Paper IA - 28 >
(2) The polarity of the voltmeter was reversed.
(3) The voltmeter was COilllecied across both R and the rheostat
A. (I) only
B. (2) only
C. (!) & (3) only
D. (2) & (3) only

149. <BKDSE Practice Paper IA- 26> :Figure (a) Fignre(b)


Two metal rods, X and r, of wllfonn cross-sectional area are made of the same material and have the same volume. The
In Figure (a), two identical resistors are connected in series to a cell of e.m.£ V and negligi.Dle internal resistance. The power
length and resistance ofX are L and R respectively. What is the resistance of Y if it has a length of2L ?
dissipated by each resistor is P. If the two resistors are now connected in parallel as shown in Figure (b}, what is the power
A. ¼R dissipated by each resistor ?
B. ½R A. 2P
C. 2R B. 4P
C. SP
D. 4R D. !6P
I DSE Physics - Section D : M.C.
EM2 : Electric Circuits
153. < HKDSE 2012 Paper IA - 26 >
R
PD-
E M2 M
-
/431
1
DSE Physics - Section D : MC.
EM2 : Electric Circuits
157. <HKDSE 2013 PaperIA-31 >
PD-EM2-M/441

.�� L,

a�b
12"
In the above network, the resistance across terminals a and bis 6 n. If the 12 n resistor is replaced by a 6 n resistor, the
resistance across terminals a and b In each of the above circuits, the cell has constant e.mf: and negligible internal resistance. When the sliding contact S of
A. becomes 2 n. each rheostat shifts from the mid-position to the right, how would the brigh1ness of each bulb change?
B. becomes 4 n. bulbLz
bulbL1
C. becomes 6 .Q..
D. cannot be found as the value of R is unkno'M.l. A becomes dimmer remains unchanged
B becomes dimmer becomes brighter
C remains unchanged becomes dimmer
154. < HKDSE 2012 Paper IA- 27 > D becomes brighter remains unchanged
What will happen ifthe variable resistor is set to zero in the circuit?
A. The light bulb will bum out 158. < HKDSE 2013 Paper IA- 32 >
B. The light bulb will not light up.
C. The brightness ofthe light bulb will increase.
D. The brightness ofthe light bulb will remain unchanged.

"'
y
155. < HKDSE 2012 Paper IA- 28 >
.___,V,____.
X

6Q In the above circuit, the cell has.negligj.Jili}�.s.istance, When switch Sis closed, both bulbs are not lit. The voltmeter
has a reading but the ammeter reads zero. If only one fault has been developed :in the circuit, which of the following is
possible?
In the above circuit. the cell has e.m.£ 12 V and internal resistance 2 n. What is the current in the 6 n resistor? A. Bulb Xhas been shorted accidentally.
A. 0.5A B. Bulb Yhas been shorted accidentally.
B. LOA C. Bulb Xis burnt out and becomes open circuit
C. 1.5A D. Bulb Y is burnt out and becomes open circuit
D. 2.0A

159. < HKDSE 2014 Paper IA - 25 >


156. < HKDSE 2013 Paper IA - 30 > 6V

X
y

z
Resistors X, Y and Zin the above circuit are identical wiu1e the battery of negligible internal resistance supplies a total power The figure shows two light bulbs P and Q connected to a cell of e.mf. 6 V and negligible internal resistance. The voltmeter
of24 W. What is the power dissipated in resistor Z? reads 6 V when the switch Sis closed. Which of the following is possible?
A 3W A. BothP and Qare short-circuited.
B. 4W B. BothP and Qare burnt out and become open circuit.
C. 6W C. P is short-circuited or Q is bumt out and becomes open circuit.
D. SW D. Pis burnt out and becomes open circuit or Q is short-circuited.
I DSE Physics - Section D : M.C.
EM2 : Electric Circuits
PD-E
M2-M
/451
1 I DSE Physics - Section D : MC.
EM2 : Electric Circuits
PD-EM2- / 46
M
II
160. <BKDSE 2014Paper IA- 24 > 163. <HKDSE 2016 Paper IA- 25 >
0- l0Mn
ion

"' lOkQ
R
+---�,
In the above circuit, the variable resistor R can be adjusted over its full range from 0 to 10 MO. What is the approximate
range of resistance between a and b ?
In the above circuit, the cell has constant e.m.£ and a fixed intemal resistance. When Sis closed, the ammeter reads 3.0A. A 00.to l0kQ
When Sis open, which of the following is apossfble reading of the ammeter? B. l0Qtol0kQ
A. 1.6A C. l0Otol0MQ
B. 2.0A D. l0kQto l0MQ
C. 2.4A
D. 3.2A
164. <HKDSE 2016 Paper IA- 26 >
161. < HKDSE 2015 PaperJA - 26 > Two :filamentlightbulbsX and Yare connected in parallel to a dry cell. Xis brighter than Y. 'Which statements are correct?
(1) In 1 s, the number of charges flowing tbroughX is greater than that flowing through Y.
(2) In 1 s, the electrical energy dissipated by Xis greater than that dissipated by Y.
R, (3} For every unit charge passing, the electrical energy dissipated by Xis equal to that dissipated by Y.
r---=}-----{A 'A. (lJ & (2) only
,'--------' B.' (1) & (3) only
C. (2) & (3) only
D. (!), (2) & (3)
In the above circuit, the cell has a finite intemal -resistance and both meters are ideal. In which situation below will the
readings of the ammeter and the voltmeter suddenly increase? 165. < HKDSE 2016 Paper IA- 27 >
A. R1 is faulty and becomes a short circuit.
B. & is faulty and becomes a short circuit.
C. & is faulty and becomes a short circuit.
D. & is faulty and becomes an open circuit.

162. < HKDSE 2015 Paper IA-25 >


For safetypmposes, the driver seat of a car is equipped 'With a seat belt warning light. When the driver seat is occupied, the s
switch S 1 -under bis seat will close. If the seat belt is not yet fastened, switch S2 will remain open and the warning light will
light up. Iftbe seat belt is fastened, the switch Sz will close and the waming light will shut off. Which circuit below is the
bi:st design ?
A. B.
In the above circuit, all the bulbs are identical If the voltage V gradually increases, which bulb(s) will bum out first?
A. PandQ
B. R
C. S
D. T

166. <BKDSE 2017PaperIA-24 >

C. D.
In the circuit, all resistors ·are identical. The internal resistance
of the battery can be neglected. Vlhat is the potential difference I 91'
Ii-- •
\s,
betweenXand Y?
A. 1.5V

I
B. 3.0V y
C. 4.5V
s, s,
X
D. 6.0V
167. <HKDSE 2019 Paper IA-24> 169. <HKDSE 2020 Paper IA-22>

prn<lllce the most


=OW 1W w same metal. Which one would
uum Ae
=
u=• �
n,sistors 0
_, _..., •
The cylindri<:al
. thetw oen dso feaeh resistor ?
powerwhen the same voltage ,s --
• app6_,.m •�- aeross

A-

B.

C.

D.

170. <HKDSE 2020 Paper JA-23>

ibiee identical resistors,. a battery ofnegligi'ble internal resistance and an ideal voltmeter are connected
168. <HKDSE 2019 Paper JA-25> to form Circuits (a) and (b) .respectively.

._______.,, ___, '------11/t------'


Cucuit(a) Cucuit(b}

Given that the voltmeter reading is & V in Circuit {a). what is the voltmeter reading in Circuit (b) ?

A. 4V
B. 6V
C. 8V
D. 12V
I DSE Physics - Section D : M.C. Solution
EM2 : Electric Circuits
PD-EMZ-MS/0111

HKEAA.'s M.axking Scheme is prepared for the :ma.tkeu' refe:ren.cc. It sboo1d not be regard.Ii as a set of model answers.
Stwients and t:cacbers who are not invo1'Vcd in tbc inarldug process are advised to inter;Ptet 1he Maddng Scheme with care. M.C. Solution
DSE Physics - Section D : M.C. Solution
EM2 : Electric Circuits
PD-EM2-MS/02

M.C. Answers 1. D
Voltage given to the upper two resistors = 6 V
1. D 11. C 21. C 31. C 41. A 51. D
Since voltage is proportional to the resistance for two resistors in series,
2. A 12. B 22. D 32. C 42. A 52. A
3. D 13. A 23. B 33. B 43. C 53. C voltage across the upper 8-n resistor = 6 x _8_ = 4V
4+8
4. C 14. B 24. B 34. D 44. A 54. C potential difference between points P and Q = 4 V
5. B 15. D 25. A 35. C 45. C 55. A
2.
6. C 16. C 26. C 36. A 46. C 56. C
When K is closed, the voltage across the upper resistor Rremains unchanged.
7. B 17. D 27. A 37. C 47. B 57. C
Thus same current flows through the ammeter A, reading of ammeter will not change.
8. C 18. D 28. A 38. D 48. C 58. A
9. A 19. C 29. C 39. B 49. D 59. C 3. D
10. D 20. A 30. C 40. D 50. B 60. A Kopen:
all voltage of the cell would. be across the voltmeter since its resistance is very large
61. D 71. A 81. D 91. C 101. C 111. A v..-2v
62. D 72. C 82. D 92. C 102. D 112. C
K closed:
i 63. A 73. C 83. D 93. A 103. C 113. C all voltage of the cell would be across the 100 n resistor but no voltage across the ammeter
64. C 74. A 84. B 94. B 104. C 114. D V=0V
65. C 75. B 85. A 95. D 105. D 115. D
4. C
66. D 76. C 86. D 96. A 106. B 116. B
As the length is halved, the resistance of the heating coil is also halved
67. A 77. D 87. C 97. A 107. D 117. C
since Ro::£:. R ➔ ½R
68. C 78. C 88. B 98. D 108. A 118. B
69. A 79. B 89. D 99. C 109. D 119. D Power: P= V2 :. p oc .!. :. P➔2P
70. D 80. C 90. A 100.B 110. C 120. A Since E = P t , as same energy is required for boiling,

121. D 131. D 141. B 151. B 161. D


.. P➔2P ⇒ f➔ ½t

122. D 132. C 142. C 152. B 162. C .. Time taken= 8 x ½ = 4min.


123. D 133. B 143. D 153. B 163. B
5. B
124. D 134. A 144. C 154. B 164. D
Since the two resistors are connected in parallel, each of them has a voltage of2 V.
125. D 135. A 145. D 155. B 165. B
vi (2)i
P=R=(4)=1W
126. A 136. C 146. C 156. B 166. B
127. D 137. C 147. B 157. A !67. C
128. D 138. D 148. C 158. C 168. B 6. C

129. C 139. C 149. D 159. D 169. B CwrentthroughtheS000nresistor = 2+8 = l0mA

130. A 140. D 150. A 160. C 170. B V;.D =IR"" (10 x 10-3) x (5000) = sov
DSE Physics - Section D : M.C. Solution PD-EM2-MS/03 DSE Physics - Section D : M.C. Solution PD-EM2-MS/04
EM2 : Electric Orcuits EM2 : Electric Circuits

7. B 13. A

The two resistors at the left hand side are in parallel, equivalent resistance is R Equivalent resistance = 2 + 3..2 = 3 n
The three resistors at the right hand side are in parallel, equivalent resistance is R Cumntflows throughR1 = !:._ = =2 A !.
3 R 3

:. R=�+�=1 .67Q 2
2 3 Current flows through� = 2 = 1 A (as�andRJ are equal.)
As D and B are at the same potential, equivalence resistances across AB and AD are the same.
Powerdissipatedby� = I2 R = (1)2 (2) = 2W

8. C
Let the voltage of the cell be V 14. B
When Kis open, the equivalent resistance of the two resistors is 2R. ✓ (1) Radius decreases ⇒ cross..secrional area decreases ⇒ R in.creases
v 2 ✓ (2) Length increases ⇒ R increases
P =
2R
(3) Resistance of wire is independent of its shape.
When Kis closed, one ofthe resistors is shorted and the resistance ofthe circuit is R
:. P ' = vi :. P' = 2P
R 15. D
'When connecting in series, same current flows through 2 resistors
9. A .·. 0 12 R o:: R ·. Bi_ = Pi = 1
is Pz
✓ {l) Weight is a type of force which is a vector that has both magnitude and direction 4
'When connecting in parallel, same voltage flows through 2 resistors
(2) Charge is a scroaI"which has no direction(+ or - only indicates two types of charge)
(3) Voltage is a scalar which has no direction P=V2 o::.!... B._= Rz =4
R R P2 R,_

10. D
16. C
Since the voltal?Y across L, rroiains ,mcbngedalthoughLJ burns out
Reading ofA1 : same voltage across and same resistance ⇒ same current :. reading remains unchanged
:. the current through Li and the power given out by L1 remain unchanged
Reading ofk: 8 closed ⇒ voltage across the resistor with.k increases ⇒ reading of/4 increases
.-. brightness of Li remains unchanged.
Reading ofAJ : S closed ⇒ no current flows through A; as it is shorted .·. reading becomes zero
(Notethat L;, will go out when L3 burns out)

11. C 17. D
Total current flows into a juncti.on point = total current flows out of the junction point
Since power: P= V
2
R decreases ⇒ power given out by the cell increases => total brightness increases
3+2 = 1+1
:.
R
As more lamps in parallel => equivalent resistance R decreases :. C gives the maximumbrightness overall. I= 4A

12. B 18. D
As the voltmeter has same resistance R, Energy given out by the heater= y •t = (200f x(3x60) = 144k1
z

R (50)
the equivalentresistmce of the voltmeter and the resistor in parallel is !!:. or O.SR
2
Euetgyabsorbed by the water = m cl:J.T = (1) x (4.2) x (2 0) "" 84 k1
Voltage acrossthevoltme1er = 6 x O.SR
I2V Energy wasted = 144- 84 = 60 kT
R+O.SR+R
DSE Physics · Section D : M.C. Solution PD-EM2-MS/05 DSE Physics - Section D : MC. Solution PD-EM2-MS/06
EM2 : Electric Circuits EM2 : Electric Circuits

19. 25. A
(a) R = Rx3R = IR WhenK1 is closed, voltmeter reads 12 V ⇒ Wlknownresistance = 6 n.
• R+3R 4
= 2Rx 2R = Both switches are closed ⇒ equivalent resistance ofparallel circuit= 3 n
(b) R R
b 2R+2R
V = 24x-3 - = SV
(c) R • = fi 3+6
4
:. &<�<.&,
26. C
Reading ofA1 : same voltage across the resistor withA1 and same resistance ⇒ same CUIIent
20. A
Reading ofAi : S closed ⇒ voltage across the resistor with Az increases ⇒ reading ofAz increases
Brightness ofP :
Reading ofAJ : S closed ⇒ shorted circuit :. no current flows throughAJ
equivalent resistance of the circuit decreases
⇒ current from cell increases
27. A
⇒ voltage across resistor increases
Assume the resistance of each resistor is 1 n.
⇒ voltage across P decreases
(3) 11, • .!.n
Brightness ofR 3
same voltage across R after the switch is closed
⇒ same brightness
28. A
21. C P is blown ⇒ equivalent resistance ofthe whole circuit increases
Since A1 indicates the current flowing through the three bulbs, ⇒ current given out by the battery decreases
butk indicates the current flo'Wing through one of the light bulb. ⇒ voltage across Q and S decreases :. Q and.Sboth become less bright (:. B, C and Dare wrong.)
·· LS • 0.6A ⇒ voltage acrossRincreases (:. A is correct)
A, •
3

29. C
22. D
There is a drop in potential when current flows through a resistor.
t oc R as same energyE is required for same heating process
.·. Q and R are at the same potential.

.-. Ra "" 400.0 As Ri>Rz


:. p.d.acrossPQ > p.d.acrossRS

23. B 30. C
• A. Resistance ofV >> resistance ofbulb :. voltage across voltmeter "" voltage across the cell Cun-ent passing through 1 11 row = current passing through 213d row = I A
✓ B. Resistance ofA is very small :. very large current flows to ammeter :. ammeter bums out
Current passing through 3ni row = I x ;: = 0.5 A
• C Very small current through the bulb :. the light bulb would not bum out but go out
• D Only the ammeter will bum out ; there is only very small current through the voltmeter
Current delivered from the battery = 1 + 1 + 0.5 = 2.5 A

31. C
24. B
✓ Length increases ⇒ resistance increases For parallel circuit, voltage acrossX = voltage across Y.
(!)
• (2) Cross-sectional area increases ⇒ resistance decreases Same voltage and same charge ⇒ same energy required, by E = Q V
✓ (3) Tincreases ⇒ resistance increases :. Ei = Ez
DSE Physics - Section D : M.C. Solution PD-EM2-MS /07 DSE Physics - Section D : M.C. Solution PD-EM2-MS/08
EM2 : Electric Circuits EM2 : Electric Circuits
32. C 38. D
Potential difference between P and Q = voltage across the 3 n resistor 2
E=Pt= V •t
3
:. VPQ = __ x 12 = 9V R
3+1 As same energy Eis required for boiling :
toc­
1
V'
33. B
t2 = v;2
IfXis closed, all the lamps will be shortwcircuited and go out. ti v;i
If Y is closed, current will only flow throughL but not the other two lamps, L would then give out maximum brightness.

34. D 39. B
Ro,:.!.. 3 4
Since
A equivalent resistance of the circuit "" 4+ ( + l)x = 6 n
(3+1)+4
For the same length : area A oc volume o:: mass m
currentthroughJ.'Y = !x! = 0.5 A
.. Rcc.!..oc..!.. 6 2
A m p.d. betweenX and Y = voltage across the 3 n resistor = 3 x 0.5 = 1.5 V
RA = me = �
RB mA 4 40. D
(1) same voltage across Li ⇒ no change in current throughL1 :• same brightness
35. C
WhenK is open, power dissipated by A :
✓ (2) Li and 13 are in series => same current flowing through ⇒ equal brightness

Pi= (V/2)
2
= 1-.v2 ✓ (3) voltageacrossL1 = 2xvoltage acrossL,, => Pi>P2(by P = V2 !R) ⇒ Li is brighter
R 4 R
When K is closed, power dissipated by A : 41. A
2 Sis closed
Pz= (V/3) = _!._V
2

R 9 R ⇒ equivalent resistance J,
⇒ current given out by the battery t (.·. D is incorrect but A is correct by P = VI)
⇒ voltageacrossPt (:. Bis incorrect)
36. A ⇒ voltage across Q J-
Reading of ammeter A ⇒ current across Q-1- (by V = IR)(:. C is incorrect.)
K is closed ⇒ equivalent resistance of circuit decreases ⇒ current through ammeter increases
Readingofvoltmeter V: 42. A
Kis closed ⇒ thepartwithKis shorted ⇒ no voltage acrossK ⇒ no voltage across V Voltmeter Vi: Sis closed ⇒ short circuit across the voltmeter ⇒ Vi = 0 V.
Voltmeter Vi :
37. C

Connect the additional resistor parallel to theresistor(s) of the largest R (excluding the bulb}
43. C
⇒ equivalent resistance of that part ofcircuit decreases
(1) Equivalent resistance "" 1 n
⇒ current given out from the battery increases ⇒ greatestbrightness ofthe light bulb
(2) Equivalent resistance "" 1 Mn (106 Q)
:. The additional resistor should be connected across CD. (3) Equivalent resistance "" l kO
(If connected across the bulb, the equivalent resistance with the bulb J, and voltage across the bulb ,I,) :. Equivalent resistance in descending order : (2), (3), (1)
DSE Physics - Section D : M.C. Solution PD-EM2-MS/09 DSE Physics - Section D : MC. Solution PD-EM2-MS/10
EM2 : Electric Circuits EM2 : Electric Circuits
44. A 51. D
✓ (1) momentum is a vector which has direction A. Large resistance voltmeter in series ⇒ current is very small ⇒ ammeter would notbum out

• �) power is energy per time, power is a scalar B. Very small current ⇒ light bulb would not bum out

• (3) voltage is energy per charge, voltagi:: is a scalar


C. Large resistance voltmeter ⇒ share all the voltage of the circuit ⇒ reading of voltmeter is 6V
✓ D. Very small cun:ent ⇒ reading ofammeter is zero

45. C
52. A
ByE=QV
Sis closed ⇒ no cummtthroughL1 as itis shorted ⇒ brightness ofL1 decreases to zero
:. (lOJ) = (lC)x(lOV)
Without Li, all the voltage of thebattery is given to k, thus brightness of k increases.
C is correct while A and B are incorrect.
Same voltage across L3 as L3 is connected in parallel to the battery ⇒ brightness ofM is no change
Voltage has no direct relationship with resistance .·. D is incorrect.

53. C
46. C
Equivalentresistance ofthecircuit: R = 4+i = 6 .Q
IfSis closed, the second row resistor and the third row resistor are shorted, 2
current flows through Ai, S, the 1st row resistor and.A, only :. A3 becomes zero
Current flows from the battery : I = � = i = 1A
As equivalent resistance decreases current given out by battery increases :. A1 increases and Az increases R 6
(A.1 = A2 after the switch Sis closed) Current flow through pointX = 0.5 A
P.d. betweenXandY= 0.5x4 = 2V
47. B
Cmrentofthecircu.it: J = .!'.:: = -'- = 0.4A 54. C
R 10+5 A. Work ➔·unit: J (watt: unit of power)
Power dissipated in the 10 Oresistor: P = /2 R = (0.4)2 (10) = 1.6W
B. Electromotive force➔ unit: V (Newton: unit offorce}
✓ C. Momentum➔ unit:N s or kg m s-1
48. C
D. Heat capacity ➔ unit: J 0C-1 Goule per kilogram: specific latent heat)
✓ (1) As they are in parallel, thus same voltage across Lz and £3 to give same brightness
✓ (2) Current through L1 is the sum of current through M and L3, thus L1 is brighter.
55. A
(3) Consider Lz and L3 as one equivalent bulb L, same current flowing Lt and L,
p.d. between P andQ = p.d. across 5 n resistor
but equivalent resistance of L <Li ⇒ P2 +P3 <P1 (by P = 12 R)
� (0.6)(5) ~ 3V

49. D
56. C
The 1st row has an equivalentresistance of 2R while the 2nd row has a resistance of R
(1) Same voltage across the light bulb withA1 ⇒ 11 : no change
As V =JR, thereading ofA2mustbe twotimes the reading ofA1
✓ (2) S closed ⇒ equivalent resistance decreases ⇒ more current given out by battery ⇒ h increases
readingofk = 2x2 = 4A
✓ (3) Sarne voltage across voltmeter after Sis closed since the voltmeter measures the voltage of the battery
k = A1 + A2 = 2 + 4 = 6A

50. B 57. C

A. The voltmeter should not be connected in series to the light bulb. • A. Charge:C=As Current: A

✓ B. Voltmeter is connected in parallel correctly while ammeter is in series correctly. • B. Work:J voltage;V
C. Ammeter should not be in parallel and voltmeter should not be in series. ✓ C. Kmetic energy : J Heat: J
D. Voltmeter should not be in series with the ammeter. • D Force :N Momentum: N s
DSE Physics - Section D : MC. Solution PD-EM2-MS/11 DSE Physics - Section D : MC. Solution PD-EM2-MS/12
EM2 : Electric Circuits EM2 : Electric Circuits

58. A 64. C
Since the resistance is 5 kn, it is a large resistance. • A. IfXburns out, mmneter becomes zero.

Circuit in A is used to measure a larg e resistance. • B IfYburns o ut, ammeter becomes zero.
✓ c. As Xis shorted, current by-passes Xand flows to Y. Thus, vollmeter gives zeroreading.
Current measured by ammeter current flowing throughR

=

D. If Y is shorted, current would flow through X and voltmeter would give a non�zero reading.
Voltage across R >> voltageacross ammeter ⇒ voltage measuredby voltmeter ,.. vol t age acrossR

65. C
59. C
Voltage across 6 n. = (0.4) x (6) = 2.4 V Current through the 12 Q resistor = 2·4 = 0.2 A
✓ A. Force x displacement = Work 12
✓ B. ½ x mass x (speed)2 = K.E. Current through the resistor R = 0. 4 + 02 = 0. 6 A VoltageacrosstheresistorR = 6-2.4 = 3.6V
• C (Cunent) x resistance = Power
2
Resistor: R = !..
= 3.6 = 60.
I 0.6
✓ D. Current x voltage x time = Power x time = Energy

66. D
60. A (1) Ammeter reads the total current through the resistorR and the voltmeter.
Voltage across the lower6 Qresistor: V =IR = (0.3)(6) = 1.8 V ✓ (2) Since the voltmeter is connect ed in par allel w ithR :. voltmeter gives the actual voltage acrossR

Currenttbroughthe4.Qresistor: I= LS= 0.45A ✓


4 (3)

ReadingofA1 = totalcurrentthroughthetworesistors = 0.3 + 0.45 = 0.75A


67. A

61. D 60
Equivalent resistanceof two 60 n. resistor s = 2 = 30 n = the resistance at the 3rd row (30 .Q r esistor)
Equivalent resistanceof (a) = 2R Equivalent resista nceof (b) = !!..
2 Current through 30 n resistor = !.
2
Ammeter reading : .
I=� oc .!_ ⇒ I'= l:!!:.....J = 41
R R R/2
68. C
Total power dissipated: p.,, v2 o::..!.. ⇒ P'=l:!!-_ ·P=4P
R R R/2 (!) For same voltage : I=! o: ..!_ :. current throughR1 is doubled thatthrough,fu
R R
✓ (2) R1 and,Rz are in parallel => same voltage acrosseachof them
62. D
✓ 6 12
✓ (3) Equivalent resistance ofPQ = x = 4 n :. Voltag e ac ross PQ and 4 n resistor are the same.
A. "!!_o::p 6+12
: . Energy dissipated by each coulomb of charge are the same. (ByE = QP)
✓ V' - p
B.
69. A
✓ C. Fv = P
✓ S closed => equivalent resistance of circuit J. => CUITeD.t t
• D m•.€ v= E
(!)

(2) For samevoltage, J=!:..o:..!... RatioofRkeptunchan ged ⇒ ratiooflisunchanged


R R
63. A Sclosed ⇒ R3 is shorted ⇒ no current throughA3 :. readmg of A; becomes zero
Let the resistance ofeach resistor be R.
70. D
Power g iven out by thebattery = VI = (9) (5) = 45 W
:. Equivalent resistance in ascending order :R1 <R2 < RJ Power dissipatedbythebulb = 45 - 18 = 27W
DSE Physics • Section D : MC. Solution PD-EM2-MS/13 DSE Physics - Section D : M.C. Solution PD -EM2-MS /14
EM2 : Electric Circuits EM2 : Electric Circuits

71. A 78. C
Alllmeter: in serie s with the bulb an drheostat ⇒ wire(1) connected to P Suppo se each length ofthe·square has a resistance of 1 n.
Voltmeter: in parallel with bulb => wires (2) an d(3) co nnected t o Q andR X= 1x3 = 0.750 Y= 2 xz = lQ Z= lx3=0.75n
Cunent flows into vol1meter via(+) tenninal => wire (2) connecte d to Q 1+3 2+2 1+3
:. X= Z < Y
72. C
✓ A Resistance = Voltage / CUIIent :. n = VA-1 79. B
✓ B Power = Energy / time :.W=Js-1 The curre nt flows through theresi stancecoil between XB only.
C = Cun:ent time
Charge C = As
= 40 x �= 30 n
X X
Resistance of coil betweenXB
✓ D Voltage = Energy/ charge V - JC-1

Current: I= f= 2_ = 0.2A
73. C R 30
✓ A As equivalent resistance of the circuit -1-, current given out by battery t :. reading ofAi t
✓ B Si ncestill same voltageacross A2 and the resisto r in series :. same current th roughA2 80. C
Q It
C Voltage betweenXand Y is equal to the voltage across the battery which is unchanged
✓ D As current given out by battery t .·. power delivered by battery t (190 X 10-J) X (}6 X 60 X 60)
= 10944C

74. A
81. D
Equivalentresistanceofthecircuit = �+3=3.670 Since the ammeter bas no resistance, thetwo resistor s at the righ t are shorted and no current flows through them.
2+1
The curre nt will flow through the resistor at the left and the n through theammeter.
Voltage ofthebattery = IR = (3) x (3.67) = 11 V
Ammeter reading = 2. = 3A
2
75. B
Slope of V-1 graph = R. Al, Yhas a greater slopethanX resistanceofYis higher than thatofX
Voltmete r reading = 6V
(1) :.

(2) The equivalent resistance ofX and Yin series must be greater :. the slope must be greater than Y.
82. D
✓ (3) The equivalent resistance ofXand Yin parallel must be smaller :. the slcpemust besmaller than X
Let the resistanceof XbeR.
v'
By P= - .-. (20) - (6)' R = un
76. C R R
When switch S is open, It = 1 A si nce same voltageofthe battery is across the3 n-resistor Equivalent resistanceofYandZ = 1.S = 0.9n
= (1)(3) = 3 V
Voltage across the 3 Q..resistor "" voltage of the battery 2
Voltage across the 6 0-resistor = 3 V V2 = (1Z) 2 = 160W
TotalpowerdissipatedinYandZ =
Current through the 6 Q..resis tor = 3 V = 0.5A
R (0.9)
6Q
Cnrrent through the ammeter k = 1 + 0.5 = 1.5A 83. D
Si nce thevoltage of thebattery 12 Vis shared betweenR1 andR2,
77. D the resistances of R1 and & have to begiven i n o rder to calculate the reading of the voltmeter.
Fo r a thinner wire, theresis tanceshoul d be greater.
Sincethe slope of the V-I graph represents the resista nce of the wire, 84. B
thus theslopeshould be greater as shown i n D. Whe n temperature 0 is increased, resistanceof the tb.ennistor is decreased, thus the current shoul d increaseas shown in B.
DSE Physics - Section D : MC. Solution PD-EM2-MS/15 DSE Physics - Section D : MC. Solution PD-EM2-MS/16
EM2 : Electric Circuits EM2 : Electric Circuits
85. A 93. A
Since the voltmeter measures the voltage across the battery, the reading should be constant.
Equivalent resistance of the whole circuit = � + 2 = 2.833 n
1+5
86. D Voltageofthebattery: V= IR= (3 )x(2.833) = 8.5V
Reading ofA2 is two times that of A1 since the resistance is halved. Thus, reading ofk = 2 A
ReadingofA3isthesumofA1andk :. readingofA3 = 1+2 = 3A 94. B
By Q = It (700mAh) •I(15h) :. I = 0.04667 A
87. C By P = VI P = ·(3.6) (0.0 4667) = 0.168 W
✓ (1) The polarity of the ammeter is wrongly connected in reverse direction..
Current should flow into the(+) temrinal and out of the(-) terminal.
95. D
(2) The voltmeter is correctly cormected. If switchX is closed, the left light bulb is shorted,
✓ (3) The voltmeter should be connected across the resistor R only. and the equivalent resistance of the circuit would decrease, thus current given by the cell is increased.
If switch Y is closed, the light bulb P would be shorted and would not light, thus Y must be opened.
88. B
(j) Capacity: Q = 800 mAh = (800 x 10-3 A) x {3600 s) = 2880 C 96. A

"' E = Q V = (2880) x (3.6 ) = 10368 J


✓ (1) From A to B, the resistance of the circuit decrease, thus the CUIIent in the circuit increases.

10368 (2) As current increases, the voltage across the rheostat increases, thus the voltage across the wire decreases.
P=!= 0.04W=40mW
t 3x24x3600 (3) Since the resistance of the wire is not constant,
the voltage across the wire will not be proportion to the current through the wire.
89. D
(j) Q =It= {0.8)x(lx60) = 4 8C 97. A

"' number of electrons = __48__ = 3.0 x 10 20


l.6xHr19
✓ (1) Since the two light bulbs are in series, same current flows through each of them.
Thus, the voltage is proportional to the resistance.
Since voltage across A is greater, thus the resistance ofA is greater than B.

90. A (2) Current must be the same for two light bulbs in series.
Current passing through point P = 4 x 1 = 4 A (3) By P = VI, since A has greater voltage and same current, power ofA is greater, thus A is brighter.
Voltage across each light bulb = 4 x 1.5 = 6 V
Current through each light bulb = 1 A 98. D
Powerofeach lightbulb = VI"" (6) x (l) = 6W Q =It = (0.225) (10 x 3600) = 8100 C OR P = VI = (1.2) (225 x lo-3) = 0.27W
E = Q V = (8100) (1.2) = 9720 J E =Pt = (0.27) (10 x 3600) = 9720 J
91. C
If filament of bulb Zbreaks, equivalent resistance of the whole circuit increases. Current delivered by the cell decreases.
99. C
Voltage across bulb X decreases, thus brightness ofX decreases.
Current through the upper 6 n-resistor = 0.3 A
Voltage across bulb Y increases, thus brightness of Yincreases.
Current through the lower 6 n-resistor = 0.3 A

92. C Current through the 12 0-resistor = 0.3 + 0.3 = 0.6 A

Voltage across the 8 n and2 Qresistors = 10 V Thus current delivered from the cell = 0.6 A
Equivalent resistance of the 8 n and 2 n resistors = 8 + 2 = 10 n Total equivalent resistance of the circuit = 12 +� = 150.
2
Current through the two resistors : I = � = � = I A
R 10 Voltage of the battery V = IR = (0.6)(15) = 9.0 V
PD-EM2-MS/17 PD-EM2-MS/18
I
DSE Physics - Section D : M.C. Solution DSE Physics - Section D : MC. Solution
EM2 : Electric Circuits EM2 : Electric Circuits
100. B 106. B
Assume the circumference of ihe circle consists of 4 equal arcs. each arc has a resistance ofR. ✓ (!) Since both of them are straight line passing through the origin, Io: V, thus they obey Ohm's Law.
X : The equivalent resistance is two resistors, each of2R in parallel, i.e. equal to R. Resistance is equal to t he reciprocal of slope, i.e. R = 11 slope.
(2)
Y : The equivalent resistance is two resistors, one of3R and the other ofR, in parallel, i.e. equal to 3 R / 4. As the slope ofPis greater, resistance ofPis smaller.
Z : The equivalent resistance is two resistors, one ofR and the other of3R, in parallei ie. equal to 3R / 4. ✓ (3) In parallel, I cc 1/R, since resistance ofPis smaller, current passing throughP is greater.
.·. X> Y=Z
107. D

101. C Assume each resistor has a resistance of2n.


When Sis open, the left resistor and the voltmeter are in series. &= 2+2=2n & = 4x6 = 2.40.
As the resistance of the voltmeter is in.finite, all the voltage would be given to the voltmeter, reading is 12 V. 2 4+6

When Sis closed, the two resistors are in series, and each resistor shares half ofthe voltage of the battery.
Re= 2xlO = 1.67 n Rn=�=3Q
2+10 2
The voltmeter measures the voltage of the right resistor, thus the reading is 12 x ½ = 6 V. Network D has the largest equivalent resistance.

102. D 108. A
X A, When both switches are open, the lamp would not light up. Voltage ofthe cell = (1) (6 + 6) = 12 V
X B. When either switch is open, the lamp would not light up. When Sis closed, total equivalent resistance of the whole circuit = 6 + � = 8 n
6+3
c. When only one switch is open and one is closed, the lamp would not light up.
X


CUtrentgivenoutbythe cell = g_
= l.SA
D. When either or both switches are open, the lamp lights up.

103. C 109. D
When the switch is open, no current is given out by the cell. IfXis short circuit, current still flows throughX, Y and the ammeter.
As no current flows through the light bulb, voltage across the light bulb is zero, thus reading ofvoltmeter Q is O V. B. If Yis short circuit, cumnt still flows throughx; Yand the ann:neter.
All the voltage of the cell is then given to the switch, thus the reading of voltmeter Pis 1.5 V.
C. IfXis open circuit, no current can flow through X, thus both ammeter and voltmeter have zero reading.
✓ D. If Yis open circuit, since ideal voltmeter has infinite resistance, no current flows from the cell.
104. C
Reading ofammeter is zero.
(!) Let the resistance of each bulb be R. However, the voltmeter will read the voltage of the cell and is non-zero.
The equivalent resist=:<: of Y and Z is R/2, which is less than that ofX
Thus the voltage across PQ should be less than 6 V.
✓ 110. C
(2) As Q and S are shorted, there is no voltage across these two points.
As 1 kn.resistor andvoluneterarein parallel, Vw1,mom = Vm = 3 V.
✓ (3) If YblllilS out, voltage across Z would increase to 6 V, thus Zbecomes brighter.
For series circuit, V ccR 15-3 3
:. R = 0.75kil
3 R
105. D
11 1 1 1 1
Equivalent resistance betweenPQ = 133 n -=-+- --=-+- :. R..,=3kQ
R Ria R v (0.75) (1) R,,
Equivalent resistance between QR = 0.83 n
Equivalent resistance between RS = On 111. A
Equivalent resistance between SP = 1.5 n ✓ Short circuit across 14 ⇒ current does not flow through L;, and £3 ⇒ Lt lights only
(!)
As the equivalentresistance across Sand P is the maximum, minimum murent would be given out by the cell,
(2) Filament-ofL;,bumtout ⇒ cunentcanflowtbroughl.\ ⇒ 14can alsolight
thus the ammeterreadillg is the minimum.
( 3) Both filament burnt out ⇒ no current flow through.Li ⇒ L1 does not light
DSE Physics - Section D : M.C. Solution PD-EM2-MS/19 DSE Physics - Section D : M.C. Solution PD-EM2-MS/20
EM2 : Electric Circuits EM2 : Electric Circuits
112. C 118. B
ByE=qV Xisthemid-pointofAB ⇒ RAx. = RXB = 60.

:. VR, "'VR, ⇒ E1 ""'- E2. (qistheohargeofonedectron) A 6xlZ


Equivalent resistance across X= = 4.Q
6+12

113. C VAX RAX

ill
=
e R,.
Equivalente.m.£ ofthetwo cells= 10 V

Equivalentinternalresistance ofthetwo cells=.±= 2 n VAX = _4_


(3) 4+6
Current through the resistor : J "" � = � = l A
R 2+8
119. D
Cu:rrentgivenoutbyeachcell = 1 x!,. = 0.SA
2 K open: Voltmeter is in series with the 10 O-resistor, thus no ct1r1cnt given out by the cell
As no voltage across the 10 !l-resistor, all the voltage ofthe cell is given to the voltmeter

114. D :. readlng = 3 V

<D When Sis open, the voltmeter is connected in series with the 12 V battery, K closed: Voltmeter is in parallel with the ammeter, all the voltage of the cell is givento the 10 Q-resistor
thus thereis no current flow as the resistance of the voltmeter is infinite. There is no voltage across the annneter
There is no voltage shared by the 3 n resistor.
:. reading= 0V
Thus all the voltage of 12 V is given to the voltmeter to give the reading of 12 V.

When Sis closed, the 3 Q and 5 .Q resistors are inseries.


120. A
,n
Voltage shared b)'the 5 n resistor = 12 x� = 7.5 V The circuit diagram can be redrawn as sh own.
3+5
LetR be the equivalent resiStance.
1 1 1 1 1 A B
ll5. D -=-+-+-+-
R (3) (3) (3) (4)
✓ 4Q
(1) e::=- :. R = 0.8 .Q
I
✓ (2) e=-
121. D
✓ Open circuit ⇒ no CUITe1lt ⇒ no voltage drop across the internal resistance ⇒ e = V The circuit diagram can be,redrmvn as shown. 3V
(3)
The three resistors are inparallel.
I,
I'
116. B Equivalent resistance : "'
The circuit diagram can be redrawn as shown: R=.®_=zn
3
"'
!+8+8
1=!..fil. I.SA
"'
--
Equivalent resistance=..L_ = 10
2
n R (2)

122. D
To measure a high resistance R., voltmeter should be connected in parallel across the resistor and the ammeter.
The ammeter canread the actual current flowing through the resistor.
117. C
Since the resistance of the an:imeter is small COillpared with the resistor, the voltage across the ammeter is negligible.
Since resistmce of voltmeter is infinite, there is no current flows through AS, thus no voltage across AS.
Thus, the voltmeter reading is close to the actual voltage across the resistor.
Voltmeter thus measm:es the voltage across the battery which is always equal to 2 V.
Circuit A is used to measure a low resistance R.
Thus, the graph should be a horizontal line. Note that CUITellt flowing through the ammeter and voltmeter must be from(+) terminal to the(+) terminal ofthe meters.
DSE Physics - Section D : M.C. Solution PD-EM2-MS/21 DSE Physics - Section D : M.C. Solution PD-EM2-MS/22
EM2 : Electric Circuits EM2 : Electric Circuits
123. D 129. C
The cell 3 V is discharging and gives out current while the 1 V cell is under charging. ✓ (!) Reading of ammeter gives the sum of current flowing tbroughR and voltmeter.
Ncte.m.£ = 3-1 =2V ✓ (2) By R = T , larger measured I => smaller measured R
:. I =-2
- = 0.333A
High resistance of resistor R => large current across voltmeter ⇒ incorrect ammeter reading
2+4 (3)
Vin = 0.333 x 4 = 1.33 V
p.d. acrossXand Y = 1.33 + I = 2.33 V 130. A
Ju:,.
= 0.3 - 0.2 "" 0.1 A (from left to right)
124. D Vsn = (0.2)(5) = 1 V Vin = (0.l)(l) = 0.1 V
££ 1 £ · 2- 2 :. The lower intersection point is at a higher potential since the potential drops only 0.1 V which is less than 1 V.
pl 2 2
P = J2R = 12·- cc 1 2·- oc -- cc I -£ where volume of wire : V = A. i.

'
A A•f V
/30 =�= 0.3A (from bottom to top)

:. h = 0.3-0.l = 0.2A(from.righttoleft)

131. D
125. D
The potential of earthed point is O V.
Note that the device does not obey Ohm's law, resistance is found by R = !..
I Before switch Sis closed, potential difference across the 10 .Q resistor - 2 V.
Thus potential at P = + 2 V.
After switch S is closed, potential difference across the 1 O .Q resistor = 3 V.
The resistance decreases by 6 0.
ThuspotentialatP = +3V.

68
126. A
132. C
The circuit diagram can be redrawn as :
When the voltmeter with infinite resistance is connected, there is no current flowing through the two l .Q resistors.
S open: Voltage of the battery is shared among 2 resistors. Since voltage across the middle 2 .Q resistor is 2 V, the battery has e.m£ of 6 V.
Th.en, voltmeter measures voltage across 1 resistor only,
When the ammeter with zero resistance is connected, the equivalent resistance ofthe whole circuit is 5 n.
which is equal to half of the e.m.£, that is, 2 V.
Current delivered from the battery is 1.2 A. Current flows fromX to Y is 0.6 A.
S closed: The right resistor is shorted, all the voltage of the battery V
S
is given to the left resistor.
133. B
Thus, voltmeter measttres the e.m.f. of the battery, that is, 4 V.
Across CD, no current flows through the 10 .Q resistor and S. B
127. D The two resistors R and R are in parallel
10n R
(1) Resistors in parallel have same voltage => same energy dissipated by the 1 C charge (by E = Q V) :.B.=25 :. R=SO.Q
2
✓ (2) For parallel circuit, equivalent resistance < resistance of every individual resistor, :. R <R1
Across AC, no current flows tbroughS. A
✓ (3) Less resistors in parallel ⇒ R t R
B=---C=}----(C 500
128. D 10n
10n R A B
Sis closed ⇒ equivalent resistance ofR1 andR:z J. ⇒ p.d. acrossR1 .J, butp.d. acrossR3 t son
Potential at the earthed point is 0 V. _J-----wQ-__:c,,,iD
Potential atX: decrease (as p.d. across Rt J,)
Potential at Y: decrease (as p.d. across & t) Equivalent resistance = 10+ � = 35 n
2
DSE Physics - Section D : M.C. Solution PD-EM2-MS/23 DSE Physics - Section D : M.C. Solution PD-EM2-MS/24
EM2 : Electric Circuits EM2 : Electric Circuits

134. A 139. C
voltage VIV ✓ A. The p.d. betweenPQ is equal to the p.d. across the battery,
14 which is equal to the e.m.:E of the battery, thus it is unchanged.
12 ✓ B. After switch Sis closed, equivalent resistance of the whole circuit decreases,
10
8 thus more current flows out from the battery, and the p.d. across R1 increases.
6
I/ Y,
As the right hand side of Ri is earthed at zero potential, the potential at P increases.
4
X C. The p.d. across R2 decreases after the switch is closed.
2 I/
I--' current I/ A As the left hand side of Rz is at zero potential, potential at Q should increase.
0 0.2 0.4 0.6
✓ D. As the p.d. across Rz decreases, the current flowing through& decreases.
Si.nee the current through the two light bulbs are the same, draw a vertical line such that Vx +Vy = 12.
The current is 0.3 A and Vx = 9V, Vy = 3V.
140. D

L½ �"?7
135. A

✓ ByR= �= 0.5 =con


(1)
I 0

X (2) WhenV=2V:R =--2-=IOOQ.


2ox10-
3

'V/henV=3V:R = __3_3 =750.


40x10- 3V 3V
The resistance is not constant.
As each resistor is connected between point A and point B, they are in parallel,
X (3) As the line does not pass through the origin, the current is not directly proportional to the voltage.
The voltage across each resistor is equal to the e.m.£ ofthe cell, that is, each has a voltage of3 V.

136. C Current through the 1 n resistor = � = 3A


1
= 270 -120
V gx = SV
270 Current through the 2 n resistor = � = 1.5 A
2

137. C
Current through the 3 n resistor "" � = 1A
3

6Q AmmeterA1 = LS+ l = 25A

Ammeter A2 = 3 + 1.5 = 4.5 A


6Q 6 Q +- shorted

141. B
a b
X (1) As they are connected in parallel, their voltages must be the same.

Equivalent resistance: R = £ = 2n ✓ (2) As the resistance of Pis greater, the current through it is smaller.
3
X
(3) By P = VI, as current I through P is smaller, power ofP is smaller, thus P should be dimmer.
138. D
✓ (1) As an ammeter is used to measure current, it should be connected in series for current to flow through it. 142. C

✓ ✓ (1) As the resistance of P is greater, the voltage across P is greater by V = IR.


(2) An ideal ammeter has zero resistance, a practical ammeter should have low resistance.


X (2) As they are connected in series, their currents must be the same.
(3) For high resistance ammeter, it would increase the equivalent resistance of the circuit,
thus it would seriously affect the current of the circuit. ✓ (3) By P = VI, as voltage to Pis greater, power ofPis greater, thus Pis brighter.
DSE Physics • Section D : M.C. Solution PD-EM2-MS/25 DSE Physics - Section D : M.C. Solution PD-EM2-MS/26
EM2 : Electric Circuits E.M2 : Electric Circuits

143. D 150. A
WhenP is atX, current flows directly fromXto P without pasSlllg through the resistance wireXY, current is maximum. X should be closed so that current would not pass through the light bulb below the switchX.
When P is at the mid point ofXY, current flows fromX and Y to P through. the resistance wire XY, current is minimum. fshould be opened so that current would not pass through the light bulb beside the switch Y.
WhenP is at Y, current flows directly from Y to P without passing through the resistance wire XY, current is maximum. If no current flows to other light bulbs, the current or voltage of P would be maximum, thus the brightness is maximum.
As a whole, current decreases and then increases when P is moved fromX to Y.
151. B
144. C Since the resistance of the voltmeter is vecy large, negligible current is drawn by the voltmeter.
✓ (1) Two resistors consume two times the power compared with that of one resistor.
Current given out by the battecy = 2.:_Q_ """ 025 A
✓ (2) Two resistors draw two times the current compared with that of one resistor. 2+10

(3) The equivalent resistance of two resistors in parallel is halved of that of one resistor. Voltagegiventotbe lightbulb = 0.25xl0 = 2.SV OR Voltage given to the light bulb = 3.0 x ....!£_ = 2.5 V
2+10

145. D
152. B
Ohm's law states th at the voltage across a conductor is directly proportional to the current
provided the temperature is constant, Voltage across each resistor in Figure (a) = ½V
which is equivalent to sey that the resistance of a conductor is constant provided the temperature is constant. Voltage across each resistor in Figure (b) = V

ex: V2
2
By p = V :. p :. V ➔ 2 V ⇒ P ➔ 4P
146. C
e = (2.0)(S+r)
e = (1.2)(10+r) 153. B
Combine the two equations : Assume the equivalent resistance of the four resistors R is R '.
(2.0){S+r) = (1.2)(10+r) :. r = 2.50. By .!.=..l.+l..
6 12 R'
147. B :. R' = 12n

Reading ofk is two times th atofA1 since the resistmce is halved. Thus, reading ofA2 = 2 A If the 12 n resistor is replaced by a 6 9- resistor,

Reading of A; is the sumofA1 andA2. Thus.reading ofA; = 1 + 2 = 3 A equivalent resistance = 6 x 12 = 4 n


6+12

148. C
154. B
✓ (1) The polarity of the ammeter is wrongly connected in reverse direction.
Current should flow into the(+) tenninal. and out of the H tem:rina1 If the variable resistor is set to zero, the light bulb is shorted,

( 2) The voltmeter is correcilyconnected. all the current will flow through the zero resistance path,
✓ The voltmeter should be connected across the resistor R only. thus no current will flow through the light bulb, the light bulb will not light up.
(3)

149. D 155. B

Equivalent resistance of the 3 n and 6 n resistors which are in parallel = 6 x 3 = 2 n


R= pL ex: f::...oc f::...x!::...oc £ex: L2 6+3
A A A L V
Total equivalent resistance oftbe whole circuit = 2 + 2 = 4 n
:.L➔2L⇒R➔4R

OR Current given out by the cell = g= 3A


4
Since the volume is the same, L ➔ 2L ⇒ A ➔ ½A Temrinalvoltageofthecell = E - Ir = 12 - 3x2 = 6V

.·. R ➔ -3.._R = 4 R Current flowing through the 6 n resistor = � = 1 A


1/2
DSE Physics - Section D : M.C. Solution PD-EM2-MS/27 DSE Physics - Section D : M.C. Solution PD-EM2-MS/28
EM2 : Electric Circuits EM2 : Electric Circuits

156. B 160. C
Let the e.m.f. of the cell be &and the internal resistance of the cell be r.
21
When Sis closed, the 3 .Q resistor is shorted, thus, the total resistance of the circuit is (6 + r) .Q.
X
y :. e=3x(6+r)
I
When Sis open, the total resistance ofthe circuit becomes (3 + 6 + r) .Q, Let the current be I.
I z :. e = Jx(9+r)
Combine the two equations :
Let the current passing through Z be I. 3x(6+r) = Ix(9+r)
Current passing through Yshould also be I. x A IfJ=l.6A,then3x(6+r)=l.6x(9+r) r is negative .-. it is impossible
Thus the current passing through Xis 2 L X B. If I= 2.0A, then 3 x (6+r) = 2.0 x (9+r) r = O.Q it is impossible
TotalpowerdissipatedbyX,YandZ =- (2I) 2R + 12 R + 1 2 R = 24 ✓ c. If I= 2.4A, then 3 x(6+r) = 2.4x (9+r) r = 6n possible
:. J2 R=4 X D. As the total resistance increases, the current must decrease and less than 3 A it is impossible
Power dissipated in resistor Z = 12 R = 4 W

161. D
157. A X A. IfR, becomes a short circuit (that is,R1 = 0 ),
current would not flow through R:i and RJ.
Voltmeter reading would become zero.
X B. IfR:i becomes a short circuit (that is, R:i = 0 ),
the voltmeter reading would become zero. R,
L, X c. IfR3 becomes a short circuit (that is, RJ = 0 ),
1---C:::::J---{ A
the total resistance of the circuit decreases. ;-----;;--1
Cw:rent given out by the cell increases.
L 1 : The above two circuits are equivalent Current would not flow to the right part of the rheostat
When the slider shifts to the right, the resistance of the rheostat increases, cun-ent decreases, tbusL1 becomes dimmer. Asterminalvoltage:V= s-1r· :.lt ⇒v-!­ "---+:
I -+. :
•---------·
Reading ofammeter would decrease.
Li : & the light bulb and the rheostat are connected in parallel to the cell, the voltage across L:i. is constant ✓ D. IfR:i becomes an open circuit (that is, R:i = oo ), the total resistance of the circuit increases.
Thus the brightness of L2 is not affected by the rheostat, and therefore brightness ofLi remains unchanged. Current given out by the cell decreases.
Astennmalvoltage:V= s-lr :.1-1- ⇒ vt
158. C As voltage across R1 increases since it is equal to the tenninal voltage, cmrent through R1 increases.
A IfXis shorted, the ammeter A should have reading but voltmeter V reads zero and Y is lit Reading ofammeter thus increases and reading of voltmeter also increases.

B. If Yis shorted, both the ammeter A and the voltmeter V have readings, and Xis lit.
✓ IfX is burnt out, no current flows through ammeter A,Xand Y, thusX and Yare not 'lit. 162, C
C.
However, the voltmeter V reads the voltage of the cell, thus the reading ofV is not zero. The light bulb will light up'only when S1 is closed (seat is occupied) and S1 is open (seat belt not yet fastened).
D. If Yis bum.t out,Xand Y are not lit, but both the voltmeter V and al]]Illeter A have no readings. Option C is correct. Ifboth switches are closed (seat is occupied and seat belt fastened),
the light bulb is shorted by S2, current would pass through the resistor, S,, ands, and the light bulb is shut off.
159. D OptionD is not a good design since the cell would be shorted and damaged when both switches are closed.
Ifthe voltmeter reads 6 V when switch S is closed, then one ofthe following may happen
Pis burnt out and becomes open circuit 163. B
The voltage of the cell would then share between Q and the voltmeter.
When the variable resistor is adjusted to O n, the equivalent resistance is 10 .Q and 10 kQ in parallel.
As the resistance of the voltmeter is much larger than Q,
all the voltage of the cell would be given to the voltmeter and the voltmeter reads 6 V. Since IO .Q is much Simllerthan 10 kn, the equivalent resistance is about 10 n.

<l) Q is short-circuited. The voltage across Q becomes O V. <l) When the variable resistor is adjusted to 10 Mn, the equivalent resistance is 10 MO and 10 kn in parallel.
All the voltage of the cell would be given to P and the voltmeter reads 6 V. Since l O kQ is much smaller than IO Mn, the equivalent resistance is about 10 kn.
DSE Physics - Secti.on D : M.C. Solution PD-EM2-MS /29 DSE Physics - Section D : Question PD-EM2-Q I 01
EM2 : Electric Circuits EM2 : Electric Circuits

164. D The following list of formulae may be found useful :


✓ (1) Since Xis brighter than Y, the power ofXis greater than that of Y.
Resistance and resistivity
As P "" VI, the two bulbs in parallel have same voltage, thus the current flowing tbroughX is greater.
Current is the amount of charge flowing in 1 s, thus, number of charges through X in 1 s is greater.
Resistors in series
✓ (2) Power is the energy given out in 1 s.
As the power ofXis greater, the electrical energy dissipated by Xis greater. 1 1 1
Resistors in parallel -=-+-
✓ R R, R,_
(3) Voltage (p.d.) is the conversion of electrical energy to other energy per unit cbaige.
As they are in parallel, they must have same voltage,
Power in a circuit
thus the elecirical energy dissipated by Xis equal to that of Y.
Energy transfer during heating or cooling E= mcM
165. B
Energy transfer during change of state
Assume each light bulb has a resistance of 2 0 and the voltage of the supply is 6 V.

The equivalent resistance of P and Q is 1 0. They are in series with R and share the voltage of 6Vin proportional.
Thus, voltage across P and Qare both 2V and voltage acrossR is 4 V. Part A: BK.CE examination questions
As T and Sare in series and connected across the supply voltage of 6 V, each of them shares 3 V.
1. < HKCE 1980 Paper I- 8 >
The highest voltage across the l i ght bulb is R, thus, R will burn out first

1.0
166. B

�--1:
In the circuit, all the resistors all identical. 9V
Assume the resistance of each resistor is 2 n. I/A
I 0.5
The voltage across R1 is 9V and is irrelevant R,

-1=:J----iY
Resistors & and R.i are in parallel,.
R,
their equivalent resistance = � = 1 n

Resistor & and this equivalent resistance are in series.


I
X
R,
VIV

A student performed an experiment to investigate Ohm's Law using the circuit as shown above. The results are sbovro. in the
l = 3V graph above.
Voltage acrossXY = 9 x__
2+1
(a) What is the range of voltages for which Ohm's Law is obeyed'! (1 mark)

(b) Suggest a reason why Ohm's Law is not obeyed outside this range. (2nwks)

(c) Find the resistance ofthe lamp when the voltage is

(i) O.SV;and

(u1 1.5 V. (2nwks)


1 DSE Physics - Section D : Question
EM2 : Electric Circuits
PD-E
M2-Q
/ 02 I I 3.
DSE Physics - Section D : Question
EM2 : Electric Circuits

< HKCE 1987 Paper I- 7 >


PD-EM2 •QI 0311
2. < HKCE 1984 Paper I - 7 >
�--111 �>---{Z}---,
.---{=:}------,----{ A

The figure above shows a circuit diagram to measure an unknown resistance R.


(a) The figure below shows the components used in the circuit. You are then given 8 pieces of conducting wires. Draw in
the figure below the wires connecting the temrina1s of the components to complete the circuit above. (7 marks)

�: The figure above shows an experiment set-up to measure the resistance ofa light bulb.

(a) Draw a circuit diagram for the experiment. Indicate on your drawn diagram the positive terminals of the ammeter and
�,witch voltmeter with"+" signs. (5 marlcs)

voltmeter

(b) State where you should set the slider of the rheostat at the beginning of the experiment. State the reason for your choice.
(3 mms)

(c) Ifthe resistor were connected in reverse direction.how would the readings of the ammeter and voltmeter be affected?
( 2,nms)

(d) Using the same components provided, draw a circuit diagram you would use to measure a resistance comparable to that
of the voltmeter. (3 marks)

(b) If the slider in the rheostat moves from A to B, how does the reading of the ammeter change ?
I
3. (c)
DSE Physics - Section D : Question
EM2 : Electric Circuits
PD-
EM2-Q
/04 1
1 I
4.
DSE Physics - Section D : Question
EM2 : Electric Circuits

< HKCE 1989 Paper I- 8 >


PD-EM2
-Q I 0
51
1

The figure shows the apparatus of an experiment to study how the resistance of a metallic coil changes with temperature.
The temperature is controlled by changing the output voltage of the DC supply.
3

DC supply

0
&!)
ammet«
(0-500mA) (0-5V)

The figure above shows the voltmeter and the ammeter used in the experiment. What is volt:mcter
(i) the ammeter reading, and �
coil
(ii) the voltmeter reading
as indicated in the diagram? Hence calculate the resistance of the light bulb at this moment.
(a) Draw the wires connecting the temrinals of the apparatus to complete the circuit for the experiment. (4 ,,,,,;,)

(b) DescnDe how to measure the temperature of the coil. (3 marks)

(d) (i) When the current increases, how does


(1) the temperature, and
(2) the resistance
(c) 'When the voltmeter reads 12V, the ammeterreads 2 A- Calculate the resistance ofthe coil at this reading. (3 marks)
of the light bulb change ? (2 """"l

(ii) Make a rough sketch of the voltage across the light bulb against the current. (2marks) (d) How would the resistance ofthe metallic coil chaz:ige with temperature ? (2 marks)

(e) For a particular output voltage, if the resistance coil were shortened, how would the an:uneter and the voltmeter reacling
cbangC?
(The :intcmal:resistance of the DC supply and the ammeter are negligible.) (3 marks)
/ ti
..
=====aa:EM2

5.
E
. aa:S=Ph=
y
: Electric

< HKCE 1991 Paper 1- 6 >


s
S Que
aa:tlaa:on= aa::aa:=snaa:·aa:on====aa:Paa:Daa:--=E-=M2=--=Q=/ 0;;:6;:a1
SICaa:aa:. aa:-aa: ;;:ecCircuits
D D

..
11

6. < BKCE 2001 Paper I -10 >


DS
E
Phsics - ectlon D : stlon
y S
EM2 : Electric Circuits
Que
PD-EM2-Q/07

6V

,.-----,111-----,
✓Spring
Figllrel
D

An electronic device Dis connected to a resistor Rand a 6 V power supply as shown in Figure I above. The resistance of R Stretching force / N
is 470 .Q. A voltmeter of high resistance is connected across R. Figure 2 shows the variation of the resistance of the device D
with temperature. A spring is ha:nging freely from the ceiling and John stretches the spring with his hand as shown in Figure I. It is known that
40

Resistance I .Q the extension ofthe spring is directly proportional to the stretching force (see Figure 2).

(a) Using Figure 2, :find the stretching foree if the extension of the spring is S em.
15

(b)

IO _.,, ,
Figure2

F.igure3 �'.1 ; 2 0 on
1
1

John wsnts to use a voltmeter to measure the force he applies to stretch the above spring, He sets up a device as shown
(a) The reading of the voltmeter is 4.7 V. At this instant, find in Figure 3. XYis a uniform.resistance wire of length 20 cm andP is a metallic sliding contact XYis fixed vertically
andP can slide smoothly alongXYas the spring is stretched. The voltage of the battezy is 4.5 V and the resistance ofXY
(i) the current flowing through R, is 20 n. The resistance of the variable resistor is set to 40 n. P touches endX of the wire when the stretchin.g force is
'""·
(i) Draw a circuit diagram for the circuit in Figure 3. (4""'1<s)
(tl) the resistance ofthe device D,

(iii) the temperature ofthe deviceD.

{b) How would the voltmeter reading change ifthe temperature of the device D increases? Explain briefly.
(ii) Show that the voltmeter reads 1.5 V when P touches end Yofthe wire.
6.
DSE Physics - Section D : Question
EM2 : Electric Circuits
(b ) (ill) Ifthevoltmeterreadingis 1.2 V, £nd
PD-EM2-Q/081

I
7.
DSE Physics - Section D : Question
EM2 : Electric Circuits
PD-E
M2-
Q/0

(b) Iris conducts the experiment for a zinc-carbon cell, an alkaline cell and a lithium. cell separately. Figure 2 shows the
911

variation of the voltage across the bulb with time fur the cells. The bulb will light up as long as the voltage across it is
(1) the distance of P from cndX, above0.6V.
(2) the stretching force. (4 """"J
Voltage/V
12

(iv) John .finds that the device is not sensitive enough (i.e. the voltmeter reading shows no observable change when be
slightly alters bis stretching force). In order to increase the sensitivity, be suggests :reducing the resistance of the Figure 2
variable resistor. Explain whether John's suggestion is appropriate. (3 marks)

-,i

0 2 4 6 7 8 9 10
Time/hour

(i) A salesman claims that the lifetime of a lithium cell for lighting up the bulb is five times that of an alkaline cell.
7. <HKCE 2005faperl-9 > Determine whether the claim is correct or not (2 marl.cs)
to be connected to a
"'
bulb
data-.logger

(ii) The prices of the three types of cells are shown in the Table below.

Figure 1
Type of cells Price per cell

.zinc..cm:bon $15

,lkalino $ 3.8
Iris uses the apparatus shown in Figure 1 to study the lifetime of AA-size cells for lighting up a bulb. She connects a cell and
a switch to the bulb and uses a voltage sensor to me3S\ll'C the voltage across the bulb. lithium $25.0

(a) Draw a circuit diagram to illustrate how the apparatus is connected. Use the symbot G) to denote the voltage sensor.
(3 """") Which type of cells is the best buy, in terms of the cost per hour for lighting up the bu.lb ? Show your calculations.
(3 """")
1
8.
DSE Physics - Section D : Question
EM2 : Electric Circuits
< HKCE 2006 Paper IM 11 >
PD-EM2 -QI 10

I 8.
DSE Physics - Section D : Question
EM2 : Electric Circuits

(c) The teacher asks Mary, ''VVhat happens ifthe variable resistor is set to zero ?''
"The light bulb will burn out," M.uy answered.
PD-EM2-Q/II

low voltage power supply Explain ml.etherMazy's answer is correct ('2 m,,k.,J
( with negligible internal resistance)

9. < HKCE 2007 Paper IM 7 >

switch I I I,.____
lightbolb
ammeter

sealed box
A teacher gives Jane a sealed box in which a light bulb is connected to a variable resistor. The teacher asks Jane to find out
how the bulb and the variable resistor are connected together inside the sealed box. Jane then sets up a circuit as shown in A teacher conducts an experiment to study the energy conversion of a filament light bulb. A simple circuit is connected as
the above Figure. She reduces the resistanceR of the variable resistor and records the changes as shown in the below Table. show:n in the above :figure and the bulb is ittmlersed into 0.09 kg of oil inside a foam cup as shown below. The bulb is lighted
up for 300 s, and the temperature of the oil is increased from 20"C to 42°C.
Data: Voltageofthepowersupply = 3V
Initial value ofR = 15 n Initial ammeter reading = 2.6 A
Final value ofR = 5 Q Final ammeter reading = 3.0 A thermometer
foam cover
Observation: Brightness of the bulb remains unchanged

(a) (i) Jane correctly concludes that the variable resistor and the bulb are connected in parallel inside the box. Give a filament light bulb
reason to support Jane's conclusion. (1 muk)
oil

In the experiment, the ammeter and voltmeter readings are 1.4 A and 12 V respectively. The specific heat capacity of the oil
(ii) Draw a circuit diagram to illustrate how the apparatus shown in the above Figure are connected, inclutling the is 2100 J kg""1 0C--1 •
components inside the box. Use the symbol � � to denote the low voltage power supply. (2 marks)
(a) Calculate the energy absorbed by the oil.

(b) Describe the energy conversion when a current passes through the filament light bulb. (1 mark)

(iii) Using the data in the above Table, find the resistance of the bulb. (3 """"'l
(c) (i) Estimate the amount of energy that is converted into light enexgy in the experiment, and state ONE asSUIJiption
made in your calculation. (4 marks)

(b) Jane's classmr,te Mary conducts the same experiment by replacing the low voltage power supply with two 1.5 V dry
cells which are connected in series. If the internal resistance of the dry cells is not negligilile, explain why the brightness
of the bulb decreases when R is reduced. (3 marks) (ii) Hence, determine the percentage of electrical energy consumed by the :filament light bulb that is converted into
light energy. (2 m arks)
10.
DSE Physics - Section D : Question
EM2 : Electric Circuits

< HKCE 2009 Paper I- 6 >


PD-EM2-Q/12

l 11.
DSE Physics - Section D : Question
EM2 : Electric Circuits
PD-EM2-Q/13

(b) (ii) Measurements are repeated with different settin gs of the variable resistor. The Table shows the data obtained.
Using the apparatus shown in the figure below, describe the procedures of an experiment to study how the resistance of a
nichrome wire depends on its thickness. Trial Voltmeter Reading VIV Ammeter Reading I I A
(4 marks)
1 1.4 022
2 2.8 0.42

3 4.1 0.64
4 5.6 0.82

Plot a graph of voltmeter reading against ammeter reading in the Figure below. Use a scale of 1 cm representing
1 VandO.lA.

11. <HKCE2011Paperl-5>
Jane wants to find the resistance of a resist'ance wire by me asuring the voltage across and current through the resistance
wire.
(a) As shown in Figure (a), Jane bas connected the resistance wire in series with a ba ttery, an ammeter, a switch and a
variable resistor. Add a voltmeter(@) in Figure (a) to complete the circuit. (l mark)

Figure(a) (ii i) From the graph plotted lll (b) (ii), find the resistance of the resistance wire.

ilr---=l---__J (c) Now the resistance ofa :filament ligh t bulb is studied using the same experimental se tup. The voltag�t graph
(b) After the teacher has checked the circuit, Jane performs the experiment.
obtained is shown in Figure (c). Explain why the graph is not a straight line. (2 marks)
(i) The ammeter readin g Ula certain trial is shown in Figure (b). In this setting, the maximum cunent that can be V
measured by the ammeter is I A What is the reading shown? (1 mark)

Figue (c)

Figure (b)

Amm.eterreadin g - _______
DSE Physics - Section D : Question PD-EM2-Q/14 DSE Physics - Section D : Question PD-EM2-Q/15
EM2 : Electric Orcuits EM2 : Electric Circuits

Part B : HKAL examination questions Part C : HKDSE examination questions

12- < HKAL 1995 Paper I - 8 > 13. <HKDSE2013PaperIB-10>


(a) In the circuit shown in the below Figure, a 12 V battery of negligible internal resistance is connected with a thermistor R
and a resistor of resistance 120 n. The graph shows the variation of the thermistor's resistance with temperature.

Thermistor's resistance
+--------{ V f----➔ ""'
""
L_c;z:S--< A
R a
The above circuit is employed to measure the e.m.f., e, and the :internal resistance, r , of a dry cell. Assume that the voltmeter
and the ammeter used are ideal. 12v -=- A �"
VIV
120n ""'
'"
" '"
Temperature / °C

(i) Find the resistance ofthe thermistor Rat 25"C. (1 mruk)

(ii) What is the potential difference VAB across A and B at 25°C ? (2 marks)

The voltmeter readings, V, and the ammeter readings, I, obtained for different rheostat settings are used to plot the above (b)
grap h.

(a) Express Vin terms of e, I and;. (1 mruk)

12v-=..

V
(b) Hence deduce from the graph the e.m.£ and the internal resistance of the cell. (2 marks) B

Kelly wants to confirm the above calculation by measuring VAB using a voltmeter of about 1 kQ resistance, She
finds that the reading registered is slightly different from the value found in (a) despite making careful measurements.
Explain why this is so. Suggest how the accuracy of the measurement could be improved (3 marks)
DSE Physics - Section D : Question PD -EM2-Q / 16 DSE Physics - Section D : Question PD -EM2-Q /17
EM2 : Electric Circuits EM2 : Electric Circuits
13. (c) (i) 14. <HKDSE2016PaperIB-7>
(a) The circuits in the above Figure each contains two resistors connected in series with a 6 V battery of negligible internal
resistance, The resistors in circuit I are IO ill each while those in circuit II are 100 n each.

6V 6V
�----,111-----�
12v-=- Al----
To
electtonic
�----,110----�
120n
switch
circuit I circuit II
B lOk.Q lOkQ

The potential difference VAB is used to drive an electronic switch connected across AB to turn on a fan if
temperature rises above a certain value such that VAB is 6.0 V or above. Using the information provided in the
graph, find the minimum temperature needed to keep the fan on. Show you working. (2 marks)

A voltmeter of intemal resistance Rv = 10 k!l is used to measure the potential difference across one of the resistors as
shown.

(i) What would be the respective voltmeter readings ?

(ii) Jn fact, the potential dif:ference across each resistor before connecting the voltmeter is 3 Vin both circuits. Explain
why this voltmeter gives a relative inaccurate value for circuit I. Hence state the general principle of selecting a
suitable voltmeter for such measurement. (2 marks)

l
(ii)

(b) Circuit ill shows a possible method for measuring resistance using a voltmeter and an ammeter. The internal resistance
for the voltmeter and the ammeter are Rv and� respectively and their readings V,,, and[,,,_ give the measured resistance
12v-=- R,,,_ = Vm • The true resistance value of the resistor isR.
1.
To
electronic
switch
--------<111-----
circuit ID Rv -
V f ----1
Without using additional components, complete the new circuit diagram below to illustrate how the circuit can be
modified to tum on a heating device when temperature falls below a certain value. Explain the action ofthe circuit.
R
No calculation is required. (3 marks) '------C:=J---(A "'

(i) State which reading(s), V.,,,


I,,, or both, do(es) NOT give the true voltage across the resistor and/or the true current
passing through the resistor. Hence write down an equation relating P.A. Rm andR. (2 marks)

(llJ Find the percentage error associated with.Rm when measuring the resistance of this resistor.
Given: Rv = 10 kn,&= 1 n andR = 10 n. (2marks)
I DSE Physics - Section D : Question
EM2 : Electric Circuits
PD-EM2-Q/18 II DSE Physics - Section D : Question
EM2 : Electric Circuits
PD-EM2-Q / 19

15. <HKDSE2017 Paper IB- 8 > 15. (c) The student claims that since the resistance of the light bulb is not a constant, the equation R = VII cannot be used to
A student uses the following apparatus to measure the resistance of a tungsten :filament light bulb. calculate the resistance ofthe light bulb. Briefly explain why his cl.aim is wrong. (1 mark)

a battery, a switch, a variable resistor, an ammeter, a voltmeter, a light bulb

(a) Figure l shows an incomplete circuit for the experiment. The '+' symbol represents the positive terminal of the
ammeter.
(d) Determine the resistance of the light bulb at V= 0.1 V and2.5 V.
Use suitable circuit symbols to complete the circuit, and mark the positive tenninal of the voltmeter with '+'. (3 marks)

+
Figure 1 A rrr,
(e) It is given that the cross--sectional area of the tungsten :filament in the "light bulb is I.66 x 10➔ and the resistivity of
tungsten at room temperature is about 5.6 x 10-a n m Estimate the length of the tungsten filament in the light bulb
using the appropriate resistance found in (d)._ (3 marks)

The table below and Figure 2 show the results obtained.

Vol e across the Ii ht bulb VIV 0 0.1 0.2 0.3 0.4 0.5 1.0 2.0 3.0 16. <HKDSE2019 Ppaper-IB-7>
Current JJ mA 0 76 112 126 133 139 170 226 273
You aro provided with a batteey (of fixed e.m.f. � and internal re,istance r), a YBl:iable resistor {with seven,!
300 known rnistance values R to be ,ele<tcd), a switcl!, a voltmotor (assumed ideal) and a few connecting wi=.

250
__,..,,,- 0--

200

!
Fignre2 ._, 150

100
(a) With tlie aid of , cin:uit diagram. doscn1,c the prooedwo of an experiment to study how the terminal
voltage I'"delivered by the batt<;:rY depends on tho remtance R comectl:d to it Stw:, ONE pm;aution of
50
Ille experimem. (5 mori<s)

(b) Describe lhe variation of I'"wi1h Rand cxprcss Vin tem>s of,; rand R. (2 morb)
0 0.5 1.0 1.5 2.0 2.5 3.0 3.5
Voltage across the light bulb VIV

(b) Briefly explain the variation of the resistance of the light bulb with the voltage across the light bulb. (2 marks)
17. <HKDSE2020Paper lB-8>

Figure 8.1 shows a simplified circuit of the lighting system of a car. Each of the taillights (Ti, T2)� high-beam
headligbts (H,, H,) and low-beam headlights (L,, .l,) has resistance 30 n, 2 n and 3 n respectively. The
internal resistance of the 1_2 V battery and the resistance of the fuse are negligible.

B
A,-------------..-------,C

L,
T, 3Q
30 n
;ure S.1

T,
30n
1s,
fus1_
L,
3Q

12V-=..
F '------<l�------_._.-------'D

Vlhen switch S1 is closed and switch S2 is set at the position shown in Figure 8.1, only 71 and 72 as well as
H1 and H2 are lit. The current drawn from the battery is at a maximum in this setting.
(a) Explain why £1 and L2 are not lit. (I mark)

(b) (i) What is the potential difference across the taillight T2 ? (I mark)

{ii) Indicate on Figure &. I the direction of current in each of the branches AB, GB and BC. Which branch
carries the largest current?. (3 marks)

(c) Calculate the power delivered by the battery and show that the equivalent resistance of the circuit is
slightly less than 1 Qin this setting. (4 marks)

(d) Based on your answer in (c). explain whether a fuse rating of l5 A js suitable for this circuit or not.
(2 marks)
DSE Physics - Section D : Question Solution PD-EM2-QS/Ol DSE Physics - Section D : Question Solution PD-EM2-QS/02
EM2 : Electric Circuits EM2 : Electric Circuits .

BKEAA's Maml1g SclJe:rne is PICPaICd for the ma.kets' teference. It should not be regatded as a set ofntOdel answers.

I
Students and teachers who are not involved in the marking process are advjscd to .lllteip:[et the Marl::ing Scheme wit.b. care, 2. (d) [3]
I
Question Solution
L (a) From.0 Vto 1 V, Ohm's Lawis obeyed [!J

(b) The temperature of the lamp increases, [ll
thus the resistance of the lamp increases. [ll
V
(c) (i) R=�=�=l.33.Q <accept1.3Qto1.4Q> [!]
I 0375
< The cells, the switch, the rheostat, the resistor and the ammeter are in series> [!J
(ii) R = !::.. = � = 1.55 n <accept 1.53 n to 1.56 n> [I] <The voltmeter is connected in parallel> [!]
I 0.97
< The voltmeter is connected across the resistor and the ammeter> [!]

2. (a)

�,1
3. (,) [5]

(b) The reading increases [2]

(o) {i) 440mA [!]

(nJ 2.6V [!]

R = !'._ = 1i_ = 5.91 Q [2]


I 0.44

(d) (i) (1) Temperature iru:Ieases [!J


< Cells, switch, rheostat, resistor and ammeter in series > [5] (;2) Resistance increases [1]
< voltmeter is connected in patallel to resistor > [2]
(u) V
(b) Tow""1,P [!]
so that the resistance is maximum [!] < initial portion is a stnlight line > [!J
and cw:rent is smallest [!]
< final portion curves upwards ? [!]
(,) Reading of voltmeter: no change [!]
Reading of ammeter: no change [!]
. DSE Physics - Section D : Question Solution PD-EM2-QS/03 DSE Physics - Section D : Question Solution PD-EM2-QS/04
EM2 : Electric Circuits EM2 : Electric Circuits

4. (a) 6. (a) F = 100N [!]

(b) (Q

DCa,pply
+
y
p
voltmeter

coil <For correct symbols> [!]


<Battery, variable resistor andXYin series> [!]
< animcter in series with the coil and supply> [1]
<Polarit.yofbattery; (+) to Y > [1]
< voltmeter in parallel with the coil> [1]
< Voltmeter in parallel with PX> [!]
< polarity of the ammeter correct > [1]
< polarity of the voltmeter correct> [1] (il) Current = f= � = 0.075 A [1]
R 40+20
(b) Immerse the coil into the water in a water bath. [1]
When P touches Y,
Then measure the temperature of the water [1]
by a thermometer. voltmeter reading = JR = (0.07S) (20) = 1.5 V [1]
[1]

(iii) (1) DistanceofPfromX - l.2 x 20


(c) R = ! [1] 1.5
[!]
I
= 16cm [1]
= .!3: [1]
2 (2) Since extension of spring "" 16 cm, FromFigure2 [!]
=6n [!] stretching force = 320 N [!]

(d) Resistance increases when temperature :increases. [2] (iv) His suggestion is apPiopriate.
Ifthe resistance of the variable resistor is reduced, the voltage acrossXYis increased. [!]
(e) Amtneterreadingincreases. [l]
Thus a change in the position of P will result in a greater change in the voltmeter reading. [!]
Voltmeter reading remains unchanged. [2]
So the sensitivity of the device is increased. [!]

5. (a) (i) Cw::rent:J= != 4 ·7


[1] 7.
R 470 (a)
= 0.01 A [!]

(it) Voltageacross the deviceD.,, 6-4.7 = 1.3 V [1]

Resistance ofthe deviceD = � = 130 n [1]


O.ol

(iii) TemperatureofthedeviceD = 20"C <from the graph> [1]

(b) When temperature increases, resistance ofthe deviceD decreases. [!] < Cell, switch and bulb in series> [1]
Voltage across the device D thus decreases. [!] < Bulb and sensor in parallel> [1]
Therefore, voltmeter reading increases. [1] < Correct circuit symbols > [1]
DSE Physics - Section D : Question Solution PD-EM2-QS/05 DSE Physics - Section D : Question Solution PD-EM2-QS/06
EM2 : Electric Circuits EM2 : Electric Circuits
7. (b) (i) lifetime oflithiumcells = 8.2 hours < accept 8 to 9 hours> 8. ( b) When the resistance R decreases, the current through the dry cells increases. [l]
lifetime ofalkaline cells = 4.4 hours < accept 4 to 5 hours > [l] The voltage across the internal resistance ofthe drycells increases. [l]
Since S.2 = 1.86 ;o 5 Thus, the voltage across the light bulb decreases. [l]
4.4
Therefore, the brightness ofthe bulb decreases.
so the claim is not correct [l]

(ti) Lifetime ofthe cells: (c) Mazy is not crurect [l ]


[I]
zinc-carbon = 1.4 hours < accept 1.3 to 1.5 hours> Reason : (ONE of the follofflllg) [l]
alkaline "' 4.4 hours <accept 4.4 to 45 hours> * since the ligbt bulb is shorted
lithium = 8.2 hours < accept 8.2 to 8.3 hours> * since there is no current flowing through the light bulb

CostperhourfurZUl.C..ca:rbon cells= LS=$ 1.07 < accept 1.00 to 1.15 > (l]
* since the voltage across the light bulb is zero
1.4

Costperhourforalkaline cells = � = $ 0.864 < accept 0.84 to 0.86 >


4.4
9. (a) E=mc!J.T [l]
Cost per hom- for lithium cells = 25 = $ 3.05 < accept 3.01 to 3.05 > • (0.09)(2100)(42-20)
8.2
= 4158J"" 4160J [l]
So alkaline cells are the best buy. [l]
(b) Electrical energy changes to heat and light energy.
<OR>
8. (a) (i) Since the brightness of the bulb is unchanged, Electrical energy changes to internal energy. [l]
thus the voltage across the bulb is unchanged when the resistance ofthe rheostat is reduced. [l] <OR>
Electrical energychanges to heat. [l]
(ii)
(o) (,) P• VI• (12)(1.4) • 16.SW [l]
+
Electrical energy: E = Pt = (16.8)(300) = 5040 J [l]
A Light energy= 5040 - 4160 = 880J <accept 5040-4158 = 8821> [l]

Any ONE of the following: [l]


* No energy lost to the surroundings.
* The voltmeter is ideal.
* The resistance of the connecting wires is negligible.

(it) Percentage = � x 100% <accept �x 100% > [l]


< rheostat and light bulb in parallel > (l] 5040 5040
< all the symbols and connections correct> Ul = 17.5 % [l]

(iiIJ Initial resistance of R = IS Q <OR> Final resistance of rheostat = 5 Q

CurrentthroughR = 2_ = 0.2A CummtthroughR = l = 0.6A [l] 10. Connect the two crocodile clips across a certain length of the thin.nichrome wire. [l]
15 5
Record the ammeterreading. [l]
Current throughthe bulb = 2.6 - 0.2 = 2.4 A Currenttbroughthe bulb = 3-0.6 = 2.4A [I]
Repeat the experiment using the thicker nichrome wire ofthe same length, [l]
Rerutallce ofthe light bulb = 2-_ = 125 .Q [l]
2.4 The ammeter reading should be larger, showing that the resistance ofthe thicker wire is smaller. [l]
DSE Physics - Section D : Question Solution PD-EM2-QS/07 DSE Physics - Section D : Question Solution PD -EM2-QS/ 08
EM2 : Electric Circuits EM2 : Electric Circuits

11. (a) 12. (a) e = V+lr


:. V=-lr+e [l]

(b) e = y-intercept = 1.6 V [l]


r =-slope= 4n [l]

13. (a) (i) R=80.Q [l]

< The voltmeter is connected in parallel across the resistor only.> [l] (ii) VAB = 12 x lZO [l]
120+80
(b) (i) Reading = 0.32 A [l] = 7.2V [l]

VIV (b) As Rv and 120 n are in parallel, their equivalent resistance is smaller than 120 n. [l]
Therefore, the voltage shared across AB is smaller th.an that expected. [l]
The accuracy can be improved by using a voltmeter with resistance mu.ch higher than 120 Q. [l]

(c} (i) VAB "" 6V


:. Vit. = 12-6 = 6V
.·.R=120f.! [l]
Minimum temperature is 16"C <accept 15 to 16 "C > [l]

(n)

120n
--
12V ::
To
IJ elewonic
R
switcl>

I/A
< circuit diagram : 120 n andRare interchanged> [l]

'When temperature falls. below a certain value, resistance ofRincreases. [l]


< Correct labels with units and correct scale > [l]
Voltage acrossRincreases to 6 V or above, the heating device is then tumed on, [l]
<Data points plotted correctly> [l]
< Best-fitted straight line drawn> [l]

(ill)R = slope of the straight line 14. (a) (i) Circuit I:R= 10 kO
[l]
= 6-0 = 6.67 n < Acceptable range ofR: 636 - 6.83 n > [l]
Equivalent resistance = !Q. = 5 kn
2
0.9
V=6x__5
[l]
{c) As the temperature of the filament increases with the current, [l] 5+10
its resistance increases with the temperature. [l] =2V [l]
DSE Physics - Section D : Question Solution PD-EM2-QS/09 DSE Physics - Section D : Question Solution PD -EM2-QS / 10
EM2 : Electric Circuits EM2 : Electric Circuits

14. (a) (i) Circuit U: R = 100 .0


15. (d) AtO.IV:R=_O._l_ = J.32!2 [I]
Equivalentresistauce = c2-+-1 -r1 = 99.01 Q 76x10-l
100 10000
At2.5 V: R = �"' 10 n [2]
99.01 2sox10-l
V = 6 x
99.01+100
= 2.985 V < acc ept 2.99 V > < accept 3 V >
(e) At room temperature, resistance is 132 n. [!]
[!]
By R=p!_
(ii) In circuit I, the resistance of the part of circuit decreases significantlyaf\er the voltmeter is connected. [!] A
OR e
(132) ""_(5,6 X }Q-8) X [I]
In circuit I, the resistance of the voltmeter is C01J1parable to the resistance of the resistor in parallel. [I] (1.66xW')

Resistance of voltmeter should be much higher than the resistance of the resistor connected in panllel. [I]
••• ,e = 0.0391m <accept0.039m> [I]

(b) (i) V,,, does not give the true voltage for the resistor. [I] 16.

[!] Solution

(ri) For circuit ID :


Rm"'-lO+I=IlD:
Percentage error = ll -l
10
O x I00% UJ + V 1---- ♦

= 10% [I]

15. (a) O=tht.wil<h""'tta>ld�"/'m,cfRrc,d;ngs t-----{V ,_____


JA
Adjust <ho resis<a,,:e R I<> low,r/o<ber valll<(s} ,ru1 - <he 1A

mep}
oxpe,imont
+
A � Firstsettbe vmable RSistcrto ifs maxir:mm:t / a IA
val� ...,,
- Opontbeswttclulter...,h.........,.,. 0"1l
Airy- reasonabW 4b5WCt
5

< correct symbols of light bulb, voltmeter and variable resistor (rheostat)> [l]
< correct positions ofthe components > [I]
< correct positive temrinal of the voltmeter> [I]

(b) As the voltage across the light bulb increases, tempcrato.re of the light bulb increases, [I]
thus its resismlce increases. [l]

(c) R = !:.. is the definition ofresistance. [l]


I
It can be applied to all conductors even if the resi$tal:l.ce is not constant.
Hong Kong Diploma of Secondary Education Examination
DSE Physics - Section D : M.C. PD-EM3-M/01
Physics - Compulsory part (;t,i,1-":>,-)
EM3 : Domestic Electricity
Section A-Heat and Gases (�:if<, $i.ll:lt)
1. Temperature, Heat and Internal energy (11J.lt • �;fn � ��)
2. TransferProcesses (?.!',-t4--;fHIMi) The following list of formulae may be found useful :
3. Change of State ($asV-JatW.)
4. G,nernl(mu,w (iH&ll.!t,t�)
5. Kinetic Theory (,£--f-i!�'l't)
Resistance and resistivity R = pl
A
Section B- Force and Motion (JJ ;fa �if,)
1. PositionandMovement(-f!I.R.:fo:fH'ii) Resistors in series
2. Newton'sLaws(4<illlt.#")
3. MomentofForce ('hi€) Resistors in parallel 1 1 1
-=-+-
4. Work,Energy andPower(fp;Q, hti!::fu/.l!ft) R RI Ri
5. Momentum.(1111:)
6. Projectile Motion(4;\t:lllti!ffi) Power in a circuit P=JV-,,,1 2 R
7. Cirou1arMotion ( 00 Jil ;t:tii)
8. Gravitation ( 51 h)
Section C- Wave Motion (ilt�)
1. Wave Propagation (itttig.:iltii!.) Part A : HKCE examination questions
2. WavePhenomena(i!tiilJl.at)
3. Reflection and.Refraction of Light (;Tc.II!! &.M.&.hrM)
1. < HK.CE 1.980 Paper Il 33 >
4. Lenses (it.it)
M

y
=
5. Wave Nature ofLight (ic,6gitttl,�•!±)
D.C.
6. Sound(*½)
Section D - Electricity and Magnetism ( 't ;fa .;Jt)
Z 210
1. Electrostatics (ff-1t4':)
2. Electric Circuits C1l!:l3-)
3. Domestic Electricity ( �� m 11!:) X 180 K
4. Magnetic Field(x&tlf)
5. Electromagnetic Induction (1tllt!i1..!&)
6. Alternating Current (Sf.ilii.1t) In the circuit shown, Yis as A fuse, Z andX are 3 A fuses. When the switchK is open the current passing through the 21 n
Section E- Radioactivity and Nuclear Energy (:$:.M-1.11../ll:fa*-«.fi'E.) resistor is 2.4 A. WhenK is closed, which of the fuses will be blown ?
1. R.adiation and Radioactivity(fflM:fo:M:.M� �) A. Xonly
2. AtomicModel(.ilT��) B. Yonly
3. NuclearEnergy(al'lll6) C. Zonly
D. XandZonly
Physics - Elective part (;J!.if":,,-)
Elective 1 -Astronomy and Space Science (��tf::fo#t.ic.tl-4!=)
L The universe as seen in different scales c;i;: JSJ !Z. r., �Ii. ""'f {i!J �'iliiliitt.) 2. < HKCE 1982 Paper Il- 26 >
2. Astronomytbroughhist:ory (?(.:$C.${i!J.ft-hl!t.) The table below shows the voltage and power rating for various electrical appliances. Which of the electric appliances has
3. Orbital motions under gravity ( i: h r lr!i M..utr.u1.,) the Slllallest working resistance 7
4. Stars and the universe(·IJ.£.ifo�'ili)
Appliance Voltage PoWer/W
Elective 2 - Atomic World (lfi. Tilt�)
1. Rutherford's atomic model (il�#>Bf,TfA-it!) A Air-conditioner 200 2000
2. Photoelectriceffect(X.'t/1.t.li) B. Television 200 250
3. Bohr's atomic model of hydrogen (�i {i!J A\,,1$ T-ffi1:!) Q Heat� 100 2000
4. !'articles or waves(Jlti.T �i't.)
D. Hair-dryer 100 20
5. Probingintonano scale(��#H�'l!t�)
Elective 3 - Energy and Use of Energy Cile."iofoif6i!li.{!{J#!.JH)
1. Electricity at home C �,I§, ffl 1t) 3. < HK.CE 1982 Paper Il - 28 >
2. Energy efficiency in building (,tffi.{i!Jli6i&tt.$-)
3. Energy efficiency in transportation(>{#;� {i!J �f;it./1.t.�) A set of Christmas tree lights consists of20 bulbs in series. Each bulb has a rating of"l0 V 5 W". One of the bulbs burns
out and Jimmy goes to buy a replacement. When he gets back, he finds that although the new bulb is .marked 5 W, it looks
4. Non-renewable energy sources ( � -r -i+!t.li6 �)
dimmer than the others when the lights are turned on. The most probable reason for this is that the new bulb
5. Renewable energy sources (,r .j!j.;£111;.i.!f,)
A. has a smaller current flowing through it
Elective 4 - Medical Physics ( ,& 4!= Jti,; J.t �)
1. Makingsenseoftheeyc(��lr!i�'ff) B. has a filament with a higher resistance.
2. Making sense of the ear (.lf-#,p;!J;. 'Ir) C. has been shorted accidentally.
3. Medical imaging using non-ionizing radiation (�F- 1@:Ml'-#.Af-l-$1${8!.$) D. is designed to work at a lower :voltage.
4. Medical imaging using ionizing radiation(il!:Nr-.ti.Afll-<!!>�i#.,Jf;)
DSE Physics - Section D : M.C. PD-EM3-M/02 DSE Physics - Section D : M.C. PD-EM3-M/03
EM3 : Domestic Electricity EM3 : Domestic Electricity
4. <HKCE1982Paperll-29> 10. < HK.CE 1986 Paper JI - 36 >
fifteen bulbs, each labelled '200 V 60 W', are connected in parallel to a 200 V supply. Which of the following fuses should P and Qare bulbs ofrating"40W, 200 V" whileR and Sare ofrating "60 W, 200 V'. 'Which ofthefollowing circuits gives
be used in the circuit? the maximum brigh tness ?
A. 2A (Assume all circuits are connected to the same voltage.)
B. 3A A.
C. 4A
D. SA
p

s. < HKCE 1983 Paper Il- 35 >


Eight 100 W Uil!J!ls and one 1000 Wheater are all connected in parallel to a l'l1llins supply of200 V. Which of the following
should be used ?
A. 5 A fuse
B: 10 A fuse B.
C. 30Afuse
D. 50Afuse

6. <HKCE1984P aperll-30>
Which of the following has the greatest current when it is operated at 200 V ?
A. a lamp with a reststance of 400 n
B. arice-cooker with rating of 400 W at 200 V
C. an electric iron with rating of400 Wat 220 V C.
D. ahair-dcyerwithratingof600 W at200 V

7. < HK.CE 1984 Paper ll- 27 >


Two heaters of rating '1 kW, 200 V' and '2 kW, 200 V respectively are connected in series to a 200 V supply. What is the
total p ower consumed by the heaters ?
A. 3kW
B. 1.SkW
C. lkW D.
D. 0,67kW

8. <HKCE 1984 Paper ll- 35 >


In the figure shown, Sis aswitch to tam on and offthe electric fan. X, Y and
Zare wires to be connected to the three terminals of aplug. How shouldX,
Y and Zbe connected to the three p ins (E, L a:ndN ) ofa given plug?
E,rth Live Neutral
A. X y z
B. y X z 11. <HKCE 1986 Pa per JI- 32 >
C. z y X
D. X z y
An electricheater of rating "2000 W, 200 V" and an electric cooker ofrating "500 W, 200 V'' are connected in parallel to
· a200 V a.c. source. The total power ofthe two appliances is
A. soow.
•• <HK.CE 1985 PaperJI-37 > B. 1500W.
C. 2000W.
Electric bulbs R are ofntin.g "40 W, 200 V' and electric bulbs Sare of rating "60 W, 200 V". 'Which of the following
D. 2500W.
combinations would produce the maximum brightness if connected to the same voltage ?
A B.
s s 12. < HKCE 1987 Paper JI - 29 >
� The circuit shows a lamp ofrating '6 V, 9 W' connected in series
� with a resistor Rand a 9 Vbattery. What should be the resistance

¾
C. D. of R if the lamp is to work as rated?
A. zn
B. 3n
C. 40
n. sn
I
13. <HKCE1987Paperll-27>
DSE Physics - Section D : M.C.
EM3 : Domestic Electricity
PD-EM
3-M
/ 04 I 17. <HKCE1991Paperll-28>
DSE Physics - Section D : M.C.
EM3 : Domestic Electricity
PD-EM3-M/05

For safety, the correct way of connecting the fuse and switch to electrical appliances should be
A. fuse in live wire, switch in neutral \\ire.
B. fuse in earth wire , switch in live wire.
C. both in neutral wire.
D. both in live wire.

-
18. <HKCE 1991 Paper Il- 36 >
In the three-pin plug as shown. X, Y and Z are respectively connected to the Two light bulbs A andB of ratings '10W, 6 V' and '5 W, 6 V' respectively are connected in series to a 6 Vbattety. 'Which

-
X y z ofthe following is/are correct 7
(1) The resistance ofA is smaller than thatofB.
A. neutral livo (2) The current throu gh A is the same as that through B.
B. live neo"'1 (3) AisbrighterthanB.
c. noutr.,J ,arth live A. (1) only
D. nou"'1 live B. (3) onfy
C. (1)&(2)only
D. (2) & (3) only
14. <HK.CE1988 Paper Il- 33 >
A 31>in plug is connected to a boiler ofrating "2000 W, 200 V". Vlhich of the following statements is/are true ? 19. < HKCE1993 Paper II- 33 >
(1) A 5 A fuse should be used in the circuit. Which ofthe following is NOT a correct unit for the corresponding physical quantity?
(2) The fuse should be placed on the brown wire ofthe cable.
Physical quantity Unit
(3) The yellow and green wire of the cable should be connected to the earth pin.
A. charge coulomb
A. (1) only
B. eurnnt ampere
B. (3) only
C. (!)& (2) only
c. resistance ohm
D. voltage joule
D. (2) & (3) only

20. < HK.CE 1994Paper 11- 30 >


15. < BKCE 1989 Paper Il- 39 >
The diagram shows the label attached to a rice cooker. Which of the
following statements is/are true when the cooker is working at its rated Operating voltage 220V/50Hz
values 7 Power 500W
(1) The resistance of the cooker is 96.8 n.
(2) The cooker draws a current of4.4 A from the mains supply.

- - -
(3) The cooker consumes 1 kWh of electrical. energy in 2 hours.
A. (1 ) only
A stanrlard three-pin socket on the wall is shown in the figure. \Vhich of the following is correct? B. {2) only
C. (1) & (3) only
Pin (1) Pin (2) Pin (3) D. (2) & (3) only
A. nou"'1 live
B. neutral livo 21. < BK.CE 1994 Paper II -29 >
c. live nou"'1
D. neutral livo Which of the following diagrams correctly shows the connection of the wires ofan iron to the pins ofa plug 7
(J,: Live, N: Neutral, E: Earth )
A. B.
16. <HKCE 1989 Paper 11-37 >
N�
Operating voltage 220V/50Hz E
p,­ 1500W
FuseRating 30A L

C. D.
The diagram shows the label attached to an electric appliance. How much electrical energy is supplied to the appliance
:in2 hours?
A. 2.0kWh
B. 2.SkWh
C. 3.0kV/h
D. 6.0kWh
DSE Physics - Section D : M.C. PD-EM3-M/06 I DSE Physics - Section D : M.C. PD-EM3-M/071
EM3 : Domestic Electricity EM3 : Domestic Electricity

22. <HKCE1995Paperll-28> 26. < HKCE 1997 Paper n - 28 >

=-
Which of the following values is equivalent to one kilowatt hour? The kilowatt-hour is a unit of
A. 1000W A. charge.
B. IO00J B.
C. 3.6xl06 W C. energy.
D. 3.6 x 106 J D. power.

23. < HK.CE 1995 Paper Il- 35 > 27. < BKCE 1997 Paper n - 30 >
Which of the following statements about the earth wire in an electric iron is/are correct?
X(Bcown (1) The earth wire should be cOJlllected to the metal body ofthe iron.
Fuse
(2) If the iron is working properly, no current pass through the earth wire.
Y(Blue) � (3) In case the neutral wire is broken, the earth wire provides a spare wire for the return path to the mains socket.
A. (I) only
Z (Green/ yellow)
B. (3) only
C. (I) & (2) only
The diagram above shows a three-pin plug and the wires connected to it To which of the pins should each of the wires X, Y D. (2) & (3) only
andZbe connected?
p Q R
28. < BKCE 1998 Paper II - 33 >
A. z y X
z
, 7_,__ ,,
B. y X Power supply
c. y z X
D. z X y
totalres.istanceR

24. < HKCE 1995 Paper Il - 30 >
An electric appliance is connectedto a power supply of voltage Vby long cOJlllecting wires of total resistance R as shown in
the circuit. It is found that the cun:ent passing through the appliance is only ½Io, where Io is the current required for the
appliance to work at its rated value. Which of the following changes could
increase the current through the appliance to Io ?
Voltage of power supply Total resistance of connecting wires
A. increases to 2V remains unchanged
B. increases to 2 V reduces to R/2
A Christmas tree is illuminated with four strings of light bulbs. Each string has ten identical light bulbs connected in series C. remains unchanged increases to 2R
as show.n fu the figure. If one light bulb suddenly bums out, which ofthe following will happen? D. remains unchanged reduces to R/2
A Only that light bulb will go out.
B. One light bulb in each string will go out.
C. One string of light bulbs will go out 29. < BKCE 2000 Paper II- 35 >
D. All ofthe light bulbs will go out.

25. < HKCE 1996 Paper n - 34 > To tnalllS supply

--1-
The above diagram shows the main parts of an electric iron. Jn which of the following situations will the fuse blow when the
switch is closed?
(1) The insulation at contact point Xis worn out so that the wire touches the metal case.
The switch S and the fuse of an iron are incorrectly fitted in the neutral wite as shoMt above. Which of the following (2) The insulation at contact point Yis wom out so that the wire touches the metal case.
statements is correct ? (3) The heating element is broken.
A The iron will not operate even when Sis on. A. (I) only
B. The iron will still operate even when Sis off. B. (3) only
C. The iron will still operate, but ifthere is a high current the fuse will not blow. C . (1)&(2)on!Y
D. The iron will still operate, but the heating element will remain at a high voltage even when Sis off. D. (2) & (3) only
DSE Physics - Section D : MC. PD-EM3-M/08 DSE Physics - Section D : M.C. PD-EM3-M/09
EM3 : Domestic Electricity EM3 : Domestic Electricity

30. <HKCE2001Paper1I-31> 34. < HKCE Z003Paper II 34 >

•--1
w

12V

C. D. R

L�

;-:---� Three light bulbs P, Q andR of ratings '24 V, 80 W', '12 V, 80 W' and '12 V, 40 W' respectively are connected in parallel
to a 12 V battery. Vlbich of thebulbs will be the brightest, and which will be the dinunest ?
The brightest The dimmest

31. <BK.CE2001Paperll-32> A. p Q
B. p R
Two electric heatersXand Y are of ratings '110 V, 40 W' and '110 V, 80 W' respectively. Which ofthe below deductions c. Q p
about the two beaters is/are correct ? D. Q R
(1) The operating resistance ofXis twice that of Y.
(2) X will consume a power of80W when it is connected to a 220 V mains supply.
35. < HKCE 2003 Paper II 35 >w

(3) Both heaters wotlc at their rated values 'When they are connected in series to a 220 V mains supply.
The photograph shows a tlireewp:in plug ofan appliance. Which of the following is a
A (1) only function ofpin P ?
B. (3) only A. It prevents the appliance from being shortweircuit.
C. (1) & (2) only
B. It protects the user from getting an electric shock.
D. (2) & (3) only
C. It provides a retum path for the current
D. It can break the circuit when the current flowing through the appliance is too large.
32. <BK.CE 2002 Paper 11- 34 >
Three identical lamps L1, Lz and L; of ratings '6 V, 12 W' are connected 36. < HKCE 2003 PaperII 36 > w

to a 6 V batteyy as shown in the figure. Which of the following statements

C tri!if
is con-ect?
ENERGY LABEL
A. The voltage across Lz is 3 V.
B. ThC current passing through L1 is 2 A. jg;;Ji
C. The total power drawn from the battery is 12 W.
Brand AA!T XXX
D. The total power dissipated in£, and¼ is smaller than that inL;.
Model ffl!!df: XXX

.Annual Energy Conswuptiou kWh/year 250


33. <HKCE 2002PaperII-36 > i!!lffiU �TlL'Nij

_,
Actual cousumptlou will depend ou where tbi: appllaucc ls
Live wire located and how itjs II.Std. Alm!111l 260 washes per yeu.
$�t•���s��•�Aa�m��-
1UU:�4ii,1¥-ilt� 260 .:J:. •
Energy Efilciency Grade

Neutral wite Washing Machine Category �!iJU XXX

EEL .Registration. Number ��tla!lltci.!.i XXX


Ifthe live and neutral wires ofan electric kettle are mistakenly interclianged inside the plug as shown above, which of the
following will happen ?
The :figure shows the energy label of a washing :machine. If the average working time per wash is 1.8 hours, estimate the
A. The kettle will not operate. average electric power consumed by the machine.
B. Thefuseofthekettlewillblow. A. 450W
C. The metal case of thekettle will still stand at a high voltage even when the switch ofthe kettle is off. B. 534 W
C. 962 W
D. The heating element of the kettle will still stand at a high voltage even when the switch of the kettle is off. D. 1731 W
37. <BKCE2004Papcril-33>
DSE Physics - Section D : MC.
EM3 : Domestic Electricity
PD-EM3-M/10

I
42.
DSE Physics - Section D : M.C.
EM3 : Domestic Electricity
< HK.CE 2006 Paper JI-21 >
PD-EM3 M
- /ll
ll

One day, Donald used the following electrical appliances at home:


1010101010101
/������"

An..fiance Ratin« Duration Cost of eJectricitv kWh


electric he ater 220V,2500W 30 minutes C, 220V SOHz
L-- circular disc
television
,___
220V,270W 5 hours C,
[JUmumonmmom11nonmo, ----1
220V 150W 8hours C,
Which of the following relationships is correct?
A CJ>C2>C3
' 600 revolutions/kWh
,
The figure shows the label of a kilowatt-hour meter connected' to a mains supply. When an appliance is switched on for
B. C2>C1>C3
2llllII.Utes, the citcular disc rotates through 24 complete revolutions. What is the electric power consuinedby the appliance?
C. C2>C3>C1
A 900W
D. C3>C2>C1
B. 1200W
c. 1800 W
38. < BKCE 2005 Paper Il- 20> D. 2400W
The following table shows three electrical appliances which Clara usedin a certain month:
43. < HK.CE 2006 Paper JI- 41 >
Appliance Rating Duration In the circuit shown, the resistots represent the heating elements in a
air--conditioner 220V, 1200W 250hours rice cooker. The resistances ofthe elements are SR andR respectively.
The rice-cooker can be operated in two modes, ru:unely, cooking and
television 220V,250W 80hours keeping wan:n. The power consumed by the cooker in the cooking
computer 220V,-150W 60 hours mode is 600 W when Sis closed. What is the power consumed by the
rice-cooker in the mode of keeping warm when Sis open?
Calculate the cost ofelectricity used. Note: 1 kW h of electricity costs$ 0.86. A. IOOW
A $6225 B. 120W
B. $73.79 C. !SOW
C. $282.94 D. 180W
D. $ 536.64
44. <HKCE2006Paperll-39>
39. <HKCE 2005 Paper 11-21 > An electric appliance draws a current2 A when it is operating at 220 V. Which ofthefollowing is the best description of the
Ifa 15 A fuse is installed in the plug of an electric kettle of rating value '220 V, 900W', which of the following descriptions cunent andthe voltage ofeach wire of the electric appliance when it is connected to a 220V mains supply?
is correct? Live wire Neutral wire Earth wire
A The kettle will not operate.
� Voltage Current � Current Voltage
B. The kettle will be short-circuited.
C. The output power of the k&tle will be increased . A 2A 220V lA 220V IA 0
D. The chance of the kettte bcing damagedby an excessive current will be increased. B. 2A 220V 2A 220V 0 0
C. 2A 220V 2A O O 0
40. <HKCE 2006 Paper JI-24> D. 2A 220V 0 0 0 0

An electrical appliance is protected by a fuse in a domestic circuit. 'When the appliance is switched on, the fuse blows 45. < BK.CE 2007 Paper II-18 >
immediately. Which of the following statements is/are possible reason(s) for this phenomenon?
(1) The resistance of the appliance is too large. A household circuit breaker is marked '220 V, 15 A'. Now an electric iron rated '220 V, llOO W' and a cooker rated
(2) The appliance is short-circuited.. '220V, 550 W' are connected in parallel to the mains socket How manylight bulbs rated '220V, IOOW' at mostcan still be
(3) The rated value of the fuse is too small . connected in parallel to the mains witb.cut triggering the circuit breaker?
A. (I) only A. 12
B. (3) only B. 16
C. (1)&(2)only C. 17
D. (2) & (3) only D. 20

46. < HKCE 2008 Paper JI - 22 >


41. <.HKCE 2006 Paper Il-40>
A lighting system consists of 3 bulbs, each rated '220 V 30 W'. These thtee bulbs should be connected in parallel to the
Two light.bulbsare marked '220 V, 50 W' and '220 V, 100 W' respectively. If they are connected in series to a 220 V
220 V mains supply. However, the bulbs are wrongly connected in series to the mains supply. What is the power dissipation
lll3llls supply, what is the current drawn from the Jl'lains supply?
of the wrollgly connected system?
A 0.ISA
A 3.33 W
B. 0.23 A
B. !OW
C. 0.46A
C. 30W
D. 0.68 A
D. 90W
I
47. <BKCE2&09Paperll-18>
DSE Physics - Section D : MC.
EM3:DomesticElectricity
PD-E
M 3-M 1
/ 211

51. <HKCE2010Paperll-16>
DSE Physics - Section D : MC
EM3 : Domestic Electricity
PD-EM3-M/13

Two •identical bulbs, X and Y, are connected in parallel to the mains. X The photo below shows the back of a microwave oven.
The current passing through Xis 0.4 A. What is the energy consumed
by the two bulbs in 5 hours ?
y Power supply: 220V - 50Hz
A. 0.44kWh
Power consumption: 1150W

"v--�
B. 0.88kWh
C. 440kWh �--220V Microwave output :
Microwave frequency :
750W
2450MHz
D. 880kWh

48. <HKCE2009Paperll-20>
The power ratings and resistances of two light bulbs are "24 W, 6 n'' and "9 W, 4 0" respectively. Ifthese two light bulbs 'iVbich of the following statements is/are correct?
are connected in parallel to a power supply, what is the maximum current drawn from the power supply so that both light
bulbs are working within the :rated power ? (1) The current flowing through the microwave oven is about 3.4 A.
A. 2.0A (2) Around 65¾ of electrical energy is converted into energy carried by microwave.
B. 2.SA (3) The wavelength of the microwave emitted is about 0.12 m.
C. 35A
A. (1) only
D. 4.8A
B. (2) only
C. (1) & (3) only
D. (2) & (3) only
49. <BKCE2009Paperll-23>

-:'t&
52. < HKCE2010 Paper n. 20 >

Which of the following is the COIIect connection ofX, Y and Z to the mains ?

"""
X y z beating element
A. live

'""' """
neutral
In the figure above, the kettle is wired incorrectly. Which of the following statements is correct when point P is accidentally

"""
B. live neutral
connected to the metal case ?
C. neutral live
D. neutral live A. The kettle will still operate at its rated value when Sis closed.
B. The fuse will blow when S is closed.
50. <BKCE 2009Paperll-41 > C. The heating element will bum out when Sis closed.

- -
D. A current will flow through the live wire even when Sis open.
A ma.ms heater has two identical heating elements of same resistance R.

- 53. < HKCE 2010 Paper II· 19 >


s, s-,-
R

!i.
· R
s,

When S1 aD.dSi are closed andS3 is open, the power of the heaterisP. What is the power of the heaterwbenS1 and S2 are In the circuit above, the rating of the bulb is "6 V, 12 W", Find the resistance of R so that the bulb will work at its rated
open and S3 is closed? value.
A. 0.25P A. 2n
B. 0.5 P B. 30
C. 2P C. 4Q
D. 4P D. 6Q
DSE Physics · Section D : M.C. PD-EM3-M/14 DSE Physics - Section D : M.C. PD-EM3-M/15
EM3 : Domestic Electricity EM3 : Domestic Electricity
54. < HK.CE 2011 Paper JI- 11 > Which of the following statements concerning the fuse in an electrical appliance are correct?
As shown in the figW'e, a 2 W light bulb is immersed into 50 g of water. (I) A �e is made ofa metal with low melting point.
The bulb is operating at its rated value. After 10 minutes, the (2) Jfthe fuse in an electrical appliance is blown, it should not be replaced by a piece of copper wire.
temperature of the water increases by 4.5°C. Estimate the amount of (3) If copper is used to replace the blown fuse, it will cause short circuit of the appliance as copper has low resistance.
light emitted during the 10 minutes.
Given : specific heat capacity ofwater = 4200 J k g-1 0
c-1 A.
B.
(I) & (2) only
(1)&(3)only
C. (2) & (3) only
A 255J
D. (!), (2) & (3)
B. 690 J
C. 945 J
D. 1200 J 59. Small light bulbs of rating "12 V, 6 VI" are to be used in a circuit. The voltage of the power supply is24 V and the cmrent
drawn from the supply is 2 A What is the number of light bulbs connecting in the circuit so that each light bulb is under
nonnal rating ?
A. 2
B. 4
Part B : BKAL examination questions C. 8
D. 16
55. < HKAL 1980 Paper I - 22 >
Torch bulbs marked ''3 V, 1.5 W'', are to be used in a circuit by using 6 V battery. What should be the number of bulbs 60. Which of the following statements concerning two light bulbs with rated values, '200 V, 100 W' and '200 V, 40 W', are
connected in the circuit ifthe steady current drawn from the battery is to be2 A and each bulb gives normal brightness ? correct?
A. 2 (1) The energy dissipated by the '200 V, 100 W' light bulb is greater than that of the '200 V, 40 w• light bulb when
B. 3 they work at their rated values.
C. 4 (2) The current flowing through the '200 V, 100 W' light bulb is greater than that ofthe '200 V, 40 W' light bulb when
they work at their rated values.
D. 8
(3) The resistance of the '200 V, 100 W' light bulb is greater than that of the '200 V, 40 W' light bulb when they work
at their rated values.
56. < HKAL 1983 Paper I- 43 > A (1)&(2)only
A set ofChristmas tree lights consists of20 light bulbs in series connected to a supply of 200 V. Each light bulb has a rating B. (I) & (3) only
of"l0 V, 5 W''. When one bulb bums out, Jenny goes to buy a replacement When she gets bac k, she finds that although the C. (2) & (3) only
newbulb is marked 5 W, the light it gives is dimmer than the other bulbs. Which of the following is a possible reason ? D. (!), (2) & (3)
(1) The supply voltage bas dropped below200 V.
(2) The current through the circuit is less than 0.5 A 61. Which ofthe following concerning the household circuit is/are correct?
(3) The rated voltage ofthenewbulb is less than 10 V. (1) Fuse.should be installed in the live wire ofan electrical appliance.
(2) Switch should be installed in the neutral wire ofan electrical appliance.
A. (1) only
B. (3) only (3) Current always flows from the live wire through the appliance to the neutral wire.
C. (1) & (2) only A (1) only
D. (2) & (3) only B. (3) only
C. (I) & (2) only
D. (2) & (3) only

Part C : Supplemental exercise 62. Which of the following statements concerning two light bulbs with rated values, '200 V, 100 W' and '200 V, 40 W', is/are
correct?
57. In a household circuit, as more lamps are switched on, which of the following statements are correct? (1) The resistance of the '200 V, 100 W' light bulb is greater than that of the '200 V, 40 W' light bulb.
(1) The equivalent resistance of the whole circuit increases. (2) The current flowing through the '200 V, 100 W' light bulb is greater than that of the '200 V, 40 W' light bulb when
(2) The total power consump6on increases. they are connected in series.

(3) The total current drawn increases. (3) The power dissipated by the '200 V, 100 W' light bulb is smaller than that of the '200 V, 40 W' light bulb when
they are connected in series.
A (1) & (2) only
A (1) only
B. (1) & (3) only
B. (3) only
C. (2) & (3) only C. (1) & (2) only
D. (!), (2) & (3) D. (2) & (3) only
DSE Physics - Section D : M.C.
EM3 : Domestic Electricity

Part D : BKDSE examination questions


PD-EM3-M/16

I
67. <HKDSE2013Paper1A-33>
DSEPhysics-SectionD: M.C.
EM3 : Domestic Electricity
PD-EM3-M/1711

Which of the following domestic electrical appliances consumes a power close to 1 kW in nonnal working conditions?
63. < HKDSE Sample Paper IA - 26 > A. an electric fan
B. a microwave oven
If a 15 A fuse is installed in the plug of an electric kettle of rating value '220 V, 900W', state what happens when the kettle C. a fluorescent lamp
is plugged in and switched on. D. aTVset
A The kettle will not operate.
B. The kettle will be short-circuited.
C. The output power of the kettle will be increased. 68. < HKDSE2015 Paper IA - 28>
D. The chance of the kettle being damaged by an excessive current will be increased. Which statement is NOT a reason why mains socket at home are connected in parallel instead of a-series circuit?
A. Electrical appliances connwted to differep.t sockets can be switched on or offindependently.
B. Voltage supplied to each socket is :fixed and all eleetrical appliances can operate at their rated voltage.
64. < HKJ)SE Sample Paper IA-25 > C. The current supplied can be reduced and thinner cables can then be used.
D. When an electrical appliance breaks dO'Wll and becomes an opep. circuit, other appliances can still work.norm.ally.
The table shows three electrical appliances which Clara used in a. certain month:

Appliance Rating Duration 69. <Bla>SE 2015 Paper IA- 29 >


air-ctmdi:tioner 220V, 1200W 250 hours An electric iron of 1800 W sold in Hong Kong (220 V SO Hz) is connected to a 110 V 60 Hz ma.ins socket in another
television 220V,2S0W 80 hours country. How does its performance compare on the same ironing setting'?
A. The electric iron does not work because the a.c. supply is 60 Hz instead of SO Hz.
comp- 220V, 150W 60 hours B. The electric iron is as hot as it is used in Hong Kong.
C. The electric iron is hotter than when it is used in.Hong Kong.
calculate the cost of electricity used. Note : 1 kW h of electricity costs$ 0.86. D. The electric iron is colder than when it is used in Hong Kong.
A $ 62.25
B. $73.79
C. $282.94 70. < Hl{DSE2016 Paper IA- 28 >
D. $ 536.64
A television set in stand-by mode consumes LSW. If it is in this mode for 16 hours a day, estimate the carbon dioxide (CO2.)
emission due to the electricity consumed.in stand-by mode in a 30-day month.
65. <llKDSEPracticePaperIA-30> Given: 1 kWh of electricity consumed corresponds to 0.8 kg CO1 emission frotn the power station.
A. 0.576 kg

· =·��;;::=
,.-----;:::;-E?-!
B. 0.720kg

L(_:::tf:��,
C. 576 k g
D. 720kg

me case heating element -l- 71. <BKDSE 2017 PaperIAR 25 >


The figure above shows the main parts of an electric iron. In which of the followmg situations will the fuse blow when the Which of'lh.e following statements about the use ofa fuse is correct ?
switch is closed ? A. A fuse should be installed in the neutral wire.
A. The heating element is broken and becomes an open circuit.
B. A fuse is not required in an electrical appliance 'With double insulation.
B. The earth wire is wom out and becomes disconnected.
C. The insulation at contact pointXis wom out so that the wire touches the metal case. C. A SA fuse is suitable for a heater of rating '220V, 1500W'.
D. The insulation at contact point Yis wom out so that the wire touches the metal case. D. The melting point of a fuse should be lower than that of copper.

66. <HKDSE 2012 PaperIA-33> 72. <Hl{l)SE2018 Paper IA-24>


The figure shows a simple domestic circuit for an electric
iron. The fuse will blow when which of the following
points are short-circuited ?
(1) XandY
(2) Y,uuiZ Three identical resistors are azr.mged as shown. The rated power of each resistor is 12 W. If no resistor exceeds its rated
(3) XandZ power, what is the maximum power dissipation in such an arrangement?
A. (1) only A. 16W
B. (3) only B. 18W
c. (1) & (2) only C. 20W
D. (2) & (3) only D. 24W
73. <HKDSE 2020 Paper IA-2 >4

The figure shows part ofa domestre


· ",,...., _ cimrl in which
switcl>S isclosed. ""=8 t the bull> no t light up when
L does

N� traI - -j--- - �d_ _

s
L
'--+"l--.J c
h

The�is�en checke d h
wit switchScJ oscd. Using avoltage tester totouc h pointsban dcin rums
the �r.mdicates
tb8: bo � points
test er mdicates only pOinta IS athighare athigh voltage. When touching point5 a and in ru.ms, the
.
v oltagt. Whichof th e follo wing can be a reason fo d
rthe fault?
A. The Switc h
Shasbe en damaged.
B . The filamen t ofbulb Lhasb eenb'1rl)toutand becomes an open cireu: it.
There is a sh
o rtcircuit!,e!Wffll a
C. and
D . There isanopen circuitbetween c andd. d.

7 4. <HKDSE 2020P aperIA -25 >

The batter y shown has a capacity of 1100 mAh. How much energy is delivered
tte s op
e ho ur ? Assume that the ba ry' eratinthe vbatt,ry
when l ta
cum,mof250 mA for on o ge
operates nOtnJally a a g
1

remainsat 3.7 V duringthatperiod

A. (3.7 X� X 3600 )]
1000
(J.7x�x3600 )J
B.
1000

C. (37 x �x
1000
l)J

(37 x � x l)J
D. . 1000
DSE Physics - Section D : M.C. Solution PD-EM3-MS/Ol DSE Physics - Section D : M.C. Solution PD-EM3-MS /02
EM3 : Domestic Electricity BM3 : Domestic Electricity
HXEAA's Marki:og Scheme is prepared for the markers' :reference. It should not be reganied as a set ofmodel answCIS.
Students and t.eachers who arc
not involved in the mamng process arc advised to interpiet the Marking Scheme with care. 4. D

I==� x 15 = � x 15 = 4.5A
M.C. Answers V, 200

I. B 11. D 21. A 31. A 41. A 51. D 61. A 71. D Value of fuse should be equal to or greater than]

2. C 12. A 22. D 32. D 42. B 52. D 62. B 72. B 5 A is the appropriate one

3. D 13. B 23. A 33. D 43. A 53. B 63. D 73. D


S. B
4. D 14. D 24. C 34. C 44. C 54. A 64. C 74. A
100x8+ 1000 =9A
I = B._ x8+ � =
V V 200 200
5. B 15. D 25. D 35. B 45. B 55. D 65. C
Value of fuse should be equal to or greater than/ ⇒ 10A is the appropriate one
6. D 16. C 26. C 36. B 46. B 56. B 66. C
6. D
7. D 17. D 27. C 37. B 47. B 57. C 67. B
By R � V. , for hair dryer: R = (20 0) = 66.7 n which is the smallest resistance.
2 2

8. C 18. C 28. A 38. C 48. B 58. A 68. C (600)


P,
9. C 19. D 29. A 39. D 49. D 59. C 69. D :. It gives the greatest current for a given voltage.
10. A 20. C 30. D 40. D 50. A 60. A 70. A
7. D
2 2
= (200) a::20O
2
Rll:W = ( 00) = 400 and �
M.C. Solution 1000 2000
kW

:. Equivalent resistance=40 + 20=60 n


1. B
:. P=£:_= ( l00) =0.67kW
2

Voltage across 18 n = voltage across 21 n = (2.4)(21) = 50.4 V' R (60)


.4
lx=S0 =2.8A<3A .-. fuseXwillnotbeblown
18
8. C
Jy = Ix+lz = 2.8+2.4 = 5.2A > SA .-. fuse Ywill be blown
Earth: Z (connected to the metal case of the electric fan)
Live : Y (this wire has a switch)
2. C
Neutral : X (giving a returning path)
A. R = V,
2
= (200)2 =200. B.
2
R = (Z00} =1600
P, (2000) (250)
9. C
R= (100) 2 02
R = (l0 ) =500 RR= (200) =1 000 0 ' Rs= (200) =667 D:
2 2
c. =50 D.
(2000) (20) 40 60
Heater has the smallest resistance. (1) smaller resistances in parallel gives smaller equivalent resistance (by ..!. "'..!....+...!.... ),
R R1 R,_
(2) smaller equivalent resistance ⇒ larger current ⇒ maximum brightness (for same voltage)
3. D
C gives the maximum brightness.
A. If cmrent is smaller cwrent, then all bulbs would become dimmer
B. If the resistance is higher, then it should give greater power as P =PR, and thus become brighter 10. A
C. Ifthe bulb is shorted accidentally, then it does not work and would not look dimmer
All resistors in parallel must give the smallest equivalent resistance, thus it gives the maximum brightness
2
✓ D. As R = V.. , for the same rated power 5 W but lower rated voltage, the new bulb has smaller resistance
P, 11. D
by P = 12 R, the new bulb has smaller power and thus looks dnnmer
For the two appliances working in rated values, P = 2000 + 500 = 2500 W.
DSE Physics - Section D : M.C. Solution PD-EM3-MS/03 DSE Physics - Section D : M.C. Solution PD-EM3-MS/04
EM3 : Domestic Electricity EM3 : Domestic Electricity
12. A 20. C
As VR = 3V 2
✓ (1) R = V, = (Z2o)2 = 96.80
500
:. I=�= 2_::a I.SA
v. 6
• (2) I,=�= S00=2.27A
220
R= VR=�=2Q
I 1.5 ✓ E =Pt = {0.5)x(2)"" I kWh
(3 )

13. B 21. A
The longest pinX i s the Earth pin.
The switch should be connected in the livewire L so th at the iron can be cut off from high volt agewhen ti is off
Pin Yis the Live pin and pin Xis the Neutral pin.
T heearth wire E should be connected to the metal caseofthe iron.

14. D
2000 = 10 A > 5 A 22. D
(1) I= � = :. 5 A fuse should not beused, otherwise, it will beblown.
V, 200 lkWh = (l000W)x(60x60s) = 3,6x106 J
✓ (2) As thebrown wire is the live wire, the fuse should be placed on itto allowthe fuse cutting off the circuit
when the current is too large. 23. A
✓ (3) Yellow and green wire is the earth wire. P, Earthwire ⇒ Green/yellow ⇒ Z

Q, Neutral wire⇒ Blue⇒ Y


15. D
R, Live wire⇒ Brown ⇒ X
Pin 1 is the Earth socket
Pin 2 is the Neutral socket.
24. C
Pin 3 is theLive socket
One light bulb burnt out ⇒ no cummt flow through that series

16. C ⇒ all bulbs in that string will go out


E = Pt
= {LS)x(2) = 3kWh 25. D
x A. S i:Son ⇒ circuit complete d ⇒ iron will operate
17. D
B. Sisoff⇒ circuit notcompleted ⇒ ironwillnotoperate
Fuseis use d to limit the CIIUellt of circuit Switch is used to tum off theappliance from the circuit.
C. Hig h cumnt ⇒ fuse will blow ⇒ circuit cut ⇒ iron will not oper ate
Both ofthem should be conne cted to the live wire,
so that the appl i ance i s cut offfrom high voltage w hen thecircuit is disconnected.
✓ D. S is not in live wire ⇒ the heating element is still at high voltageeven when Sis off.

26. C
18. C
2 kWh• kW•h • (PJ(� • Ene,gy
✓ (1) By R cc v; oc 2.. :. A: larger rated power P, ⇒ smallerresistanceR
P, P,
27. C
✓ (2) In series ⇒ same cunent pass ing through
✓ (1) Presence of Earth wire connected to metal case
(3) By P=l 2R, A; smaller�esistanceandsamecurrent ⇒ sma l ler power output ⇒ dimmer ⇒ provides a path for conduction of current. to the Earth
⇒ prevent electric shock
19. D ✓ (2) Current returns back to power supply v i a neutral wire instead.
Voltage : unit V (volt)
(3) Earth wire is not connected to znams socket, fi neutral wire is broken, no current flows.
DSE Physics - Section D : M.C. Solution PD-EM3-MS/05 DSE Physics - Section D : M.C. Solution PD-EM3-MS/06
EM3 : Domestic Elec1ricity EM3 : Domestic Elec1ricity

28. A 34. C
1 Light bulbPis not un derrated con d ition, since P oc V2
I = ; ⇒ resistanceof ap plia nce = resistance of wire s = R
:. V➔½V ⇒P➔¼P :. actual powerofPis20W
:. equivalent resistanceof thee lectrical appliance and the con necting w ires = 2R
Light bulb Q is under rated condition, thus the actual power is 80 W
By V=IR Light bulb R is alsounder rated condition, thus thea ctual power is 40 W.
I f voltageofpower supply : V➔ 2V and no change to the resistance The brightestlight bulb isQ and the dimmest light b ulb i s P.
:. It gives a current of J••
35. B
29. i.. Pin Pis the earth pin which is connected to the metal case of ht e electric appliance.
✓ (1) Current flows to the ground clirectly ⇒ no cunent through the beating element Whe n the live wire accidentally touches the metal case, theearth wire p rovidesa low resistance path
⇒short circuit ⇒ current becomes very large ⇒ fuse blow s to conduct the c urrent totheearth, thus prevent theelectric shock ofhuman bodies.

(2) Current flows through the heating element beforereaching the metal case => no increase in J
36. B
(3) Heating element broke n ⇒ the circuit not completed ⇒ no cuuent flow s through the fuse
No. of wa shesperyear = 260 Av eragetime pe rwash = l.8hours
Total operating time = 260 x 1.8 = 468 h ours
30. D
By E = Pt
Both the switch and the fuseshoul d be con ne cted to the live wire
(250 kWh) = P (468 h)

31. A P = 0.534kW = 534W


2
✓ (1) By R = V. the rated power of Xishalved ofthat of Y, thus theres is tanceis twice ofthat of Y.
37. B
,

Electric heater: E = 2SOO kW x 30 h = 1.25 kWh


Since P = V
2
(2) :. P cc V2 :. when voltage is doubled,. the powershould be 41:imes. 1000 60
R
(3) Since their resistances are not equal, the voltageacross each beater will not be 110V Television : E = �kWx5h = I.35kWh
1000

32 D E = �kWx8h = 1.20kWh
1000
Resistanceoflamp = 6'/12 = 30
Since the cost is p roportional to the amount o felectrical energy used
Total resistance = 3 / 2 -+ 3 = 4.5 n
Cw:re n tflow:from the battery = 6 / 4S = 1.33 A
38. C
VoltageacrossL; = I.33x3 = 4V
Totalelectrical energyused = 1.2kWx250h + 0.25kWx80h + 0.1SkWx60h = 329kWh
Voltagea c rossL1andLi. = 6-4 = 2V A isnot correct.
CmrentthroughL 1 = 133/2 = 0.67 A B is not correct. Cost ofe lectricity = 329 x $ 0.86 = $ 282.94
Total power drawn from the battery = 6 x 1.33 = 8 W C is not correc t.
Total power dissipated in.Li andLz = 1.332x1.5 = 2.7 W 39. D

Power dissipated in.L3 = 1.332 x 3 = 5.3 W .·. Di s correct. A. The kettle will op erateno matter which fuse is installed .
B. The kettle will not be short-circuited unles s ht elive touche s the neutral.
33. D c. The kettle will still work properly with therated power of 900W.
Ifthelive and neutral wires are interchanged, then the switch woul d become connected on the neutral wire.
✓ D. Since the rated current = 900 / 220 = 4.09 A. The fuse value of 15 A istoohigh.
In this case, when the switch i s off, thekettlewill still stand at a high voltage. Ifthe current exceeds the 4.09 A b ut les s than 15 A,
However, the kettle would work properly when the switch is on. the fuse will not blow and the ket lt e may bedamaged.
DSE Physics - Section D : MC. Solution PD-EM3-MS/07 DSE Physics - Section D : M.C. Solution PD-EM3-MS/08
EM3 : Domestic Electricity EM3 : Domestic Electricity
40. D 46.
(220/ = 1613 Q.
2

X (I) If the resistance is too large, the cmrent would be too small, and the fuse would not blow. Resistanceofeachb ulb = ...t...
v.
= --
P, (3 0)
✓ (2) Ifthe appliance is short-circuited, the current would become very large and the fuse willblow.
Total equivalent resistance for three bulbs in series = 1613 x 3 = 4840 n
✓ (3) If the rated value ofthefuseis too small, then the rated cmrentmay exceed the fuse value
V2 = <220f
and the fuse will blow. Totalpowerdissipated = = IO W
R (4840)

41. A
47. B
Resistance of the '220 V, 50 W' light bulb = (2 Z0)2 = 968 n Totalcur.rentgiven outbythe mainssupply = 0.4x2 = 0.8A
(50)
Total power given out by the mains supply: P = VI = (220 ) (0.8) = 176 W
Resistance of the '220 V, 100 W' light bulb = (2lo)2 = 484Q Totalenergyconsumed: E =Pt= (0.l76kW)x(5h) = 0.88kWh
QOO)
Totalequivalent resistance of the two light bulbs in series = 968 + 484 = 1452 n
48. B
Current drawn from the mains supply = 22 0 = 0.15 A 2
1452 By R= v,
P,
By (6)=� V. = 12V :. Ratedvoltageofthe6Olightbulbis l2V.
42. B
(24)
According to the label, 600 revolutions indicate an electrical energy of 1 kWh, i.e. 3600 000 J.
By (4) = V,.2 .. V. = 6 V :. Rated voltage of the 4 n lightbulb is 6 V.
For24revolutions, E = 24x� = 144000] (9)
600
As the two light bulbs are in parallel, theymust have the same applied voltage V.
Electric power: P = !. = l44000 = 1200W If the applied voltage is 12V, then the 4 n light bulb would be burnt, thus the applied voltage should only be 6 V.
t 2x60
Equivalent resistance ofthe circuit: R = � = 2.4 n
6+4
43. A
When Sis closed, the SR resistor isshorted. The equivalent resistance of the whole circuit is R. Current delivered from the power supply: I = KR = ..!.. = 2.5 A
2 .4
When S is open, the SR and R resistors are in series. The equivalent resistance ofthe whole circuit is 6R. OR
V' I
By P=-cc­ :. Power consumed in the mode ofkeepingwarm = 600 x .!:... = 100 W Current delivered from the power supply: I = � + � = 2.5 A
6 4
R R 6R

44. C 49.· D

Live wire : current drawn is 2A Yislive as switch and fuse are on the live wire.
voltage is at high voltage of220 V Xis neutral to complete the circuit
Neutral wire : current is equal to that of live wire, i.e. 2 A Zis the earth that is connected to the metal case of tho appliance.
voltage is at low voltage of0V
Ear th wire: no current through earth wire when operating, i.e. 0 A so. A
no voltage in the earth wire, i.e. 0V
When S1 and� are closed and 83 is open, the two resistors are in parallel, equivalent resistance is ½ R.
45. B When Si and 82 are. open and 83 is closed, the two resistors are in series, equivalent resistance is 2R.
The equivalent resistance is increased 4 times.
Maximum power given by the mains so·c1cet without triggering the circuit breaker = VI_ = 220 x 15 = 3300 W

Assume n light bulbs at most can be connected. :. 1100 + 550 + 100 n = 3300 :. n = 16.5 By P = v2 , as power is inversely proportional to tho resistance, the power should become .
R .
¼.
Thus, maximum number of lightb ulbs that can be connected = 16. Thus the power is 0..25 P.
DSE Physks - Section D : M.C. Solution PD-EM3-MS/091 DSE Physks - Section D : M.C. Solution PD-EM3-MS/10
EM3:DomesticElectricicy EM3: Domestic Electricicy

51. D 57. C

(1) ByP = VI .·. (1150) - (220) I :. 1=5.23A (1) As more lamp s are switched on in parallel, the equivalent resistance should decrease.

✓ Percentage of energy carried by the microwave = 750 / 1150 = 65%


✓ (2) As more power is given to the lamps, the total power consumption increases.
(2)
✓ (3) Ai; the equivalent resistance decreases, IIlore current is drawn from the supply.
✓ (3) Wavelength: ?,. = v/ f"' (3 x 108) / (2450 x 106) = 0.12m

58. A
52. D
✓ (1) Fuse should be made of metal with low melting point
When S is closed, current would flow from live t o the metal case and then to earth as it is a shorted path,
so that it would be melted by the heating effect of the excess cµxrent
thus no current would flow through the heati ng element and the kettle will not operate.
✓ Copper has a high melting and cannot be melted easily,
B. As the cwrent would not flow through the heating element and the fuse, the fuse would not blow.
thus even the cmrent exceeds the rated current of the fuse, copper would not be melted and blown.
C. As there is no cmrent flowing through the heating elemen t, the heating element would not bum out
(3) Both the fuse and the copper wirehave negligr.ble resistance,
✓ D. When S is open. current can still flow through the live wire to the metal case, as they are connected in series to the appliance, they would not cause short circuit ofthe appliance.
and then to the earth wire to complete the circuit.

59. C
53. B
For the bulb being fully lit,
Rated current = !i.,_ = g
= 2A
(!) Number ofbulbs in each series path =
24
=2
V, 6
12
Voltage across the two resistors = 9 - 6 "" 3 V
(2) Current flowing through each parallel circuit= current flowing through eachbulb= � = £ = 0.5 A
V 12
Equivalent resistanceof the two resistors = !'.'.:. = ! = 1.5 n
I 2 Number of parallel circuit connected to power supply=..!_= 4
0.5
As the two resistors are in parallel: � = 1.5 :. R = 30
2 .-. Total numberof bulbs= 2 x 4 = 8
OR
54. A Power giveno� by the power supply = VI = (24) x (2) = 48 W
Electrical energy supplied to the light bulb = Pt = (2)(10 x 60 ) = 1200 J Number ofl ightbulbs = 48 = 8
Heat energy given out by the light bulb = m c !:i.T = (0.050) x (4200) x (4.5) "'.' 945 J
Lightenergygiven outbythe light bulb = 1200-945 = 255 J
60. A
✓ (!) When they work at their rated values, power given out by the '200 V, 100 W' light bulb is l 00 W
55. D
which is greater than the 40 W given out by the '200 V, 40 W' light bulb.
Total power givenout by the battery = VI= (6) x (2) = 12 W
✓ (2) By P = V1, as the power given outby the '200 V, 100 W' light bulb is greater,
For each li ght bulb to give normal brightn ess , the P?Wer given out by each lightbulb is 1.5 W.
the cmrent drawn is greater.
Total number oflight bulbs in the circuit = E..
1.5
= 8
(3) By R = )i'?J Pr , asthepowergivenoutby the '200 V, 100 W' light bulb is greater,
its resistance is smaller.

56. B
61. A
(!) Voltage droppedbelow 200 V ⇒ all light bulbs would become dimmer
✓ (!) Fuse should be installed in the live wire,
(2) Smaller current ⇒ all ligh t bulbs become dimmer
so that the appliance is cut offfrom high voltage when the fuse is blown.
✓ As R =
V' , for same rated power 5 W but lower rated voltage, the new bulb has smaller resis tance
p
(3) (2) Switch should also be connected to the live wire,
so that the appliance is cut off from high voltage when the switch is off.
As P = 1 R for light bulbs in series, P«:R :. the new bulb has less power than theother bulbs
2

(3 ) Since household circuit is an a.c. circuit, Cl.llieDt flows from L toN and fromN to L alternately.
DSE Physics - Section D : M.C. Solution PD-EM3-MS/11 DSE Physics - Section D : M.C. Solution PD-EM3-MS/12
EM3 : Domestic Electricity EM3 : Domestic Electricity

62. B 70. A
(1) By R = Vl I Pr, as the ratedpower P,ofthe '200 V, 100 W' light bulb is greater, E =Pt= (0.0015kW)X(l6bx30) = 0.72kWh
its resistance is smaller.
Carbon dioxide emission = 0,72 x 0.8 = 0.576kg
(2) When they are connected in series, the current flowing th rough each ofthem must be the same.
✓ (3) As the resistance of the '200 V, 100 W' light bulb is smaller, 71. D
by P = 12 R, same current I in series, thus, P cc R, therefore, the power d issipated i s small er.
A. Fuse should be installedin the live wire.

63. D B. The fuse is still required to limit the current to avoid overheating.

The kettle will operate no matter which fuse is installed.


For electrical appliance with doubleinsulation, there is no metal case, thus, earth wire i s notneeded
A.
B. T he kettle will not short-circuited u nlessthe live touches the neutral. C. Fora heater ofrating '220 V, 1500 W', the rated cmrent: Ir = 1500/220 = 6.82 A
A SAfuse i s not suitable as it would be blown vm.en the heater is switched on.
c. The kettle willstill work properly with the rated p ower of 900W.
✓ ✓ D. Fuse has a l ow m eltingpoint that it can melt and break the circuit once cur.rent exceeds th e rated current
D. Since the rated current = 900/220 = 4.09A :. thefusev alueoflSAistoohigh.
Thus, its melting po n
i t mustbe lowerthan that of copper'Wire.
Ifthe current exce eds 4.09 A but less than 15 A, the fuse will not blow and the kettle may be damaged.

64. C 72. B

Total electrical energy used= 1.2 kW x 250 h + 0.25kW x 80h + 0.15 kWx60 h = 329 kWh Since the voltage ofR1 is equal to the sum of voltage ofRz and&,

C ost of electricity = 329 x$ 0.86 = $ 282.94 R1 should work underratedpower t o give Pi = 12 W.


Since th e v oltage of Rz ishalved of R1, R, R,
65. C by P = V2!R o:: V2,thepowerof&:isone-fourthofR1
If the insulation atXis worn out, once the switch si clos ed,
P2 = 12x(½J = 3W
current would flowfrom the Live wire through the metal case to the Earth wire.
As th eresistance of themetal cas e si negl g
i l.Ole, current would be vezy larg e and the fuse would blow. Sincethevoltage of.R; is equal to that of&:, thusP3=P2 = 3 W
Totalpower = 12+3+3 = 18W
66. C
✓ (1) lfXand Y are short circuited, current would notflow through the resistor and becomes very large.
✓ (2) If Y and Z are sh ort circuited, current would notflowthrough the resis t ro and becomesvery large.

(3) IfXand Z are short circuited, current must still flow through the resistor and wouldnot b e too large.

67. B
A microwave ovennor.mally has apower around 1000 W.
Other appliances : electric fan, fluorescentlamp and TV, have power much less th.an 1000 W.

68. C
Inparallel circuit, current supplied by the mains is the sum of current in each parallel path.
Current cannotbe reduced by usingparallel circuit

69. D
A. The :fre quency of the a.c. mains docs not affectthe working ofan electric iron.
B. As the appliedvoltage is smaller th.an that in HongKong, thus the power cannot be the same.
C. As the applied voltage is smaller than that in Hong Kong, thus the power cannot be hi ghe r.
✓ D. For th e s ame res istance of an appliance ; P = V2 IR :. p 0:: V2
Since the mains voltage of country Xis lower, thepower given out is smaller, thus the iron si colder.
DSE Physics - Section D : Question PD-EM3-Q/01 DSE Physics - Section D : Question PD-EM3-Q/02
EM3 : Domestic Electricity EM3 : Domestic Electricity

The following list of formulae may be found useful : 1. (b) (i) A 15 A fuse is connected to thesocket Which line : earth, neutral and live, should the fuse be placed? (1 mark)

Resistance and resistivity


(ii) Findthe maximumnumber of kettles that can be joined in parallel to the socket without blowing the fuse.
(3 nra,ks)
Resistors in series

Resistors in parallel 1 1 1
-=-+-
R R, R,
(c) (i) If the efficiencyofthekettleis 80%,howlongwillittaketo heat 1 kg ofwaterfrom20°C to 100°C? (4 marlcs)
Power ina circuit

Energy transfer during heating or cooling E= mcl:::..T

Energy transfer during change of state E=l&n


(ii) If the cost of electricity is S0.80 per kWh, how much does thls heating process cost?

Part A : HKCE examination questions

-
1. <HKCE 1981 Paper I- 8>

Socket Plug 2. < HKCE 1983 Paper I - 7 >

nrntr.,l[ 0 OO ] liv, fuse box i:�_ _;'


heating lighting
unit unit

c-----'
A household electric circuit consists of a heating unit ( 220 V, 1100 W) and a lighting unit ( 220 V, 220 W) connected in
A ___,,!,-. parallel to themains of220 Vas shown in the above :figure.
(a) Find the maximum cummt drawn from the mains. (2nra,ks)
Electric kettle

The above figure shows a220 V supply socket, a 3-pin pl ug, and an electric kettle with a rating: '220V 1.65 kW'. The kettle
has 3 wires A, B, C leading from it. Wire A is joined through a switch S to the heating element Hofthe kettle ; wtte B
completes the circuit of the kettle; wire C is joined to the metal case ofthe kettle.
(Given: Spel?ifiC heat capacity ofwater = 4200 Jkg-1 0C-1) (b) Is a 5 A fuse suitable for use in the fuse box? Explain briefly. (2 marks)

(a) (i) To which of the pinsX, Y, Z of the plug should each of the wires A, B, Cof the kettle be connected? (3 nra,ks)

(c) If ele ctrical energy costs $0.90 per kWh and the whole system is switched on for 150 hours, what will be the cost of the
electricity used? (2 marks)
(il") What is the fimction of the 'earth' tenninal in the socket? (1 mark)
DSE Physics - Section D : Question PD-EM3-Q/03 DSE Physics c Section D : Question . PD-EM3-Q/04
EM3 : Domestic Electricity EM3 : Domestic Electricity

3. < HKCE 1986 Paper I- 7 > 4. < HKCE 1988 Paper I- 7 >
The figure shows a simplified system of a 220 V domestic circuit N and L denote the neutral and live wires respectively. Given socket A and socket B, an unqualified electrical technician wires the two sockets to the live L, the neutral N, the earth
B of the 220 V mains supply for a heater rated at "220 V, 1000 W" and a cooker rated at "220 V, 800 W'' as shown in the
:figure below. (The sockets are viewed from the front)

5A Fusebox

NeutralN LlveL

(a) (i) If either one of these appliances is plugged into one of the sockets, would there be any current drawn from the
mains with the switch ofthe appliance on? Explain briefly. (2marks)
(a) Give a reason why the lamps are all connected in parallel instead ofin series.

(b) The light bulbs of the circuit shown in the above :figure are all marked "60 W 220 V''. Suppose that all the light bulbs (ii) Ifboth appliances are plugged into the sockets, would the appliances work at'the rated power (1000 W and 800 W)
are switched on. (5 marks) with switches on? Explain briefly. (2 marks)
(i) What is the total resistance of the lighting circuit ?

(iliJ Draw a diagram to show how the two sockets should be connected to the live, the neutral and the earth of the mains
supply with a fuse placed at a proper position. (4 marks)
(ii) What is the total current drawn from the power supply'!

(c) (i) Explain the use of the fuse in the circuit. (2m,oo)

(ii') Give a reason why the switches should be connected to the live wireL instead ofto the neutral wireN. (2 marks)

(iii) What physical quantity does the meter measure ? (2 m,oo) (b) Suppose the heater and the cooker are switched on in the correct wiring circuit for 2hours, calculate
(i) the total current drawn from the mallls supply, and (3=ks)

(d) The household voltage supply in Hong Kong has changed from 200 V a.c. to 220 V a.c. in 1995. Give one reason for
the change. (2 marks) (ii) the total cost of electricity if one kilowatt-hour of electric energy costs $0.90. (2m,oo)
DSE Physics - Section D : Question PD-EM3-Q/05 DSE Physics - Section D : Question PD-EM3-Q /06
EM3 : Domestic Electricity EM3 : Domestic Electricity

5. < BK.CE 1992 Paper I - 5 > 6. <BKCE1998Paperl-4>


A student uses the following components to measure the resistance of a light bulb : A battery, an ammeter, a voltmeter, a
switch, a varial;,le resistor and the light bulb. An incomplete circuit for the experiment is shown below:

I
The above Figure shows a 3-pin plug and a kettle.

(a) To which ofthe pins A, Band C ofthe plug should each of the wires x; YandZ ofthe kettle be connected? (2 marks)

(a) On the above figure, use suitable circuit symbols to complete the circuit Indicate on your diagram the positive terminals
ofthe ammeter and voltmeter with''+'' signs. (5 marks) (b) (i) Explain why itis safer to have pin A ofthe plug longer than the other two pins.

(b) What is the :function of the variable resistor in the circuit?

(il) Explain why switch S ofthe kettle is connected in wire X instead of wire Y.

(c) The below figure shows the result obtained in the experiment.

,. 30
(c) The ratingofthe kettle is '220 V, 2000 W'.
(i) Ifthe kettle is switched on for halfan hour, calculate the cost of electricity.
(Given: One kilowatt-hour of electricity costs $0.9.) (2 marks)

Voltmeter (0-1 V) Ammeter (0 - 50 mA)

(i) Whatis (1) the voltmeter reading, (2) the ammeter reading?

(n) Calculate the resistance of the light bulb. A housewife plugs the kettle and an oven of rating '220 V, 2500 W' into a 15 A socket as shown in the above
figure. Explain why this connection is dangerous. Show your calculations. (3 marks)

(d) Theratingofthelightbulbis"220V, llOW".


(d) A student makes the following note in his book:
(i) Calculate the resistance of the bulb when it is working at its rated value.
In case either wireX or Ytouches the metal case of the kettle accidentally, the kettle will stop working.
Explain whether the student's note is correct

(ii) Explain why the resistance found in (d)(i) is much greater than that found in (c)(i:i). (2marks)
DSE Physics • Section D : Question PD-EM3-Q/07 DSE Physics • Section D : Question PD-EM3-Q/08
EM3 : Domestic Electricity EM3 : Domestic Electricity

7. < HKCE 1999 Paper I-1 > 8. (b) Consumel" unit


E L N

M
OiiilJ
=N OA 30A !SA

Figure z
Figure 1 shows the front viewofa socket and theearth (E), live (L) and neutral (N) wires of the 220 V mains supply.
\ To water
. heater=
(a) On F'igure 1, show how the socket is connected to the mains supply. (2 marks)

(b)
ModelNo. : CS301
Ring cifcui.t in
F>gw-,2 220V SOHz the kitchen
2500W

Figure 2 shows the label attached to an electrical appliance. If the appliance is switched on for 150 hours in a month,
calculate the cost of electricity. (Given: I kWh ofelectricity costs$ 0.87.) (2 marks) Figure 2 shows a household electrical wiring circuit The roams cable (containing live and neutral wires) is connected to
a consumer unit via a meter M At the consumer unit, the wires branch out into a number of parallel circuits. Figure 2
also shows the power circuit in the kitchen. It is in the fonn of a ring circuit with three sockets tapped off from the ring.

(i) Name the meter M What physical quantity does the meter record ? (2mm)

8. < HKCE 2001 Paper I- 9 >


(,) (ii) The following appliances are connected to the ring circuit in the kitchen:

Appliances Rating
a refrigerator 220V, 600W
an electric kettle 220 V , 2000 W
Flg,rel
an oven 220V, 1500 W

If the appliances are all switched on, find the total current drawn :from. the mains supply. (3 marks)

Mazy wants to estimate the efficiency of an electric water heater in her kitchen. She uses a container to collect the water
and a thermometer to measure the temperature (see Figure 1). She finds that when the heater is switched on, 1.6 kg of
water at 23°C is heated to 67°C in one tninute. The rating value of the heater is '220 V, 6000 W' and the specific heat
capacity of water is 4200 Jkg- 1 •c-1 •
(iii) Explain why the water heater mentioned in part (a) is not connected to the sockets in the rings circuit but directly
(i) Fmd the energy absorbed b y the 1.6 kg ofwater in one minute. (2 marks) connected to the m.ains via a separate circuit (2 marks)

(ii) Estimate the efficiency ofthe heater.

(iv) State one advantage of the ring circuit arrangement.

(Iii) State one reason to explain why the efficiency found in (ii) is less than 100%. (lmmk)
DSE Physics - Section D : Question PD-EM3-Q/09 DSE Physics - Section D : Question PD-EM3-Q/10
EM3 : Domestic Electricity EM3 : Domestic Electricity

•• <HKCE 2002 Paper I- 7 > 10. (b} Assume that 80% of the power ou1put ofthe ixonis used to generate steam. Estimate the maxml.um mass of steam that
can be generated by the iron in l s. (2 marks)

(c) The ixon is designed to operate at220 V or 110 Vwith the same poweroutputofll00 W.
�---i Heating coil and fan I----
(i) Ineach of the following cases, find the resistance ofthe heating element of the iron:
In a science project c ompetition, a student constructs a hand-dryer. He connects an electric fan of rating '20 W, 24 V' and (1) whenoperatingat220V,
a heating coil to a 24 V power supply as shown in the Figure l. When switch Sis c ol sed, the fan will operate at its rated
v,Jue. (2) when operating at 110 V. (3nwks)
(a) Are the fan and the heating coil connected in series orin parallel? Explain your answer.

(ii)

(b) If the outputpower of the heating coil is 200 W, find

(i) the operating resistance of the heating c oil,

(u') the total cmrent drawn from the power supply when Sis closed. (4 marks)
Figure2

L (Live wire)
N (Neutral wire)
E (Earth wire)

The heating element of the ixon consistsof two identical resistance wires as shown inFigure 2.

(1) Draw two diagrams to show how the resistance wires are connected when the ixon is operating at220 V and
at 110 V respectively. (3 marks)
10. <HKCE2003Paperl-8>

water tank

Figure 1

(2) What is the resistance of each resistance wire? (1 mark)


Figure 1 shows a travc:l steam iron with a rated power output of 1100 W. The water tank in the iron is filled with water.
When the iron i s turned on, water drips continuously from the tank to a hot plate inside the iron, generating steam for
ir ozring clothes. Assume the initial temperature ofthe water drops is 20°C.
(iii) A tourist switches the iron to the 220 V mode but conne cts it to a 110 V supply. Explain whether the iron can
Given: Specific heat capacity ofwater = 4200 J kg- 1 0C-1, :functionnormally. (3 marks)
specific latentheat of vaporization of water = 2.26 x 106 Jkg-1 •

(a) Calculate the energy requjrcd to vaporize 1 kg ofwater at 20°C into steam. (2mmsJ
DSE Physics - Section D : Question PD-EM3-Q/11 DSE Physics - Section D : Question PD-EM3-Q/12
EM3 : Domestic Electricity EM3 : Domestic Electricity
11. < HKCE 2006 Paper 1- 6 > 12. < HKCE 2008 Paper I- 7 >
Figure 1 shows a ceiling lamp in Jack's home. The lamp has two filament light bulbs, each rated "220 V 40 W''. The lamp is
tumed on or off by a switch (see Figure 2) on the wall.

filament
light bulbs

Figure! Figure2

__
----�....., . ..
.,.....,.
__,wi...,,i,
,.,,

.....,,
Rated Capacity

Heating time
24.1 litres
(a) Give two advantages of connecting the two light bulbs to the 220 V mains supply in parallel. (2 marks)

�:;ri:--l\".....�.. 24.3 minutes


·::...-&-· ..:::u-�7"�-=
...... ...,._:::;:;:..."'="'..=.:::
(15°C ➔ 65 °C)

(b) Explain why the switch should be connected to the live wire of the mains supply. (2m,m)
The above Figure shows the energy label ofa water heater from which some information is listed in the above Table.
Given: mass ofl litre of water "' 1 kg,
specific heat capacity of water = 4200 J k.g-1 0C-1

(a) The heating element ofa water heater is usually installed on the lower position ofthe water tank. Suggest one reason for
this design. (1 mark)
(c) Jack decides to replace each :filament light bulb Vii.th an energy saving bulb of the same brightness. The Table below
shows the details of the two kinds of bulbs. Considering the_ price of the bulbs and the electricity fee, find the total
money saved per energy saving bulb after operating for 4000 hours.
(b) Using the information in the above Table,
Price per bulb Powe, Electricity fee
(i) estimate the energy required to heat a full tank of water from 15°C to 65 °C, (2 marks)
:filament light bulb $5 40W
$0.95/kWh
JI
l
energy saving bulb $35 SW

(ll') hence estimate the current drawn by the water heater when it is operating at 220 V. (3m,m) (3 marks)

(c) Explam why thick wires are used to connect the water heater to the mains-supply. (2marks)
DSE Physics - Section D : Question PD-EM3-Q/13 DSE Physics - Section D : Question PD-EM3 -Q / 14
EM3 : Domestic Electricity EM3 : Domestic Electricity

13. <BKCE2010Paperl-9> Part B : BKDSE examination questions


A hair dryer with a plastic case has three settings of power output : high, medium and low. The settings are selected by the
use of a switch device. The Figure below shows the circuits ofthese three settings in random order. All the resistors in the 14. < BKDSEPractice Paper m - 9 >
heating element have the same resistance of 50 .Q. The hair dryer is connected to 220 V a.c. mains supply.

l
,----------------- ,-----------
' '

l
' ' I


12 V d.c. power supply
,,<r..;..---
circuitX to 220 Va.c.
--- mains supply
p-;-
' heatoc
' '
I ________________ : I•• •--••••• I

heating element switch device joulemeter

,-----------------''
:
'
A 12 V heater is operated Uildet a steady d.c. voltage of12 V. The energyconswned by the heater in 2 minutes is measnred
' by ajoulemeter as shown in the Figure. The initial and final readings of the joulemeter are 126 J and 2525 J respectively.
'
circuity ' to 220 V a.c.
' mains supply (a) Estimate the electrical power of the heater.
'
'
' '
' ' '
I-••••• •••••--•••: ! ••••-••••• I

heating element switch device

l
,-----------------'
' '',-----------'
"o�--j
circuitZ to 220 V a.c.
- _.___ mains supply
-,,
: (b) Hence, find the current through the heater. (2 marks)
' ________________ : '
I•-----•••• I
'

heating element switch device

(a) Find the p<1wer delivered by circuit Y. (3 marks)

(c) A 5 A fuse is installed in the power supply. Explain whether the fuse will blow if another identical heater is connected
in parallel with the original heater. (2 marlcs)
(b) Fill inX, Y and Zin the appropriate spaces in the below Table. (1 mmk)

Power output settings Circuit


L=
Medium
!Egh

(c) Explain why the hair dryer does not riced an earth wire. (1 mmk)
DSE Physics - Section D : Question PD-EMJ-Q/15 DSE Physics - Section D : Question PD-EMJ-Q/16
EM3 : Domestic Electricity El'v.I3 : Domestic ffiectricity
15. < HKDSE2012 Paper IB - 8 > 16. (b) What is the total power dissipated when mode Xis _selected ? Assume that the resistance of each heating element
remains unchanged. (2 marks)
In the circuit shown in the Fi gure, resistors R1 and&. represent the heating elements in a heater using mains supply. Both
resistors arc immersed in water.

·----------,'
:
rs 'v 220Va.c. (c) Without the need of calculations, explain 'Which mode of operation has the largest total power dissipation. (2 """"')

(d) (i) Iffuses 3 A, 5 A and 13 A are available, determine which will be the most suitable to limit an excess etment. Show
yourwork. (3 marks)

The heater can be operated in two modes, namely, heating and keeping warm, and it is controlled by the switch S. The power
consumed by the heater in the heating mode is 550 W and in the mode of keeping warm is 88 W. The mains voltage
is 220 V a.c.

(a) In which mode is the heater operating when switch Sis open? (1 mrul<) (ii) A student claims that since a.c. is used for the heater, the switch S can be installed in either wire A or wire B.
Comment on this claim. (2 marks)

(b) Find the resistance ofR1. (2 """"')

(iii) Ifa fault resulted in the live wire having contact with the metal case of the heater, which wire, A, B or C, could
prevent an electric shock of a person touched the case of the heater ? Explain. (2 marks)
(c) When switch Sis closed, calculate the ClllTent passing through resistor&. (3 marks)

16. <HKDSE2014PaperIB-8> 17. <HKDSE2018PaperIB-8>

The Figure shows the schematic diagram of an electric heater consisting of four identical heating elements, each having a (a) Figure 1 shows the schematic diagram of an eleetric heater which can operate in two modes, namely, 'heating' and
'keeping warm'. The heating elements of resistances 4R and R are connected to the mains supply via a 3-way switch
rated value of '500 W 220 V'. A user can use the rotaty switch to select one of the three modes of operationx; Y, Z. Wrres
with its two poles tied together. That is, both poles can be connected to one ofthe three pairs ofterminalsX, Yor Z.
A, B, Cfrom the heater are connected to the 220 V a.c. mains via a 3-pin plug.

'
3.way sw·tch

·- metal.case
4R
--;--
metal case
ofelectric
ofeleclri1/
heater
L
7• I' -
x\
�:L=-·�4s~��=r�oA Y,
z,
�-------------t------=B fuse

'-------------" -------c -
mains supply

(a) Find the resistance R ofa heating element. (1 mrul<) (i) To which pairs oftenninals, X, Y. or Z, should the switch connect to when the heater is in 'heating' mode ?
(1 mrul<)
I
17.
DSE Physics - Section D : Question
EM3 : Domestic Electricity
(a) The power consumed by the heater in 'heating' mode is 800 W.
PD-EM3
-Q/
1 71 1
DSE Physics - Section D : Question Solution
EM3 : Domestic Electricity
BKEAA's Matkjng Scheme is prepared for the markers' reference. n
PD-EM3 - S I 01
Q

should not be regarded as a set of model answers.


$tlldents and =chels who are not involved in the marking process are advised to inte:l:piet: the Marking Scheme with cme.

(ii) Calculate the current drawn from the 220 V roams supply when the heater is in 'heating' mode. (2 mark s)
Question Solution
I. (a) (i) A ------+ Y [l]

B----->Z [l]
(iil) Find the power consumed by the heater in the mode of 'keeping wann'. (3 """"J
c-----,x [l]

(ii) To prevent electric shock if there is any leakage ofele ctricity to the metal case. [l]
OR
To prevent electric shock by conducting the current to the Earth through the earth wire. [l]

(b) (i) Live [l]


(b) Figure 2 shows a simplified domestic circuit connected to an electrical appliance via a fuse, a meter M, a residual current
circuit breaker (RCCB) and a switch.
(ii) Rated current of each kettle p

=3--------
[l]

I::
V
fuse M switch

r=i = 1650 = ?.SA [!]

D
Figurel 220
:.....,.__.....:
+ Maximum number ofkettle 15
7.5
� 2 [l]
(i) What physical quantity does the meter Mrecord?
(c) (i) Actualpowergiventothewater = 1650x 80% = 1320W [l]

(ii) An RCCB is a kind of safety device that cuts off the supply automatically whenever there is a small difference Energyrequiredtoheatlk.gofwater = (l)x(4200)x(100-20) = 336 0001 [l]
between the currents in the live (L) and neutral (N) wires. State, in each of the following situations, which
device(s) will respond (i.e. the fuse blows and/or the RCCB cuts off the supply). Time taken = 336000 [l]
1320
(1) A short circuit occurs between points X and Y. (1 ""'1<)
= 255 s [l]

(2) A short circuit OCCUIS between point Y and the metal case of the appliance. (1 ""'1<) (h) E = 1.65kWx�h [l]
3600
""0.117kW h [l]
OR

E
= 336000 x 100
[l]
3600000 80
= 0.117kWh [ l]

Cost= 0.117x0.80
= $ 0.0936 < accept $ 0.0933 > [!]
There is question in next page

2. (a) Totalpower = 1100+220 = 1320W [!]


I=!_ = 1320
220
= 6A [l]
V
18. <HKDSB 2019 Ppaper-IB-8>
Flgure 8.1 shows • -.ild electrical wiring ci.rouit. The mains cable (contllini:ng live wire L and
neutral wire N} :is connected to a oonsumer 1lilit '\'J.a a lilowatt--bour meter M. At the consumer unit the wires.
branch out into a. number of pamJJeJ circuits..
Consumenurlt

M fuseK
IIlilll
30A !SA
To2'.20V Earth
mains supply

lire 8.1 Klrohen unit

mains soekot (iJ


(frontview) �

X YZ ligb6"8 setL,,

(a) Indicate on F� 8.1 how tho mains sockat should l>e connoctod to wiresX, Y and Z. (l marl:)

(b) Liglmng sets L1 oad L2 of power ratings 300 W and 450 W respe<tively are oollI!eCtOd in parallel to the
braDch with fuse K.

(i) State one advantege of connecting L, and L, in parnJ]el instead of in series to the branch. (l mark)

(ii) Iffuses nmrked3 A, 5 A, 10 A and 13 A are available, which oueis themostsui:tablem befusoK?
Explain yom choioe. · (3 marks)

(c) The kitohen unit incl•des 1he following electrical appliances:

rating effective time of operation


at .rated value per day
• refiigemor 220V,500W 8hour,;
an electric kettle 220V,2000W OShoi,r
an induction cooker 220V,3000W 2hour,;

Bow much should be peid perdayt<> rua these appliances if I kWh ofe!eotrical eoergy cosu S0.9 ?
(3 mmts)
DSE Physics - Section D : Question Solution PD-EM3-QS/02 DSE Physics - Section D : Question Solution PD-EM3-QS/03
EM3 : Domestic Electricity EM3 : Domestic Electricity

2. (b) No! [!] 4. (a) (ill') E L


The fuse will blow. [!]
[4]
(c) E = 1.32kWx 150h = 198kWh [!]
Cost= 198 x0.9 = $1782 [!]
fuse

3. (a) Any ONE of the reasons below: [2]


* Each lamp can be switched on and off independently
* Even if one lamp is burnt, the other lamps can still operate properly (b) (i) [: !_ [!]
* The voltage across each lamp is 220 V to give out rated power.
V
8 0
Total current= 1000 + 0 8.18 A [2]
i 220 220
(b) (i) Resistanceofeachlightbulb = v [!]
p
(ii) Energyused""' (1.0+0.8)kWx2h = 3.6kWh [!]
= C2Z0) 2
= so1 n [!] Cost = 3.6 x 0.9 = $ 3.24 [!]
(60)
= SO?
Total resistance
5. ( a}
= 2690 [!]

(il') Totalcwrent = 3x� [!]


220
= 0.818A [!]

(c) (i) To prevent the overloading of the wire. [ 2]


OR
< Battery, ammeter, switch, variable resistor and light bulb in series with correct symbols > [3]
To limit the flow of current in the circuit. [2] < Voltmeter in parallel with the light bulb> [!]
(ii) To ensure that the lamps are cut off from live when the switches are apen. < +ve tenninal of voltmeter and ammeter correct > [!]
[2]

(iii) E nergy [2] (b) The variable resistor is used to vacy the current flowing through the bulb. [2]

(d) EitherONE ofthe followings: [2] (c) (i) (1) 0.84 V [!]

* To match with the international standard (2) 24mA [!]


* To increase the power output
(11) R= �I =
4
O.S =3Sn [2]
0.024

(d) (i) Operatingresistance = V = (2Z0}


2 2
4. (a) (i) No ! [!] [!]
P (110)
Sockets A and B are connected in series to the mains. [!]
= 440n [!]
(ii) No! [!]
(ii) When the bulb is working at its rated value, its temperature is much higher, [!]
When both appliances are plugged in, the voltage across each socket is less than 220 V. [!] so resistance increases. [!]
DSE Physics - Section D : Question Solution PD-EM3-QS/04 DSE Physics - Section D : Question Solution PD-EM3-QS /05
EM3 : Domestic Electricity EM3 : Domestic Electricity

6. (a) XisconnectedtoB. 8. (a) (i) E =mct:.T [!]


Y is connected to C. = (l.6)x(4200)x(67-23) = 2960 00J <accept 295680J or 296kJ> [!]
Z is connected to A.
< 1 mark: for one ofthem correct> [!] (it) Power output = != 295680
=- 4930 w <accept 4928 W > [!]
t 60
< 1 mark for the other two correct> [!]
Efficiency Power output
Power input
OJ
(b) (i) To ensure themetal case ofthe kettle is earthed
before the heating element is connected to the live wire. [2] 4928 x
= 100 % = 82.1% <accept 82.2% > [!]
OR 6000
The long pin opens the shutter that blocks the other two apertures (iii) Any ONE ofthe following : [!]
to prevent accidental insertion of metalobjects. [2] * Some energy is lost to the surroundings
(:h) Sis connected in wireX * Some energy is absorbed by the container
because the heating clement will be cut offfrom the live wire Vihell Sis switched off. [2]
(b) (i) It isakilowatt-hourmeter. [!]
OR
IfS is connected i.n w:ire Y, It measures the electrical energy used. [!]
the heating element will still be at live wire even when Sis switched off. [2] (n) Totalpower = 600+2000+1500 = 4100W [!]

(c) (i) E=2kWx0.5h= lkWh [I] == !_ 4100


I == [!]
Cost= $0.9xl = $0.9 V 220
[!]
= 18.6A [!]
(ii) Current drawn from the mains supply
2000
(riI) The water heater draws a large CU1Tent from the mains supply. [!]
2500
[2]
= 220 +220 Jfother electric appliances are connected together, overloadmg may occur. [!]
= 20.S A (iv) Any ONE ofthe following: [2]
Since the current exceeds 15 A, the connection is daugerous and the fuse: will blow. [I] * Ifthe ting circuit is broken at one point, the ring circuit can still :fim.ction.
(d) Ifwire Xtouches the case, a short circuit will be set up between the live wire and the earth, [!] * Current. is divided into two halves viatwo paths, thus thinner cables can be used.
the fuse will blow and the kettle will stop working. [!] * Since current is divided into two halves, the chance of overloading is reduced.
Ifwire Ytouches the metal case, the circuit is still completed, so .the kettle willcontinue to work. [!]

9. (a) The fan and the heating coil are connected in parallel [!]
7. (a) So that they can operate at their rated values. [!]
E L N <For earth wire>

--V'
[!]

<For neutral and live wire > [!] (b) (i) R [!]

= 24 = 2.88 n
1
[!]
200

(ii) J=!.. [!]


V
(b) Energy used = 2.5 kW x 150 h = 375 kWh [!] = 20+200
9.17A [!]
Cost of electricity = 375 x 0.87 = S 326 <accept S 326.25 > [!] 24
DSE Physics - Section D : Question Solution PD-EM3-QS /06 DSE Physics - Section D : Question Solution PD-EM3-QS 107
EM3 : Domestic Electricity EM3 : Domestic Electricity

10. (a) E-mlr+mct:.T 11. (b) (ii) E = Pt [l]


4
= (1)(2.26xl06) + (1)(4200)(100-20) [!] (5.06 X 106) = p (2 ,3 X 60) P = 3470W
= 2 596 000J [!] P = VI [l]
(3470) = {220) I :. I= 15.8 A [l]
(b) Usefulenergyoutput inl s = (1100)(l)x80% = 880J [!]
(c) Thick wires bas smaller resistance, [l]
Maximum. mass of steam generated in 1 s = � = 3.39 x 104 kg [!]
2596000 thus reduce the heating effect of current on the wires. [I]
OR
V'
(o) (i) (1) R [l]
p Thin wires are not used since their resistance is greater [l]
and thus the wires may be over�heated. [l]
(220)
2
44 n
= = [l]
1100

(2) R - (110)' -ll<l [l] 12. (a) Both light bulbs can work underrated voltage. [l]
1100
When one light bulb burns out, the other light bulb can still work. [l]
Cu) (1) 220Vmode 110Vmode
(b) When the switch is o� the light bulb is cut off from high voltage. [I]
Thus no electric shock occurs when the light bulb is touched. [l]

{c) Total cost of filamentlight bulb "' 5 + 4000 x � x 0.95 = $ 157 [l]
1000

Total cost of energy saVlllgbulb "' 35 + 4000 x _S_ x 0.95 "' $ 65.4 [l]
1000
L (Live wire) L (Live wire) Total money saved = 157 - 65.4 = $ 91.6 [l]
.____.,.__ N (Neutral wire) '---...-- N (Neutral wire)
E (Earth wire) B (Earth wire)

< In 220 Vmode, the two resistors are in series> [l] 13. (a) Equivalent resistance: R = � "' 25 n [l]
50+50
<In 110 Vmode, the two resistors are in parallel> [!]
V'
< All the connections are correct> [!] p [l]
R
(2) Resistance of each wire = 22 n [l] = 2202 ""1936W <accept 1940W> [l]
25
(iii) The iron cannot function properly. [l]
When the iron is in 220 Vmode, its resistance is 44 n. (b) [l]
Power output settings Circuit
(110)' = 275 W
Thus the power output = -- Low z
[!]
44 Medium X
This power is much smaller than the rated value. [l] High y

{c) AllfONEofthefollowing: [l]


11. (a) The water can. be heated uniformly by convection. [l] * The hair dryer has a plastic case.
(b) (i) E = mcllT [l] * The case of the hair dryer is an insulator.
(24.1) (4200) (65 -15) = 5.06 X lQ� J [l] * The hair dryer has double insulation.
I
14. (a) p = !!._ = (2526-126)
DSE Physics - Section D : Question Solution
EM3 : Domestic Electticity
PD-EM3
-QS/0
811
DSE Physics - Section D : Question Solution
EM3 : Domestic Electricity
16. (c) ModeZhasthelargestpowerdissipation,
PD-EM3-QS/09

[!J
t 2x60 since the equivalentresistance of mode Z is the smallest [!J
= 20W [!J
(d) (i) ModeZdrawsthelargestco.rrent [!J
(b) ByP= VJ
I= ZZO x2 = 4.SSA VJ
•·• (20) - ( 12) I [!J 96�
:. I= 1.67A [!J OR
I= SOO x2 = 4.55A [!J
{c) Totalcurrent = l.67x2 = 334A [!J 220
As the total current is less than 5 A, the fuse will not blow. [!J The 5 A fuse would be the most suitable one: VJ
(ll) The claim is not correct The switch mustbe installed in the wire A which is the live wire [!J
so that the heater is cut offfromhigh voltage when the switch is off. [!J
15. (a) keeping Wllim
(iil.) Wire C is the Earth wire that could prevent an electric shock [!]
z
(b) By p = v since current would be conducted from the case through this wireto the Eartll. [!J
R
2
:.(88)= (220) [!J
R, 17. (a) (i) To X [!J
Ri = sson [!J
(ii)P=VI ;. (800) - (220) I [!J
(c) Powergivenoutbytheresistor� = 550-88 = 462W . [!J :. I= 3.64A [!J
ByP=VI
i
(iii)ByP= r-2+ v = sv
2

.·. (462) - (220)/, [lJ


R 4R 4R
.·. h=2.1A [!J
(800) = 5 <22W [!J
4R
R = 75.625 .Q
2
16. (a) R = (Z20) = 96.8n [IJ To keep warm:
(500) i
P= v = c22o)2
2 4R 4 (75.625)
Cb) P = v = (220)2 [!J
(96M96.8J = 160W

= 2S0W VJ OR
i i
OR (800) = V2 + v = 5v [!J
R 4R 4R
i
[IJ Pwo:m. = v [!J
4R
Total power = 125 x 2 = 250W [!J = (800)x.!. = 160W
5
OR
(b) (i) electrical energy [!J
(220) - 1.136 A
(96.8+96.8) (ll) (1) onlythefuseblows [!J
p = J'Z R"'l = {l.136)� x (96.8 x 2) = 250 W [!J (2) only the RCCB cuts off the supply [!J
Hong Kong Diploma of Secondary Education Examination DSE Physics-SectionD :M. C. PD -EM4 -M /01
. -··-"
18. (a) Banl!NL EM4 :Magnetic Field

�=-•-
fJ.J
The following list of formulae may be found useful :
JillQQS socket
Z(L}
IA Force onamoving � inan:egwtic.Eeld F ""BQvsinO

\ Force on a dIT� c:cohclor · in a 110&D=tic fieli F "' Bllsin 0


l'(NJ µ,,r
Yagnetic fieli due t a lag sliaiiiit wue, Bo
21,r

-H_.,.. _ _,,,_,,_._ ... }


l
Magnetic field insire a l.a::g s:>laoid µ,, LNI
Bo - -
(b) (i) IA
(jn panillei} <:alt slill op-, i.e. both wwk Any
independently. ONE Use the following data whereve·r·necessary :
- Both cmworlc at the rated power.
Perireability of fne � 1-4, = 4nx10-7Hm· 1
- &ry "'8S<lllable answer
l (barge of elec!:rcn e "' 100 X 10-19 C
Electr:m re;t mass mo "" 9.llX 10-31 kg
(u) P=IY Acceleraticn due "b g!Mty g " 9. 81 m s£ (close "b ire F.arth) ,
f .. !1.+!.. .. �+ 450
(300+450) = 1(220) V Y 220 220 lM
/=3A-0909lA•3.41 A lA
Part A :HKCE examination questions
Thus s A fllsc soould i;., used. lA
:
3' l < HKCE 1981 Paper ll-33 >
Which of tie foll.ariig will be <Eflectei by a imgnetic fieli ?
(c) BlecUicaleneigy llSed perdsy 0) Electrai:egretic waves
•O.SOOkW x 8h+2kWx 0.5 h+3 kWx 2h lM 00 A beam of elec!nrs
=llkWh @ A beam of pro1:lm
Cos1=$0.9/kWh>< II kWh lM A. Cl) cnly
:$9.9 IA • O)&@a,Jy
C Iii)& (3)a,Jy
3 D. CU 12) & (3)
2 Astronomy - -Clct�
a O:bi:tal m:rticns utder grs;ity (ti"FWJ.fkif�) 2 < BKCE 1981 Paper ll -32 >
. • -""'11e"""'""c�-.;)
Elective 2-A'tomic World (4�-JJ.)
l Ruthrlortf s atcmic ml (Jl��) A
2 llD1relofuc """(�'ll>tll;)
a Boor's atcmic model of� (3E.il{IIJU-'fU)
4 Rrliclfs er 18,lS Otr.�ti.)
5 Probi»< "'"''"" - (-:JI.)
Elective 3 - Energy and Use of Energy (g'l,:;f,,�aa!J{Ul)
I Tuctriciv atb=('.Uffljl;)
2 ,._ - ;. bnldm< -•) The fipe sb:Jws two iim :rai5 A andB � by two• ligt slri:rg; S'.l 1tet ttrey· ::re clae � Th::ir lam- mis a-e
3 ,,,__ ;.-(-'!'-) m:.uE aS':lle:cid When ammnt flOIIS � 1te S'.llemid what will happen bA and.B?
4 Non---:renewable eoergy a:ims (:;:i::,;J"MU) A B
5 Renewable energy mms ('iJ"�) A. moves bh left moves biterigd:
Ele:::tive 4- Medical Pbysics (ll�Jlt,f:) B. moves bh rigd: moves b 1m left
1 Making i:mre of 'Ire � -(lllW»&.T) C moves bhri# moves b ire rigJt.
2 Making sense of 1ie e:- (�) D. moves b 'Ile kft moves b 1te kft
a Jemca1 ineg-uig�n:n-i� :rmiatin($��)
4 Hedical � us� kiriziig ra:1iati.cn ( ��)
PD-EM4-M/02 PD-EM4-M/03
1
DSE Physics - Section D : M.C. DSE Physics - Section D : M.C.
EM4 : Magnetic Field EM4 : Magnetic Field

3. <HKCE 1984 Paper ll- 28 > 7. <BKCE 1991 PapcrJI.32>


P and Qare two long parallel straight wires e&rymg currents as shown.
What is the direction of the force on Q?
A. to the left
B. to the right
C. out of this page
D. into this page

4. < HKCE 1986 Paper Il - 33 > B D


Two long parallel wires AB and CD are connected to batteries as shown in the figure. The force acting on CD is
X A in a di:reetion to the left.
d.c. sOlllCe B. in a direction to the right
y C. in a direction out of the paper.
D. in a direction into the paper.

Two close, long, parallel, straight metal wires X and Yform part of the circuit shown above. Xand Y
A. attract each other. 8. <HKCE 1992 Paper II· 33 >
B. repel each other. A srnal.1 compass is placed near a long current carrying wire. In which of the folloWlllg diagrams is/are the compass needle
C. first repel and then attract each other. pointing in the correct: direction ?

--e-----
D. first attract and then repel each other. (I) Compass above the wire

5. < HKCE 1988 Paper II - 32 >


(2) Compass below the wire
' ''
Pt------iQ

'' (3) Current flowing out of paper


___ .!R

The figure shows the path of an electron in a magnetic fieldPQRS. What should be the direction ofthe magnetic :field? A. (1) only
A. PQ B. (2) only
B. QR c. (1)&(3)only
C. into the paper D. (2) & (3) only
D. out ofthe paper

9. <HKCE 1993 PaperJI.3S>


6. < HKCE 1990 Paper II - 31 >
A motor lifts a load of40 N verticallyupwards at a steady speed of0.2 m s·1• The voltage applied to the motor is 12 V and
the current drawn is 2 A Find the efficiency ofthe motor.
A C A 12%
X X X X X X B. 16.7%
X X X X X X C. 33.3%
B D
D. 66.7%
X X X X X X
X X X X X X
10. < HKCE 1994 Paper JI· 33 >
The diagram shows three long straight wires .X, Y and z. Xand Y carry
currents flowing into the paper mrile Z carries a current floWlllg out
� of the paper. The ounents are all equal in magnitude. What is the
In the figure shov.n above, a copper rod is suspended horizontally by two insulating threads AB and CD, and the direction of direction of the resultant force actmg onX? z0
the magnetic field is into the paper. What happens when the switch of the circuit is closed? A. p
A. The copper rod will move into the paper.
B. The copper rod will move out ofthe paper. B. Q
C. The tension in each thread is decreased. C. R
D. The tension in each thread is increased. D. S
DSE Physics - Section D : M.C. PD-EM4-M/04 DSE Physics - Section D : M.C. PD-EM4-M/05
EM4 : Magnetic Field EM4 : Magnetic Field
11. < HKCE 1995 Paper II - 31 > 15, < HKCE 1998 Paper II -32 >
Which of the following involve( s) the application of electromagnets ? Upwmds

t--+-
(1) Anelectricbell
(2) A telephone receiver
0 0 ®
i
(3) A moving-coil loudspeaker Left +- rught
A. (3) o,Jy X y z
B. (!) & (2) o,ly DownWMds
c. (2) & (3) only
D. (!), (2) & (3)
The above diagram shows the cross-section of three parallel strai ght 'Wires x; Y and Z. X and Y car:ry currents flowing out of
the p aper and Z carries a current flowing into the paper. What is the <lirection of the resultant force acting on Y?
12. <HKCE1995Paperll-34> A. towards the left
B. towards the right
C. upwards
soft iron core
D. dovmwards

lei. < HKCE 2001 Paper II- 36 >

soft iron core


The above diagram shows a simple electromagnet. Which of the followin g can increase the strength of the electroma gnet ?
(1) Decreasing the resistance of the variable resistor.
(2) Replacing the soft iron core with one made of steel
(3) Replacin g the battery with a 50 Hz a.c. source. 12V
A. (1 ) only II
B. (3) only
C. (1 ) & (2) only
D. (2) & (3) onlr
1----G
The figure shows an electromagnet which is used
the electromagnet ?
to pick up iron ob jects. Which of the following can increase the strength of

±
(1) increasing the number ohurns of the coil
13. < HKCE 1996 Paper II - 35 > (2) reducing the resistance of the variable resistord--l
(3) replacing the battery with a 12 V a.c. power supply
A. (1) only
B. (3) only
C. (1 ) & (2 ) only
D. (2) & (3) only

Two long insulated wires canying equal CUIIents are placed perpendicular to each other on a table as shown in the figure.
The points P, Q, R and S are all of equal distances from the wires. At which point(s) is the resulting magnetic field pointing 17. <HKCE2002Paperll-32>
out of the paper ?
A. Ponly
B. Qonly
C. Ronly
D. Sonly

14. <HKCE1998Paperll-31> P, Q and R are three parallel straight wires carrying equal cw:rents flowing out of the paper. R is equidistant from P and Q.
!OQ
What is the direction ofthe resultant force acting on R by P and Q ?
A.
12V M

.....
B

A motor is connected in series with a 10 n resistor and a 12 V power supply as shown. Jfthe current in the circuit is 0.5 A,
t
C.
find the power consumed by the motor.
A. 3.5W
B. SW
C: 6W D.
D. 7W
PD-EM4-M/06 PD-EM4-M/07
1
DSE Physics - Section D : M.C. DSE Physics - Section D : M.C.
EM4 : Magnetic Field EM4 : Magnetic Field

18. < HKCE 2003 Paper II- 37 > 21. < HKCE 2005 Paper Il- 22 >
Qg

Which of the following diagrams shows the magnetic field pattern formed around a flat circular current-canying coil, in the
planeas shown above ?
Two compasses P and Qare placed near thepoles of an electromagnet as shown above. In which of the following diagrams A. B.
are the northpoles of the compass needles pointing in the correct directions ?
p Q
A.
CD CD
B
.
CD CD C. D.

C.
CD CD
D.
CD CD
19. < HKCE 2004 Paper II- 30 > 22. < HKCE 2005 Paper Il- 23 >
David wants to design a battery-powered toy car. Which ofthe following circuits should he use? The figure shows a simple motor. Which of the
A. B. following cha nges can increase the turning effect of
the coil?
(I) usingastrongermagnet
(2) reducing the resistance oftbe rheostat
(3) using a coil with a smaller number oftums
C. D. A. (1) & (2) only
. (1) & (3) only
B
C. (2) & (3) only
D. (1), (2) & (3)

23. < HKCE 2006 Paper II. 36 >


20. < HKCE 2004 Paper Il- 34 > The diagram shows a home.made device which can
produce sound. In the device, a magnet is fixed to the signal input
base of a plastic cup. When a signal passes through the
coil, the base vibrates to produce a sound. Which of the magnet fixed
following methods can make the sound louder? on the base
(1) using a stronger magnet b"'
(2) inserting a copper rod into the coil plastic cup
Two parallel straight wires P and Q carry equal currents flowing into the paper. A compass is placed at a point R where
PR = QR. In which of the following diagrams is the north pole of the compass needle pointing in the corr� direction ? (3) increasing the number oftums in the coil
Tb,e effect ofthe Earth's magnetic field may be ignored. A. (1) & (2) only
A.
e B
.
8
C.
CD
D.
CD
B. (1) & (3) only
C. (2) & (3) only
D. (l), (2) & (3)
DSE Physics - Section D : MC. PD-EM4-M/08 . DSE Physics - Section D : M.C. PD-EM4-M/09
EM4 : Magnetic Field EM4 : Magnetic Field

24. < HKCE 2008 Paper ll -19 > 27. <HKCE20091'aperII-22>


I A current carrying wire, peipendicular to the plane of the paper, is located at P. P is in the midway between two identical bar
magnets with unknown polarities X and Y as shown in the figure. The current in the wire is flowing into the paper. The
magnetic force F acting on the wire is downward. Which of the following statements are correct ?
I
(1) Xis a north pole and Yis a south pole.

1
Two parallel wires carry equal currents flowing in the same direction. Which of the following statements are correct ? p
(2) Fis reversed if the current direction is reversed.
(1) The magnetic forces acting on the two wires form an action-reaction pair. (3) Fis larger if stronger bar magnets are used. y
(2) The two wires attract each other. A. (l} &(2) only
(3) If the directions of the current in the two wires are both reversed, the wires will repel each other. B. (1) & (3) only
A. (1) & (2) only C. (2} &(3) only F
B. (1) & (3) only D. (1), (2) & (3)
c. (2) & (3) oruy
D. (1),(2)&(3) 28. < HKCE 2009 Paper II- 21 >

25. < HKCE 2008 Paper n- 42 >


p

The figure above shows two parallel straight wires carrying equal cunents. Which of the following diagrams correctly shows
--' --+➔- uniform magnetic field the resultant magnetic field lines?
-+ l---
A. B.

''
Q
� observer's eye
C. D.
The above figure shows a rectangular loop of wire carrying a steady current I. The rectangular loop can rotate freely about
PQ. Ifa uniform magnetic field to the right is applied, which of the following is correct?
Resultant magnetic force acting Rotation of the rectangular
on the rectangular loop loop about PQ
A. clockwise
B. anti-clockwise
C. clockwise 29. <HKCE2010Paperll-22>
D. anti-clockwise Q

26. < HKCE 2008 Paper II- 43 > F


A compass is placed inside an air-cored solenoid XY which is connected to compass
a d.c. supply with terminals P, Q as shown.. If the endX of the solenoid y
behaves as magnetic north pole, what are the polarity of the tenninal P and

--J
the direction in which the north pole of the compass needle points?
p
X �y

L
(The tip ofthe arrow represents the north pole of the compass needle.)
A ClllTCnt carrying wire Y is placed between two bar magnets as shown above. The wire experiences an upward force. Which
Polarity of P Compass of the following is/are the posSible combination(s) of the direction of the current in Y and the magnetic poles ofX and Z?

A. + e pole X current direction in Y pole Z

B. + e p Q (1)
(2)
N
S
fromPto Q
fromPto Q
S
N

e (3) N fromQtoP S
C. A (1) only

e
B. (2) only
D. C. (1) & (3) only
D. (2) & (3) onl y
DSE Physics - Section D : M.C. PD-EM4-M/10 DSE Physics - Section D : M.C. PD-EM4-M/11
EM4 : Magnetic Field EM4 : Magnetic Field
30. < HKCE 2011 Paper n - 21 > 33. < HKAL 1980 Paper I- 46 >
Rectangular coil PQRS is carrying a current flowing in clockwise direction. It is placed inside a uniform magnetic field Which of the following affects the magnetic field strength on the axis of a long solenoid?
pointing into the paper as shown in the figure below. (1) The diameter ofthe solenoid
x x x x x x (2) The number of turns per unit length of the solenoid
Q�--R (3) The current flowing through the solenoid
X X X X X X A. (!) only
X X B. (3) only
C. (!) &(2) only
X X X X
magnetic field direction of cUtrent
x x x x x x D. (2)&(3)only
X X X X X X
P�--�s 34. < HKAL 1981 Paper I - 44 >
X X X X X
A free electron travelling horizontally with speed v enters a uniform vertical magnetic field B. Which of the following
X
Which of the following statements are correct? statements is/are correct?
(1) A magnetic force pointing to the right acts on RS. (1) The path of the electron is circular on a vertical plane.
(2) No magnetic force acts on QR. (2) The speed oftbe electron remains constant.
(3) The resultant magnetic force acting on the coil is zero. (3) The radius ofcurvature ofthe path ofthe electron is inversely proportional to the magnetic field B.
A. (1) & (2) only A. (1)only
B. (!) & (3) only B. (3) only
C. (2) & (3) only C. (1) & (2)only
D. (1),(2)&(3) D. (2) & (3) only

31. < BKCE 2011 Paper n - 41 > 35. < HKAL 1981 Paper 1- 13 >

In the figure below, the arrow shows the direction of the magnetic field at Z due to a i:.mrrent--canying solenoid. A vertical wire of 0.4 m Jong carries a constant current of 5 A. It is placed in a magnetic field of strength I o-3 T, which dips
at an angle of30° to the horizontal. Detennine the magnetic force acting on the wire.
z---. A. 5.0 x 104 N
B. 8.7 x 104 N
C. I.5x10-3 N
D. I.7xl0-3 N
X
rrrrrrrTTl,�ftrronoo,o
36. < HKAL 1982 Paper I - 43 >

A solenoid with a solid core has a diameter d and n turns per length. It carries a current!. The magnetic :fieldB inside is
The direction of current through the solenoid and the magnetic north pole ofth.e solenoids are (1) independent ofd.
Direction of current Magnetic north pole (2) proportional to n.
(3) independent of the material ofthe core.
A. fromPto Q X A. (I) only
B. fromPtoQ y B. (3) only
C. fromQtoP X C. (1) & (2) only
D. fromQtoP y D. (2) & (3) only

37. <HKAL 1982 Paper I- 19 >

Part B : HKAL examination questions

32. < HKAL 1980 Paper I- 44 >


Two charged particles and Y enter a region where a magnetic field acts perpendicular R S
X
to the plane of their motion. The resulting paths shown in the diagram may be affected X and Y are identical flexillle conducting wires, suspended from fixed points Pand Q. The bottom parts of the wires R and S

n
by the mass, charge and initial speed of the particles. Which of the following quantities are also fixed. When a current 2 A is passed from R to P through X, and a current 1 A is passed from Q to S through Y,
alone could cause the observed difference in the paths ? which of the following diagrams best represents the shapes of the two wires?
(1) Xhas a smaller mass than Y. A. B. C.

1t
D.
(2) X1"'sa small"' ,iwg, than l'. �
Y X
(3) Xhas a greater speed than Y.
A. (!) only
B. (3) only
C. (!) & (2) only
D. (2) & (3) only
DSE Physics - Section D : M.C. PD-EM4-M/ 12 DSE Physics - Section D : M.C. PD-EM4-M/13
EM4 : Magnetic Field EM4 : Magnetic Field

38. <BKAL1983Paperl-18> 42. < BKAL 1988 Paper I - 38 >


Two parallel wires attract each other with a force Fwhen the same current passes through them. Ifthe current is doubled and
the distance between the wires is also doubled, the force ofattraction will become
A. ¼F.
B. ½F­
lo R-S Lx
t I,
e. F. Q
D. 2F. In the above figure. PQ is a fixed long wire carrying a current It . RS is another wire perpendicularto PQ. When a current l2
flows through RS in the direction shown, the magnetic force on the wire RS
A. acts in the +y direction.
39. < HKAL 198S Paper 1- 25 >
B. acts in the-y direction.
Which of the following graphs best represents the variation of the strength of the magnetic field B along the axis of a long C. acts in the + x direction.
solenoid carrying a constant current, with the distancex from the centre of the solenoid along the axis to one of its end?
D. acts in the -x direction.
A a
B

.t=:::
43. <llKAL 1988 Paperl-37>
A particle ofmass m and charge q moves in a circular orbit inside a magnetic field B. The time taken for a single orbit is
I X I X A .!!.!L. B. 2•m
2sm Bq

J
C. D.
B 2mq Bm

.t�
c _ D.
2 •q

I X \ I X
44. < HKAL 1989 Paper I -32 >
p
.q
I
40. <BKAL 1985 Paper I- 27>
A small particle with mass 32 x 10-26 kg and charge -1.6 x 10-19 C
enters a unifunn magnetic field of flux density 0.08 Tat a speed of
10 5 m s· 1, as shown in the above figure. It will
-x
X X

X
X

X B into paper B
-0 6--
A pass undeviated through the magnetic field.
B. be deflected upward in a circular arc of radius 0.25 m. X X X
A square loop canying a current! is placed in a unifonn magnetic field B in the xy plane as shown in the figure. If the loop is
C. be deflected upward in a circular arc of.radius 0.50 m. free to rotate, the magnetic forces acting on the loop will cause it to
A. rotate about the y-axis as indicated byp.
D. be deflecteddownward in a circular arc of radius 0.25 m. B. rotate about the y-axis as indicated by q.
C. rotate about the x-axis as indicated by r.
D. rotate about the x-axis as indicated bys.
41. < IIKAL 1987 Paper I-34 >

45. < RKAL 1990 Paper I- 45 >

c
As shown in the figure, Sis an a.c. supply of frequency 50 Hz connected to a resistor R via two long, parallel, straight metal A beam of parti les, of different masses, charges, polarities and speeds, travels long PQ and passes through a narrow slit S1.
wires X and Y. The magnetic forces acting onX and Y In the region between S1 and S1, an electric field E and a magnetic field B are directed perpendicularly to each other. The
A. are always equal to zero. E�eld acts vertically upward and theB-field acts out of the plane of the paper. The particles that are undeflected and emerge
from slit S2 must have the same
B. always attract A. polarity.
C. 'always repel. B. speed.
C. charge.
D. sometimes attract and sometimes repel; the frequency of variation is 50 Hz.
D. mass.
46.
DSE Physics - Section D : MC.
EM4 : Magnetic Field
< BKAL 1992 Paper I - 36 >
PD-EM4-M/14

Two parallel straight wires separated by a distance r carry currents in the same direction. Which of the following statements
I
50. <BKAL1994PaperilA-32>
DSE Physics - Section D : M.C.
EM4 : Magnetic Field
PD-EM4-M/15
11

path of X X X X X
is/are correct ?
(1) The two wires attract each other, the electron

::(J:
(2) The force acting on each wire is inversely proportional to r 2• X
(3) The current in two wires produce a magnetic field with maximum flux density midway between them. uniform magnetic field
A. {I) only field
B. (3) only X
C. (I) & (2) only
D. (2) & (3) only X X X X X
An electron moves in a circular path of diameter 0.01 m in a plane with a uniform magnetic field of 0.02 T directed
47. < HKAL 1993 Paper I - 49> perpendicular into the plane as shown in the figure. Find the speed and the direction of circular motion of the electron.
X X X X
A. 1.76 x 107 m s-1 in anticlockwise direction

(J):Q
X X X X B. 1.76 x 107 m s·1 in clockwise direction
C. 3.52 x 10 7 m S-1 in anticlockwise direction
X [sX X X X
D. 3.52 x 107 m s· 1 in clockwise direction
X X X X X

Two particles P and Q of the saIIle charge and mass but moving vr.ith different speeds Vp and VQ respectively enter a region 51. <HKAL 1995Paper IlA- 34 >
of uniform. magnetic field B directed into the plane of the paper. The subsequent circular paths are as shown in the figure . B
'Which of the following sta±eoieu.ts is/are correct?
(1) Both particles P and Qare positively ch arged. y
(2) Speed ofparticleP is smaller than that of Q. ''

. .:
(3) The period of circular motion ofPis shorter than that of Q. '
A. (I) only
B. (3) only
C. (I) & (2) only
. ----------'
\ Uniform magnetic field
•T (outofpaper)

D. (2) & �) only


Particles A and B moving at the same speed enter a square region of unifonn magnetic field as shown. Particle A leaves at X
whileparticleB leaves at Y. Iftbe charge to mass ratio ofparticleA isk, then the charge to mass ratio of particle B would be
48. < BKAL 1993 Paper I - 36 > A. k
IA 2
k
i
0.2m
B.
4
C. 2k
D. 4k
IA
Two long parailel straight wires, each cam.es a current of 1 A in opposite directions, are separated by a distance of 0.2 m as
52. <HKAL1996PaperilA-26>
�hown in the figure. The magnetic field at a pointXmid-way between the two wires is
A. 2 x lo--6 Tout of paper.
B. 2 x 10 -' Tinto paper.
C. 4 x 1Q---6 T out of paper.
D. 4 x 10➔ Tinto paper.

49. < HKAL 1994 Paper JIA- 31 >


For two long, straight parallel conducting wires canying the same current, the magnitude of the magnetic force acting on a
section of the wires would be affected by Four infinitely long straight parallel wires P, Q, R, S carrying equal currents are situated at the four comers of a square as
(I) the distance between the wires shown. The currents inP, Qare into paper and those inR, Sare out of paper. 'What is the direction of the resultant magnetic
(2) the length of that section of the wires :field at the centre of the square ?
(3) the directions of cwrent flowing in the wires
A. I
A. (I) only
B. {3) only B. IT
C. (1) & (2) only C. ffi
D. (2) & (3) only D. N
. DSE Physics - Section D : M.C. PD-EM4-M/16 DSE Physics - Section D : M.C. PD-EM4-M/17
EM4 : Magnetic Field EM4 : Magnetic Field
53. < BKAL 1998 Paper IlA- 30 > 56. < HKAL 2000 Paper IlA - 25 >

N s

Four parallel long straight wires carrying cummts of equal magnitude pass vertically through the four comers of a square
The above diagram shows a rectangular current--canying coil ABCD in a uniform magnetic field between two pole pieces. PQRS. In one wire, the current is directed into paper. In the other three wires, the currents are directed out of paper.
The magnetic field is perpendiculartci the plane of the coil. Which ofthe following statements is/are correct? Which of the following can produce a resultant magnetic field with the indicated direction at the centre O 1
(1) There is a magnetic force acting on the side BC of the coil. Current into paper Cw-rent out of paper
(2) The magnetic forces acting on the coil tend to reduce its area.
(3) There is a resultant force acting on the coil. A. p Q,R,S
B. Q P,R,S
A. (1) only C. R P,Q,S
B. (3) only
C. (1) & (2) only
D. s P,Q,R
D. (2) & (3) only
57. < HKAL 2000 Paper IlA - 27 >
54. < HKAL 1998 Paper IlA - 26 > A beam of charged particles passes through a region of crossed unifonn electric and magnetic fields without deflection.
X X X Which of the following quantities must be the same for the particles making up this beam ?
A. charge to mass ratio
B. velocity
C. mass
D. sign of charge
X X X X

The above figure shows a charged particle moving in a circular orbit with a constant speed v on a plane perpendicular to a 58. < BKAL 2002 Paper IIA- 29 >
unifom1 magnetic field directed into the paper. Which of the following graphs represents the relation between the time Tfor
the particle to complete a circle and its speed v ?
A. B. c. D.
T T T

,t=_,
o�,

55. < HKAL 1998 Paper IlA- 27 >


o�,

Two long, straight parallel wires, each carrying a current I into paper, are
separated by a distance r as shown in the figure. What is the magnitude and
direction of the resultant magnetic field at the point P at the same distance r
from both wires ?
o�,

59.
A.
-
Three long straight parallel wires P, Q and R carrying currents of the same magnitude are situated at the vertices of an
equilateral triangle as shown. The currents in wires P and R are directed out of the paper. Which of the following indicates
the direction of the resultant magnetic force acting on the wire P 1

< BKAL 2003 Paper IIA- 31 >


B. C.
t D.

A charged particle enters a region ofuniform magnetic field whose direction


A. µ .I to the left is normal to the initial velocity of the particle. The subsequent path of the i-�1
, ',
':
2n, particle is as shown in the figure. Which ofthe following may be the reason
to accollllt for this shape of the path 1
''' ''
B. ✓3µ.J to the left (1) The magnitude ofthe magnetic field decreases gradually. '''
'' ''
'''
2n,
(2) The particle loses its charge gradually.
c. µ,[ to the left (3) The particle loses its kinetic energy gradually. ' ----------------' ''
"' A. (1) only Magnetic field
B. (3) only normal to paper
D. ✓3µ.I to the right C. (1) & (2) on ly
2K, D. (2) & (3) only
PD-EM4-M/18 PD-EM4-M/19
1
DSE Physics - Section D : M.C. DSE Physics - Section D : M.C.
EM4 : Magnetic Field EM4 : Magnetic Field

60. < HKAL 2003 Paper IlA- 30 > 63. <HKAL 2006PaperIIA-17 >
In a vacuum, an electron moves in a circle vvith speed v in a uniform magnetic field of flux density 1 mT. If an ct..y.micle
¼
with speed v is to follow the same path, what magnetic flux density in the opposite direction is required ?
(Given: mass ratio ofan C1'-particle to an electron is 7200 : 1; charge ratio ofan ct-particle to an electron is 2 : L)
20cm 10cm p A. 0.9T
B. I.ST
C. 3.6T
B! D! D. 7.2T
In the above figure, AB and CD are two parallel long wires with separation 20 cm carrying currents 11 and h respectively.
The resul:tantmagnetic field at the pointP 10 cm from wire CD is zero. Ifhis equal to 0.6 A, determine the magnitude and
direction of the current Ji in the wire AB. 64. < HKAL 2006 Paper IIA-18 >

A 0.2 A flows in the same direction as I,. A long straight wire carrying a Ctll'rent J is placed at a distance r from the point P.
Both the wire andpointP are in the plane of the paper. When the current I increases long str,ight-
B. 0.2 A flows in the opposite direction ash by 0.5 A, the magnetic flux density Bat pointP increases by 5.0 x I� T. Find r.
C. 1.8 A flows in the same direction as h I p

,_•
A !cm f
D. 1.8 A flows in the opposite direction ash B. 2cm
C. 4cm
D. 8cm
61. < BKAL 2004 Paper IlA - 29 >
65. < BKAL 2006 Paper IIA-16 >

f;
Three long, parallel, straight current-carrying wires P, Q and Rare placed in the same plane as shown in the figure.
IA
► p

::: Q

R
For two long, parallel, straight Wlles placed a distanced apart and each carrying a current of I A, the magnetic force per unit
Two long, parallel wires PQ and.RS are connected to a sinusoidal a.c. supply as shown in the figure. Which of the following
graph bestshows the time variation of the magnetic force Fbetween the two wires?
A
. F B. F
length is F. What is the resultant magnetic force per unit length acting on the wire R shown in the above figure ?
A. 0
B. F
C. 2F
D. 3F

.�------+ o '-------->
62. < BKAL 2005 Paper IIA -18 >
a-particle's path
c. F
D.

Magnetic field
out of paper

proton's path
A proton and an ex-particle move in a uniform magnetic field as shown in the above figure. The magnetic field is directed 66". <HKAL2007PaperIIA-20>
out of the plane of the paper. Within a square region WXl'Z, the proton takes time ti to complete a half circle fromXto Y A beam of charged particles passes through crossed uniform electric and magnetic fields vvithout deflection. If the electric
while the a-particle follows a quarter circle fromXto Zin time tz. What is the ratio t1 : t2 ? field is removed, the particles will splitup into several beams. This splitting may be due to the particles having different
(Given: mass ratio ofan o:-particle to a proton is 4: 1; charge ratio ofan ct-particle to a proton is 2 : L) (1) charges
(2) masses
A. 1 :2 (3) incident velocities
B. 1: 1 A. (1) only
B. (3) only
C. 2: I C. (1) & (2) only
D. It cannot be determined as the ratio ofthcir speeds is not given. D. (2) & (3) only
DSE Physics - Section D : M.C. PD-EM4-M/20 DSE Physics - Section D : M.C. PD-EM4-M/21
EM4 : Magnetic Field EM4 : Magnetic Field
67. <HKAL 2007 Paper IIA- 18 >
Q <p----------� R 70. <HKAL 2009 Paper IlA- 30 >

'' ''
'' '''
''' ''
'' ''
P(;)-----------©s
Four long straight wires perpendicular to the plane ofthe paper are placed at the four comers of a square PQRS as shown in
the figure. Same current I flows in the wires at P, Q and S directed into the paper while the =ent fl.owing along the wire at
R is in the opposite direction. Ifthe wire atP experiences no net magnetic force, find the current flowing in the wire atR.
A. Il.fi
B. 1/2 A square coilPQRS, each side has a length of0.15 mis placed in a uniform magnetic :fieldof0.4 T as shown in the figure.
The number of turns in the coil is 20 and the current in the coil is 1.2 A. The magnetic field isparallel to the plane of the
C. .fiI coil. The side PQ makes an angle of 30 ° with the magnetic field. Find the magnetic force acting on the side PQ of the coil.
D. 2I A. 0.7Noutoftbepaper
B. 0.7Nintothepaper
68. <HKAL 2007 Paper IlA - 19 > C. 1.3Noutofthepaper
A segment of a vertical wire 0.50 m long carrying a current of 8.0 A is horizontal
D. 1.3 N into the paper
placed in the Earth's magnetic field. The direction of the field dips at an
angle of20 ° to the horizontal. If the magnetic force acting on the wire is �
7.5 x 10-5 N, find the magnitude ofthe Earth's magnetic field.
"""''·
71. < BKAL 2010 Paper IIA- 28 >
A. 6.4Xl0➔T � magnetic
� When moving charged particles enter a muform magnetic field at right angle, they are deflected. This deflection can be
B. 1.8 x 10-5 T
increased by
c. 2.0 x 10-sT ------._ field
D. 5.5 x 10-S T (1) increasing the mass m of the particles

.,.
(2) increasing the charge Q oftheparticles
<HKAL 2009 Paper IIA- 29 > (3) increasing the magnitude B of the magnetic field
solenoidX

{ __________,;. ·--------- - ------+


A. (1) only
solenoid Y
B. (3) only
common axis---------- E
C. (1) & (2) only
D. (2) & (3) only
xQ
72. <HKAL 2011 PaperllA- 25>
The data about two coaxial long solenoids Xand Yare tabulated below : X y

jj
solenoidX solenoid Y
radius 5cm 3""
tum density 1200 m·l 240om·1
cnrrent LOA 0.S A
(cloclcwise as viewed fromE) (anticlockwise as viewed fromE)

Point P is on the common axis while point Q is 4 cm from the axis. Both P and Q are well inside the two solenoids.
Which of the following statements is/are correct? 'Iwo Jong straight parallel wires, X and Y, carry equal cun:ents in the same direction as shown in the figure:. W= X
(1) The resultant magnetic :field at P is zero. experiences a magnetic force of 0.1 N. Ji now a unifonn magnetic field pointing into the paper is applied to both wires
(NOT shown in figure), the resultant magnetic force acting on wire X becomes 0.5 N. Find the resultant magnetic force
(2) The magnetic field at Q is 1.5 mT. acting on the wire Y. (Neglect the Earth's magnetic field.)
(3) The magnetic field at Q points to the left. A. 0.3Nm-1 to the left
A. {1) only B. 0.3Nm-1totheright
B. (1) & (2) only C. 0.6 Nm-1 to the left
C. (2) & (3) only
D. (1), (2) & (3) D. 0.6Nm-1 to the right
PD-EM4-M/22 PD-EM4-M/23
1
DSE Physics - Section D : MC. DSE Physics - Section D : M.C.
EM4 : Magnetic Field EM4 : Magnetic Field
73. < HKAL 2011 Paper llA- 30>
• • • • • • • • • • Part C : HKDSE examination questions

• • • ·---4-�· • • • • 76. <HKDSESamplePaperIA-29>

-----�···✓-
• • .,x. • ;'!il... • • • '
'
0 ,
/' '
A positively charged particle enters a unifonn magnetic :field B with a speed v making an angle 0 (0 ° < 0 :,:; 9CP) with the
boundazy of the field. The magnetic field points out of the paper and the particle leaves the field at the same boundary as
shown. The time of transit for the particle inside the magnetic field is
0) proportional to the angle 0.
(2) depettdent on the speed v.
(3) inversely proportional to the field strength B. Which diagram shows the magnetic field pattern fonned around a flat circular clllTent--carrying coil, in the plane shown ?
A. (!) & (2) only A. R
B. (!) & (3) only
C. (2) & (3) only
D. (!), (2) & (3)

74. < HKAL 2012 Paper 11A - 23 >


C. D.

I,

Two long straight parallel wires carrying currents Ii andh (with h > Ii) is shown in the figure. The separation between the 77. <HKDSESamplePaperIA-28>
two wires is r. If now another wire canying a current I in the same direction is placed midway between the two wires, what
would be the magnetic force per unit length experienced by this wire, in both magnitude and direction?

A. µJ(l2-fi) totherigbt
X,

B. µJ(I2-fi) totheleft
X,

C. A l(fi +Iz) to the right


X,

D. µ I (Ji + 12) to the left


0

X,

75. < BKAL 2013 Paper IIA - 28>


The figw-e shows a simple motor. Which of the following changes would increase the ti.traitlg effect of the coil?
The magnitude ofthemagneti.c fieldB inside a very long solenoid can be increased by
(1) usingastrongermagnet
(1) in.creasingthe CUIIent through the solenoid
(2) increasing the number of turns per unit length of the solenoid (.2) reducing the resistance ofthe rheostat
(3) decreasing the cross.sectional area of the solenoid (3) using a ci:,il with a smaller number of turns
A. (!) & (2) only A. (!) & (2) only
B. (!) & (3) only B. (1) & (3) only
C. (2) & (3) only C. (2) & (3) only
D. (!), (2) & (3) D. (!), (2) & (3)
DSE Physics - Section D : M.C. PD-EM4-M/24 DSE Physics - Section D : M.C. PD-EM4-M/25
EM4 : Magnetic Field EM4 : Magnetic Field
78. < BKDSE Practice Paper IA - 31 > 81. < HKDSE 2013Paper IA- 27 >
In the figure, four long straight wires P, Q, R and Sin the same plane
carry equal currents in the directions shown. The wires are insulated P---8-·
from each other. 0 is a point on the same plane and is equidistant from

--��=-----
each wire. Removing which wire would increase the magnetic field
SmTunifonn
► magnetic:field A. _,
strength at a ?

R VlireQ Q--�-
' ''
C. Vlll'eR '' '
R S
D. 'Wire S
The figure shows a current of 1.0 A flowing in a metal rod oflength 0.5 m. The rod is placed inside a region with a unifonn
magnetic field of strength 5 mT. What is the direction and the magnitude ofmagnetic force acting on the rod ? 82. < HKDSE 2013 Paper IA- 26 >
Direction Magnitude
A. into the paper 1.25x10- 3N
B. out of the paper 1.25 x 10- 3 N
C. into the paper 2.17x10-3N
D. out ofthe paper 2.17xlQ-3N

79. < BKDSE 2012 Paper IA- 29 >

A straight wire carrying cuuent I pointing into the paper is placed in a magnetic :field between pole pieces X and Y. The
figure shows the resultant field line pattern.. What is the polarity of pole piece X and in what direction is the magnetic force
acting on the wire? Ignore the effect ofthe Earth's magnetic field.

In the above figure shown. X, Y and Z are three long straight parallel wires with Z placed midway between X and Y. X and polarity ofX direction of magnetic force
Z carry currents of 1 A in the same direction while Ycarries a CUtTent of3 A in the opposite direction. The magnetic force A. N to right
per l.lllit length experienced by wire X due to 'Wire Z is of magnitude F. The magnetic force per unit umgth acting on wire Z B. N to left
due to both X and r is C. s to right
A. 2F to the right. D. S to left
B. 2Fto the left.
C. 4Fto the right. 83. < HKDSE 2013Paper IA� 28 >
D. 4Ftothe left. magnetic
fiold

80. <HKDSE2012PaperIA-30>
An electron enters a region in which both a unifotm electric field E and a uniform magnetic field B exist The magnetic field
Bis pointing into the paper. In which direction should the electric field be applied so that the electron could beundeflected?
A. a
electric field electric field

e- �
elcctro11

X X X X
e-�
clectronxxxx

� A copper rod XY is placed on a pair of smooth inclined conducting rails which are located in a magnetic field applied

r·:-1}.r':·
X X X X
vertically downward. The rails make an angle 0 to the horizontal and a battery is connected to the rails as shown above.

:-
·�·��"r-
Which diagram shown below represents the magnetic force FB acting on the rod when viewed from end Y?
C. D.

:! ill:'1:'
el� xxxx
e
elc��xxxx
X X X X e:,
X X X X
X X X X X
X X X X
DSE Physics - Section D : M.C. PD-EM4-M/26 .DSE Physics - Section D : M.C. PD-EM4-M/27
EM4 : Magnetic Field EM4 : Magnetic Field
84. < HIIDSE 2014 Paper IA- 26 > 87, < lEDSE 2018 Paper IA - 27 >
A straight wire carrying a current J pointing out of the paper is
placed in a uniform magnetic field between two pole pieces as
shown. At which point, P, Q, R or S, can the resultant magnetic
field be zero ?
Neglect the effect of the Earth's magnetic field.
r;}
A, p
B. Q
C. R
D. 8
Four long straight parallel wires P, Q,R and S carrying currents of equal magnitude are situated at the vertices of a square as
shown. P, Q and R each carries a current directed into the paper while S canies a current directed out of the paper. The 88. < HKDSE 2018 Paper IA- 28 >
direction of the resultant magnetic field at the centre O of the square is along
A. OP. wufmm ""''""' fiold
B. OQ.
C. OR. . . poim:ing out ofthe paper

D. as.
charged
partiell:S p Q
85. < HIO)SE 2014 Paper IA- 28 > P and Q are two particles carrying the same amount of charge but of different masseS. They travel with the same speed and
The figure shows a closely packed long solenoid of cross-sectional area enter a unifonn magnetic field pointing out of the paper as shown. Semi-circular paths with. different radii are descnbed
A and length L having a total of N turns. If the solenoid carries a before they emerge from the field. Which descriptions below-are correct?
constant direct current throughout, which of the following changes can (1) Both P and Q are positively charged
increase the magnetic flux density Bat its central cross-section?
(2) P and Q emerge from the field with the same speed.
length cross-sectional area total number of turns (3) The mass ofQ is greater than that of P.
A
A. 2L 2A 2N A, (I) & (2) o,,Jy
B. L 2A N B. (I) & (3) only
C. 2L A N C. (2) & (3) only
D. L A 2N D. (!), (2) & (3)

89. <HKDSE 2019 Paper IA-26>


86. < HKDSE 2017 Paper IA- 26 >
The figure shows a simple d.c. motor, the coil ABCD is mounted between the poles of two slab-shaped magnets.

Which of the following statements is correct?


A. The turning effect is zero when the coil is vertical.
B. The magnetic force acting on BC is the greatest when the coil is horizontal.
C. The direction of the magnetic furce acting on AB remains constant.
D. The direction of the current in the coil remains unchanged
90. <HKDSE 2019 Paper IA-27> 92. <HKDSE 2020 Paper IA-27>

1x x x x
negatively x x x x x
1/ uniform magnetic
field pointing
charged o---;-;--;tx--x-- into the paper
particle

A negatively charged partic1e goes undeflected through a region in whieh a uniform electric field and a
uniform magnetic field arc set up as shown. The electric field is set up by the potential difference
across the two parallel metal plates:. Which of the following changes may cause the charged particle to
deflea downward ? Neglect the effects of gravity.
(I) increasing the potential difference across the phnes
(2) increasinglhemagnitude of the charge on the particle
(;) increasing the particle", sp<e<i entering 1he n,gion
A, (!)only
B. (3)only
C. (1) and (2) only
D. (2) and (3) only

93. < HKDSE 2020 Paper 1 B .9 >


91. <HKDSE 2020 Paper IA-26>

• ....thoffl on a bcrizOntal surface around a long vertical


The figure below shows the magnctu: fie!d .r-r4 !the The Earth's magnetic field is NOT Two small metal spheres are attached to tlle ends of an insulating rod oflength 5.0 cm. They carry charges +Q
straight wbe cmying a st:cady' cunent /poinbDg o o paper
and -Q respectively of equal magnitude as shown in Figure 9.1. The insulati_ng rod is suspended horizontally
ucgl«!<d. between two parallel metal plates. A and B, which are connected to- an EJLT. (extra high tension) supply.

-----
What arc the clucdions ofthe following ?
ure 9.1

the horizontal component or the mapdie !Grt=c aperier,,«d


the Esrth's mapetlc field by the turrent--carrying wire ToE.H.T. +
A. � ,j,
B. � t
C. ➔ ,j,
o. ➔ t
The rod is paraUeI to the metal plates when the E.H.T. is off, After the E.1-I.T. is switched 011, an electric field

!
is set up between the plates and the rod is twisted by an angle of20 ° as shown in Figure 9.2.

'' ,,
Top view
ho�
::
+Q

sure 9.2 A B

-0
- ,' '
,,
,'

{a) On Figure 92, sketch the electric field lines due to the potential difference across the plates. (2 marks)

{b) The potential difference across A and Bis 5.0 kV and the separation between the metal plates is 10 cm.
The force due to the electric fieJd acting on each sphere is 2.0 x l 0-5 N, find

(i) the moment acting on the rod as shown in Figure9.2 due to the electric forces on the charged spheres.
(2 marks)

*(ii)the strength of the electric field E due to the potential difference across the metal plate,s.

(iii) tl1e magnitude of the charge Q on the spheres.


. DSE Physics - Section D : M.C. Solution PD-EM4-MS/0l DSE Physics - Section D : M.C. Solution PD-EM4-MS /02
EM4 : Magnetic Field EM4 : Magnetic Field
HKEAA's Marting Scheme is l)reJ)aied tor the marlrers' reference. It should not be reganlad as a set of model answers.
Studems. and teachers who are not involved in tho mm:kina process are advised to in� the M.arkm.s: Scheme with care. 2. A
When current flows through the solenoid, there is magnetic field inside the solenoid.
M.C. Answers
The two iron rods are then magnetized to become electromagnets.

1. C 11. B 21. A 31. A By use of Right hand grip rule, the direction of magnetic field is upwards.
41. C
Both the upper ends of the two iron rods are N•pole and lower ends are S•pole.
2.
'�
A 12. A 22. A 32. D 42. A
�'
Since like poles repel, they repel away from each other.
3. B 13. B 23. B 33. D 43. B Tl:m, A moves to the left and B moves to the right.
4. B 14. A 24. A 34. D 44. D
s. C IS. A 25. C 35. D 45. B 3, B
There exists repulsive magnetic force between two currents flowing in opposite directions.
6. C 16. C 26. C 36. C 46. A .-. Magnetic force on Q is to the right
7. B 17. D 27. D 37. A 47. C
8. B 18. D 28. D 38. D 48. D 4, B
For a d.c. source, there are 2 possiOle cases.
9. C 19. A 29. D 39. D 49. C Case I ; Cllil'ent from wire Xto wire Y
10. B 20. B 30. B 40. D 50. B Case II : Current from wire Yto wire X.
In both cases, directions of current in the 2 wires are opposite ⇒ repulsion
51. A 61. D 71. D 81. C 91. A

52. A 62. B 72. B 82. C 5.


B
�-;y
53. C 63. A 73. B 83. C 17
54. B 64. B 74. A 84. A
55, D 65. C 75. A 85. D (1) From the diagram, a downward force is acting on the electron.
(2) Current carried by the electrons is opposite to direction of motion of electrons.
56. D 66. C 76. A 86. A
By Left-band rule, the magnetic field B should be pointing into the paper.
57. B 67. D 77. A 87. C
58. A 68. C 78. A 88. D 6. C
59. B 69. D Consider the rod BD,
79. D 89. C

k
Direction of current: from B to D, F
(1)
60. D 70. A 80. C 90. D
(2) Direction of B-field: into paper.
By Left-band rule, an upward force acts on the copper rod
M.C. Solution :. tension in each thread decreases

1. C 7. B
Charged particles will be deflected by a magnetic field.
Wire AB : CUIIent flows from B to A
• (1) Electromagnetic waves : contain. no charged particles.
W'rre CD: current flows from CtoD
✓ (2) Beam of electrons : negatively charged particles
Since the two cum:nts flow in opposite directions, the forces between them are repulsive.
✓ (3) Beam of protons : positively charged particles Thus the force acting on CD is towards the right.
DSE Physics - Section D : M.C. Solution PD-EM4-MS/03 DSE Physics - Section D : M.C. Solution PD-EM4-MS/04
EM4 : Magnetic Field EM4 : Magnetic Field

8. B 15. A
Direction of needle gives the direction ofB-field, which canbe found by Right-hand screwmle, ConsiderXandY,samediTI:ctionofCUtteI1t ⇒attractio n⇒ Ytotheleft
X (1) the needle should point upward Consider Y and Z, opposite direction of current⇒ repulsion ⇒ Yto the left
✓ a downward B-field acting on the compass
(2) :. Combining the tvro results, Yexperiences a net force to the left.
X Q) the needle should point upward
16. C
9. C ✓ (!) Increase the number of turns ⇒ strength of magnetic field is increased

" p""'"' F-v


F• V-l
(40)(0.2) - 33.3%
Q2)(2)
✓ (2 ) Reduce the resistance ⇒ Inc rease the current ⇒ strength ofmagnetic field is increased.
a.c. sOlll'Ce gives the same strength of B-field but the direction of the magnetic field would V8I'f
(3)

10. B 17. D
For Z andX, current in opposite direction ⇒ repulsion ⇒ %experiences a force to the right Currents in wires .flowing in the same direc tion will attract each other
For Y and.X, current in same direction ⇒ attraction ⇒ X experiences a downward force The force acting on R by P will act towards P
.-. Resultant force acting o n Xis the vector sum of the aboveforces, i.e. Q. The force acting on R byQ will act towards Q
The resultant force acting on R by P and Q is downwacl.
11. B
✓ (1) An electricbell contains a soft iron core. 18. D
'When current flows through the coil, the soft ironbecomes an electrmnagnet. ByRight hand screw rule,
✓ (2) A telephone receiver makeuse of theelectromagnet the magnetic pole at P is North, as the compass needle points a"Way from North, it points upwards ;
to give varying magnetic force to make the iro n diaphragm. vibrate. the magnetic pole at Q is South, as the compass needle points towards South, it points downwards.
X
(3) Moving-coil loudspeaker is an application of magnetic force acting on current inside a magnetic field.
19. A
12. A Since the toy car is battery operated, option B and D are not crurect as the supply is a.c.
✓ (1) R J, ⇒ rt⇒ strength of magnetic field is increased The switch should be connected in series as sh.own in A
X (2) Steel : difficult to magnetize and dimiagnetize⇒ strength ofB-field is decreased. Option C is not correct as thebattery is shorted when theswitch is closed.
X
(3) a.c. source gives the same strength of B-fi.eld but the direction of the magnetic field would vaxy
20. B
13. B
Consider the vertical wire : For a downmrd current, P and S : B-:field into paper ; Q and R : B-fieid out ofpaper
Consider the horizontal wire : For a current to the right, R and S : B-field into paper ; P and Q : B-field out of paper
For the resulting field out of paper, B-field from both wires should be out of paper, i.e., the case for Q.
The resultant magnetic field due to Bp and.BQ is the vector sum of them and points towards the right
Thus thecompass needle would point along the B-:field direction towards the right
14. A

Power given out by the cell= c I·= {12) x (0.5) = 6 W 21. A


Power dissipated by the resistor = I2 R = (0.5)2(10) = 2.5 W By usingRighthand screw rule, themagnetic field lines point into the loop.
Powerconsumedbythemotor = 6 - 25 = 3.5W
22. A
OR
✓ (1) Using stronger magnet can increase the strength of the magnetic :field, thus increasing the turning effect.
Voltage across the motor = e - IR = (12) - (0.5) (10) = 7 V ✓ Reducing the resistance of the rheostat can in'vttaSe the current, thus increasing the turning effect
(2)
Powerconsumedbythemotor = VI= (7)x(0.5) = 3.SW (3) Using a coil with SIIllll.ler number oftur.ns would decrease the turning effect
DSE Physics · Section D : M.C. Solution PD-EM4-MS/05 DSE Physics . Section D : M.C. Solution PD-EM4-MS/06
EM4 : Magnetic Field EM4 : Magnetic Field
23. B 29. D
✓ (1) By using a stronger magnet, a greater wagnetic force is produced to give a louder sound. • (1) Ifthe pole Xis N, then magnetic field is towards the right, thus current should be from Q to P.
• (2) Copper is not a magnetic material, it cannot increase the strength ofthe magnetic field. ✓ (2) If the pole Xis S, then magnetic field is towards the left, thus current is fromP to Q.
✓ (3) By increasing the number ofturns, the coil can give a greater magnetic furce to have a loud sound ✓ (3) Ifthe pole Xis N, then magnetic field is towards the right, thus current is from Q to P.

24. A 30. B
✓ (1) These two magnetic forces are equal and opposite and act on each other. ✓ (1) By Left hand rule, the magnetic force acting onRS is towards the right
✓ (2) Two cunents flowing in the same direction attract each other. • (2) By Left hand rule, the magnetic force acting on QR is pointing upwards, not zero.
• (3) Hthe directions of the two cutrents are both reversed, they are still in the same direction and attract ✓ (3) The four magnetic forces acting on the four wires balance each other, thus there is no resultant force.

25. C 31. A
p
By Lefthand rule, the magnetic force acting on the right wire
is into the paper and that on the left wire is out ofpaper.
X y
Since these two forces arc oquaJ in magnitude ' ' 11F
and opposite in direction, their resultant force is zero. I ' ✓ uniform magnetic field
' '
p
However, these two forces give a clockwise turning effect

F
The magnetic field line is completed as shown in the figure.
on the coil to make it rotate.
End Xis the North pole as magnetic field lines come out here.
By Right hand screw rule, the cunent is from P to Q through the solenoid.
� observer's eye
32. D
26. C
From the diagram, the circular path ofXhas a greater radius, i.e. rx > ry
Since the left hand side ofthe solenoid is N-pole, compass
2 m,' mv mv
,=-oc-
by Right hand screw rule, current in solenoid is as sho\Vll. ByF=BQvand F = mv BQv=-
' BQ Q
Since the current flows frm:n (+) tennina1 to (-) tenninal of a N�S • (1) 11lx < my ⇒ rx <ry

L0
d.c. supply, Qis (+) and Pis(-). ✓ (2) Qx < Qy ⇒ rx > ry
Direction of magnetic field lines inside the solenoid is ✓ (3) Vx > Vy ⇒ rx > ry
towards the left, thus the compass needle is towards the left.
p Q
33. D
27. D
Fora long solenoid, B = µonl
✓ (1) By using Left hand rule, thumb representing magnetic furce F points downwards, • (1) Bis independent ofthe diameter ofthe solenoid
the middle finger representing current J points into the paper,
✓ (2) B is proportional to the number of turns per unit length n
thus the .finger representing magnetic fieldB should point to the right
Therefore, Xis North pole and Y is South pole, to give magnetic :field pointing to the right.
✓ (3) B is proportional to the current I

✓ (2) Ifthe cUtteirt direction is pointing out of paper, the tnagnetic force points upwards, thus reversed.
34. D
✓ (3) The magnetic force depends on the strength of the magnetic field, • (1) By Left..hand rule, the magnetic force actihg on the el�n horizontal.
thus stronger magnets can increase the furce F.
:. The path of the electron should be circular on a horizontal plane.
✓ (2) Since magnetic force is always perpendicular to the direction ofmotion
28. D :. no work done on the electron by the magnetic force :::,. constant speed of electron
Current out of paper should give magnetic field in anticlockwise direction.
✓ mv'
By BQv = - m, 1
r=-<X:-
Both currents give anticlockwise magnetic field, and give the resultant pattern as shown in option D. (3) -
, BQ B
DSE Physics - Section D : M.C. Solution PD-EM4-MS/07 DSE Physics - Section D : M.C. Solution PD-EM4-MS/08
EM4 : Magnetic Field EM4 : Magnetic Field
35. D 42. A
The horizontal component of the magnetic :fie!dBcos 8 is perpend icular to the vertical current
F =
B cos 8 xJL = (10- 3 cos30°) x (5) x (0.4) =
1.7 x J0--3 N
k_h
ByRigbt-,hand screw rule, B-:fieldproduced by current in wire PQ at position RS is Bi into the paper.
36. C
By Left-hand rule, the magnetic force acting on the wire RS is in +y direction.
For a long solenoid, B = µ, n I
✓ (1) Bis independent of the diameter d ofthe solenoid B
43.
✓ (2) Bis proportional to the number oftums per unit length n By BQv =mrw'- and v = r@
x (3) Bdepends on the material of the core, µ, represents the permittivity of air BQ=mai=m 21t

37. A T= 2nm
BQ
Two cmrents in opposite directions have repulsive forces between them.
These two forc es are action and reaction that have th e same magnitude, 44. D
thus two wires have the same change ofshape. By Left hand rule, magnetic force acting on the upper wire is out ofpaper and that on the lower wire is into the paper.
Thus the loop would rotate in anticlookwise direction when v ei wing from right handside indicated bys.
38. D
F = µofilz L ex; l1·l2 45. B
2Nr r
Electric force = qE
F' = (2)(2) -F =2F Magnetic force = Bq v
(2)
Forundeflect edmotionofachargedparticle: qE = B q v v= -
39. D
:. Only those particles that have the same speed v equal to BIB can hav e no deflection in the cross-field region.
Since the nragneti.c field inside the solenoid is unifonn, Bis constant and thus be a horizontal l in e.
Near the end of the solenoid, the :field then decreases gradually to zero outside the solenoid. 46. A
✓ (1) Two currents in the same direction ⇒ attraction between the two wires
40. D
F = µ. 1 � oc _!_
x I l L
By Left-hand rule, direction of magnetic force on the particle is downwatd. (2)
21tr r
m,'
BQv=-
• (3) There should be a neutral po ni t in midway between them where the magnetic fieldis zero.
-26
3 )(lO�)
(0.08)(l.6xl0-1g) - ( . 2xl0 r = 0.25m
C
r 47.
✓ (1) From the diagram, the magnetic force is upwards.
41. C By Left handrule, the current is in the same direction as the motion.
:. Both P and Q are positively charged.
When the upper part is at a higher potential,
current flows from left to right alongX and flows :from right to left along Y ✓ (2) By F"" BQv andF=-­
=' BQv= m,; . mv
·· r=-ocv
r r BQ
:::::,. current flows in opposite direction along the 2 w i res ⇒ repulsion

<l) When the lower part is at a higher potential, • (3) ByF=BQvandF=mrw'- mr@w = BQv 2
m•v- n: =BQv
T
cun:ent flows from right to left alongX and flows from left to right along Y
⇒ current flows in opposite direction along the 2 wires ::::;. repulsion
T = 2 nm which is independent ofthe speed of the particle
BQ
DSE Physics - Section D : M.C. Solution PD-EM4-MS/09 DSE Physics - Section D : M.C. Solution PD-EM4-MS /10
EM4 : Magnetic Field EM4 : Magnetic Field

48. D 53- C
ByRight-hand screwrule: ✓ (1) BC is perpendicular to the magnetic field, thus a magnetic force acts on it.
At.,\", the B-tield by the upper wire is into paper and that by the lower wire is also into paper. ✓ (2) By Left-band rule, magnetic force acting on AB is downwards and that on CD is upwards,
: . The resultant B-field is into paper. magnetic force on BC is out of paper and that on AD is into paper.
:. The magnetic forces on the four wire s tend to reduce the area of the coil.
2 X (4nxl0-7)·{1) =4xlo-6T
2,(0.l) (3) As the magnetic forces actillg on AB and CD balance each other,
and that on BC and.AD balance, there is no resultant force acting on the coil..

49. C
54. B
For two long, straight parallel conducting wires: F = µ. 11 12 L = µ I2 L /2 L
21tr oc ByF=BQv andF=mr(J) 2
21tr 21tr
✓ The force is inversely proportional to the distance between the wires r.
mra>OJ = BQv
(I)
2
✓ (2) The force is proportional to the length of that section of the wires /1,. m•v• 1t = BQv

(3) The directions of current flow only affect the direction of the force but not the magnitude. 21tm
T ""
BQ
50. B Tis independent of the speed v and is a constant, thus the graph is a horizontal line.

BQv = mv'
55 D
· v BQr 0.02xl.6x10·19 xQ.005
.. = = - l.?6xl01ms•l B,
m 9.lxl0-31

l>- B

' 'i'
For the direction, consider the point at the rightmost position
r ,' '
By Left hand rule, since current is upwards, velocity of electron is downwards B
' \
:. the electron moves in clockwise direction w
11 --------@h
- '-->I
51. A
By Right-hand screw rule, the B-field by each wire is given as shown.
2
By F = B Qv and F = mv
' Current11 produces the magnetic fieldB1 perpendicular to P 11.
Currcntlz produces the magnetic fieldBz perpendicular to PI,,.
mv' k=.fL,..2.... .!.
BQv=- o: The resultant magnetic fieldB is the sum of the two horizontal components ofB1 and }h.
m Br r
B = B1 cos 30 ° + B2 cos 30 °
k' = .:!,_,k = !:_
'• 2 µ I ✓3 ✓3 I
==2B1 cos30 ., =2• � · µ
= 1t; (totbenght)
2 r z Z


52. A
56. D
In order to have a B-field along the diagonal RP,

(1) B-field byP andR should be eliminated one another (as B-fi.eld produced by P andR at Ois along QS),

(2) B-field byQ and S should be from Oto P (as B-:field produced byQ and Sat O is along PR).
By filght-band screw rule, the B-field by each wire is given as shown.
Same current carried bythe 4 wires ⇒ same magnitude ofB-field To satisfy (1), current of P and R should be in the same direction (either into or out of paper).

:. The resultantB-field is given byI. To satisfy (2), by Right-hand screw rule, current ofS: into paper; CUITent ofQ : out of paper.
DSE Physics - Section D : M.C. Solution PD-EM4-MS/11 DSE Physics - Section D : M.C. Solution PD-EM4-MS/12
EM4 : Magnetic Field EM4 : Magnetic Field

57. B 62. B
Electric force "" Q E Magnetic force = B Q v In a magnetic field, charged particle performs circular motion with period T.
21t =
Crossed£.. andB- fields ⇒ both forces in opposite directions ⇒ no deflection if the 2 forces equal By BQv = mr(})2 .·. B Q=m(})=m (asv r(}))
T
... E
v= - 2 1t·m
:. T =
B·Q
:. With same E-field and B-field, the ions should have the same velocity.
For the proton iP , it performs halfofa cycle 11 =.!.r=.!.x �=�
2 2 B-Q B·Q
58. A

Magnetic force between two parallel currents in same direction is attractive.


For the alpha ia, itperlbnns a quarter ofa cycle ti = .!. T = .!. x 21t•(4m) = �
4 4 B-(2Q) B-Q
Magnetic force between two parallel currents in opposite direction is repulsive.

59. B 63.
mv'
Inside the magnetic field, the magnetic force provides the ceniripetal force By BQv=-- ( ris the same)
.
· · BQv
='
=-- :. r=!!!!..
BQ
B . = m. � xQ,
x
B0 m0 V 0 Q0
(!) IfthefiUK density B has decreased gradually, the radius r should be gradually increased.

(2) Ifthe charged particle has lost its charge q gradually, the radius r should be gradually increased.
✓ (3) When the charged particle has lost its kinetic energy gradually, its speed vis gradually decreased,
thus the radius r would also gradually decrease. 64. B
µ,, ·I
By B•-
60. D 21t·r
•M
b.13 µo
=
Bp = µJ1 - µ� (0.6) = 0 2x·r
2•(30) 2•00)
{41txl0-1)·(0.5)
(S.0xl0-.,1) =
Ii is in the opposite direction so that the direction of the magnetic field produced is opposite to that by h 21t·r
r = 0.02m= 2cm
6l. D
IA 65. C
p
d Since the current through PQ and the current through RS must be always in opposite direction,

p "-
2A
force between the two wires must always be repulsive. I
2A µJ1I2 L
R F= cx::I'-

. V
2,., O ,_____,,___ I\
F,
The shape of the graphFshould be same as the graJXl fl • t

Forceperunitlength: !._ = µ,,-1,-12 Fa µ,•0)·0)


L 21t·r 2n·(d) 66. C

ForceactingonRbyP: R = µ.· (lH,"2) = p (upward s) Ifthe charged particles passes crossed field without deflection, then Q E =BQ v :. V = !
I 21t•(2d) B
The beam of charged particles must have the same velocityv.
ForceactingonRbyQ: p2 = µ.•(2H2) = 4 F (downwards) ' = �
21t•(d) If only the D1agDetic :field is present, B Q v = � :. r
BQ
Netforce onR = 4F-F = 3F
Different masses m or different charges Q can give different radius of curvature r, and thus they split up.
DSE Physics - Section D : M.C. Solution PD-EM4-MS/13 DSE Physics - Section D : M.C, Solution PD-EM4-MS /14
EM4 : Magnetic Field EM4 : Magnetic Field

67. D 71. D
Ifthe wirePhas no net force, then the resultant magnetic field due to Q, R, S must be zero atP. mv'
By BQv=-
The magnetic field due to the three wires is shown in the figure. '
Letthe separation between QR be r, then the separation between PR is ✓2 r. mv
.. r=-
BQ
BQ=Bs = B
TheresultantofBQandBsis .fi.B. For greater deflection, radius r of the circular path should be decreased.
B,
B, • .fiB • (!) By increasing the mass m, radius r increases, thus deflection decreases.

µ,I' ✓2 .&!_ ✓ (2) By increasing the charge Q, radius r decreases, thus deflection increases.
✓2
X
2,( ,) 2n,
✓ (3) By increasing the flux density B, radius r decreases, thus deflection increases.
:.1'=21

OR 72. B

Let the distance of each side ofthe square be r. The magnetic furces betweenX and Y are attractive.
Q (v----------{8) R Magnetic force on Xby Y is 0.1 N rightwards and magnetic force on Yby Xis 0.1N leftwards.
Magnetic forces between any two currents are given by !.... = µ. 11 Ii ,\
:1

r--,
L 21tr When the magnetic field into the paper is applied. by use of the Left hand rule, X y
F\ :

� �
For two currents flowing in the same directl.on, attractive forces exist. I the magnetic force is in rightward direction.
For two currents flowing in the 0PPos1te direction, repulsive forces exist. l \ Since the resultant magnetic force onXis 0.5N,
F- -- - --@ S

r-7
Consider • the te$Ultant of attractive forces by Q and by S balances the repulsive force by R. p • the magnetic force due to the applied magnetic field is 0.4N rightwards.

p

-.fix µ0 I
= µ0 ·I' l
The same magnetic force of 0.4 N in rightward direction also acts on Y.
2'1tr 21tx✓Zr F,
The resultant magnetic force on Y = 0.4- 0.l = 0.3Nrightwltrds
:.1'=21

73. B
68. C
Consider the horizontal component of 1he Earth's magnetic field that is perpendicular to the current.
Horizontal component of the magnetic field is B cos 0. 0
ByF=Bcos0·I-L
(J.5 x 10--s) = (B cos 20°) (8.0) (0.5)
B = 2.0x 10- 5T

69. D
✓ (!) B = !,.lo 11xlx - µ,, n.,, ly = µ.,(1200xl - 2400x0.S) = 0

✓ (2) The magnetic field at Q is due to solenoidX only, as no magnetic field outside a solenoid. Let r be the radius ands be the arc length oftransit

B = µo nx_ Ix = (41t x 10-7) x (1200) x (l) = 1.5 mT By BQv = mrco 2 BQ=m(J)=m


21t
(asv = rai)
T
✓ (3) By right hand grip rule, magnetic field due to solenoidXat Q is towards the left.
21t·m
T•--
B· Q
70. A
✓ (!) Timeoftransitalongthe arc:t = Tx l!......
360�
I oc 0
F = BILsin0xN

= (0.4)(1.2)(0.15) sin30° X ( 20) = 0.7N


• (2) Period Tandt should 00 independent of the speed v.

✓ (3) toe To:


By Left hand.rule, the magnetic force is out ofpaper. B
DSE Physics - Section D : M.C. Solution PD-EM4-MS/15 DSE Physics - Section D : M.C. Solution PD-EM4-MS/16
EM4 : Magnetic Field EM4 : Magnetic Field

74. A 79. D

Magnetic force per unit length between two parallel currents : !... "° µ,, l; 12 Magnetic force per unit length between two parallel currents : F = A 1112
L 21tr 2s,
Magnetic forces between 2 parallel currents : Moreover, magnetic forces between 2 parallel currents in the same direction are attractive,
same direction are attractive ; opposite direction are repulsive. and magnetic forces bet\Veen 2 parallel cnrrents in the opposite direction are repulsive.

Magnetic force per length on the middle win: by /1 = µJ1 I


= µ,, I,, l Assume the separation betweenXZis r. Separation between yz is also r.
(to the left)
2n(rl2) 1tr

Magnetic forceper length on the middle wire b y ]z = µ,, 12 1 = &.!J!.. ( to the right)
2,r(r/2) 1tr

Since Ii > 11, the magnetic forceper length by his greater,

.,
R.ewltant magnetic force per length on the middle wire = µ,, I2 I Al; I µJ(Ia-I1) (totheright)
1tY 1tT

By Newton's third law, the magnetic force per unit length on Zby Xis also F .
75. A
Magnetic force per unit length on Zby X - A (1) (l) ... F ( to the left)
✓ (!) Bis proportional to I 21t(r)
✓ (2) B is proportional to n, the :number of turns per unit length
Magnetic force per unit length onZby Y = Po (l)( 3) = 3 F (to the left)
(3) B is independent of the area A for a long solenoid 21t(r)
Resultantforceperunitlength onZbyXand Y = F+3 F = 4F (to the left)
76. A
By using Right hand screw rule, 80.

:j:;:j:
the magnetic field line at the centre of the loop points into the loop
as indicated in A

77. A X X Fa X X
✓ �) Using stronger magnet can increase the strength of the magnetic field, thus increasing the turning effect. When the electron moves towards the right, it represents a murent to the left
✓ (2) Reducing the resistance of the rheostat can increase the current, thus increasing the turning effect Since the magnetic field ispointing into the paper,
by Left hand rule, the magnetic force Fa is downwards.
(3) Using a coil with smaller number of turns would decrease the turning effect
To make the electron be undeflected, the electric force FP. must be upwards.
As the electron canies negative charge, the electric field should be downw.trds
78. A
so that the electric force is upwards.

C
� By the use of Right hand screw rule to find the direction ofB-field due to a straight wire current:
Bsin30�
Direction ofB-field at point O due to P: out of paper
Direction ofB-field at point O due to Q : out of paper
Consider the component of the magnetic field peipendicular to the current, th at is, B sin 30".
Direction ofB-field at point O due to R : into paper
The direction ofmagnetic force is into the paper by use of the Left hand rule.
Direction ofB-field at point O due to S: out ofpaper
F = Bsin30° xJxL
TheB-field given by R is in opposite direction to that of the other wires.
= (5 x 10-3}sin30 ° x (LO) x (0.5) = I.25 x 10-3N Thus, removing wire R can increase the resultant magnetic field at 0.
DSE Physics - Section D : M.C. Solution PD-EM4-MS/17 DSE Physics - Section D : M.C. Solution PD-EM4-MS /18
EM4 : Magnetic Field EM4 : Magnetic Field
82. C 86. A
✓ A. When the coil is vertical, the magnetic forces on the founvires are either directed away from the centre
or towards the centre. Thus, the magnetic forces do not have moment to give turning effect

B. The magnitude of magnetic force acting on BC is constant at any position of the coil
=
since F BI l, magnetic field B, current!, and length l are all constant.

c. The direction of magnetic force acting on AB reverses every half cycle


since the direction of current through AB reverses every halfcycle due to the commutator.

D. The direction of cunent in the coil reversed ever:ybalf cycle due to the commutator.

From th e pattem. of catapult field, the magnetic force is towards the right
87. C
From the figure shown, the direction of current is into the paper. The magnetic field produced by cun-ent I is anticlockwise around I. At point R, the B is towards the right.
By using the Left band ra1c, the direction of magnetic field is upwards.
The magnetic field produced by the poles of magnet is towards the left (from North to South).
As direction of magnetic field is from N-pole to S-pole, the magnetic pole at Xis South.
Thus, at R, the resultant magnetic field can be zero.

8 3. C
88. D
Direction of current :fl.owing through the rod is from Y to X,
✓ (1) Consider the charged particles entering the magnetic field.
into the paper as seen by the eye.
The magnetic field B is out of paper, magnetic force Fis towards the right,
Direction ofmagnetic field is downwards. by using Leftband rule, the current is upwards.
By using Left hand rule, the magnetic force is towards the left. As current is the flow ofpositive charge, the charged particles must be positive.
✓ (2) Since magnetic force is always perpendicular to the motion, no work is done,
84. A thus the kinetic energy and speed must remain unchanged after emerging from the magnetic field.
By using Right band grip rule, ✓ m vi asB, q, v are the same, mass mis proportional to the radius r.
(3) By B q v "" ,
B-field due to Pis along OS at O.
AI; the radius of Q is greater, the mass of Q must b e greater than that of P.
B-field due to Q is alollg OP at O.
B-:field due to R is along OQ at O.
B-field due to Sis along OP at O.
The B-fields due to P and R are in opposite direction
and they balance each other.

The resultant ina.gnetic :field due to Q andS are in the same direction and thus theirresultant magnetic field is along OP.

85. D

Magnetic field inside the solenoid is : B == A NI


L
The magnetic field is independent of the area A.
A. The magnetic field is unchanged as both N and L are doubled.
B. The magnetic field is unchanged as both N and L are unchanged.
c. The magnetic field is halved as Lis doubled.
✓ D. The magnetic field is doubled as N is doubled.
PD-EM4-Q/01 PD-EM4-Q/02
1
DSE Physics - Section D : Question DSE Physics - Section D : Question
EM4 : Magnetic Field EM4 : Magnetic Field
The following list of formulae may be found useful : 1. (a) Whatis the polarity of the solenoids

Force on a moving charge in a magnetic field F = B Qv s:in B (i) atB, and

Force on a current-carrying conductor in a magnetic field F=B/lsin0 (ii) at C? (2,narl<,)

Magnetic field due to a long straight wire (,)

(tl) -----------------------------
Magnetic field inside a long solenoid
(b) 'Wbatis the direction of rotation ofthe coilPQRS

(i) in the first min.Ute,


Use the following data wherever necessary:
(ll) in the second minute, and
Permeability of free space
(ili) in the third minute? (3 marl<,)
Charge of electron e = 1.60 x 10-19 C

Electron rest mass m, = 9.11 X 1Q~3l kg (i)

Acceleration due to gravity g = 9.81 ms -2


(close to the Earth) (ii)

(ml _____________________________
Part A : HK.CE examination questions (c) What would happen to the rotation of the coil PQRS if the input voltage supply reversed at a high frequency
(e.g.50Hz)? (2�)
1. < BKCE 1984 Paper IM 8 >

(d) Suppose that instead of being connected to the battezy V, the tenninal of the solenoid at A is cmtnected to F and the
tenninal of the solenoids atD is connected to G. F and G remain connected to the external voltage supply as :indicated
in the above voltage-time graph.

(i) What is the direction of rotation of the coil PQRS

(1) in the first minute,

(2) in the second minute, and

(3) in the third minute? (3 marl<,)

(!)
�observer
(2) --------------------------
The figun: above shows a simple experimental set-up to smdy the motion of a motor. AB and CD are solenoids connected to (3)
a battery V. F and Gare connected to an external voltage supply. Its variation with time is shown in the following figure.
(Positive voltage Vindicates the current flows from F to G via the coil.) (ii) What would happen to the rotation of the coilPQRS if the input voltage :reversed at high frequency (e.g. 50 Hz) ?
(2 marl<,)
V
--
(e) State 3 me1hods to increase the turning speed of this motor. (3 marl<,)
time/minute
1 2 3

-
DSE Physics - Section D : Question PD-EM4-Q/03 DSE Physics - Section D : Question PD - EM4 - Q / 04
EM4 : Magnetic Field EM4 : Magnetic Field

2. < HKCE 1993 Paper I- S > 3. < HK.CE 1993 Paper I- 5 >
A student designs a simple door bell as shown in the below figure. When switch S is pressed and then released, two notes The figure below shows a solenoid passing through a piece of horizontal cardboard. A Wl'llct current passes through the
"ding-ding'' are heard. solenoid from A to B to produce a magnetic field.

/t'. A.LJF:.X1:
-Iron spring
- Ip

;r'! ,(1 I Q

/ I
r-- (a) DescnOe a method to find the magnetic field pattern on the cardboard using iron filings.
SofUron corJ

Metal plate__J Metal plate

(•) State the polarities at the two ends of the soft-iron coreP and Q. (2marks)
(b) Draw a diagram to show the pattern and direction of the magnetic field on the cardboard. (2mm)

(b) Explain how the two notes are produced. (4 marks)

4. < HKCE 2000 Paper I - 6 >

(o) Explain why soft iron is used as the core in the above design. (2 marks)

(d) Suggest one way to modify the bell so that two notes of different frequencies "ding-dong'' are produced. (2 marks)

A rectangular coil can rotate in a magnetic field as shown in the above figure. Initially the coil lies horizontally. The switch
is now closed.
(o) Comment on the follo'Wing two statements :
Statement 1 : The bell does not work Uthe spring is made of copper. (a) State the initial direction of rotation ofthe coil as seen by the observer.
Statement 2 : The bell does not work if the polarities of the battery are reversed. (4mm)
(b) The coil turns, oscillates a few times about the vertical position and then comes to a rest Explain the motion of the coil.
(4nwk,)
PD-EM4-Q/05 PD-EM4-Q/06
1
DSE Physics - Section D : Question DSE Physics - Section D : Question
EM4 : Magnetic Field EM4 : Magnetic Field

5. < HKCE 2005 Paper I - 11 > 6. <BKCE2006Paperl-7>

.. ,

moostat
main switch p Q

to22ovJ R
m,in,

Figure 1 Figure2
Figure I shows a simple hmdryer designed by Joseph. He makes use ofa motor-driven fan and a heating element to generate
warm air. Figure 2 shows the circuit diagram of the dryer. The motor and the heating element are connected to the 220 V
chain of iron clips
mains. The switch S can be connected to either contact P or Q.
In a physics lesson, you are asked by the teacher to investigate the relationship between the strength of an electromagnet and
(a) Carmen uses the dryer to dry her wet hair. Explain, in terms of molecular motion, how the dryer can speed up the rate of the number of turns of its coil by using the apparatus shown in the above Figure. Describe the procedure for the experiment
evaporation of water from wet hair. (2 marks) you should conduct State clearly how you can measure the strength of the electromagnet. (5 marks)

(b) Switch Sis connected to contactP and the following data are given:

Resistance ofthe heating element = 50 n


Rate ofair flowing through the dryer = 0.05 kg s- 1 < Hl{CE 2007 Paper 1-11 >
Temperature of air flowing into the dryer= 20 0C A copper rod PQ is hung at rest by insulating threads in a unifonn magnetic field pointing into the paper as shown in the
below Figure. The other ends of the threads are connected to a spring balance fixed on the ceiling. The two contacts P and Q
Specific heatcapacityofair = 1000Jkg·10C-l at the ends of the copper rod can slide smoothly along two fixed vertical conducting rails AB and CD. The rails AB and CD
are connected to the positive and the negative terminals of a d.c. power supply respectively. As a result, a current/ passes
Estimate the temperature of the air flowing out of the dryer, and state one assumption in your calculation. (4marks) through the copper rod.
Assume that the copper rod always remains horizontal and does not leave the magnetic field throughout the experiment.
ceiling
_...,,,,-

spring balance
--_

insulating thread
(c) If switch S is connected to contact Q instead, explain whether the temperature of the air flowing out of the dryer would
be higher than '\WenS is connected to contactP. (3 marks) A C

XX X XXXXXXXXXX
p xxxxxxxxxxxxxxxx Q
xxxxxxxxxxxxxxxx
\ xxxxxxxxxxxxxxxx
xxxxxxxxxxxxxxxx
B xxxxxxxxxxxxxxxx D
connected connected
tothe+ve copper rod tothe-ve
temrinal tennmal
I I
DSE Physics - Section D : Question PD-EM4-Q/07 PD-EM4-Q/08
I
DSE Physics - Section D : Question
EM4 : Magnetic Field EM4 : Magnetic Field
7. (a) (i) In the above Figure, indicate the direction of the force F acting on the copper rod due to the current passing from p 8. < HKCE 2008 Paper I - 8 >
toQ. (!Wll'k)
Figure 1 shows the simplified structnre of a motor with the plane ofthe coil at horizontal position. At this moment, it carries
a cwrent in the direction indicated by the arrow.
(ii) Suggest THREE me1hods to increase the force F. (3nwks)

(iii) Express the magnitude of force F in terms of the reading R of the spring balance and the weight W of the copper
rod. (1 mark)
Figure!

(b) A teacher conducts an experiment with the setup in the above Figure to find out how the reading R of the spring balance
changes with the current!. The Table below shows the data collected.

RIN 1.4 1.1 0.8 0.5


I/A o.o 0.5 1.0 1.5 + -

(i) Plot a graph ofR against[ in the below Figure. A scale of 1 cm representing 0.25 N and 0.25 A is used. (a) Mark the direction of the magnetic force acting on the side AB in Figure 2. (1 mark)
(4marks)
(b) Explain how the commutator helps to keep the coil rotating in one direction.

(c) When the coil reaches the vertical position, the current is zero. Explain why the coil keeps on turning even no magnetic
force is acting on it. (1 mark)

Figure2
N

A
(11) Findtheweight ofthecopper rod. (! Wll'k)
/
commutator
/

(fu) Find the maximun1 value of/ such that the insulating threads remain taut. (1 mark)
+ -

(iv) Ifthe experiment is repeated with a heavier copper rod, sketch a graph of R against I you would expect to obtain in (d) If the coil is not fully inserted between the magnets as shown in Figure 2, describe and explain how this would affect
the above Fi gure, and label it as L. (2 marks) the motion ofthe coil. Assume the current is the same as before. ( 3 marlcs)
DSE Physics - Section D : Question PD-EM4-Q/09 DSE Physics - Section D : Question PD-EM4-Q/10
EM4 : Magnetic Field EM4 : Magnetic Field

•• <HKCE 2069 Paper I- 8 > 10. < BK.CE 2010 Paper I - 12 >
The Figure below shows two long resistance wires X and Y which are connected in series to a d.c, power supply. X and Y are Figure 1 shows a simple motor that contains two electromagnets.
made ofthe same material but Xis thiilner than Y.

electromagnet\
electromagnet
/

wi,eX wueY
coil
soft:ironcore
soft iron core

- + connected to a d.c. power supply

(,) State the direction oftbe magnetic field at Y due to the current passing throughX (l mark)

(a) State the polarities (north/ south) ofthe electromagnets at Q and Rand the direction (up/ down) of the electromagnetic
force acting on side AB of the coil at the instant shown. (2 marks)

(b) There are magnetic forces Fx and Py acting onXand Y respectively due to the current passing through them.
Q R ______
(i) Indicate the direction of Fy in the Figure below. (I mark)
AB---------
(b) Figure 2 shows the instant when the coil has rotated by 180 °. By considering the electromagnetic force acting on side
AB, explain why the coil can continue to rotate in the same direction. (4 marks)

wueX wireY

coil
soft iron core
-+ A soft iron core
connected to a d.c. power supply

b,ttccy
(ii) Compare the magnitudes of Px andFy and explain briefly. (2 """")

...... ,
(o) Explam \Wether X and Y 'Will attract and repel each other alternatively when the d.c. power supply is replaced by an a.c.
power supply. (2 marks)
(c) Suggest two methods to increase the speed ofrotation ofthe motor.
.// r;:====================================,:;i PD-EM4-Q/ 11 PD-EM4-Q/12
1
DSE Physics - Section D : Question DSE Physics - Section D : Question
EM4 : Magnetic Field BM4 : Magnetic Field
11. <HKCE:2011PaperI-6> Part B : HKAL examination questions
The magnetic field between two slab-shaped magnets with unlike poles facing each other is uniform. Describe how to use the
apparatus in the Figure below to illustrate this. (4 marks) 1:2. < HKAL 1996 Paper I- 8 >
simple current
balance rid�

The above :figure shows a simple current balance consisting of a horizontal rectangular metal loop ABCD pivoting on the
axis XYwhich is at the middle between AB and CD. Part of the current balance is inside a flat solenoid such that CD is
perpendicular to the axis of the solenoid. The length of CD is 20 cm. When current I flows through the current balance
YCDX and then to the solenoid, a rider of mass 0.1 g has to be placed on AB to restore equilibrium. The length, Z, and the
number oftums, N, ofthe flat solenoid are 50 cm and 600 respectively.

(a) Indicate on the Figure below the direction of the magnetic field inside the solenoid. (1 mark)

simple cunent
brum� rida

(b) (i) Express, in tenns of I, an expression of the magnetic field strength inside the solenoid. Hence, find the magnetic
force acting on arm CD in tenns ofJ. (3 marks)

(u1 By considering the equilibrium of the current balance, deduce the value of the current I.
DSE Physics - Section D : Question PD-EM4- QI 13 DSE Physics - Section D : Question PD-EM4-Q/14
EM4 : Magnetic Field EM4 : Magnetic Field
13. < HKAL 2006 Paper I - 4 > 14. (b) (i) By considering the equilibrium of the current balance, find the average magnetic fieldB in the gap between the two
In a vacuum, a beam of electrons with a horizontal velocity 3.7 x 101 m s-1 enters midway into a region of electric field solenoids. (2 marks)
between two horizontal square metal plates as shown in the Figure below. The length ofthe side ofthe plates is IO cm. A
voltage of320 Vis applied across the plates and the separation between them is 1.6 cm.

Diagram NOT
drawn to scale (ll) By considering the current flowing in the two solenoids, calculate the magnitude B' of the magnetic :field at tite gap
produced by the two solenoids. (2 marks)

10cm

(a) Find the electric field strength between the plates.

(fu) State TWO possible reasons to account for the discrepancy between the two values ofB andB'.

(b) A uniform magnetic field normal to the paper is applied between the two plates so as to make the electron beam travel
horizontally. Find the magnitude ofthe magnetic :field applied. (Neglect the weight ofthe electron.) (2 marks)

15. < HKAL 2010 Paper I 4 > w

{a) Particles with the speed v of 1.63 x 107 m s- 1 are directed into an evacuated region with a uniform magnetic field B
14. <HKAL2007PaperI-3>
of 0.5 T perpendicular to the initial velocity of the particles as shown in the Figure below. Given that the charge to
The Figure below shows a current balance which consists of a copper wire loop PQRS balanced on two razor blades. Two mass ratio ofa particle is 4.82 x 107 C 1cg-1 , find the radius of the path descnOed by the particles in the field region.
identical solenoids, each of 1500 ttnns, are placed coaxially such that one ann RS ofthe current balance is in the narrow gap (2 nwk.s)
between the two solenoids. Each solenoid is 30 cm long and has a square cross-section of3 cm x 3 cm.
X X X X X X
rider particles X X X X X

solenoid x x
uniform.magnetic
X X X X x -;---_ fieldB pointing
X X into the paper
X X X X
copper wire frame

to rheostat and
smooth d.c. supply 2•0 A
:__...=;:::;...- (b) Explain whether the particles would emerge with a greater speed from the region ofmagnetic field.
When a current of2.0 A flows in the arm RS of the current balance, and the same current flows in the two solenoids in the
same direction (not shown in the figure), placing a rider ofniass n mg on the arm.RS can restore the balance.

{a) Indicate in the above Figure the direction of the magnetic force acting on the arm RS and the direction of the current in
the solenoids. (2 marks)
DSE Physics - Section D : Question PD-EM4-QI 15 DSE Physics - Section D : Question PD-EM4-Q/16
EM4 : Magnetic Field EM4 : Magnetic Field
Part C : BKDSE examination questions 17. A unifonn magnetic field pointing into the paper is applied over the region ABCD containmg part of the rod. When switch S
is closed, the rod 'kicks' out and leaves the liquid surface.
16. < HKDSE 2013 Paper m- 4 > (a) State the direction ( to the left / to the right / out of the paper) that the rod 'Icicles' and describe the subsequent motion
An electron moving with speed 1.2 x 10 7 m s· 1 enters a square region.AB CD with a uniform magnetic field of0.01 T pointing of the rod. (3 marks)
into the paper as shown in the figure below. The electron descnbes a quarter circle from A to C and it emerges from C with
the same speed. Neglect the effects of gravity.

electron O
A B
X7.-XXJ11
\>< X X X X X X >11
�xxxx X
XX X X XX X � umfonn magnetic field (b) When swimhSis closed, the initial moment aboutpointP that makes the rod 'kick' out is 7.2 x 10-4 Nm. Assume that
!>< X X X X X X � pointing into the paper the magnetic force acts at the midpoint of the part of the rod within the magnetic field.
�xxxxxxx
:><xxxxxxx {i) Calculate the magnetic force acting on the rod at that instant. (2 marks)
Xxxxxxxx
D Kxxxx.x.x..K c

(a) (i) Find the magnitude of the magnetic force acting on the electron at pointP on its path. (2 marks)

(ll) Hence, find the strength B of the magnetic field if the current flowing through the rod is 3.2 A when the circuit is
closed. (2 marks)

(ll1 Indicate in the above Figure the direction ofthe electron's acceleration at the pointP. (lmarlc)

{b) Although the electron accelerates due to the magnetic force, explain why it emerges from the magnetic field with the
same speed. (2 marks)

(c) Now the uniform.magnetic field is removed and a bar magnet is placed underneath the container as shown in Figure (b).
The rod is held tilted at an angle to the vertical but with its lower end still in the conducting liquid.

(c) Deduce the speed of the electron when entering the magnetic field such that it would descnbe a semi-circle from A to D
instead. (2 marks) �server'seye
s p

Figure(b)
) Diagram.NOT drawn to scale I
17. <BKDSE2015PaperIB-9>
Figure (a) shows a set-up for demonstrating one of Faraday's discoveries. A light metal rod is free to rotate about point P
while its lower end just touches some conducting liquid in a metallic container.
'-=HI\
p battery
Diagram NOT drawn to scale

Figure (a) (i) Sketch on Figure (b) the field lines around the rod due to the bar magnet. (1 mark)

(ii) After closing switch Sand the rod is released from rest, descn"be its subsequent motion viewed from above.
metallic (1 mark)
'-=HI\ container
battery
. DSE Physics - Section D : Question PD-EM4-Q/17 DSE Physics - Section D : Question Solution PD-EM4-QS/0l
EM4 : Magnetic Field EM4 : Magnetic Field
HlCEAA's Marking Scheme is prepared for the marked reference. It should not be regarded as a set of model answers.
18. <HKDSE2017PaperIB�9> Students and t.eachers who are not involved in the marking :PJ;OceSS are advised to i:ntcip.ret the Marking Schelne with care.
(a) Two long straight current carcying wires, P and Q, are parallel tci each other and lie on the plane of the paper as shown
in Figure 1. The currents in the wires, /p and IQ , flow in the same direction. Question Solution
wireP wireQ

'j ·j
1. (•) (,) S-pole [1]

I I
(ii) N-pole [1]
Figure 1
� right
(b) (i) clockwise [1]

( ii) anticlockwise [!]

(iii) clockwise [1]


(i) State the direction (to the left/ to the right/ into the paper/ out of the paper) of the magnetic field at Q due to P.
(1 m,,I<;)
(,) No rotation [2]

(li) In Figure 1, draw the direction ofthe magnetic force acting on Q due to P. ( d) (i) (1) clockwise [1]
(1 m,,I<;)
(2) clockwise [1]
(ili) Show that the magnitude of the magnetic force per unit length F1 acting on Q due to P is
(3) clockwise [1]
f J
F. - µ. P Q.
I - 21tr (n) The coil would rotate continnously. [2]
whereµ,, is the permeability of free space and r is the separation between the two wires. (3 marks)
(e) AnyTHREEofthefollowing: [3]
* Increase the voltage supply
* Increase the number oftoms in.PQRS
* Increase the number of turns in the solenoids
(iv) For the magnetic force acting on Q due to P and the magnetic force acting onP due to Q, if/p :f:;lQ, briefly exp].WD.
whether the two forces are equal in magnitude. (2 marks)
* Insert iron rods into the two solenoids

2. (a) Pis S-pole [1]


(b) Figure 2 shows a metal slinky spring.
Q isN-pole [1]

(b) When S is pressed, CUil'ent flows through the coil and the soft iron core is magnetized. [1]
It attracts the iron spring towards the left. [1]
The hammer strikes the left metal plate to produce the first note. [1]
Fignre2
iVben Sis released, the iron core is demagnetized.
The hammer then springs back to strike the right metal plate to produce the second note. [1]

(c) Soft iron is a magnetic material. [!]


(i) If a direct current passes through the spring, briefly explain whether the spring will be compressed or stretched due
to magnetic force. {2 marks) It can increase the strength of the magnetic field. [I]

(d) Any ONE ofthe following : [2]


* Replace one metal plate with another made ofa different metal
(ii) A student suggests that the spring w:ill be compressed and stretched alternately due to magnetic force when an :+: Change the length of one of the metal plates
alternating current passes through. Briefly explain why he is wrong. (1 mark)
:+: Change ofthethickness of one ofthemetalplates
* Stick a lump of plasticine to one plate

There is question in next page


19. <HKDSB2020 Paper lB-3>

Read the fol1owing passage about a magnetic-,dly levitated {maglev) train and answer the questions that
follow.

'A maglev train car is just a box with magnets on the four comers; says Jesse Powell, the son of the
maglcv trai.11 inventor. The electromagnets employed have superconducting coils (i.e. coils with extremely
low resistance). They therefore can generate magnetic fields 10 times stronger than ordinary
electromagnets, enough to levitate and propel a train.

train floats above


magnetic poles P and Q
guideway

magnetic
-'--�-'-----'--�pole Q
LEVITATION

Two sets of magnetic fields are set up for different functions. One is to make the train float a few
centimetres above magnelic poles P and Q as shown while the other is a propulsion system run by an
alternating current for moving the train car along the guideway by magnetic attraction and eyulsion. This
floating design enables a smooth movement of the train. Even when the train travels up to 600 knt per
hour. passengers inside experience less vibration than travelling on traditional trains.

(a) Explain why electromagnets employing superconducting coils can produce much stronger magnetic fields.
(2 marks)

(b) State the polarities of the magnetic poles P and Q and explain how this arrangement enables the train to
float. (2 marks)

(c) Referring to the resistive forces experienced by the train, explain why a maglev train ride is {i) smoother
and (ii) raster. (2 marks)
DSE Physics - Section D : Question Solution PD-EM4-QS/ 02 DSE Physics - Section D : Question Solution PD-EM4-QS/03
EM4 : Magnetic Field EM4 : Magnetic Field

2. (e) Statement 1 is correct. [1] 5. (b) Power given out by the heating element:

Copper is not a magnetic material, it cannot be attracted by the electromagnet. [1] V'
p O

Statement 2 is not conect. [1]


0 (220)'
If the polarities ofthe b;dtely are reversed,
(50)
the soft iron core can still be magnetized and attracted by the electromagnet. [1] = 968W [1]

Assume no heat lost to the sutroundings. [1]

3. (a) Sprinkle some iron filings on the board.


E =Pt = mcl::..T [1]
[1]
Tap the board gently. [1]
(968)(1) 0
(0.05) (1000) (B-20)

The magnetic field pattern is shown by the pattern of the iron filings. 8= 39.4 c c <accept B= 39.36° C> [1]
[1]

(c) If Sis connected to contact Q, the current flowing through the motor is reduced [1]
(b)
The speed ofthe rotation of the fan is decreased. [1]

As the rate of air flowing through the dryer is reduced,

the temperature of air flowing out would be higher. [1]

6. Use the electromagnet to attzact the iron clips. [1]


Record the number of iron clips when the chain just falls down. [1]

Change the number of turns of the coil and repeat the above procedure. [1]

< Direction magnetic field lines correct> [1] Record the change of the number of iron clips when the chain just falls down. [1]

< Pattern of magnetic field lines correct> [1] In each trial, the current should be kept constant. [1]

4. (a) The coil rotates in clockwise direction. [1] 7. (a) �) F [1]

(b) When the switch is closed, current flows through the coil. As the coil is placed in a magnetic field,
P-----'------Q
there are magnetic forces acting on the wires to rotate the coil and the coil turns clockwise. [1]

When the coil turns to the vertical position, the turning effect becomes zero.
(ii) (D Increase the strength ofthe magnetic field. [!]
Due to inertia, the coil shoots tbrough the vertical position to the other side. [1] @ Increase the current. [!]
The direction of the turning effect acting on the coil reverses @ Widen the magnetic field so that the length of the rod in the magnetic field is increased. [!]
and the coil rotates back in the opposite: direction (anticlockwise). This process repeats. [1]
< Do not accept: increase the length of the rod>
As energy is lost against friction during the motion, the coil will finally stop in the vertical position. [1] < Do not accept: decrease the resistance of the rod>
< Do not accept : decrease the weight of the rod >

(ili) Balance offorces: F + R = W


5. (a) More water molecules gain enough energy to escape from the water surface. [1]
:. F= W-R [1]
The water molecules after escaped from the 'Water surface would be blown away by the wind from the dryer. [1]
DSE Physics - Section D : Question Solution PD-EM4-QS/04 DSE Physics - Section D : Question Solution PD-EM4-QS/05
EM4 : Magnetic Field EM4 : Magnetic Field

7. (b) (i) 9. (a) out of paper [1]


R/N
2. (b) (i)

wire Y
1----►Fy
l
[1]

'----- + connected to a d.c. power supply

(11) The magnitudes ofthe two forces are equal ( OR Fx = FY) [1]
0. because they are action"1"eaction pair. [1]

(c) The directions of the cmrent in X and Y are always opposite. [1]
The forces are always repulsive. (OR The forces will not attract and repel alternately.) [l]
I/A

< Correct labelled axes with units > [l]


< Correct scale> 10. (a) Q north < accept N >
[l]
< Correct points plotted > [l] R south < accept S > [l]
< Correct straight line through the points> [l] AB down [1]
(ii) W = 1.4N <accept 1.35N to l.45N> [l] (b) The cunent in the coil is reversed [l]
(iii) "WhenR= 0, I= 2.35 A < accept 2.3 A to 2.4 A> [l] due to the commutator. [l]
(iv) <The line is above the original line> [l] But the direction of magnetic field produced remains unchanged. [l]
<The line is parallel to the original line> [l] So the force acting on side AB points up and the coil continues to rotate. [!]

(c) PJ;yTWOofthefollowings: [2]

8. (a)
* Use a battery ofhigher voltage. (OR Increase the current.)
* Increase the number ofturns ofthe coil.
[l] * Increase the area ofcoil in then:tagDetic :field.
* Insert a soft iron core in the coil.

(b) Commutator can reverse the direction of the CUtTent through the coil
* Increase the number of turns of the winding in the solenoid.
[l]
whenever the coil hasrotatedhalf cycle. [l]

(c) duetoinertia [1] 11. Put the cardboard on top ofthe magnets. [!]
Sprinkle some iron filings onto the cardboard. [1]
(d) Since length of wire inside the magnetic :field is decreased, [l]
magnetic force acting on the wire decreases, Tap the cardboard gently to show the magnetic fiel d pattern. [!]
[l]
thus the rotation speed ofthe motor decreases. [l] The iIOn filings will form evenly spaced parallel lines between the magnets. [1]
-
DSE Physics - Section D : Question Solution PD-EM4-QS/06 DSE Physics - Section D : Question Solution PD-EM4-QS/07
EM4 : Magnetic Field EM4 : Magnetic Field
12. (a) simple current rider 14. (b) (ii) B'=µcnl
1500
= (4xx!0-')( )(2) [!]
0.3
flat solenoid [!]
B = 0.0126 T [!]

(rii) AnyTWOofthefollowings: (2]


* The solenoids are not infinitely long.
* There is air gap between the two solenoids.
* The Earth's magnetic field may affect the result
* The aim RS may not be exactly perpendicular to the magnetic :field.
4 x10-7 x600xl
(b) (i) B = :1r [!]
* Since it is difficult to balance, some error may occur for the mass of rider obtained.
0.5
F =BIL
1
= 41tx10- x600xl xlx(0.2)
(a) BQv = mv
[!] i
15.
05
' gBr =v [!]
= 3.02 x Icr-4 J2 <accept 3.0x I0--4[2 > [!]
(4.82 X 107) (Q.5) r = 1.63 X 107
(ii) F = mg [!] r = 0.676m [!]
(3.02 x lo--4 I� = (0.1 x 10-3) (9.81) [!]
(b) No.
I= I.BOA < accept 1.81 A> [!]
Since the magnetic force acting on the particle is always perpendicular to its velocity, [!]
no work is done on the particle by the magnetic force. UJ

13. (a) E = y_= 320 [!]


d 0.016
16. (a) (i) F ""BQv
= 2xl04 Vm-1 [!]
"' (0.01) (1.6 X I0-1!l) (12 X l07) (!]
(b) BQv=QE (!] ""- 1.92 x 10-14 N [!]
4
B = !!_ = 2xl0 7
v 3.7xl0
=: 5.41xl0-4T [!]
(ii) o-+----.'""'""'"'"""""'I
},cxxx XX>f
><xxxx >o
P<XXXX :,{
kxxx x �
},cxx xxx
14. (a) �x xxxx
p(X(l<XXXXX
b5'..X.A::z!..¼.X.Al!:

I
< direction of a: towMds the centre ofthe circular arc> [!]
< Magnetic force Fis upwards > (!]
(b) Fis always peq,endicular to the velocity of the electron, [!]
< CurrentJ is anticlockwise when viewed from left> [!]
thus, no work is done and the kinetic energy remains unchanged. [!]
(b) (i) F=mg=BIL OR
(72 x lo-') (9.81) = B (2) (0.0 3) [!] Electron only changes direction while speed remains unchanged, [!]
:. B = 0.0118 T [!] no work is done and the kinetic energy remains unchanged. (!]
DSE Physics - Section D : Question Solution PD-EM4-QS/ 08 DSE Physics - Section D : Question Solution PD-EM4-QS/09
EM4 : Magnetic Field EM4 : Magnetic Field
i
16. (c) F=BQv= mv [I]
18. (a) (iii) ThemagneticfieldatQduetoP:

B - µ.I, [I]
:.v= BQr Q - 21tY

m Magneticforce on Q with length l :


As BQ is constant, if r is halved, v is also ha lved. F = BQlQl [I]
m
= µo Ir -1
:. v=0.6x107ms""1 <accept6xl06ms-1> l
[I] 21tr Q.
Magnetic force per length:

F A IP IQ
Fi =1 =

[I]
17. (a) The rod 'kicks' to the right. [I]
The rod then leaves theliquid and the circuit is not complete.
(iv) The two forces form. an action and reactionpair, [I]
Therefore, current stops flowing and the rod swings back t o the original position. [I] thus they are equal inmagnitud e. [I]
The above process then repeats so the rod continually kicks out and thenreturns. [I]
(b) (i) As current passes inthe same direction between two adjacent wires, [I]
(b) (i) Moment=Fxd the wires attract each other, thus the solenoid is compressed. [I]
:. (7.2 X 10-4) = px (0.06 + 0.03) [I]
(ii) Currents between two adjacent wires always flows in the same direction at any instant, [I]
:. F = 8 x 10-:i N [I] thus, the solenoid will always be compressed.

(ri)ByF=BIL

(8 x 10- 3) = B (3.2) x (0.06) [I]


B = 0.0417T <accept0.042T> (l]

�---s ---------c
(c) (i) �server's eye
P

'-=HI[
bat tery

(ii) The rod rotates in anticlockwise direction. [I]

18. (a) (i) The magnetic field at Q due ot P points out ofthe paper. [I]

(ii) [I]
Hong Kong Diploma of Secondary Education Examination DSE Physics - Section D : M.C. PD-EM5-M/Ol
Physics - Compulsory part (;t,iH1'1l-) EMS : Electromagnetic Induction
Section A -Heat and Gases (#.:::fl'�fflt)
1. Temperature, Heat and Internal energy (�Lt , $'fl' � �)
2. Transfer Processes (#.:#$.i/!.U.) The following list of formulae may be found useful:
3. Change of State (m.669tt �)
/:J.(J)
4. GeneraIGasLaw (f--l&Att�#) Induced e.m.f. c""N
S. Kinetic Theory ( �7:!l$'Jtj,) 6t
Section B -Force and Motion (}]ift,i(t;) Force on a current-carrying conductor in a magnetic field F=Bllsin0
1. Positio n and Movement (-fir..:f:f,,$-t;)
2. Newton's Laws ( 4-.tfPt#)
Magnetic field due to a long straight wire
3. Moment of Poree (:h,1£)
4. Work,EnergyandPower(ft.>Jli, �:i:fv.>h4l-)
5. Momentum (to:IJ Magnetic field inside a long solenoid
6. Projectile Mo tion (t.!!f:t�j/J )
7, CircularMotion(OOffil�t;))
8. Gravitatio n (31 71)
1
Section C - Wave Motion (it.1») Use the following data wherever necessary:
1. Wave Propagation (ilt#,$:i@.)
2. WavePhenomem(.!t:-.!R.l.) Permeability of free space µ, = 4n x 10-7Hm- 1
3. Reflection and Refraction of Light (;\:.69.&.M.&.#M)
4. Lenses (iUt) Charge of electron e = 1.60x 10- 19 c
5. Wave Nature ofLigh11 (;;lc.#,/lt:li,.'/fo/i.) Electron rest mass m0 = 9.11 X lQ-ll kg
6. Sound ( *i-)
Section D - Electricity and Magnetism ( 11::fu@) Acceleration due to gravity g = 9.8Ims-2 (closetotheEarth)
1. Electrostatics C*11t�)
2. Electric Circuits (ilt4)
3. DomesticElectricity(t':-®-Jll'lt)
4. MagneticField(.Jllf:Jf-) Part A : HKCE examination questions
5. Electromagnetic Induction ( 11t.@H)i�)
6. Alternating Current ( ;t1:t {t)
1. < HKCE 1981 Paper II - 34 >
SectionE -Radioactivity and Nuclear Energy (1.t:MlR.�,f<>;/'i/t)
I. Radiation and Radioactivity ($£1/t;ftt.a,ttl'J!.t.) Which of the following statements concerning the generator shown in the
:figureis/are correct?
2. Atomic Model ()!j. 'T.ffl;1'.!)
3. NuclearEnergy(:ltht) (1) The direction of the current through the load reverses periodically.
(2) The maximum magnitude of the current depends on the speed of
Physics -Elective part (±aft--��*) rotation of the coil.
Elective 1 -Astronomy and Space Science (3'tX."f,>fl';lln:3't#*) (3) The rnaxhnum magnitude of the current depends on the resistance
1. The universe seen in different scales (;;i- J;;J 1t r.rn�& Til9 ':f" 'iii imtt) oftheload.
2. Astronomy through histozy (Jt:lt �il91i-Al.ll:)
A (I) only
3. Orbital motions undergraVity(:!thTil9�ia.lU&)
B. (3) only
4. Stars and the universe (•IJ.£.,f,,*W) C. (1) & (2) only
Elective 2-Atomic World (,l!'tTif--n.) D. (2) & (3) only
1. Ruthetford's atomic model (li:;E;/8,)!j.7"��)
2. Photoelectric effect (;le 111:tl:t.4)
3. Bohr's atomic model ofhyrlrogen (Jtii lro i.&7"#&1'.!) 2. < HKCE 1983 Paper Il- 33 >
4. Particles or waves (�'T�.it)
5. Probingintonanoscale(»l,�h,;i/<1!t$)
Elective 3 -Energy and Use of Energy (Jl5:!tfP"�E.ii.€r:1-ftM)
I. Electricityathome(*,§,ffl'lt) e------------lN_ sI
2. Energy efficiency in building (¢11fil9��tt.$)
3. Energy efficiency in transportation (il#r#ilro Jlt�tt,ll.)
The figure shows a bar magnet moving along1he diameter ofa metal ring. Which ofthe following will happen?
4. Non-renewable energy sources (;;i-�*!U'6�)
5. Renewable energy sow:ces (� *.i.Jlt�) A. An induced current will flow in a clockwise direction in the ring.
Elective 4 - Medic3l Physics ( i}�4tg�) B. An induced current will flow inan anticlockwise direction in the ring.
1. Making sense of the eye (11.tU{l � 't) C. An altcmatingcurrent will be produced in the ring.
2. Makingsenseoftheear(.lf��'t) D. No induced current will be produced
3. Medical imagingusingnon-iocizingradiation U�'llt,tfE.fiRH}c'f>Wil!f>)
4. Medical imaging using ionizing radiation ( 11!:lllft-tiMU-<'ffe��lj/:)
DSE Physics - Section D : M.C. PD-EMS-M/021 DSE Physics - Section D : M.C. PD-EMS-M/03
EMS : Electromagnetic Induction EMS : Electromagnetic Induction
3. <BKCE 1984 Paperll- 25 > 7. <HKCE 1989 Paperil-35>
Which of the following devices is designed to convert mechanical energy into electrical energy? A>-
A. dynamo
B. transformer
C. motor
D. electric cell

4. < HKCE 1985 Paper n- 42 > �


A copper rod AB is free to move on two parallel conducting wires. It is pulled D-�----l"A____
by a force F and moves in the direction shown in the figure. The direction of
the magnetic field points into the paper. Which of the following statements X X X X Figure{l) Figure (2) Figure (3)
is/are true? p Q --+F
(1) A current will flow from CtoDthroughP. A magnet is allowed to fall through a copper ring. What is the direction of the induced current (if any) on the ring observed
(2) A current will flowfromB to A through Q. X X X X by the eye when the magnet is in the position as shown in Figure (1), (2) and(3) '1
(3) An induced voltage will be set up across AB. Figure (2)
A. (2) only C B Figure (1) Figure (3)
B. (3) only A. Clockwise No current Anticlockwise
C. (1) & (3) only B. Anticlockwise No current Clockwise
D. (2) & (3) only C. Anticlockwise No current Anticlockwise
D. Anticlockwise Anticlockwise Clockwise

••
5. < HK.CE 1987 Paper II - 34 >

-
< HKCE 1990 Paper II- 36 >

[Q
Direction
X y of motion

""""' Figure 2
In Figure 1, the galvanometer deflects to the left with a reading of IO µA. The north pole of the same magnet is now pushed
towards the coil from the other side at a faster rate as shown in Figure 2. The deflection on the galvanometer will be
A light coil of wire XY is suspended by insulating string such that it can swing freely. A solenoid connected to a cell with a A. more than IO µA to the right.
switch Sis placed near to the end Y ofthe coil as shown. What would happen to the coilXYjust when the switch S is closed ? B. less than 10 µA to the left.
A. The coil moves towards the solenoid. C. more than 1 O µA to the left.
B. The coil moves away from the solenoid. D. less than IO µA to the right.
C. The coil would move down.
D. The coil would move up.
9. < HKCE 1990 Paper II - 39 >
6. < HKCE 1989 Paper ll - 42 >

51ill

5
The figure shows the display on a CRO with the time base set at 1 ms cm-1 and Y-gain at 0.5 V cm-1. The peak voltage and 10@1
frequency of the a.c. voltage applied across the Y--plates are
peak voltage frequency
A. 1.5V S00Hz
In the figure above, the peak current through the resistor is
A. 2mA
= Y-gain IV cm-1

B. UV 250Hz B. 4mA
C. 3.0V 50Hz C. 8mA
D. 3.0V 2S0Hz D. 4A
. DSEPbysics-SectionD: M.C. PD-EMS-M/04 DSE Physics - Section D : M.C. PD-EMS-M/051
EM5 : Electromagnetic Induction EM5 : Electromagnetic Induction
10. <HKCE1993Paperil-34> 13. < HKCE 1994Paper II- 36 >

-E-- SofHron core

soft iron core

'-----C=J------"
coil
R
p Q
R
Two coils of conducting wires are wrapped on a soft-iron core as shown in the above figure. Switch Sis closed and after a pL--C:::J-___JQ
while re,.opened. Which of the following statements is/are true?
A magnet is initially placed between the ends of a soft iron core as shown above. The magnet is then quickly rotated
(1) At the moment when Sis closed, a current flows through R fromP to Q.
clocl:cwi3e through one complete revolution. Which of the following statements correctly descn"bes the induced Cllirellt
(2) When S remains closed, there is no current flowing through R.
(3) At the moment when Sis re-opened, a current flows throughR fromP to Q. flowing through the :resistorR?
A. (1) only A. The current flows throughR fram.Pto Q, andthen reverses its direction.
B. (3) only B. The current flows through RfromQ to P, and then reverses its &rection.
c. (!) & (2) only C. The current flows through R from P toQ.
D. (2) & (3) only D. The current flows tbroughRfromQtoP.

11. <HKCE1993Paperll-37> 14. <HKCE1994Paperll-32>


Which ofthe following correctly shows the major enetgy change in the device when it is working?
Device From To
A A microphone electrical soond
B. A loudspeaker sound electrical
C. Adynamo electrical electrical
D. Amotor electrical mechanical

15. < HKCE 1994 Paper ll- 37 >

The above diagram shows a simple generator. Which of the following graphs below shows the time variation ofthe voltage
The diagram shows the trace ofa signal on a CRO with the time base switched off
The Y-gain is set at 1 V cm·1• Which of the following statements correctly describes i-:tt-�ir.1t;:f ��� ! 1 cm
;::�"?� -t ��
produced by the generator? the input signal?
.J.w..;.. ·,·
A. B. A. It is an a.c. ofpeak voltage 1 V.
voltage

P�v
B. Itis an a.c. ofpeak voltage2 V.
C. Itis ad.c. of constant voltage 1 V.
D. It is a d.c. of constant voltage 2 V.
0

16. < HK.CE 1995Paper ll- 37 >


c. D. voltage The diagram shows the display on a CRO with the time base at IO ms cm·1
and Y-gain at 0.5 V cm·1• Find the peak voltage and frequency of the

,rv/b
signal applied across the Y·plates.
(\ Peak voltage / V Frequency/ Hz
A. 1 16.7
B. 1 25
c. 1 50
12. < HKCE 1994 Paper Il- 31> D. 2 25
Which of the following statements about direct current (de.) and alternating current (a.c.) is/are correct?
(I) All d.c. sources produce constant voltages. 17. <HKCE1996Paperll•3l>
(2) The direction of current in an a.c. circuit changes with time.
(3) Both d.c. and a.c. produce a heating effect in a resistor. Which of the following devices is not an application of electromagnetic induction?
A, (!) only A. a bicycle dynamo
B. (2) only B. a magnetic tape playback head
C. (!) & (3) only C. amoving coil microphone
D. (2) & (3) only D. a moving coil loudspeaker
DSE Physics - Section D : M.C. PD-EMS-M/06 DSE Physics - Section D : M.C. PD-EMS-M/07
1
EM5 : Electromagnetic Induction EMS : Electromagnetic Induction
18. < BKCE 1997 Paper II- 36 > 21. <HKCE 1998 Paper II- 34 >
X X X X X X
Metal rod
p X X X X X X Upwwds

X X X X X X
j____.,.
l
Left +- Right
X X X X X X

G Downwards
Induced current

In the � e circuit, the terminal Q and the sliding contact R of the variable resistor are connected to the f-input of a In the above diagram, a metal rod is placed inside a magnetic field pointing into the paper. In which direction should the rod
CRO. IfR ts moved towards P, how would the amplitude and period of the trace displayed on the CRO be affected ? be moved in order to produce an induced cmrent as shown in the diagram ?
A. into the paper

"""""
Amplitude of the trace Period of the trace B. out ofthe paper
A. remains unchanged C. upwards
B. increases increases D. downwards

====� ..
c. decreases remains unchanged
D. decreases decreases
22. < HKCE 2000 Paper II- 38 >
A bar magnet is placed near a solenoid. Which of the following correctly shows the direction of the induced current in the
0 Metalring

n:
19. < HKCE 1997 Paper Il- 34 >
solenoid ? (Note : The arrow " + " indicates the direction of motion ofthe magnet or solenoid.)
B.
A.
j .. ...

� �
I 4>--1 ill�
L@-J'

· n: · n: n: · n: w,
� metal ring is released and falls vertically around a magnet as shown in the above diagtam. Which of the following C. D.
diagrams correctly descnOes the directions ofthe induced current, if any, in the ring at positions Xand f?

C
23. <HKCE2001P3perll-35>
20, < HKCE 1997 Paper ll- 37 >

Load
long solenoid is placed in a vertical position and its two ends are connected to the Y-input of a CRO (with the time base
A
The diagram shows an electricity generator connected to a load. Which of the following can increase the voltage produced switched on). A bar magnet is released above the solenoid so that it falls through the solenoid. Which of the following
by the generator ? figures best represents the trace shown on the CRO ?
(1) Rotating the coil at a greater speed B.
(2) Reducing the resistance oftbe load
(3) Replacing the coil with one of larger area
• '' H-f
'
' '
�j,4\-J
A. (1) only

1±1:' '
B. (2) only
'
C. (1) & (3) only
D. (2) & (3) only '
. DSE Physics - Section D : MC. PD-EM5-M/08 DSE Physics - Section D : M.C. PD-EMS-M/091
EM5 : Electromagnetic Induction EMS : Electromagnetic Induction

□➔
24. < HKCE 2003 Paper II - 37 > 25, < BKCE 2004 Paper II - 35 >
A xxxxxxxxxxxx B
xxxxxxxxxxxx ,-­
'
'''
xxxxxxxxxxxx
xxxxxxxxxxxx
''
xxxxxxxxxxxx
xxxxxx:xxxxxx
xxxxxxxxxxxx
xxxxxxxxxxxx
'
R xxxxxxxxxxxx
uniform magnetic field
pointing into the paper

A rectangular coil is moved with a uniform speed from position A to position B as shown above. 'Which of the following
graphs represents the variation of the current induced in the coil 'With time?
A. B.
I I
Two coils Pand Qare wound on a soft-iron core as sho'Ml. above. Switches Sis closed and then opened again. Which of the
foUowing shows the directions of the induced cun:ent flowing through the resistor R?
At the Instant Sis closed At the instant Sis opened again

C. D.

B.

R
26. < HKCE 2004 Paper ll- 36 >

C.

IJ IJ;. -
D. The diagram shows a generator connected to a load. Which of the following statements is/are correct?
(I) The generator produces an alternating cumn.t through the load.
·_,:.:\''. (2) At the instant shown, a current is flowing through the load fromPto Q.
R ::-::,:;::/. .·. R (3) The current produced reaches a maximum when the coil is vertical.
,•,,,;-·
,, ...
,c•,:··"''
_-; __
·'-•' •'
A.
B.
(1) only
(3) only
:>,s,,
(: C.
D.
(l)and(2)only
(2) and (3) only
DSE Physics - Section D : M.C. PD-EMS-Mi 10 DSE Physics - Section D : M.C. PD-EMS-M/11
EMS : Electromagnetic Induction EMS : Electromagnetic Induction

27. <BKCE2004Paperll-38> 30. < HK.CE 2007Paper Il- 40 > metal rod
Voltage per cm Time per cm /
xxsxxxxxx Pxxxxxxxxxxxxx
xx�"''"""''""•"'cx-,r.<,<cx-;n<,<,_x-,"'-- metalrail
xx xxxxxx xxxxxxxxxxxxxx
xx xxxxxx xxxxxxxxxxxxxx unifoxm:�gn�
xx xxxxxx X X XXXXXXXXXX X X - eldpomtmgmto
c

xx xxxxx x xxXxx XX XX XX X x -- ! epaper


xx xxxxxx x·xxxxxxxxxxxxx
The photographs show the 1race ofa signal on a CRO and some settings of the CRO. The voltage gain is set at 0.5 V per cm
xx xxxxxx xxxxxxxxxxxxxx
and the time-base is set at 10 ms per cm. Find the frequency and peak voltage of the input signal
xx xx xxxxxxxxxxxxxx
xx Qxxxxxxxxxxxxx
Frequency / Hz PeakVoltage/V
A 10 2 Under an external force, a metal rodPQ is moving with a uniform speed v on a metal rail placed in a uniform magnetic field
B. 10 4 as shown above. Whic.h of the following descriptions is/are correct?
C. 20 2 (1) Current flows through the galvanometer fromQ to R.
D. 20 4 (2) The metal rod will accelerate to right ifthe direction of the magnetic field is reversed.
(3) The pointer of the galvanometer will deflect to opposite drrection if the direction of the magnetic field is reversed.
A. (1) only
28. < HK.CE 2005 Paper n- 42 > B. (3) only
C. (1) & (2) only

:1: : i--
)C " " " )( >< X >< >< >< X X X >< ><

: xCx " " "-B>< D. (2) & (3) only


c�� x

�,ex X
><� ➔ 31. < HK.CE 2007 Paper Il- 42 > direetioo. of
rotation
D ><£Y- " " "A" ><D>< "
,/0�
)C � )C

>< ><A>< A
" " " )C

positionP positionQ positionR


A conducting rectangular coil ABCD is moved across a unifonn magnetic field pointing into the paper as shown above.
Which of the following statements is/are conect 7
,.'
,/ N
(1) The induced current flows from A to C through B when the coil is at P. / coil
(2) The magnitude of the induced current is the largest when the coil is at Q.
,.
(3) The direction of the induced current when the coil is atR is the same as that when it is at P.
A {I) only
B. (3) only
C. {I) & (2) only
D. (2) & (3) only
X ""' y

The above figure shows a simple structure ofa d.c. generator. 'Which of the following statements is/are correct?
(1} The CUIIent delivered to the load is constant
29. <BKCE2007Paperil-43>
(2) The current generated in the coil is alternating, but the current delivered to the load is unidirectional.
(3) The current flows through the load fromXto Y.
A. (I) only
B. (3) only
C. (1) & (2) only
closed-loop D. (2) & (3) only
copper coil

32. < HK.CE 2008 Pap:er Il- 40 >


In the figure, a rectangular coil is pulled to the right with unil'orm. speed in
CiiiliJ a uniform.magnetic field pointing into the paper. Which of the following
xxxx
descriptions about the current induced in the coil and the magnetic force
x x xx
electronic balance acting on the coil is correct ?
x x xx
A. No current is induced in the coil and no magnetic force is acting on
XXX X --+- right
An electronic balance reads X when a closed-loop copper coil is placed on it as shown above. A bar magnet drops from the coil.
xxxx
cenain height vertically above. Just before the magnet reaches the coil, the reading of the electronic balance is B. A cunentis induced in the coil but the rcsultmrtmagnetic force acting
XX XX
A. the saJJle as X on the coil is zero.
xxxx
B. first smaller tban.X and thengreaterthanX C. A current is induced in the coil and a resultant magnetic force is acting
xxxx
C. smallerthanX on the coil to the left.
D. greaterthanX D. A current is induced in the coil and a resultant magnetic force is acting on the coil to the right
33, <BKCE2008Paperll-39>
DSE Physics - Section D : M.C.
EMS : Electromagnetic Induction

-----1,-'o-i-- ✓o7 �A"""",1.--Y


.
PD-EMS-M/12

11
36. <BKCE2010PaperII-42>
, DSE Physics - Section D : MC.
EMS : Electromagnetic Induction

A copper ring R is falling through a solenoid along the ax.1S as showtt. The
PD-EMS-M/Bll

� observer's eye

� '/
ODTL\
� soft iron rod plane of the ring is kept horizontal throughout The solenoid is carzying a
' ;..\,..\, .\,-- "" steady current I. Yis the centre of the solenoid c:::t==>R
4
-W
�J Which of the following combinations about the directions of the induced IX
coil A coilB current in the ring (if any) atXand at Y as observed from O is correct?
An experiment about electromagnetism is designed as shown above. Coil B has a greater number of turns th:m coil A. Coil atX at Y
Bis connected to a sensitive ammeter. Which of the following statements is/are correct?
(1) 'When the switch is closed, a steady Clltrent flows through the a.mmeter. A. clockwise clockwise
(2) At the moment the switch is opened, a current flows through the ammeter momentarily. B. clockwise no current
(3) If the soft iron rod is replaced by a glass rod, the ammeter will have a greater deflection at the moment the switch C. anticlockwise anticlockwise
is opened. D. anticlockwise no current
A. (I) only
B. (2) only
C. (1) & (3) only 37. <BK.CE 2011 Paper II -44>
D. (2) & (3) only X X X X X X X ' X X X X X X X X

X S p< ' X X ' X X X ' X X X X X ' metal rail


34, < HKCE 2009 Paper II- 40 > X X X X X X X ' X X X X ' X X uniform magnetic
field pointing :into
V
X X X X X X X X X X X X X X X the paper
X k ' X X X ' '
metrurod
X X X X X X X X

X X X X X X X X X X ' X X X X X

Q
A metal rod PQ is moving with a uniform speed v on a metal rail placed in a uniform magnetic field as shown in the figure
above. A resistor is connected across RS. Which of the following descriptions is/are correct?
(1) Kinetic energy of the rod is converted into electrical energy.
(2) CWTent flows through the resistor from S to R.
(3) The induced cunent will be reversed ifthe rod moves in the opposite direction.
A. (!) only

·.rv. ·.�. ".�. ·.�.


The :figure above shows the position of the coi l in a generator at time t = 0. The current is taken to be positive when it flows B. (3) only
from P to Q through the load. Which oftbe following graphs best represents the variation of current/with time t as the coil C. (1) & (2) only
rotates ? D. (2) & (3) only

Part B : BK.AL e�aminaiion questions

38. < HKAL 1980 Paper I - 23 >

- w -�- - - - -�-
The magnetic flux through a coil ofN tums increases at a uniform rate from zero to 11> in time t. What is the magnitude of
the e.m.£ induced in the coil ?
35. < BKCE 2009 Paper II- 39 > A. N(J)t
X B. (J)t/N
Q
y pillillill]_
(9 N(f)/t
magnet -----------� D. Nt/(J)

When a magnet is moving towards end for a solenoid, it is found that a north pole is induced at end Q. As shown in the
39. <HKAL 1982 Paper I - 25>
figure above, the magnet passes throughQand moves away from the solenoid, what are the polarities of end P of the solenoid
and endX of the magnet?
Sis a long solenoid. L1 is a wire loop just inside the solenoid, and
14 is a wire loop just outside the solenoid The current in the solenoid s
is increased at a steady rate, such that the e.m.£ induced in Li
poJarity of end P polarity of end X .is--:\_.2 V. Find the e.m.f. induced in 4
A. s s ¼J av. L,
B. s N B. 0.6V. �£30
C. N s c. 1.2V.
D. N N D. 2.4 V.
40.
DSE Physics - Section D : M.C.
EMS : Electromagnetic Induction
< BKAL 1983 Paper I - 21 >
PD-EMS-M/14
1
I
45.
DSE Physics - Section D : M.C.
EMS : Electromagnetic Induction

< HKAL 1987 Paper I- 38 >


P D -EM M
5- / 15
I
Rectangular coil PQRSis driven with constant velocity towards the right in a uniform X X X X
magnetic field directed peq,endicular into paper as shown in the figure. Which of the

Q R
following statements is correct at the instant shown in the figure 7 X X X ..:_..
A. Toe magnitude of the magnetic flux through the coil changes with time.
B. An induced cunent is flowing in the coil in the anticlockwise direction. X X X X
C. An electromagnetic force acts on the side PQ in a direction opposing its motion. p C]s
D. There is no induced current flowing in the coil. X X X X

A uniform magnetic field acting perpendicular into paper is inside a circular region of radius 8 cm. A circular loop of radius
41. < IIKAL 1984 Paper I- 27 > 1 0 CJXl is placed outside the field region as shown in the figure. If the magnetic field is now decreasing at a constant rate of
0.01 T s-1, what will be the magnitude and direction of the induced e.m.f. in the loop ?
Magnitude Direction
A. 2.0 x 10 -4 V fromA to Bvia the loop
B. 2.0 x 10 -4 V fromB to A via the loop

��--;Q���g-, " Q
A large copper disc mounted on a horizontal axle is spun in the clockwise direction between the poles of a horseshoe magnet. C. 3.1 x 1Q-4V fromA to B via the loop
D. 3.lxlo-'V from B to A via the loop

46. < HKAL 1988 Paper I - 42 >

42. <HKAL 1984 Paperl-22 >


A solenoid bas a length of 0.30 m and cross-sectional area of 3.2 x 10-1 Dr. There are 1000 turns of wire wound on it.
When the solenoid carries a current of 1.5 A, the magnetic flux through the solenoid is
A rectangular wire frame is placed outside a uniform magnetic field which is confined to a square region as shown in the

·.lL, ·,�, '.lJL, '•hr'


A. 6.0 x lo-7 Wb.
B. 2.0 x 10-6 Wb. figure. The direction of the magnetic field is perpendicular into paper. If the wire frame moves to the right with a uniform
C. 5.7 x lo-4 Wb. velocity, which of the graphs below best represents the variation of the induced current I with time t ?
D. 2.0 x 10-3 Wb. (The anti-clockwise direction of the current is taken as positive.)

43. < BKAL 1984 Paper I - 23 >


A sinusoidal voltage is gen erated by an a.c. generator. If the speed of rotation of the coil is increased, what will happen
to the frequency and the peak voltage generated ?
frequency peak voltage
A. increase no change
B. no change m=
C. :increase decrease 47. < HKAL 1989 Paper I - 36 >
D. increase increase

44. < BKAL 1986 Paper I - 37 >

6 Two rectangular wire loops P and Q are placed in t he same plane side by side. Loop P includes a battery and a switch S,
A short bar magnet moving with uniform velocity passes through an air-cored solenoid of connected to a galvanometer as
which is initially open. IfSis suddenly closed, what is the direction of the induced current in loop Q? Are the magnetic

·.�, ·,�, ·.6-L, . ,b_,


shown. Which of the fullowing graphs best represents the variation of the current[ in the solenoid with time t?
forces between the two loops attractive or repulsive ?
Direction of induced current Nature of force
A. clockwise attractive
B. clockwise repulsive
C. anticlockwise attractive
D. anticlockwise repulsive
DSE Physics - Section D : MC. PD-EMS-M/16 DSE Physics - Section D : MC. PD-EMS-M/17
EMS : Electromagnetic Induction EMS : Electromagnetic fuduction

48. <BKAL 1989 Paper I- 3S > 52. < HKAL 1995 Paper IlA - 35 >
A certain length of a copper wire is bent to fonn a circular coil of one tarn. The coil is then placed in a uniform magnetic A bar magnet is moved perpendicularly towards a copper disc. Which of the folloWUlg statements are correct ?
field with its plane normal to the direction of the magnetic field. The flux linkage through the coil is <I>. The same length of (1) Eddycunent is induced in the copper disc.
wire is now bent to form a double loop of smaller radius. The flux linkage through the coil would become
(2) Temp erature ofthe copper disc incxcases.
A $/4.
(3) A repulsive force is experienced by the magnet.
B. ¢12.
C. ¢. A. (1) & (2) only
B. (I) & (3) only
D. 20. C. (2) & (3) only
D. (1), (2) & (3)
49. < HKAL 1992 Paper I- 38 >
X
53. < HKAL 1997 Paper IIA- 23 >
X

X Q X X X

A metal rodPQ moves with constmt velocity across a uniform magnetic field directed perpendicularly into paper as shown in
the above figure. A voltage is induced across the rod. Which of the following statements is/are correct?
{I) The magnitude of the voltage depends on the length of the rod
(2) PointP is at a lower potential than Q.
(3) A magnetic force is acting on the rod to oppose its motion.
A (I) only The above figure shows a small coil, connected to a light beam galVSilometer, placed in a region of uniform magnetic field
B. (3) only between the poles of a magnet. The plane of the coil is parallel to the pole faces. Which of the following actions would
C. (1) & (2) only produce a deflection of the galvanometer?
D. (2) & (3) only
(I) Moving the coil to and fro between the poles.
(2) Moving the coil away from the region between the pole faces.
50. < HKAL 1992 Paper I - 40 >
(3) Rotating the coil about a diameter through an angle of 180°.
A. (I) only
B. (3) only
C. (I) & (2) only
D. (2) & (3) only

-
54. < B]{AL 1997 Paper llA - 31 >
X X X X
An aluminium ring floats above a coil cany:ing alternating current. Which ofthe following will affect the height of the ring?
(1) theresistivityofthering A B
X X X X
(2) the density of the ring
(3) the frequency ofthe alternating current X X X X
A. (1) & (2) only
B. (1) & (3) only X X X
C. (2) & (3) only D
D. (1), (2) & (3) X X X
l X
A rectangclar m etal wire :frameAJJCD moves to the left with a uniform speed across a region ofunifor.m magnetic field acting
51. < HKAL 1994 Paper IIA- 37 > perpendicularly into paper. Which of the following is/are true at the instant shovm in the figure ?
The magnetic flux linkage for a coil placed in a urtifor.mmagnetic field depends on (1) A current is flowing in the clockwise direction.
{I) the resistance ofthe coil (2) The electric potential atB is higher than that at C.
(2) the mmiber oftl:l:llls of the coil
(3) the angle between the nor:mal of the coil and the direction of the magnetic field (3) The side AD experiences a magnetic force acting to the right
A. (I) only A. (1) only
B. (3) only B. (3) only
C. (1) & (2) only C. (1) & (2) only
D. (2) & (3) only D. (2) & (3) only
DSE Physics - Section D : M.C. PD-EMS-Mi 181 DSE Physics - Section D : M.C. PD-EMS-M/19

□S
EMS : Electromagnetic Induction EMS : Electromagnetic Induction
55. < BKAL 1999 Paper IIA- 36 > I 59. <BKAL2003Paper�-34>
Loft Right
A coil ofmetal wire is placed on a plane perpendicular to a uniform magnetic field. ,.!) 180°
The coil is rotated through 180° about a diail1eter as shown. The induced e.m.f. in X X x' X X I
the coil is independent of p
A. the area of the coil. X X
-' ►
B. the flux density of the magnetic field. X X
C. the nu:mberoftu.rnsofthecoiL
D. the resistance of the coil. X x
l
X X Q R
.,,,,
The figure shows a rectangular metal coil PQRS moving from left to right with a unifonn speed across an insulated metal
56. < HKAL 1999 Paper wire canying a steady cuttentl. Which of the following gives the correct sequence for the direction of the current induced in
IIA -::stra
ight the coil PQRS ?
A. Clockwise and then anticlockwise
wire canyizzg search
B. Anticlockwise and then clockwise
ana.c. current f6-�;2-� C. Clockwise, then anticlockwise and finally clockwise again
---<:l'['il==::l;i=t
To CRO D. Anticlockwise, then clockwise and finally anticlockwise again
:;

A long straight wire canying an a.c. cunent lies on the plane of the paper as shown. A sear ch coil connected to a CRO with
the time-base off is used to measure the peak value of the magneti c field produced by the a.c. current When the search coil is 60. < HKAL 2004 Pape r IIA- 28 >
placed atB, the length ofthe trace on the CRO is 2 cm. If the search coil is placed at A, the length of the trace would be A search coil is placed at one end of a solenoid as shown in the figure. The solenoid is connected to an a.c. somce so that an
A. 0.5 cm. a.c. cw:rent is flowing in the solenoid. The induced voltage on the search coil is shown on the CRO connected to the search
B. 1 cm. coil.
C. 2cm. solenoid
CRO

0
D. 4cm.

57. < HKAL 2002 Pap er IIA- 27 >


A metal wire oflength.L is inclined at angle 60° to railPQ as shown. X X X X X X
It is moved rightwards with a uniform velocity v across a uniform
magnetic field directed into paper along the two horizontal railsPQ p Q
and RS. The strength ofthe magnetic field is B. What is the e.m.f. X X X X X
induced in the rod ?
A. � Which of the following changes will NOT affect the amplitude of the induced voltage in the search coil?
2 X X X X X A. Increase the frequency of the a.c. current in the solenoid.
B. BvL B. Increase the number ofturns on the search coil..
R C. Place the search coil in the middle of solenoid, without changing its orientation.
c. 2BvL @ Replace the solenoid with one of greater cross-sectional area while keeping the same magnitude of a. c. current.
7, X X X X X X
✓3B vL
D.
2 61. < BKAL 2005 Paper IlA - 19 >
A rectangular coil is placed adjacent to a straight wire MN cazzying a Cllll'Cllt I which varies with time t as shown in the

□ □
58. < HKAL 2002 Paper IIA- 28 > X X X X X
graph. The wire MNis in the planeof the coil and the current[is positive (+) when it is in the direction fromMtoN.
N I
X x unifonn magnetic field
/ into paper
X X

X X
T/2
T
X X X X X

A circular frame and a square frame, made from the same type of metal wires, are placed in a uniform magnetic field as
shown. The length of each side of the square is equal to the diameter of the circle. When the flux density of the magnetic M
�d is increased at a steady rate, find the ratio of theinduced cunent in the cll"cular frame to that in square frame. Starting from t= 0, how does the direction oftbe current induced in the coil vary in one period?
'� 1:1 A. Clockwise first and then anticlockwise
B. 1 :1t B. Anticlockwise fust and then clockwise
C. 1t:4 C. Clockwise -+ anticlockwise ➔ clockwise ➔ anticlockwise
D. 2:n D. Anticlockwise ➔ clockwise ➔ anticlockwise ➔ clockwise
DSE Physics - Section D : M.C. PD-EMS-M/20 DSE Physics - Section D : MC. PD-EMS-M/21
EMS : Electromagnetic Induction EMS : Electromagnetic Induction

62. < HKAL 2005 Paper IIA- 16 > '. 65. < BKAL 2008 Paper IIA -18 >
X X R X X Q X X Jxxxxx:
uniform p QX :
X l
X X X X X magnetic
1/field X X,
--';>-
X X ;
X X X X X X
X X '
R :
X X s X X p X X xx x xx
)_ __ _ __ __ :
Two conducting rods PQ and RS are placed on two smooth, parallel, horizontal conducting rails. A uniform magnetic field is In the figure, PQRS is a metal plate placed perpendicularly to a uniform magnetic field directed into the paper. At the instant
directed into the plane of the paper as shown. Ifnow rod PQ is given an initial velocity to the right, which of the following showJ'l, the magnetic field is moving to the right and eddy current is induced in the metal plate. Which of the following
statements is NOT correct? diagrams best represents a possible path ofthe eddy current induced in the plate ?
A. The induced cummt is in the direction ofPQRS. A R
B. The magnetic force acting on the rod PQ is towards the left.
:xxxxx-:
,-----------------
'xxxxx:
C. Rod RS starts moving towards the right. '
D. Rod PQ would keep on moving with a uniform speed. p QX : 0 :
X\

X x
�X X,
xi
� f-+
63. <HKAL200SPaperllA-17> X X( X X:

X X• X x:
s R J: ' R
x
: X__ X _ X __
'
;_ X__ X __X __ X__ X
__ X :

: 2n :
c. D.
� -----------: fxxxxx: :xxxxxl
generator p QX X:: p QX :
X \
Two '12 V, 6 W' lamps are operating at their rated values. The internal resistance of the generator is 2 Q. What is the X x,
percentage ofthe electrical power generated by the generator dissipated by the two lamps ? X x,
A 75% X X f-+ X X!

B. 86%
i
X X ' X X(
C. 92%
D. 100%
R
:_ X__ X _ X __ X__ X :
l s ' R
,xxxxx•
'
, ________________ J

64. < BKAL 2006 Paper IlA - 19 >


66. < HKAL 2009 Paper IIA -32 >
X X X X X X X X
A metal wire in shape of a square with each side 15.7 cm is placed in a uniform magnetic field of 0.6 T directi!:d into the
paper as shown. Suppose its shape is now changed into a circle within a time of0.5 s. Find the average induced e.m.f. and

L,
X X X X X X X X
the direction of the induced current in the :frame during this period.
X X X X X X X X X X X X X X X X X X X X
p Q
X X X X X X X X X X X X X X X X X X X X

XO
X X X X X X X X X x x X After0.5 s X X

X X X X X X X X X X X X X X X X X X
X

In the figure, a thin metal rod PQ is placed along the x--direction and it is at right angles to a uniform magnetic field pointing X X X X X X X X X X X
into the plane ofthe paper. In which of the following cases will there be an e.mi. induced along the length ofthe metal rod?
X X X X X X X X X X X X
(1) Rotating the rod about an axis through its centre along they-direction
(2) Moving the rod in thex--dlrection average induced e.m.L / mV direction of the induced current
(3) Moving the rod in the y-<lirection
A. (1) only A 8.1 clockwise
B. (3) only B. 8.1 anticlockwise
C. (1) & (2) only c. 4.0 clockwise
D. (2) & (3) only D. 4.0 anticlockwise
DSE Physics - Section D : M.C.
EMS : Electromagnetic Induction
PD-EM5-M/22
i
Ii DSE Physics - Section D : MC.
EMS : Electromagnetic Induction
PD-EMS-M/23 i
o
67. < HKAL 2010 Paper IIA- 26 > : 69. < BKAL 2010 Paper IlA - 30 >

i
X X X X X 'X X X X X X

'
X X X X X X X X X X
X X X X
X X X X X X X X X
Q Q s X X X X
X X X X X X X J x X R
X X X X
' X X X
L
! sX X
X X X
X X X
X
X
0• X
R
X X X X X X X X X X o·
y In the above :figure, PQRSis a small rectal:!.gular metal frame suspended from a fixed point Oby a plastic string. The frame
X X X X X X X X X X X
is released from the position shown and it swings across a uniform magnetic field pointing into the paper within the dotted
rectangle. Neglect air resistance and friction. Which of the follo'Wing is/are correct '1
In the figure, PQ and R£ are two parallel metal rails with separation L. A metal rod Xf resting on the rails moves with (1) A current is induced in the frame in the direction PQRSP when it is entering the field.
velocity v peipendicular to its length across a uniform magnetic field B pointing into the paper. If the rod makes an angle of (2) The current induced in the frame is at a maximum when it passes 00 '.
30° with the rails, what is the potential difference across Q and S? (3) The direction of the magnetic force experienced by the frame is opposite to its motion when it passes 00 '.
BL, A. (1) only
A B. (1) & (2) only
oos30°
C. (2) & (3) only
B. BLv cos30 ° D. (1), (2) & (3)

yj
C. BL,
sin30° < HKAL2011 PaperIIA- 27 >

D. BLv sin30" S P
-
current-carrying wire
68. < HKAL 2009 Paper IIA-12 > □
X R Q
A Jong straight wire XY canying a steady current lies in the plane of the coil. A rectangular coil PQRS is moving to the right
with constant·speed. Which of the following gives the correct direction of the induced current in the coil and the resultant
magnetic force acting on the coil at the instant shown in the above figure '1
s direction of induced current in the coil resultant magnetic force acting on the coil
A. anti-clockwise to left
B. anti-clockwise "''
C. clockwise to left
D. clock.wise "''
71. <HKAL2011PaperIIA-31>

R
'
A bar magnet is suspended by a spring from a rigid support. It is oscillating above a hollow aluminium cylinder placed below s'
/ uniform magnetic field

its lower end. Which of the following statements is/are correct? p


(Neglect air resistance.) A rigid rectangular conducting loop PQRS is connected to a cell as sho"1n. It is held at rest horizontally within a uniform
(1) The amplitude of oscillation of the magnet remains unchanged. magnetic field which is parallel to the plane of the loop and perpendicular to its side R.S. Which of the following statements
is/are correct?
(2) The force between the bar magnet and the aluminium cylinder is always attractive. (1) The side RS of the loop experiences an upward magnetic force.
(3) Eddy currents are induced in the aluminium cylinder. (2) The loop experiences a turning moment due to the magnetic field.
(3) The magnetic flux linkage through the loop is zero.
A. (1) only A. (1) only
B. (3) only B. (3) only
C. (I) & (2) only C. (1) & (2} onl y
D. (2) & (3) only D. (2) & (3) only
DSE Physics - Section D : MC. PD-EMS-M/24 DSE Physics - Section D : M.C. PD-EMS-M/25
EM5 : Electromagnetic IndUciion EMS : Electromagnetic Induction

72. < HKAL 2012 Paper IIA- 24 > I Part D : BKDSE examination questions
(I) (2)
X X X X X X X X X X X X X X X X . 75. < BKDSE Sample Paper IA- 31 >

X X X X X PX X X X X X X PX X X X R X X Q X X
X X X X X X X X X X X X X uniform
X X X X X X
---- magnetic
X X X X X X X X X X X X ➔ ficld
X X X X X X
X X
Q -- ------------------
0 X X X X X X X
Q -- ------------------
0 X X X X X

X X X X X X X X X X X X X X X X X X s X X p X X

In the above figures, a metal rod PQ making an angle 0 with the horizontal moves with speed v across a uoifonn magnetic The figure shows conducting rods PQ and RS are placed on two smooth, parallel. horizontal conducting rails. A uniform
field pointing into the paper in two different directions shown. What is the ratio of the e.m.f. induced across the rod in magnetic field is directed into the plane of the paper as shown. PQ is given an initial velocity to the right and left to roll
Figure (I) to that in Figure (2)? Which statement is INCORRECT?
A. The induced cur.rent is in the dir ection PQRS.
A. 1:sin0
B.
B. I ' sin 0
2
C.
The magnetic force acting on the rod PQ is towards the left.
Rod RS starts moving towards the right
C. sin B : I D. Rod PQ moves with a uniform speed.
D. sin20: I
76. < HI{DSE Practice Paper IA - 33 >
A square metal frame of side length L is placed inside a uniform
magnetic field B as shown. What is the change in magnetic flux x x f x
x,x x
Part C : Supplemental exercise through the frame when it is rotated about the axis � by 90° and
180° respectively?
73. A solenoid has a length of 0.30 m and cross-.sectional area of 3.2 x I0--4 m2• There are 1000 turns of wire wound on it.
'When the solenoid catties a current of 1.5 A, the magnetic flux linkage through the solenoid is 90° 180°

A 6.0 x 10-1Wb. A. 0 0

B. 2.0 x J0-6 Wb.


B. 0 2BL2
c. BL' 0
C. 5.7 x 10-1 Wb. D. BL' 2BL2
D. 2.0 x 10-l Wb.

] 77. < HKDSE 2012 Paper IA- 31 >


74.
''
:x x x x x x:
''
'
1 X X X X X X 1
:
'1 x x x x x x1
:
''
magnetic field 1 x x x xr
'
x x
'
:x_x__ x_ x_x_x:
X X X xBX X C
SL
A rectangular coil ABCD is placed in a mrifonn magnetic field dµected perpendicular into paper. When the ·coil is set into A square metal frame of length of side L moving with constant velocity v passes through a region of uniform magnetic field
motion, an induced current flows from A to Bas shown. 'Which of'tJie follo\Vlllg statements are correct? of width SL as shown. What is the total time period during which a current is induced in the frame?
(I) Thecoilismov.ingtowardstheleft. "'-, A. L
V
{2) The electric potential at A is higher than that atB. 2L
B.
'
\
(3) There is a magnetic force acting on AB towards the right
A (1) & (2) only c. 3L
B. (1) & (3) only V
C. (2) & (3) only D. 4L
D. (!), (2) & (3) '
DSE Physics· Section D : MC. PD-EMS-M/261 DSE Physics - Section D : M.C. PD-EMS-M/27
EMS : Electromagnetic Induction EM5 : Electromagnetic Induction

l______
78. <BKDSE2012PaperIA-32> I

-
j 81. < HKDSE 2015 Paper IA-24 >


p _____ Q
• .

• � magnetic field
:_ • • oftheEart:h
magnetic field

A copper rod PQ is placed horizontally as shown below. It is released and then falls vertically, cutting across the magnetic
field of the Earth pointing out ofthe paper. Neglect air resistance. Which of the following statements is/are correct?
(1) A voltage is induced across PQ.
Y,
(2) A steady induced CU1:Ient is generated in the rod.
(3) Due to the effect of the Earth's magnetic field, the copper rod falls with an acceleration less than the acceleration A rectangular metal frame is made to rotate steadily about its axisXYin a unifonn magnetic field. At the instant shown, the
due to gravity. frame is in the plane ofthe paper and side Pis moving out of the paper while side Q is moving into the paper. Which
statement is INCORRECT at this instant ?
A. (l)only
A. The induced e.m.t: in the frame is at a maxmrom.
B. (3) only
c. (1) & (2) only B. The induced cuncnt produced in the frame is flowing in anti-clockwise direction.
D. (2) & (3) only C. The magnetic force acting on sideP is in a direction pointing into the paper.
D. The magnetic forces acting on the frame produce a moment opposing the frame's rotation.

79. <BKDSE2013Paper1A-29>
A metal rod DP is rotated about O in a clockwise direction in the plane of xxxxxxxxx 1 82.. < HKDSE 2016Paper IA - 29 >
the paper with a uniform magnetic field pointing into the paper. Which
statement is correct ? xxxx xx x A student uses.a search coil to study the strength of the magnetic field inside a l�ng solenoid which is connected to an a.c.
signal generator set at a certain frequency. The search coil is connected to a CRO with time-base switched of:E When the
A. An induced current flows in the rod from O to P.
xxxxx� xx magnetic field is detected, a vertical trace on the CRO is displayed. Which of the following can improve the accuracy of this
XXXX XX . X experiment ? \
B. An induced current flows in the rod from P to 0.
xxxixxxxx (1) Rotate the plane of the search coil until the length of the vertical trace on the CRO is the maximum.

A
r�
E.m.f. is induced in the rod with end Oat a higher electric potential.
E.m.f. is induced in the rod with endP at a higher electric potential.
xxxxxxxxx
xxxxxxxxx
(2) Increase the signal generator's frequency and use the same cummt as before.
(3) Set the axis of the solenoid along an east-west direction to avoid the effects of the Earth's magnetic field.
A. (1) & (2) only
B. (1) & (3) only
80. <BKDSE2014PaperIA-27>
C. (2) & (3) only
I D. (1), (2) & (3)
N

83. <HKDSE2017h.perIA-28>
A metal rod PQ of length 1 is moving along smooth p
X X X X
T horizontal metal rails X and Y with constant speed v in a X
Figure (a) I Figurt (b)
0 uniform magnetic field of magnetic field strength B pointing
into the paper. The met.al rails X and Y are separated by a X X X X
distance of d and are connected to a resistor of resistance R R d l 1---->v
as shown. Which of the following descriptions about the X X X X
induced current is correct?
y
M magnitude direction X X
A. Blv :fromXto YthroughR
A long straight cunent-carrying wire MN and a rectangular coil PQRS are :fixed in the same plane as shown in Figure (a).
The cunent I is taken as positive when it flov,,s from M to N and it varies with time t as shown in Figure (b). The direction of
the induced current in the coil during the time interval O - Tis B. Blv from Y to X throughR
A. first anti-clockwise and then clockwise.
C. Bdv fromXto Y throughR
B. first clockwise and then anti-clockwise.
R
C. anti-clockwise throughout. Bdv
D. from ftoXthroughR
� clockwise throughout R
DSE Physics - Section D : M.C. PD-EMS-M/28 DSE Physics - Section D : MC. Solution PD-EMS-MS/01

=�-
EM5 : Electromagnetic Induction EMS : Electromagnetic Induction
�•s Matkin sScbemeis pre,pazed Qfflhe maikers' It showd .not be :regardi,das a setof:snodel answers.
·84. <HKDSE2017PaperIA-27> Studen:tsand t:eac: hel::avhoueno· :ib.v , QW.in'the-markingproc.ess are.advised to.lllft:Il)tettbe Matking:Scheme'With care.
A metal disc is rotating about its centre C with constant speed. Part of the metal disc is inside a uniform magnetic fiel
'
pointing into the paper as shown. AD. eddy current flows in the metal disc. 'M.C. Answers
D]. I 1. B 2!. D 3!. D 41. D SI. D
·2. D 12. D 22.f 32. C 42. B 52. D
3. A 13. A 23. A 33. B 43. D 53. D
4. D !4. D 24. C 34. D 44. A 54. A
X X X X X X X s. B JS. A 2S. C JS. D 4S. A 55. D
After which of the following changes will the eddy current increase?
i 6. B !6. B 26. A 36. D 46. D S6. Ii
A. Reverse the direction of the magnetic field B. Increase the strength of the magnetic field
7. B !7. D 27. A 37. D 47. D 57. D
s. C !8. A 28. A 38. C 48. B 58. A
'9. B !9. B 29. D 39. C 49. A S9. C
IO. C 20. C 30. B 40. D so. D 60. D

; 6!. A 7!. D SI. B


C. Apply the magnetic field over the whole metal disc D. Cut several sliis from the metal disc ' 62. D 72. A 82. A
X X X X X 63. B 73. D 83. C
X
64. B 74. B 84. B
X X 65. D 75. D 85. D
X X
X X 66. B 76. D 86. D
X X X X X X X
67. C 77. B 87. D
68. B 78. A
85. < HKDSE 2018 Paper IA -29 >
69. -A 79. D
A stone and a slrong magnet of the same size and shape are released from rest into a hollow aluminium tubing respectively.
Which of1:he following is correct ? Neglect a ir resistance. 70. C 80. D

,,; stone magnet


M.C. Solution
I. D

Q) Since commutator is used, itis a d.c. generator, thus the current always flows in the same direction.
✓ (2) !he maximum current depends on the induced voltage which depends on the rotational speed ofthe coil.

✓ (3) By J = � , nwcimum. magnitude of current depends on the resistance ofR.


R
drops slower reason
A. stone the stone is more massive 2. D
B. magnet the stone is more massive
C. stone There is no magnetic flux passing through the coil,
the magnet induces eddy current in the aluminium tubing
D. magnet the magnet induces eddy current in the aluminium tubing thus there is no induced current in the coiL

There is question in next page


86. <HKDSE 2019 Paper IA-28>

87. <HKDSE 2020 Paper IA-28>

f"alling hollow
magnet aluminium tube

When a small strong magnet falls through a hollow aluminium tube as shown. eddy c!D'mll.s are
induced. Which of the following oom:ctly dcscnl>es the mction cf current induced in the tube when
-viewed by an observer :from above 7

A. clockwise bothllbove and below the -


B. and-cloelcwise both above and below the magnet
C. clockwise above the magnet and anti-clockwise below the magnet
D. anti-clockwise above the magnet and clockwise below the magnet
DSE Physics - Section D : M.C. Solution PD-EMS-MS/02 DSE Physics - Section D : M.C. Solution PD-EMS-MS/03
EM5 : Electromagnetic Induction EMS : Electromagnetic Induction

3. A 9. B
Dynamo (generator) changes the mechanical energy of the rotating coil into electrical energy ofinduced current Peak voltage : Vo "" 2 cm x 10 V cm·1 "" 20 V

4. D
Peakcmrent: 1., = "t =
5
:�o' = 4mA

10. C
✓ (1) When Sis closed, current flows in primary coil to give magnetic field lines directed towards the right.
Current is induced in secondacy coil from P to Q through R
to give magnetic field lines directed towards the left to oppose the change.
(1) By Right-hand .rule, current should flow from D to C thr ough P.
✓ (2) When S remains closed, there is no change of current, thus there is no change of magnetic field,
✓ (2) By Right-hand rule, induced current flows from B to A through Q.
therefore, no current is induced in the secondary coiL
✓ (3) By Faraday's Law, induced voltage (e.m.f) is setup across AB where e = B v 1.
(3) When Sis opened, cummt stops flowing in the primary coil and the magnetic field decreases to zero.
Current is induced in the secondary coil from Q to P through R
s. B to give magnetic field lines directed towards the right to oppose the change.
When the switch Sis closed, current starts to flow in the solenoid
which produces amagnetic field with N-pole at the left hand side. B
An induced current would then flow in coil.XY to oppose the change such that the pole at Y is North. As commutator is used, it is a d.c. generator,
The two solenoids thus experience opposing magnetic forces so the coil would move away from the solenoid. d.c. generator gives a vmying de. voltage (unsteady voltage) as shown in option B.

6. B
12. D
Peakvoltage: Vp=05Vcm-1 x3cm= 1.5V
(1) A d.c. generator is a de. sourcebut it produces varying d.c. voltages.
Period: T = 1 ms cm-1 x4cm = 4ms
✓ (2) All a.c. sources wo uld give out current with changing d irections.
Frequency: f = l.. = _l_3 = 250 Hz
T 4xl0- ✓ (3) Both d .c. and a.c. give heating effect by P"" 12 R.

7. B 13. A
Figure (1): As the N-pole of magnet moves towards the ring, N-pole is induced at the upper end ofthe ring, At the time shown, when N-pole of the magnet moves away from the right hand side ofthe core,
thus the induced current is in anticlockwise direction observed by the eye. S-pole is induced on the right hand side ofthe core, thus induced current flows fromP to Q through.R.

Figure (2): As the magnet is inside the ring, there is no change of magnetic field , thus there is no induced current. After half of a cycle, when N-pole ofthe magnet moves away from the left hand side ofthe core,
S-pole is induced on the left hand side ofthe core, thus induced current flows from Q to P throughR in reverse direction.
Figure (3): As the S-pole of magnet moves away from the ring, N-pole is induced at the lower end of the ring,
thus the induced current is in clockwise direction.
14. D

8.
X A. microphone : sound energy ➔ electrical energy
C
In Figure 1, as the S -pole of magnet moves towards the coil (solenoid) from the left, X B. loudspeaker: electrical energy ➔ sound energy
the left side ofthe coil is induced to be S-pole, X C. dynamo : mechanical energy ➔ electrical energy
and the induced current flows through the galvanometer and d eflects to the left. ✓ D. motor: electrical energy ➔ mechanical energy

In Figure 2, as the N-pole of magnet moves towards the coil (solenoid) from the right,
15. A
the right side of the coil is induced to be N-pole, that means the left side of the coil is induced to be S-pole,
thus the induced current flows in the same direction as Figure l, As the trace of the signal is a line, it is an a.c. voltage.

and as the magnet moves at a faster rate, the induced current is greater and is more than 10 µA. Peak voltage: Vo = 1 cm x 1 Vcnr-1 = 1 V
DSE Physics - Section D : M.C. Solution PD-EMS-MS/04 DSE Physics - Section D : MC. Solution PD-EMS-MS/05
EM5 : Electromagnetic Induction EMS : Electromagnetic Induction

16. B 22. C
Peak voltage = 2 cm x 0.5 V cm- 1 = 1 V A. As they move away from each other, the left side of the solenoid should be induced in N-pole,
1
thus induced current should flow from left to right through the galvanometer.
Period: T = 4cmx lOmscm- = 40ms
B. As the solenoid moves away from the magnet, the left side of the solenoid should be induced in S-pole,
Frequency: f = ..!. = __l_ = 25 Hz thus induced current should flow from right to left through the galvanometer.
T 40x10 -3
✓ C. As the solenoid moves towards the magnet, the left side oftbe solenoid should be induced with S-pole,
thus induced current flows from right to left through the galvanometer.
17. D
D. As the magnet moves towards the solenoid, the left side of the solenoid should be induced with N-pole,
✓ A. A bicycle dynamo gives induced voltage, which is an application ofelectromagnetic induction. thus induced current should flow from left to right through the galvanometer.
✓ B. When a magnetic tape moves across a playback head, a.c. current is induced.
✓ c. When sound wave is incident onto a microphone, the coil is set into vi'bration :inside a magnetic field, 23. A
thus a.c. cur.rent is induced. (i) When the magnet is approaching, an induced voltage is set up in the solenoid.
D. Loudspeaker makes use of the magnetic force produced by the current inside a magnetic field. @ When the magnet is inside the solenoid, no induced voltage is set up.
@ When the magnet is leaving, an induced voltage is set up in the solenoid but in the opposite direction.

18. A
When R moves towards P, the resistance of RQ increases, 24. C

thus voltage increases, amplitude of the trace increases.

Period (or :frequency) depends on the a.c. source, thus it remains unchanged.

s R
19. B

When the ring moves towards the magnet at.X: cunent is induced so that the lower side of the ring is N-pole.

When the ring moves away from the magnet at Y, cunent is induced so that the upper side of the ring is N-pole. I,
By using Right hand grip rule, the induced current in both cases can be determined. When Sis just closed, the current through coilPproduces a magnetic :field downwards and upwards through Q.

In order to oppose the change of magnetic field, coil Q induces a cunent which produces a magnetic field in oPPosite
20. direction (downwards), thus by Right hand grip rule, the current through R is in upward direction.
C
✓ Rotating the coil at a greater speed can increase the rate of change of magnetic flux, When Sis just opened, current through coil P suddenly stops flowing,. thus the magnetic field through Q decreases.
(1)
thus induced voltage by the generator increases. In order to oppose the change of magnetic field, coil Q induces a cut:rent which produces a magnetic :field :in the same
direction, thus the induced current should be opposite to that when Sis closed.
(2) Reducing the resistance can increase the induced CUlTellt,
but the resistance would not affect the induced voltage.
✓ A coil of greater area contains more magnetic flux, thus the induced voltage increases. 25. C
(3)
When the coil just enters the field region, an induced current flows in anticlockwise direction{+).
When the coil moves inside the field region, there is no induced current
21. D Metal rod
x/ When the coil just leaves the field region, an induced current flows in clockwise direction (-).
The induced current in the metal rod flows from left to right. X X X J X

In order to give this induced current, X X X ,J, X X X


26. A
M,t!on
the rod should move downwards, by using Right-hand rule. X X X X X ✓ (1) Since slip rings are used, alternating current is produced.
(2) By rught-band-rule, a current is induced in the coil and flows through the resistor from Q to P.
G
Induced current (3) The current is maximum at the instant shown, but becomes zero when the coil is vertical.
DSE Physics - Section D : MC. Solution PD-EM5-MS/06 DSE Physics - Section D : M.C. Solution PD-EM5-MS/07
EM5 : Electromagnetic Induction EMS : Electromagnetic Induction

27. A 33. B
Since the time base is !Oms per cm and the time for one cycle is 10 cm X (I) When the switch is closed, only a pulse of induced current flows momentarily through the ammeter.
period T = 10 cm x 10 ms cm- 1 = 100 ms ✓ (2) At the moment the switch is opened, an induced current flows through the ammeter momentarily.

Frequency: f = ..!_ = __I__ = 10 Hz X


(3) Since glass rod is not a magnetic material, the magnetic field will become weaker,
T lOOxl0-
3
thus the induced CU1TeDt should be smaller.
The amplitude of the trace is 4 cm and the voltage gain is 0.5 V per cm
.·. Peakvoltage = 4cmxo.svcm-1 = 2v 34. D
At this position, the magnetic field is pointing towards the left from N-pole to S-pole.
28. A
The left arm of the coil is moving upwards.
✓ (1) By us.ing Right hand rule, induced current flows along AB in anticlockwise direction.
By using Right hand rule, the induced euttent in the left ann of the coil is flowing ont of paper,
X (2) There is no induced current in this position.
and then from P to Q through the load, thus the current is positive and maximum at t = 0.
(3) By using Right hand rule, induced Clllreilt flows along DC in clockwise direction.

29. D 35. D
Since the bar magnet is moving towards the coil, :induced current flows to oppose the change. When the magnet is at the right side and moving towards the solenoid,
The top of the coil becomes South-pole due to the induced current. as N-pole is induced at Q, thus endX should be N-pole, as opposing forces should exist between them.
Action and reaction pair then exists between the magnet and the coil. When the magnet is at the left side and moving away from the solenoid,
For the magnet, an upward force acts on it by the coil. as end Yis S-pole which is leaving, endP should become N-pole as attraction forces should exist between them.
For the coil, a downward force acts on it by the magnet
The reading of the balance is then greater due to the dovmward force on the coil 36. D
The top of the solenoid is S-pole and the bottom is N-pole, by Right hand screw rule.
30. B As the ring moves towards the solenoid atX, by Lenz's law, to oppose the change,
By Right hand rule, the induced current in the rod is from Q to P, a current is induced so that the bottom of the ring is S-pole, and by Right hand screw rule,
X
(1)
thus current through the galvanometer is fro� R to Q. the induced current is in anticlockwise direction observed by the eye.

X (2) If the magnetic field is reversed, the direction of the induced current would reverse. When the ring is at Y, as there is no change of magnetic field, thus no current is induced at Y.
However, the opposing force is still towards the left, and the rod should still move in unifonn speed.
37. D
✓ (3) Since the direetion ofthe induced cunent is reversed,
the pointer of the galvanometer should deflect to the opposite direction. X (I) Since the rod is moving with uniform spe� its kinetic energy remains unchanged.
✓ (2) By Right hand rule, the induced C1UTent flows from Q to P, then flows from S to R through the resistor.
31. D ✓ (3 ) If the rod moves to left, by Right hand rule, the induced cwrent flows from P to Q through the rod.
X (1) The current delivered to the load by a d.c. generator is an unsteady d..c., thus the current is not constant.
✓ (2) Due to the commutator, cummt delivered to the load is a d..c., i.e. the �nt flows in one direction only. 38. C
✓ (3) By use ofthe Right hand rule, the induced current in the coil is in anticlockwise direction,
and thus the current flows fromXto Ythrough the load.
s = �(N<l>)
.,
= !!..!!'.:::!!. = !!.!!_
32. C
XX XX
By Right hand rule: xxxx
xxxx 39. C
induced current flows upwards in the left wire. X X
-to,right The magnetic flux through the coil k is equal to the magnetic flux through the coil Li,
By Left hand rule : XX XX as both of the contams the same number of magnetic field lines inside the solenoid..
xxxx
this current gives a magnetic force towards the left. XX XX
Thus,both of them induce the same e.m.f ofl2 V.
_ DSE Physics - Section D : M.C. Solution PD-EMS-MS/OS DSE Physics - Section D : M.C. Solution PD-EM5-MS/09
EMS : Electromagnetic Induction EMS : Electromagnetic Induction
40. D 46. D
An e.m.f. is induced from P to Q as PQ cuts the magnetic field lines. In the position shown, there is no cutting of magnetic field lines, thus no induced currenL
Similarly , an e.m.£ is also induced from S to R as SR cut the magnetic field lines.
As the frame moves, during the time interval that theleft wire of the frame cuts across the magnetic field,
Since there is no net e.m.f. in the circuit, thus no induced current flows in the loop.
current is induced upwards and flows in the frame in c lockwise direction, which is negative,
and since the frame moves with Ullllorm velocity, the induced current is constant.
41. D
The magnetic field due to the magnet is perpendicularly into paper.
47. D
At the right hand s ide of the magnet, the disc moves from a region without field to a region with magnetic field,
When the switch Sis suddenly closed, current flows in loop Pin anticlockwise direction.
to oppose the change. eddy current is induced in anticlockwise direction to produce magnetic field out ofpaper.
As the current in the right wire of loop Pis upwards, it then produces a magnetic field into paper at loop Q.
At the left hand si& oftbe magnet, the disc moves from a region with magnetic field out to a re gion without field,
to oppose the change, eddy current is induced in clockwise direction to produce magnetic field into paper. When loop Q suddenly experiences a magnetic field into paper,
a current is induced in anticlockwise direction so as to produce a magnetic field out of pa per to oppose the change.
42. B Asthe current in the right wire ofloop P is upwards and the current in the left wire ofloop Q is downwardll,
For a solenoid:B= µ.N I repulsive force occurs between them as currents in owosite directions repel each other.

Magnetic flux through the solenoid : ct> = B A 48. B

4> =BA::: µ.
'
NI
.A Forthe samelength.ofwireformingthe doubleloop:21tr = 2(21tr') :. r' = r/2
The radius of the new loop is halved, thus the area of the new loop becomes one-quarter.
Flux linkage =
( 41txl0-1)(1000)(1.5)
= (0.3)
_(3.2xl 0-4) NBA cc NA
As N ➔ '1N and A ➔ ¼ A, the flux linkage becomes ½ $
=2.0xlO...s Wb

43. D 49. A

Frequency ofthe voltage is equal to the rotational frequency of the coil, thus the fre quency increases. ✓ (1) = B-�
"'
e = ;il'J) = BvL cc L
t,J t,J
Peak vol tage is proportional to the rotational speed of the coil, thus the amplitude increases.
(2) By Right�hand rule, induced e.m.f. is from Q to P, thus P is at a higher potential

44. A (3) As the circuit is not complete , there is induced e.m.f. but no induced current,
thus no magnetic force acts on the rod to oppose its motion.
When the bar magnet moves towards the solenoid, induced current in the coil is in one direction.
When the bar magnet moves away from the solenoid, induced current is in the opposite direction.
50. D
Ocly the graph in option A shows that the induced cur:rents are in two directions, thw; it is the answer. ✓ If the resistivity of the ring is smaller, then the resistance of the ring is smaller,
(1)
thus greater eddy CUIIent can be induced in the ring and the ring would float at a greater height.
45. A ✓ (2) If the density of the ring is smaller, then the mass or weigh t of the ring would be smaller,
Note that only the small loop contains magnetic field. thus the same upward ma gnetic force due to eddy current would make the ring float at a greater height

Thus, in calculating the magnetic flux: ct> = BA, the area A should be the smaller loop. ✓ (3) If the fr equen cy ofthe a.c. increases , then the rate of change of magnetic flux in the coil increases,
thus greater eddy current is induced, and the ring would float at a greater height
By e = ii!l> = AM
t,J 6t

/J = [ 1t X (0.08)2 ] X (0.01) 51. D

""20 X 104 V (1) Resistance would not affect the flux linkage of a coil.
As the magnetic field is decreasing, to oppose the change., ✓ (2) Flux linkage is proportional to the number oftums of the coil.
the induced cm:rent would produce a magnetic field in the same direction, that is, into paper, ✓ Since the magnetic flux through a coil depends on the component of the magnetic field through the coil,
(3)
thus, the induced current is in clockwise direction. the angle would affect the flux linkage of the coil.
DSE Physics - Section D : M.C. Solution PD-EMS-MS/10 DSE Physics - Section D : M.C. Solution PD-EMS-MS/11
EM5 : Electromagnetic Induction EM5 : Electromagnetic Induction

52. D 58. A
✓ (!) When th e magnet moves towards copper, the copper experiences a change in magnetic field, M
(j) 6 : A oc A il) R = pt oc t
thus eddy current is induced in the copper disc. A
✓ (2) As eddy current is induced, the current produces a heating effect, thus temperature increases. J=!.oc:'!-..
✓ The eddy current .flows in the direction to give a repulsive force to oppose the motion of the magnet. R l
(3)
le _ Ac. f 8 _ (1tr2) _ {4x2,j _
1
I� As Pc (2r) 2 (21t r)
53. D
• (!) It gives no change ofmagnetic flux through the coil, thus no e.m.f. is induced.
59. C
✓ (2) The flux through the coil changes to zero, thus e.m.f. is induced.
(j) When the coil is at the left and moves to the right,
✓ (3) The flux through the coil changes, thus e.nt..f. is induced
the coil experiences an increase ofmagnetic flux out of paper.
In order to oppose the change, a clockwise cun:en.t is induced to produce a magnetic field into paper.
54. A

"' When the coil is justabove the wire,
(!) By Right-hand rule, indu.ced current flows fromB to C, and then to D and A in clockwise direction. the coil experiences a change of magnetic flux from out of paper to into paper.
(2) The induced e.m.f. is from B to C, thus C is at a higher potential. In order to oppose the change, an anticlockwise cnrren.t is induced to produce a magnetic field out ofpaper.

Side ADisnot in the magnetic field, it eXperiences no magnetic force. When the coil is at the right andmoves to the right,
(3)
the coil experieo.ces an decrease ofmagnetic flux into paper.
In order to oppose the change, a clockwise current is induced to produce a magnetic field into paper.
55. D
h.(1)
s "" N == � 60. D
f,J t
The secondary coil is a search coil which gives the peak voltage by Vo = 2 nfN Bo A
s depends onN,B,A, tonly
✓ A. /t ⇒ v,t
& is independent of the resistance ofthe coil.
✓ B. Nt ⇒ Vot

56. D ✓ C. B..fieldinthemiddleofthe solenoidisgreaterthanB-field atthe end :. Bot⇒ Vo t


D. By B = µ,,nl ,BisindependentoftheareaA :. sameB ⇒ same Vo
For a search coil, the length£ of the trace on the CRO is proportional to the peak induced voltage in the coil,
which is proportional to the peak value of the varying magnetic field.
61. A
For a straight wire carrying current, magnetic field produced is:
From t"" Oto T/4, current flows fromMto N and is decreasing, magnetic .field through the loop is into paper
I and is decreasing. Thus, the induced current in the loop flows in clockwise direction to oppose the chattge.
B = µ" o:: .!_
2:irr r
From t = T/4 to T/2, currentflows from N toMand is increasing, magnetic fi eld through the loop is outof paper
As the distance at A is halved that of B, the magnetic field at A is doubled that ofB,
and is increasing. Thus, the induced current in the loop flows in clockwise direction to oppose the change.
thus the length of trace is doubled.
From t= T/2 to 3T/4, current flows fromNto Mand is decreasing, magnetic field through the loop is out of paper
: . L' = (f }2 = 4cm and is decreasing. Thus, the induced current in the loop flows in anticlockwise direction to oppose the change.
From t = 3114 to T, current flows fromMto N and is increasing, magnetic field through the loop is into paper
and is increasing. Thus, the induced cunent in the loop flows in anticlockwise direction to oppose the change.
57. D

Since the component ofv perpendicular to the wire is v sin60° 62. D


Inducede.m.£ : ✓ A. By Right-hand rule, induced current flows along PQ and in the anticlockwise direction.
e = B(vsin60°)L ✓ B. By Left-hand rule, magnetic force acting onPQ is towards the left.
= ✓3BvL ✓ C. By Left-hand rule, there is magnetic force acting on RS towards the right
D. Due to the opposing magnetic force acting on PQ, the rodPQ would decelerate.
. DSE Physics - Section D : M.C. Solution PD-EMS-MS/12 DSE Physics - Section D : M.C. Solution PD-EMS-MS /13
EM5 : Electromagnetic Induction EM5 : Eleciromagnetic Induction

63. B 69. A
Tota.lpower consumedbythetwo lamps = 6+6 = 12W ✓ (1) When the frame enters the field, to oppose the cliange,
Current passittgtbrough eachlamp = £ = 0.5 A the frame induces a c:unent in anticlockwise direction to produce a magnetic field out of paper,
12 thus the induced current is in the directionPQRSP.
Power dissipated by the internal resistance of the generator = 12 r = (0.5 + 0.5)2 (2) = 2 W (2) There is no induced current when it passes 00' as there is no change of magnetic flux.
Perctntage of power dissipated by the lamps = �xlOO¾ = 85.7% = 86% (3) As there is no induced current when it passes 00 ', there is no magnetic force acting on the frame.
12+2

70. C
64. B
Direction of magnetic field due to the current along XYis into the paper by Right hand grip rule.
(1) There is no cutting of magnetic flux when the rod rotates about the axis along y.
As the coil moves away, the strength of B gradually decreases and thus the flux also decreases.
(2) By right.hand-rule, the induced e.m.t: isbetween the top and the bottom of the rod, not along the rod.
To oppose the change, the coil induces a current in clockwise direction to produce a flux into the paper.
✓ (3) By right-hand-rule, there is an induced e.m.f along the length ofthe rod.
Since the induced cunent along RS is upwards, the magnetic force between XY and RS is attractive,
thus the magnetic � on the coil is to left.
65. D
When the field moves away, at the left hand side of the field, the metal there experiences the loss of the field.
71. D
To oppose the change, current is induced in clockwise diteciion to produce a magnetic field into paper.
(1) The cunent is from S to R, by Left band rule, the magnetic force should be downwards.
The eddy current should be at the edge of the field, since the change ocCUIS there.
✓ (2) Magnetic force on RS is downwards and force onPQ is upwards, they fonn a couple to give a moment
✓ (3) There is no magnetic field lines passing perpendicularly through the loop, thus the flux is zero.
66. B

Toe circumference of the square and the circle must be equal.


n. A
:. 15.7 x4 = 21tr :. r = 10 cm Figure 1: e = BvL Figure2:e= B(vsin0) L
Areaofthe squaze = 15.7xl5.7 = 246.Scm.2 :. Ratio = l:sin0
Areaofthecircle = 1t(l0)2 = 314cml
73. D
& = t:,,¢> = BM = (0.6) {314-246.S)xl0
4
S.lmV -
t,J M (0.5) N<b = NBA = Nµ0 NI· A = (l000)x (4�xlO ') (lOOO)(l.5) ·(32x10-4 ) = 2.0 x 10- 3Wb
Ju the area is increased, the coil experiences an increase of flux into pap er. ' (0.3)

By Lenz's law, to oppose the change, the induced current acts to give a magnetic field out of paper.
74. B
Thus the induced current flows in anticlockwise direction.
✓ (1) By Right band rule, B is into paper, I is downwards, thus, motion is leftwards.

67. C (2) By Right hand rule, induced e.m.L is downwards from low to high potential.
Potential difference across Q and S indl.lced e.m.f: along the rod between QS Thus, potential of Bis higher than that ofA.

Bvl = BvL ( l is the length of the rod) ✓ (3) By Left hand rule, Bis into paper, I is downwards, thus, magnetic force Fis rightwards.
°
sin3O OR
For induced current, magnetic force must oppose motion, thus magnetic force is rightwards.
68. B

(1) As eddy current is induced in the aluminium cylinder, 75. D


the amplitude ofoscillation of the magnet gradually decreases ✓ A. By Right band rule, induced current along rod PQ is upwards, thus the Cllitellt is in the directionPQRS.
due to the opPosUlg force of the eddy current. ✓ The magnetic force must be oppose the motion, thus it is towards the left.
B.
(2) The force should act always to oppose motion, thus the force may be attractive or repulsive. ✓ C. Current along rod RS is from R to S. By Left hand rule, the magnetic force on RS is towards the right.
✓ (3) The change of magnetic flux experienced by the cylinder induces eddy current Ul it. • D Due to the opposing magnetic force, the rodPQ should decelerate to rest, not uniform speed.
"\
DSE Physics - Section D : M.C. Solution PD-EMS-MS/14 DSE Physics - Section D : M.C. Solution PD-EMS-MS/15
EMS : Electromagnetic Induction EM5 : Electromagnetic Induction

76. D 81. B
The flux through the metal frame: rJ> = BA = B L2 ✓ A. The two sides of the frame P and Qare moving pe.tpendicularly to cut the magnetic field lines,
When the frame is rotated by 90 °, the flux changes fromB L2 to zero, thus the change isBL 2• thus the induced e.m.f.is at a maximun:i. at this instant.
When the frame is rotated by 180°, the flux changes fromB L2 to -B L2, thus the change is 2B L2 • • B Consider the sideP, it is moving out ofpaper. The direction of magnetic field is towards the left.
By Right hand rule, the induced current inP is in upward direction.
77. B The induced current in Q is in downward direction.
The!Cfore, the induced current in the frame should be in clockwise direction.
When the square metal frame moves the distance L into the field region, current is induced.
✓ C. Whenever there is induced current, there must be a magnetic force opposite to the m otion.
When the square metal frame moves inside the field region, no current is induced.
As Pis moving out of paper, the magnetic force onP is into the paper.
When the square metal frame moves the distznceL away from the field region, cllirCJlt is induced. ✓ D. The magnetic force on Pis into the paper and that on Q is out of the paper.
Thus the total time that current is induced = d = 2L These two magnetic forces form a moment to oppose the rotation of the frame.
V V

78. A 82. A
✓ (!) By Right hand rule, an e.m.f. or voltage is induced across PQ. ✓ (!) Rotate the search coil so that the search coil has different orientation angle with the magnetic field.
• CZ) As the rod is not a complete circuit, there is no current.
"When the vertical trace on the CRO is rnax:immn, the plane of the search coil is petpendicular to the field.

• (3) Since there is no current, there is no opposing m agnetic force, thus the rod falls with acceleration g.
If the vertical trace is not the maximum. the search coil only measures a component of the field.
✓ CZ) Since the induced voltage is proportional to the frequency of the a.c., high.er fuquency gives longer trace,
79. D thus, percentage error of the length ofthe trace is reduced, and accuracy is improved.
Since the rod is isolated, there is no complete circuit, thus there is no induced cumnt • (3) The Earth's magnetic field would not affect the experiment,
By Right hand rule, the induced e.m.f is from Oto P, thus the endP is at a higher electric potential, as search coil cannot detect steady magnetic field such as that of the Earth.
as induced e.m.f. points from low to high potential
83 . C
80. D By using Right hand rule, indllced cumnt flows upwatds along the rod, thus it flows fromXto Ythrough R.
From t= 0 to T/2, the current is positive and flows from MtoN. Since only the length d ofthe rod can have current flow, induced cunent is B d v IR.
N
By Right hand grip rule, themagne6c field in the coilPQR.S
due to Iis directed into paper. magnetic fieldB
p �ueto �t / 84. B
As current is decreasing, magnetic :field B in the coil is decreasing. · X X
IS deccea.smg
• A. Reverse the direction of magnetic field can only reverse the direction of eddy current,
Byleru:'s law, to oppose the change of magnetic field, I � but cannot increase the magnitude of eddy ewtent.
the coil induces a cWTentto give aB-field in same direction, xx
magnetic field ✓ B. Increase the strength oftbe magnetic field can increase the rate of change of magnetic flux,
thus, theB-field by the induced current is also into the paper. Q R due to induced
cuxrertt in the coil thus, the induced eddy current increases.
By Right hand grip rule, the induced current in the coil is in • c When the whole disc is inside the magnetic field, there is no change of magnetic flux,
clockwise direction of PSRQ. M
thus, no eddy current is induced.

From t"" T/2 to T, the current is negative and flows fromNto M. N • D. Cut several slits give lamination that will reduce the induced eddy cun:ent.

The magnetic field in the coil PQRS due to I is directed out of paper. magnetic field B
As eurrent is increasing;, magnetic fieldB in the coil is increasing. P ?u_e toc�tJ 85. D

..
1sm�g
• As the magnet moves down along the aluminium tubing,
By �•slaw, to oppose the change,
the coil induces a current to give aB-field in opposite direction
I B:X the alumiDium experiences a change of magnetic flux of the magnet,
thus, the B-field due to the induced cutrent is into the paper. magnetic field thus, eddy current is induced in the alumfn.ium tubing.
Q R due to induced
By Right hand grip rule, the induced CWTent in the coil is in =t in the coil As eddy current is induced, there is an opposing magnetic force acting on the magnet
clockwise direction ofPSRQ. M Thus the net downward force acting on the magnet is less than mg, its down-ward acceleration is less thang,
In conclusion, the induced current in the coil is in clockwise direction throughout the time interval. therefore, the magnet drops slower.
DSE Physics - Section D : Question
EMS : Electromagnetic Induction

The following list of formulae may be found useful :


PD-EMS-Q/011

I
2. < HKCE 1981 Paper I- 7 >
DSE Physics - Section D : Question
EMS : Electromagnetic Induction
PD-EMS-QI 0211

�,
A light rectangular conducting loop EFGH moves from left to right with mrifoxm velocity across a uniform magnetic field

-
Induced e.m.f.

....
e = N t,.<ll pointing into paper. Figure1, 2 and 3 show three subsequent positions of theloop during its lllOtion.

g,:
" " " " < < < <

)�
Force on a current-carrying conductor in a magnetic field F=Bllsin.0
• <

Magnetic field dueto a long straight wire


" " ,. G
,. II f
p
Magnetic field inside a long solenoid
..,.,., Flgure2 Figure3

Use the following data wherever necessary: (a) What can you say about the induced current in each of the three cases shown in Figure 1, 2 and 3 ? (3 marks)

Permeability of free space Ji,,= 4rc x 10--7Hm-1


Charge of eleci:ron e = 1.60 x 10-19 c

Electron rest mass me= 9.11x10-31kg


(b) State whether an external force is required to maintain the motion of the loop with uniform velocity in each of the above
Acceleration due to gravity g = 9.81 ms-l (close to the Earth) three cases. Explain briefly. {6 marks)

Part A : HKCE examination questions

L < HKCE 1980 Paper I- 9 >


Thefigure below shows a simple current generator.

3. < HKCE 1982 Paper I- 7 >


If an a.c. signal is inputted to the CRO with the time-base set at 1 ms cm- 1 and the voltage sensitivity set at 2.5 V cm-1,
the wave pattern displayed is shown in the below :figure.

(a) In the figure shown below, sketch a graph to show how the induced voltage varies with time when the coil is turned
through one revolution, starting: with the plane of the coil lying parallel to the field. (2 marks)
Induced voltage
(a) From the wave pattem obtained, estimate the peak voltage of the input signal. (1 mark)

{b) Estimate the time taken for the spot on the screen to produce a complete cycle.

(b) The generator is used to deliver a d.c. cur.rent. What modification to the generator is necessary to enable this to be
done 1 (2 marks) (c) What is thefrequencyoftheinputsignal?
DSE Physics - Section D : Question
EM5 : Electromagnetic Induction
PD-EMS-Q/03

I DSE Physics - Section D : Question


EM5 : Electromagnetic Induction
PD-EMS-Q /04 II
4. < HK.CE 1982 Paper I - 9 > S. < HK.CE 1983 Paper I- 8 >
The diagram below shows a solenoid with an iron core, a bar magnet and compasses A and B with needles pointing in the The below figure shows an alternating current generator.

,,J{i,�r-- \
directions indicated. Neglect the effect ofthe Earth's magnetic field.

solenoid ✓ llVL
iron ,-,c;.;+-H-+H+H-+-,
"'" X

(a) Sketch on the diagram above, the magnetic field pattern near endX and end Y.
(a) Descn'be briefly with the aid of a diagram how the alternating current generator can be converted into a direct current
generator. (4 marks)
Indicate

(i) the polarities of X and Y

(ii) the neutral point, and

(iii) the direction of the current in the solenoid. (8 marks)

(b) What happens to the neutral point in each of the following cases :

(i) the iron core is taken out of the solenoid at the left hand, and, Cl =kl
(b) Sketch a graph ofthe output voltage of the a.c. generator against time. Indicate on the time axis ofyour graph the times
at which
(i) the plane of the coil is parallel to the magnetic :field (using the letter H), and
(ii) the plane of the coil is perpendicular to the magnetic field (using the letter V). {5 m arks)
(n) the battery is replaced by a centJ:e..zero galvanometer? (2 =rl<,)

(,)

(c) Describe what happens to the output voltage if (6 nmk,)

(i) the generator rotates at double its original speed.

(ti) the generator rotates in the opposite direction.


After the two changes descnbed in (b) have been made, the bar magnet is moved tmwrds the solenoid and passes
through it. Indicate on the given diagrams above the positions of the galvanometer pointer when

(i) the magnet is just entering the solenoid,


(rit) the number of turns of the coil is doubled.
(ii) the magnet is inside the solenoid, and

(iii) the magnet is just leaving the solenoid. (3 nmk,)


I
6.
DSE Physics - Section D : Question
EM5 : Electromagnetic Induction

< HKCE 1985 Paper I- 7 >


PD-EMS-Q/05

II I DSE Physics - Section D : Question


EM5 : Electromagnetic Induction
PD-EMS-QI 06
I
7. < HK.CE 1988 Paper 1- 9 >
The below figure shows a simple experimental set--up to study the induced current in a wire moving in a magnetic field. The
wire is connected to a galvanometer. If current flows through the galvanometer from A to B, the pointer will deflect to the
right.

~----spiral spring

pointer----tt➔·-t---:(
N
11�----- magnet

(a) Draw a diagratn to :indicate the directions of motion ofthe wire, ofthe tnagneti.c field and of the induced current (if any)
(i) if the wire is moving quickly upwards.
A student sets up an apparatus as shown in the above figure and claims that it can be used to measure a current flowing from
(ii) if the wire is moving quickly sideways towards the north pole. tenninal A to B through the solenoid.
Descnlle briefly what happens to the galvanometer pointer in each case. (6nwks)
(a) (i) Explain why this set-up can be used to measure current. (3nwks)
(i )

(i.11 Suggest TWO methods to increase the sensitivity of the set-up. (2marks)

(it) (ill) Can this set-up still measure current if the magnet is replaced by a soft iron bar 7 Explain briefly.

(b) A andB are now connected to a centre-zero galvanometer. The magnet is set to viotate up and down. It is kept out of
the solenoid Is there any cur.rent passing through the solenoid when the magnet

(i) moves towards the solenoid '1

(ii) is at its lowest point '1


(b) State THREE methods of increasing the indticed current in the experiment. (3nwks) If there is a current, state its direction. Explain briefly in each case. (5 nw:ks)
1 DSE Physics - Section D : Question
EMS : Electromagnetic Induction
PD-EMS- Q/07 I I DSE Physics - Section D : Question
EM5 : Electromagnetic Induction
PD-EMS-Q/0811

8. <HKCE1989Paperl-7> 9. < BKCE 1996 Paper I - 7 >


A student uses the set-up as shown in th e figure to investigate the generation of electrical energy from wind. The blades are (a)
turning in the direction shown.

F"ignrel
A bar magnet is pushed with constant speed from left to right through a solenoid as shown in Figure 1. Descdbe the
change in the direction of the CUitent passing tbrough the galvanometer during the motion of the magnet (3 matks)

(b)
R j
(a) What is the name of the parts labelled
(i) A, \R
I" /
(Li) B, and p
(iii) C? '=
I= f
Figwe 2 Figure 3
Figure 2 shows the structure of a simple a.c. generator. A voltage is induced when the coil is set into rotation. The
(b) Sketch the variation of the output current against time and determine the direction of the current passing through the output ofthegcneratoris displayed on a CRO as shown in Figure 3. The time base of the CRO is set at 20 ms cm- 1 and
resistor R at the momer,t shown in the figure. (3 marks) Y-gainat SOmV cm-1.

(i) Which points (P, Q, R and S) shown inFigure 3 correspond to instants at which the plane of the coil is parallel to
the magnetic .field? ( 2 m arks)

{it) Find the peak voltage and :frequency of the output of the generator. (3 awk,)
(c) Yon are given a voltmeter and an ammeter. Draw a circuit diagram to show how they can be used to measure the power
output by the generator. (3 marks)

(iii) Describe what happens to the peak voltage and frequency of the output of the generator in each of the following
cases:
(1) Increasing the speed ofrotation ofthe coil.
(2) Winding the coil on a soft-iron core. (4 m,rk,)
(d) The ctllW3t output is 0.7 A when the voltage betweenXY is 12 V. Determine the power output by the generator at that
moment (2 marks)

(e) State TWO advantages and TWO disadvantages of using wind to generate electrical energy. (4 marks)
(Iv) Steam is commonly used to drive generators in power statiOllS to generate electricity. Suggest two other practical
means of driving generators. (2 marks)
I DSE Physics - Section D : Question
EM5 : Electromagnetic Induction
PD-
EM
5 -Q
/ 09 I I DSE Physics - Section D : Question
EM5 : Electromagnetic Induction
PD-E
M 5- /
Q 10 I
10. < HKCE 1998 Paper I - 5 > 11. < HKCE 2002 Paper I- 6 >
Aluminium ring

paper cone
A soft-iron rod is inserted into a solenoid AB, which is connected to a battery and a switch Sas shown in the above figure.
InitiallySis open. An aluminium ring i s also inserted into the rod and placed beside the solenoid. Sis now closed.

(a} State the polarity at endB of the solenoid. (1 mar k)

The Figure above shows a type of motor. PQ and RS are solenoids. The solenoids and the coil ABCD are connected in
parallel to a battery. (b) Explain why the aluminium ring will move away from the solenoid. {3 marks)
(a) State
(i) the polarity at end Q ofthe solenoid PQ,
(11) the direction ofrotation of the coil as seen by the observer. (2uwks)

(b) Name the component£ and explain its :function. (3 mru,k,)

12. < BKCE 2003 Paper I - 6 >


Figure 1 shows a battery-powered electric fan. Mary wants to construct a simple generator from the fan. She removes the
motor of the fan and cormects it to a light bulb (see Figure 2). When the blades of the fan are turned rapidly, the bulb lights
up.
(c) Suggest two � of increasing the rotating speed of the coil. (2uwks) Light bulb

(d) A student says "If the battery in the above Figure is replaced by a SO Hz a.c. supply, the coil will only oscillate to and
fro. Hence the motor will not function properly."
Explain why the student is incorrect. (4marks)

(e) Desmibe, with the h elp of a diagram, how the motor in the above Figure can be converted to a <lirect current generator.
Figurel Figure2
(3 mmks)
(a) Name two essential.components ofa motor.

(b) Explain why the bulb lights up when the blades are turning. (3 marks)
DSE Physics - Section D : Question PD-EMS-Q/11 DSE Physics - Section D : Question PD-EMS-Q/12
EM5 : Electromagnetic Induction EM5 : Electromagnetic Induction

13. < HKCE 2008 Paper I" 11 > 14. < BI(CE 2010 Paper I" 13 >
A student uses the setup in Figure 1 to study the current induced in a solenoid when a magnet is falling through it 'When a A hand•shaken torch does not require any battezy. Shaking it for a short while will produce a current and give out bright
cutren:t is passing through the current sensor from A to B, a positive reading is obtained Figure 2 shows the result after the light. Figure I shows the structure of the torch.
magnet is released at a certain height

Figure I

light omitting rubber cushion


component

(a) Describe how current is produced in the fixed coil when the torch is shaken. (2marks)

0.9 J.0 1.1 1.2 1.3 I.4 1.5 1.6


Time/s
(b) State one method to increase the crurrent without modifying the torch. (lm&k)
l<igure2

(a) State the polarity of end Yofthe magnet (I mark)


(c) Descnl,e the energy conversion when thls torch is in operation. (2 marks)

(b) Explain why the reading of the induced current is negative when the magnet leaves the solenoid. (3 marks)

{d) In another design as shown in Figure 2, the fixed coil covers the whole length of the path ofthe movable magnet It is
found that the torch becomes dimmer than the original design when operating it in the same way. Explain briefly. (The
resistance of the fixed coil can be neglected.) (2 marks)

Figure2

(c) Explain why the magnitude of"negative peak" is greater than that of"positive peak''. (2 marks)
light emitting rubber cushion
emitting
component
DSE Physics - Section D : Question
EMS : Electromagnetic Induction
PD-EMS-Q/131

I DSE Physics - Section D : Question


EMS : Electromagnetic Induction
PD-EMS -Q / 14 II
Part B : BKAL examination questions 16, (b) If now the d.c. supply is replaced by an a.c. supply.

(i) What would be observed if the switch is closed ? (1 mark)


15. <HKAL 1996PaperI-7>

(ii) Suggest a practical use of this experimental result. (1 mark)

(iii) The heat capacity of the ring is 7 .8 J K-1 and its temperature is found to rise from 25°C to 40°C during the first 50 s
Figtae 1 Figure2 when the a.c. supply is on. Find the average rate of increase in internal energy of the ring. (2 marks)

A centre.zero galvanometer A is connected in series with a resistor, R. and a 1.5 V cell as shown in Figure 1. The pointer of
A deflems to the left. The galvanometer is now connected to a coil as shown in Figure 2 When a bar magnet is moved with
uniform speed by a student towards the coil. the pointer ofA deflects to the right.

(a) On Figure 2, indicate the direction of the induced cumnt in the coiland also the poles of the bar magnet. (2 marks)

(b) Where does the electrical energy in the circuit come from ? (lm..-k)
17. <HKAL2000Paperl-7>
(a) In the figure below, there are two parallel wires carcying currents in opposite dttections. The current in the left wire is
perpendicularly out ofpaper while the cwrent in the rlgbtwire is perpendicularly into paper. Sketch in the space below
the magnetic field pattern around the two wires. (2 marks)
(c) Suggest THREE waY5 to increase the deflection ofA. (2 marks)

0
16. < HKAL 1996Paper I- 7 >

retort stand

(b) In the figure below, if the current flowing through both wires is 5 A and the separation between them is 0.05 m, find the

rn
d.c. supply magnitude of the resultant magnetic field due to these two wires at pointX, which is 0.02 m from wire Q. (2 marks)

A coil and a retort stand made of iron are arranged as shown. The coil is connected to a d.c. supply via a switch S. Wben the
switch is pressed on, the allllllinium ring placed on top of the coil jumps up momentarily and then falls back to the top of the ----◄ X
coil. 0.02m
(a) Briefly explain this observation. (3 marks)

Q
0.05m
DSE Physics - Section D : Question PD-EMS-Q/15 DSE Physics - Section D : Question PD-EMS-Q /16
EMS : Electromagnetic Induction EMS : Electromagnetic Induction

17. (c) Two long parallel current-carrying wires are now connected to a signal generator giving out a.c. current. A search coil 19. < HKAL 2001 Paper I- 4 >
connected to a CR.O is used to investigate the magnetic field around the two wires. With the time base of the CRO
The figure below shows a solenoid of diameter 5.0 cm and length 50 cm. The solenoid has 1000 tums and it carries a
switched off; a vertical trace is observed on the screen of the CR.O.
steady current of 60 mA.
----soon---+

solenoid

(a) Calculate the magnetic field strength at the centre O of the solenoid. (2marks)

(i) Explain what is represented by the length ofthe trace observed on the CRO. (2 marks) (b) Calculate the magnetic flux fulkage through the solenoid.

(ii) Explain whether the :measurement of the magnetic field by the se3Ich coil would be affected by the Earth'sll:laglletic
field. (2 marks)

20. < HKAL 2003 Paper I- 4 >


The figure below shows a circular coil ofl00 turns and radius 5 cm pivoted by two smooth vertical supports. It is placed .in a
region with a 'Ullifunnmagnetic field of0.l T towards the right. The ends ofthe coil are joined together and the resistance of
the coil is 10 n.
18. < HKAL 2000 Paper I w 8 > .
In the figw-e below, a space shuttle carries a satellite attached to itby a 20 km long conducting cable. The two move together
around the Earth above the equator in orbital motion. In the orbital motion, the cable sweeps through the magnetic field of
the Earth.
'
. unifonn magnetic field
X X X
s{tellite ;,<
X X X X

X X X X
I .
20km
X l Not to scale
X X
cable
X
/j
X X X X
The coil is tumed through 90° by an extema1 force until its pl.an.e is pexpcndicular to the magnetic field.
shu�
X X
(a) Would the coil resist bclng turned? Explain briefly.
X X X X X

Given : Magnetic field strength in the orbital region = 30 x IO� T


O.tbitalposition ofthe shuttle = 6.8 x 106 m from the centre of the Earth
(b) If the coil is rele ased from rest at the new position described above, would it move back to its original position ?
MeanradiusoftheEarth"" 6.4x !06 m Explain briefly. (2 marks)
(a) Calculate the orbital speed of the shuttle at this position. (3 marks)

(c) The coil is now fixed with its plane perpendicular to the magnetic field. The flux density of the field is increased at a
uniform rate of0.3 T s-1 • Find the magnitude of the current induced in the coil. (3 marks)
(b) Find the e.m.£ induced across the cable. State one assumption that yau have made in the calculation. (3 marks)
DSE Physics - Section D : Question PD-EMS-Q/17 DSE Physics - Section D : Question PD-EMS-Q/18
EMS : Electromagnetic Induction EMS : Electromagnetic Induction

Part C : HKDSE examination questions 23. <BKDSE2014PaperIB-9>


(a) You are given a low voltage d.c. supply, an aluminium.ring, a switcll, a coil of 1100 turns and a retort stand arranged as
21. < BKDSE Sample Paper IB- 7 > shown. Use three connecting leads to complete the connections among the apparatus in the figure and descnbe bow to
demonstrate Lenz's law in electromagnetic induction. State and explain the observation. (6 marks)
Amy uses the motor of a toy fan as a simple generator. She connects a bulb to the two terminals of the motor. This is shown
in the Figure below.

retort stand
(uon)
"''"" aluminium low voltage
ring d.c. supply

connecting
The bulb lights up when the blades are turned rapidly. Explain why and state the energy conversion talcing place in this t=,lnru,
process. (4marks)

22. < HKDSE 2012 Paper IB -10 >


You are given a long conducting wire, a pair of slab-shaped magnets on steel yoke and a light-beam galvanometer for
detecting small currents. With the aid of a diagram, descn'be an experiment 1o investigate TWO factors affecting the e.m.f.
induced in a conductor when it I110ves in a magnetic field. (J marks)

light-beam
galvanometer

conducting wire
slab-shaped magnets
on steel yoke (b) Descnbe what would be observed if the experiment in part (a) is repeated with

(i) a low voltage a.c. supply; (1 muk)

(ii) a low voltage a.c. supply and an aluminium ring with a slit cut through it as shown [ 0 ]. (1 muk)
24.
DSE Physics - Section D : Question
EMS : Electromagnetic Induction

< .HKDSE 2016 Paper m � 8 >


Read the following passage about lightning and answer the question that follow.
PD-EMS-Q/19

I
24. (b) (i)
DSE Physics - Section D : Question
EMS : Electromagnetic Induction
PD-EMS-Q/20

State the direction of the magnetic field ( to the left/ to the right/ into paper/ out of paper) produced at point Oby
the lightning current. Estimate the magnetic field strength's peak value at 0.
I
(.3 ma:rks)

thundercloud
(i.l.J Explain why within the very short duration of lightning, an induced cumnt :first flows in the coil in a certain
direction and then reverses. Your answer should include the directions of the induced current in the coil.
(3 ""'1<,)

induced charges dl lightning =nt (iii) Among the physical C(Ualltities related to lightning, electric fiel� in the atmo �here, ligh�g cu �t :m,d
magnetic field due to lightning, suggest which one can be morutored so as to give fore-warmng of lightnm g.
'½ground Explain your c hoice. {2 marks)
++++++++++++++++

Lightning occurs when diarges accumulate in the clouds to such an extent that the electric :field in the atmosphere is
strong enough to cause the air to Jose its insulating properties. The threshold electric field for 'breakdown' to occur is
about 3 x 10s V m-1 above which electrons or ions in the atmosphere can pass through the air between clouds and the
ground or between clouds and clouds. The peak cummt ofa typical lightning bolt can reach about 30 000 A. How the
charges are separated and accumulated in the clouds is not fully understood yet. In most cases, negative charges are at
the base of the cloud and positive charges are induced on the ground.

�,�� : . · 1· . . •:.!
25. < HKDSE 2018 Paper m - 9 >
p/""________
(a) (i) What is the meaning of'breakdown' in the passage? (I """")
magnetic J 1

:'' . . . . . .:''
fieldB�• • • • •

: L --i,,V : resistor
(ii) The thundercloud's base and the ground can be modeled as two parallel plates with opposite charges. If the
negative charges distributed at the cloud's base are about d = 2 k:tn from the ground, find the potential difference
between the cloud and the ground when the electric field in the atmosphere just reaches the threshold of
' I •
' e • e e el
L------------ _____________ J
-wn•. (2 ""'1<,)
Q '---�-----

The Figure above shows a metal rod PQ of length L moving with constant velocity v across a uniform magnetic fieldB
pointing out of the paper. An e.m.f? is induced across rodPQ as it cuts the field lines. When the rod is connected to a
resistor outside the field, a current I flows in the circuit.
A lightning detector having a small square coil inside is placed at point O which is 1.S Ian from the lightning bolt The coil
and the lightning's direction are in the same vertical plane as shown. Assume that the lightning current flows verticaily (i) Indicate the direction of/in the above Figure. (I mark)
upwards to the thundercloud from the ground.
(ti) Explain why an external force Fis required to maintain the uniform motion of rod PQ. Find Fin terms of the
/ Diagram NOT drawn to scale physical quantities given. (3 matks)

square coil
u
pw.u-d �
lightning 1.Slan----a
(iii) This set-up works as a generator, By considering the mechanical power input by external force F to the set-up,
eumnt
show that ? =BL v. (2 marks)

ground
I
25.
DSE Physics - Section D : Question
EM5 : Electromagnetic Induction
PD-EMS -Q /2111
(b) At a certain place the Earth's magnetic field runs along the S-N llirection such that the field lines make an angle 0 with
the horizontal as shown inFigure (a) .
DSE Physics - Section D : Question Solution
EM5 : EJectromagnetic Induction
PD-EMS-QS/01

BKEAA's Maik:ing Scheme is im,pa:red for the marker.s' refe:rcnce. It should not be regarded as a set of model answers.
Students and t.eachets who are not involved in the marldng process are ad'Vised to interpl'etthe Marking Schemewithcare.

X Question Solution
Earth's magnetic field
NO !. (a) Induced voltage

horizontal
• t

Figure(11) Figure (b)


A ship with a vertical abm:rinium mast sails at sea along a straight course to the west as shown in Figure (b). As a result,
an e.m..t: is induced across the mast.XY.
� /
< the induced voltage is maximwn at t= 0 > [l]
{i) Explain why it is only the horizontal component of the Earth's magnetic field that is cut by the mast which gives
rise to this induced e.m..£ (1 mark) < a complete cycle shown above is drawn> [!]

(b) Change the slip rings [!]


to a commutator. [!]
(ii) Given: lengthofmast.xi' = 20m < accept the use of diagram to show the commutator >
"speed of the ship = 6 m s-1
Earth's magnetic field= 50 µT
0 = 30°

Referring to (a)(ili), calculate the e.m.f: induced acrossXY and state whether the distribution of free electrons along
the mast is more at end X, more at end Y or uniform along XY. (3 marks)

2. (a) In figure 1, induced current flows indirection ofGFEH. [!]


(ili") Suppose X and Y are connected by a cable running side-by-side withthe mast so that they fonn a complete circuit. OR
Explain whether there will be a C'tllrent passing through it. (2 marks)
In figure 1, induced current flows in anticlockwise direction. [!]

In figure 2, no induced current flows. [!]

In figure 3, induced current flows in the duection ofHEPG. [!]


OR
In figure 3, induced current flows in clockwise direction. [l]

(b) In figure 1, an applied force along1:he direction ofmotion is required to maintain the motion ofthe loop [11
since there is an opp osite magnetic force acts on the wire GFwhen current flows in magnetic field. [11
Jn figure 2, no force is required [1]
sfuce there is no llllIIent flowing in the loop. [1]
There is question in next page
In figure 3, an applied force also along the diiection ofmotion is required to maintain the motion of the loop [!]
since there is an opposite magnetic force acts on the wire HE when cur.nmt flows in magnetic field. [!]
26. <HKDSE 2019 Ppaper-IB-9>
A reotangulerwilP.QRSof:IO turns, each having an S1eaof0.00S m', is placed in a unifonnmaglleticiieldB (c) Figure 9.3 shows a thln I<� aluminfum plale suspended by a long siting. the plare is partly
of .-gth 0.3 Tpomnng into the papor as sbown in Figuro 9.1. . inside a miliimn magnetic field )IM'ided by a strong magnet
X X ·x X X X
X uniform. magnetic field B
X X X X X X X
p
X > Q -'x
•x -
X X X X
a!Uminiumplare X X X:
ne9-l X X X X X X X
!l)re9.3 unili>rm magnetic field
X X X;
x § x x . x suddenly shifled to the right
":'
x
b...;;...., ...,�
X X X X X X X
x x x x x x x x,''
•X X ·x X X•
'
(a) The strength ofthe magne:tic field decreases uniformly to :zero wilbm 0.5 s. ., ______ . ---- .--- t

(i) Explain why a �ent would be ind uced in the coil. The magnet, wllicb is :not in conw:t wifu.11,e plai<\, is mddonly shifted'° !he right

•(ii) Calrolo!e the chan,.., in total magnotic tllllt linkage throogh the coil and the >alue of tho induced (i) On Figure 9.3, draw a ,mall circle at !he !ooatioa whe<o eddy cwrcn!S
· an, indn1'od w, !ho
e.m.f {m !he coll. (3 IIllllks) aluminium plate. Use an orrowt<> indicate 1ho diiccoon. of =reJII. (2 marl3)

(ii) Dmcnoe tho su!>seqneat moti011 oftho alumin!Ulll plare, if any. (l Ill31"k)
(b) Now lhe coil is cototed unifurmly about on axis through 130 ° as shown in Figures 92(a) and 9.2(b)
witlrin. 0.5 s.

X X X X X X �form magnetic field� X X X X X X


XXX XXXX X:<XXXXX
x'kxxxfx ��x x xfx
-------� ... --x -�--x--�- 1(--�---ft,. c:::::> ----::<'--� -X--�--,>(-- �--,>;.---· axis

XXXXX� X ]8()C' X XXXX" X


s----R P"=��"""'Q
X X-X XXXX X XXXXX X

X X :X · x X X. X x x x x x x x
F"1gUIO !l.2(a) F(gllre 9.2(b)

(i) Stare the •alue ofthe cllange in total magnotic J'itn< Iinkago through the ooil in tbis case (I marlc)
(i,) At the moment when 1he roil rotaJed tlttolJgh 90°, would the induced Cllrl10lt llow in the dm:<:tion
PQRS, PSRQ or is ll1ere no iod!lced cum,nt in tho coil? · (1 made}
DSE Physics - Section D : Question Solution PD-EMS-QS/02 DSE Physics - Section D : Question Solution PD-EMS-QS /03
EM5 : Electromagnetic Induction EM5 : Electromagnetic Induction

3. {a) Peak voltage = 3 cm x 2.5 V cm- 1 = 7.5 V [I] 5. (a) Replace the slip rings by a commutator. [2]

(b) Time taken = 4 cm x 1 ms cm- 1 = 4 ms [I]


Connect the coil to the commutator via 2 carbon brushes. [I]

(c) Frequency = _I_ [I]


4xl0-3
= 250Hz [I] [I]

4. (a)

• se,
� barmagnet

re:=]
(b) voltage
<Voltage is maxiz:num att= 0 > [!]
n�tr,!
point
;;:,--- < Both positive and negative voltage shown>
< Sh ap e correct>
[!]
[!]

�J/�
' t
<CorrectH> [!]
< Correct V> [!]
< Correct pattern.> [2]
< Indication of direction ofmagnetic field> [2]
(c) (i} The output voltage increases and the frequency also increases (OR double) [2]
<Both of the poles are South> [2]
< Neutral point between two compasses> [!] (ii) The amplitude of the voltage is unchanged but the phase is reverse. [2]
< Correct direction of current> [I] (iii} The output voltage increases (OR double). [2]
(b) {i) The neutral point moves towards X [I]

(tl) The neutral point disaPPeaIS [2]


6. (a) (i)
-�•on
(,)

[I] [2]
magnetic
field "'='

The pointer deflects to the left. [!]

Ll""'
(ri)

ma�tlc�
[2]
fiold
The pointer does not have any deflection. [!]

(b) AnyTHREEofthefollowing: [3]


[!] * Move the wire with a greater speed
* Use a stronger magnet
* Replace the wire with a coil of more number of turns
* Replace the wire with one of lower resistance
DSE Physics - Section D : Question Solution PD -EMS-QS/04 DSE Physics - Section D : Question Solution PD-EMS-QS/05
EM5 : Electromagnetic Induction E"M5 : Electromagnetic Induction

7. (a) (i) Cutrentf1owing through the solenoid will give a magnetic field to attract the magnet [l] 8. (e) Advantages: (AnyTWO) [2]
The magnet will be pulled down * Wind is cheaper
*
[1]
and the spring will cxten.d. [l ] Wind is ofunlimited supply

(:b.) Any TWO ofthe following: [2]


* Wind has no pollution

* Increase the number ofturns in the solenoid Disadvantages : (Any TWO)


*
[2]

* Use a strongermagnet
*
Unsteady output (no output when no wind)

* Use a weaker spring


*
Lawpower output
Suitable onlyfor windyplaces
(lll) The set�up can still work. [1] * Direction ofwind always changes
Since the soft iron will be magnetized by the magnetic field ofthe solenoid, (1]
attraction always�. (1]
9. (a) Whenthemagnetmovestoi.vardsthe solenoid, aCll!tentpasses through GfromXto Y. [1]
(b) (i) Yes! [1] When the magnet is inside the solenoid, there is no current. [l]
FromBtoA (1] When the magnetmoves away from the solenoid, a cutTent passes through. G from Yto X. [1]
There is a change of magnetic field and thus has an induced current. [1]
(b) (i) The plane of the coil is parallel to themagnetic field at points Q and S. [2]
(ii) No! [1]
(ii) Peak voltage = 1 x SO = SO mV (OR 0.05 V) [l]
There is no change in magnetic field at the lowest point as the magnet is momentMily at rest. [1]
Period : T = 4 x 20 x 10-3 = 0.08 i: [1]

1
8. (a) (i) A is coils.
Frequency: / = ...!. = - - = 12.5 Hz [l]
(1] T 0.08

(ii) B is carbon brush [1] (iii) (l) Peak voltage increases [l]
Frequency increases [l]
(ill) Cisconunutator (OR twosplitrings) (OR twohalf�rings) [1]
(2) Peak voltage increases [1]
(b ) Frequency remains unchanged. [1]
[2]
(iv) Any TWO of the folloWlllg: [2]

> I * Wind (ORmoving air)


* Running water
Current flows fromX to Y. [1]
* Seawave
(,) [3]

OR 10. (a) (i) End Q is a south pole. [1]

(ti) The coil rotates in a clockwise direction. [!]

(b) Eis a commutator (OR 2half-rings) (OR 2 split-rings) [1]


(d) Power= VI= (12)x(0.7) [1] It is used to reverse the direction of current flowing through the coil when the coil has rotated halfcycle. [1]
""8.4W [1] Hence the coil will continue to rotate in the same direction. [1]
I
10.
DSE Physics - Section D : Question Solution
EMS : Electromagnetic Induction

(o) AilynY.Q of the following:


PD-EMS-QS / 06

[2]
I .,
DSE Physics - Section D : Question Solution
EMS : Electromagnetic Induction

(•) Any TWO of the following:


PD-EMS-QS/07

[2]
* Increase the voltage of the battery' (OR Increase the current passing through the coil ) * coil
* Increase the nl.Ullber oftums of the coil * magnet
* Increase the area of the coil * commutator

* Increase the number of turns of the solenoid * softiron core

* Insert a soft iron core inside the paper cone


(b) When the blades are turning, the coil inside the motor will rotate [l]
* Wind the coil on a softMiron core in the magnetic field of the magnet. [l]
A voltage is induced in the coil [l]
(d) If the direction of current is reversed, the polarities of the solenoids [l]
and thus current passes through the light bulb to light it up.
and the current flowing through the coil are reversed at the same time. [l]
So the direction of rotation of the coil will remain unchanged. [l]
Hence the motor will still fimction properly for a.c. supply. [1] 13. (a) North ( OR N•pole) [l]
OR
(b) By Lenz's law, the induced current flows in a direction to oppose the change of magnetic field. [1]
If the direction of current is reversed,. the direction of current flowing through the solenoid [l]
When the south pole leaves, current is induced so that the lower end of the solenoid becomes north. [1]
and the current flowing tbmagh the coil are reversed at the same time. [l]
By Right hand grip rule, the induced CllrieUt then flows in an opposite direction through the sensor. [l]
So the direction ofthe force,s acting on the coil will -remain unchanged. [l]
Hence the motor will still :fimcti.on properly for a.c. supply. [1] (o) The magnet is accelerating. (OR The speed of the magnet is increasing.) [l]
Thus the rate ofchange of magnetic field is greater. [1]
(,)

[1]
14. (a) When the torch is shaken, the movable magnet moves across the fured coil.
The magnetic field through the :fixed coil changes (OR fue coil cuts the magnetic field lines) [l]
and a current is induced. (OR by Lenz's law, a curront is produced.) [1]

(b) Any ONE ofthe following : [l]

* Shake the torch with a greater speed.


The wires connecting the coI!ll1lUtator are disconnected from the battery. [1] * Shake the torch more times.
The other arrangements remain unchanged. [1]
(o) Kinetic energy (OR KE.) changes to electrical energy, [1]
and then changes to light ( OR light and heat) [1]
11. (a) B is South pole. [1]
(d) Because the magnet cannot leave the coil, (OR the magnet is always inside the coil) [1]
(b) WbenS is closed, a magnetic field will be built up in the solenoid. the coil experiences weaker (OR no change) ofmagnetic field. [1]
There will be an induced current flowing in the aluminium ring. [1]
<OR>
By Lenz' s Law, the induced current flows in a direction such that it produces an effect to oppose the change. [1]
Only when the magnet is entering or leaving the coil, [1]
So the end ofthe ring near the solenoid becomes a south pole.
there is a change in magnetic field. [1]
The aluminium ring will move away ftom the solenoid under the action of the repulsive force acting on it. [1]
DSE Physics - Section D : Question Solution PD-EMS-QS I 08 DSE Physics - Section D : Question Solution PD-EM5-QS/09
EM5 : Electromagnetic Induction EM5 : Electromagnetic Induction

15. (a) 17. (c) (i) The length of the trace represents the peak-to-peak voltage ofthe induced voltage in the search coil [I]
< Correct direction of current> [!] It is proportional to the peak value of the magnetic field produced by the ac. currents. [I]
< Correct poles ofmagnet> [!]
(ii) The working of the search coil would not be affected by the Earth's magnetic field. [I]
Since the Earth'smagnetic field is steady, no e.m.f. can be induced in the search coil [!]

(b) Work done by the student in pushing the magnet.

(c) * Movethemagnetmorequicltly 18. (a) g = � GM= g&; = (9.81) x(6.4x 106)2

* Useastronger magnet
R ,'

* Use a coil with more tums By G


Mm m,l
[l]
*
=
r' r
Insert a soft iron in the coil
v = ✓GrM = 9.81x(6.4xl0 6 i
< any one or two correct> [!] 6.8xl0 6
< the third one correct> [1]
= 7690 mg-1 [!]

(b) e = BvL
16. (a) WbenS is closed, current starts to flow in the coil and producesmagnetic field.
= (30 X 10-6) X (7690) X (20 X 103_) (!]
Thus an increase of themagnetic field occurs through the ring.
= 4610V [I]
By Lenz Law, an eddy cumnt is induced in the ring to oppose the change.

Opposing magnetic force acts on the ring to make it jump up momentarily. (!] Any ONE ofthe followings: [!]
However, when the current reaches its steady value, no change of.field results and the ring klls back. [!] * The cable is always peipendicu1arto the B-field

(b) (i) The ring will float steadily above the coil in air. (1]
* The magnetic field is uniform over this-20 km cable

(u, Practical use: magnetic levitation of attain (!]


* The satellite and the shuttle move with the same speed

(iii) Rate ofincrease in intemal energy = 7.8x(40-25) [!]


50
7
( a) B - ANI (41rxHr )(lxla3 )(60xIO-:i)
2.3 J S-1 [!]
19. (!]
- -,- = 0.5
= 1.51 x I0-4 T [!]
17. (a)
(b) N¢> = NBA [!]
"' (Ix 10 ) (1.51 x 10 ) (-,,; x 0.D252)
3 -4

< direction correct > [!] = 2.96x I0-4 \1/b

< pattern correct> [I]

� � 20, (a) Yes, the coil would resist being turned. [!]
When the coil is turned, a current is induced.

By lenz's law, magnetic forces acts on the coil to give a moment to oppose its rotation. [!]
= 41tx10-1xs _1__ 1
(b) B ( __)
0.02 0.07
(!]
21t (b) No. (!]
= 3.6x10-ST [1] Since there is no induced cummt through the coil, no magnetic forces act to rotate the coil [!]
DSE Physics - Section D : Question Solution PD-EMS-QS/ 10 DSE Physics - Section D : Question Solution PD-EMS-QS/11
EM5 : Electromagnetic Induction EM5 : Electromagnetic Induction

M> M 23. (a)


20. (c) e = N - = NA - = (100) (n: x 0,05 2) (0.3) = 0.236 V [1]
At At re1Drt stand
(iron)
J=!..= 0.236
[1]
R 10 alummi.Ulll low voltage
ring d.c. supply
.·. I= 0.0236A [1]

21. When the blades are tumed, the coil inside the motor will rotate in the magnetic field ofthe magnet [1]

An induced e.m.£ will be set up in the coil. [!]


connecting
The induced current flows through the bulb and lights up the bulb. [!] t...,,,.,aJ,

Kinetic energy is converted to electrical energy and then to heat and light energy. [1] < the connection of the 3 wires is ALL correct > [1]

Place the aluminium ring on the top oftbe coil through the rod ofthe retort stand. lll

22. When the switch is closed, the ring jumps up momentarily and falls down. [1]
As the aiunriniUlll ring experiences a sudden increase of magnetic field produced by the coil at the start, 111
according to Lem's Law, eddy cumnts are induced in the ring to oppose this cllange. 111
A repelling upward magnetic force then exerts on the ring to push itup.
When the current and magnetic field becomes constant, the ring :fulls back as eddy current no longer flows. 11]

(b) (i) The alllllliniumring floats above the coil in the air. [1]
< The conducting wire connected to the light-beam galvanometer > [1]
<The wire placed between the pairs of magnets> [1] (ll) The ring retnains at rest on the coil without moving up. [l]
< Two hands holding the wire> [!]
Connect the long wire to the galvanometer and place the wire across the magnetic field.

To investigate the factor affecting the induced e.m.f. : (Any TWO of the followings ) 24. (a) (i) AnyONEofthefollowings: [1]

* * The air loses its insulating properties.


*
Rate of movement
Move the wire slowly act0ss the field and then quickly across the field. The air becomes conducting.
[!]
The galvanometer would deflect more for a faster rate. [!] * Charges can pass through the air between the clouds and the Earth.

* Number of turns
(ii) E = !'.:_ [1]
Move the wire across the field. Then wind the wire into a few number of turns and move it again. [1]
The galvanometer would deflect more for more number of turns of the wire. [!] V =Ed= (3 x 105)x (2000) = 6x 108 V [1]

* llelative movement (b) (i) Magnetic field into paper [!]


Move the wire vertically down across the field and then up across the field. [!]
The galvanometer would deflect t o one side and then to the OpPosite side. [!] B = µ.I [1]
2sr
* Polarities of magnet 41txlQ- 7 X 30000
Move the wire vertically down across the field, the galvanometer would deflect to one side. [1] 21txlSOO
Reverse the polarities of magnets and repeat the experiment, the galvanometer would deflect to the opposite side. [l} 4x 10-ti T [1]
i
DSE Physics - Section D : Question Solution PD-EM5-QS/ 121 i Hong Kong Diploma of Secondary Education Examination
EM5 : Electromagnetic Induction ±6 Physics - Compulsory part (,i;,j/J-<fl¾)
Section A - Heat and Gases (�)ft' i31t)
1. Temperature,Heatandinternalen.ergy(i;.&, :Af.'f"��)
24. (b) (ii) When the lightning current isincreasing, the induced current flows in the anticlockwise d irection [1] 2. Transfer Processes (?li#:f?i&:m)
so as to oppose the increllSe ofma.gnetic field. [1] 3. ChangeofState(,e.@119i:t!!)
4. General Gas Law ( -t 3& JiliJU: #)
After reaching� the lightning current is d ecreasing, 5. Kinetic Theory (�T;;{W.,tlJ, )
the induced current flows in the clockwise direction (OR opposite direction). [I] Section B...., Force and Motion (iJ�;(:tlJ)
1. Position and Movement (-!JtJ:,fa,tjt& )
(i ii) Elec tric field [IJ 2. Newton's Laws (4:-t.ri�'/:l't)
3. MomentofForce(:'1�)
E-field increases (OR builds up) before lightning occurs [I] 4. Work,EnergyandPower(ft.;,>;, �t"'f"'.;,';41)
OR 5. Momentwn(:t;i!:)
6. ProjectileMotion(t.U#:�til)
Lightning current and magnetic field onlyexist during lightning. [I] 7. Circular Motion (/.ID,ll).;ft/,)

:::_J--;----.----; --;----;----;.;
8. Gravitation ( � J :'1 )
Section.C - Wave Motion ()£1/J)
1. Wave Propagation (i&:1194/ti!)

�-.
25. (a) (i) p -
2 Wave Phenomena (i&:W.,.!Jl..l-)

.. .. .. ..
1
[IJ 3. R eflection and Refraction ofLlght (*Im R...W Jl##)

:'
4. Lenses (.i!;l;t)
'1 5. Wave Nature ofLlght (�iroit.!;-\1:l"-fi.)

.:''
6. Sound ( 4-¼)
:
L ---;i. V , resistor

''' '
Section D- Electricity and Magnetism ('llt't"Zi)
1. Electrostatics (ff-11!:*)
I ■ • ■ ■ • ■I
�------------ -------------J 2. Electrii:: Ciicuits ( it113-)
Q'-----�---- 3. DomesticElectricity(�.§Jll 11;)
4. Magnetic Field (Zit*)
(ti) By l.eJ?Z's law, a magtietic force Fs acts on the rod to oppose its motiOD. [!] 5. Electromagnetic Induction ( 't..Jtlitl&)
6. Altemati:ngCunent(.!tl)it'll!;)
An applied force Fis needed to balance Fs to maintain uniform motion. [!]
Section E -Radioactivity and Nuclear Energy (M:M�&t;ft,;tt�)
:. F=BJL [IJ L Radiation and Radioactivity (-f!Ut'f0'1A:Af-JJ!..t.)
2. Atomic Model (l!i-f�iD
(lll) Mechanicalpowerinput: P = Fv = (BIL)v [!] 3. NuclearEncrgy(#:As)
P ower input = electrical power output
B/Lv = l;l Physics - Elective part (il!ii!-<!' 5/-)
[!]
Elective I -Astronomy and Space Science (�S.:.*'fl'>tt.�#"f>)
:. l;=BLv
I. The universe seen indifferent scales (;i:::I;J �r.1#&Tli9*'ili"W.tt)
2. Astron omythroughhistozy (�:st.*i¥,;.g;.A.i:_)
(b) (i) The horizontal COlllponent is peipendicularto the mast [!] 3. Orbital motions under gravity ( :t-fJ r 6tr.;Jr.it�f/J)
OR 4. Stars and the universe (•/I..l,,fa �'iii)
Elective 2-Atotnic World (/,f.T'l!!:-.rf..)
The vertical component is parallel o
t the mast
I. Rutherford's atomic model (.lt:!i-1:Z.&-f:tit�)
(ii) I; (Bcos30°)Lv
*
2. Photoelectric effect ( 1l!: rt_,m.)
3. Bohr's atomic model of hydrogen (Iltjjij 119 it.&-f.fit�)
- (50 X Io-6 cos 30°) (20) (6) 4. Particles or waves ($:r.-f�it)
5.12 x 10 -3 V ( accept 5.12 mV)
5. Probing into :Mll.O scale Cl1Uliii ffe'!!!:-4-)
[!]
Elective 3 - Energy and Use of Energy ($itifll,ij"6i!ltf/,J-ftffl)
Moreelectrons at endX [I] 1. Electricity at home(*,§ Jli 11!:)
2. Energyefficiency in building (Mt 1Mi{l ��rt$)
(iii) No current [JJ 3. Energyefficiency in transportation (i(.ffl-"1t6tr�,lj,tt,;J:)
4. Non -renewable energy sources (;:i;:� -ii}!!. "/Mi)
Both the cable and the mast cut the field lines, both have same e.m.f. induced 5. Renewable energy sources (""of .i!J-!!.��)
and the two induced e.m.f. oppose each other. [IJ
Elective 4 - Medical Physics ( It $#1 J.f *)
OR 1. Makingsenseo ftb.ecye(mliitrlti,'3)
2. Making.sense ofthe ear (JJ:i¥,;� '5)
There is no change of magnetic flux through the loop of the circuit. [IJ 3. Medical imaging using non-ionizing radiation (.•j/<'Ql;M/l:ti.MU-,$lHJ.,$,)
4. Medical imaging using ionizing radiation ( 'lit $#.M II,} :!f!,M{t *)
I DSE Physics - Section D : M.C.
EM6 : Alternating Current
PD-EM6-M/01
11
I DSE Physics - Section D : M.C.
EM6 : Alternating Current
PD -EM6-M/ 02 II
The following list of formulae may be found useful : 4. < HKCE 1982 Paper II-34 >

Powerin acircuit P=IV=-P.R

Ratio of secondary voltage to primary voltage in a transfonner

In the transformer shown, the ratio of the number oftums on the primazy c011 to that on the secondary coil is 10: 1. If the
power input is 50 W, find the current in the secondary coil, assunring that the transformer has a 80 % efficiency.
Part A : BKCE examination questions A. 1.0A
B. LSA

. .DO»•
1. C. 2.0A
< BK.CE 1980 Paper II• 45 >
-v- D. 2.SA

5. < HKCE 1984 Paper II - 26 >


A

B C

A$ shown in the diagram, a 200 V mains supply is stepped down to 10 V by a transformer. If the output current is 3 A and
Three coils A, B and C are wrapped around an iron core as shown. Coil A has N turns while coil B and C both have N ½ the efficiency of the transformer is 75%, what is the current in the primary coil?
A. 0.1 A
turns. Ifan input voltage Vis applied across A, what will be the voltage acting across coil C?
B. 0.2A
A. ¼V C. 0.3A
D. 0.4A
R ½V
C. 2V 6. < HKCE 1985 Paper II- 39 >

m_,_bln, _._'_"'_�I ;g�;;,


D. 4V

2. < BKCE 1980 Paper II· 34 >


300 V ' _J-----=_· _,,_,__• "_b_Io_,_-[___
'�--•-rn_,_"'_'°__
What is/are the advantage(s) of using high voltage in power transmission over long distances? The 300 V generator in the above diagram is supplying electrical power to operate a piece ofmachinery which has a rating of
(1) Current in the transmission cables can be reduced. 10 kW at 200 V. The total resistance of the transmission cables is 2 n. What is the power loss in the transmission system?
(2) Less energy is wasted as heat A. 2.SkW
B. SkW
(3) Higher voltage can be used in household circuit.
C. lOkW
A. (1) only D. 20kW
B. (3) only
C. (1) & (2) only

D
D. (2) & (3) only 7. <HKCE 1985 PaperII-41 >

1000
3. <BKCE 1980 Paperll• 41 >

.;�lye=]
Which ofthe following statements concerning direct current (d.c.) and alternating current (a.c.) is/are correct?
(1) The magnitude ofvoltage in de. isconstant while that in a.c. varies.
(2) The tlircction ofcurrent in d.c. does not change while that of a.c. reverses periodically. The diagram shows a transformer with the prilllary coil connected to an a.c. voltage of 200 V while the secondary coil is
connected to a resistor of 100 fl. If there are l 0 0 turns in the primmy coil and 10 turns in the secondary coil, what is the
(3) Both clc. anda.c. can have beatillg effect on a resistor. current passing through the resistor? (A$sume that the efficiency ofthe transfonneris 100%)
A. (I) only A. 0.0SA
B. (3) only B. 0.1 A
c. (!) & (2) only C. 02A
D. (2) & (3) only D. lA
I
s.
DSE Physics - Section D : M.C.
EM6 : Alternating Current

< HKCE 1986 Paper II - 29 >


PD-EM6-M/03
11
I
13.
DSE Physics - Section D : MC.
EM6 : Alternating Current

< BKCE 1990 Paper ll - 37 >


PD-EM6-M/04
11

A transfoaner can be used to


(1) step-up the voltage ofa given a.c. supply.
(2) increase the power ofa given a.c. supply.
(3) step-up the voltage ofagivenbattery.
A (1) only
B. (2) only
C. (1) & (2) only
D. (2) & (3) only
In the circuit shown, the efficiency ofthe transformer is 80%. If the two lamps are to work at their rated values, what is the
current i.n the primary coil and what kind oftransformer is being used?
9. < HK.CE 1986 Pa per Il- 34 >
Current in primary coil Transformer
The efficiency of a transformer is 50% and the input and output voltages are 200 V and 12 V respectively. IT the power
output is 30 W, the input current is A. 0.2A 20 : 1 step down
A. 0.15A B. 0.4A 20 : 1 step dovm
B. 0.3A C. 05A 20: 1 step down
C. I.SA D. 0.4A 10: 1 step down
D. 2.5A

14. < BKCE 1991 Paper Il- 38 >


10. <HKCE 1988 Paper II- 29 >
""'
200VD ®M
ac. boll

In the circuit diagram shown, the transformer is 100% efficient Which of the following statements is/are correct?
(1) The current in the primary coil is 05 A
(2) The resistance ofthe door-bell is 20 n.
(3) The ratio of the number of turns in the primary coil to that in the secondary coil is 20: L
A. (1) only
B. (3) only In the above circuit diagram, the transformer is 100% efficient 'What is the maximum number of identical light bulbs, each
C. (1) & (2) only ofrating '40 W, 10 V' that can be connected in parallel across the secondary coil without blowing the fuse ?
D. (2) & (3) only (Assume that the fuse will blow ifthe current flowing through it exceeds 15A.)
A 3
B. 20
11. < HKCE 1989 Paper II- 36 > C. 60
transmission cable D. 75
p Q factory

15. < HKCE 1992 Paper II - 34 >


The :figure above shows how electrical power can be transmitted to a distant factory. The transfonne.rs P and Q should be
p Q
A
B.
stepup
stepup
""'step"down
'"
C. step do'M! stop up
D. stepdown step down

U. < HKCE 1989 Paper Il - 41 >


Which ofthe follo'IVing correctly shows the major change of energy in the devices? In the circuitshown, the rating of each light bulb is '20 W, 10 V'. The cu:rrentin the primacy coil is 0.S A Ifall the bulbs
Device From To work at their rated values, find the turns ratio and the efficiency ofthe transformer.
(I) an electric motor electrical mechanical Tnrns ratio Efficiency
(2) a loudspeaker sound mechanical
(3) a traosfomter electrical electrical A 5 : 1 step down 80%
A (1) only B. 20 : 1 step down 20%
B. (2) only c. 20 : 1 step do'M! 40%
C. (l) & (3) only D. 20 : 1 step down 80%
D. (2) & (3) only
I DSE Physics - Section D : M.C.
EM6 : Alternating Current
PD-EM6-M/05
11
I DSE Physics - Section D : M.C.
EM6 : Alternating Current
PD -EM6 -M / 06 II
16. < HKCE 1992 Paper ll - 35 > 2L <HKCE1998Paperll-35>
Which of the following can increase the efficiency ofa transformer? A 4 V a.c. supply is stepped up to 20V by a transformer. If the cuttent in the primary coil is 1 A and the power loss of the
(1) Increasing the number of tums of the secondary coil. transformer is 0.8 W, find the CUIIent in the secondary coil
(2) Using a laminated iron core. A. 0.04A
(3) Using a thicker copper wires to make the coils. B. 0.16A
A (l)only C. 02A
B. (3) only D. 0.24A
C. (I) & (2) only
D. (2) & (3) only
22. < HKCE 1999 Paper ll-34>
Which of the following is/are the advantage(s) of using high voltages for long distance transmtssion ofelectricity?
17. <BKCE 1994 Paperll-34 > (1) The transmission speed c an be increased.
An electrical power of 100 kW is tranmiitted at 20 kV through cables of total resistance of 4 Q. Find the voltage drop and (2) Some heavy industries operate at high voltages.
(3) The energy loss in the transmission cable can be reduced.
power loss in the cables.
A. (1) only
Voltage drop Powerlosi B. (3) only
A. 20V 20W C. (1) &(2) only
B. 20V lOOW D. (2) & (3) only
C lOOV 20W
D. 100V lOOW 23. < HKCE 1999 Paper Il- 31 >

. u LI
18. < HKCE 1994Paper II- 35 >
In the circuit shown, the rating of th e light bulb is '40 W, 10 V'.
The efficiency of the transformeris 80%. lithe bulb works at its
Q

rated value, :find the current in the primary coil 200V �40W
, !OV
A 0.16A
B. 0.2A Two bulbs P and Q are connected to an ideal transformer as shown. 'Which of the following statements is/are true when
C. 0.25 A switch S is closed ?
D. 1.6A (l) The brightness ofbulb P decreases.
(2) The reading of the ammeter increases.
(3) The reading of the voltmeter remains unchanged.
19. <BK.CE 1996Paperll-36> A. (1) only
B. (3) only
C. (1) & (2) only
D. (2) & (3) only
200VGE'-:
90-
24. <HKCE 1999Paperll-36>
R A '6 V, 6 W' light bulb is to be operated at its rated value. Three circuits are setup as shown below.
The primary coil of a transformer has 3600 turns and is connected to 200 V a.c. supply. The secondary coil has ISO turns, (1) (2)
which can be tapped at different points as shown above. A '40 W, 10 V' light bulb is connected to the transfomter so that it r-----,.---�
works at its rated value. Which of the following statements is/are correct ?
(1) The bulb should be connected to points P and Q.
(2) The cummt through the bulb is 4 A.
(3) If the efficiency of the transfomier is 80%, the current in the primary coil is 0.25 A.
A. (1) only
B. (2) only (3)
c. (!) & (3) only Number ofturns
D. (2) & (3) only in primacy coil"' 200
in secondru:y coil = 100
20. <HKCE1997Paperll-32>
The power supplies all have negligl.Ole intemal resistance. In which of the above circuits is the bulb working at its rated
Which ofthe following devices converts mechanical energy into electrical energy when it works ? value?
A. a transformer A. (1) only
B. adynatno B. (3) only
C. amotor C. (1)&(2)only
D. a microphone D. (2) & (3) only
1 DSE Physics - Section D : M.C.
EM6 : Alternating Cw:rent
PD -EM6 -MI 07 I I DSE Physics - Section D : M.C.
EM6 : Alternating Current
PD-EM6-M/08
11

25. < BK.CE 2001Paper II -34> 29. <HKCE2006Paperll-37>

A 12 V a.c. supply is connected to a transfonnet with turns ratio 1 : 1. The secondary coil is tapped at equal intervals as

"ULi ·u� "U�


In the above circuit, the t:ransforn:l.er is ideal (efficiency= 100%). At the beginning, the switch Sis closed and the light bulbs
shown above. Two '6 V, 0.5 W' light bulbs are connected to the secondary coil of the transfonner. In which of the following X and Y are operated at their rated values. What would happen to the brightness of the bulb Xand the ;unmeter reading if S is
cases will the two bulbs work at their rated values ? now opened?
Brightness ofX Ammeter reading

....
•=•
A increases increases
B. increases
C. remains unchanged . ,,,.
D. remains unchanged remains unchanged
A. (3) only


B. (l) & (2) only
C. (2) & (3) onl y 30. < HKCE 2007Paper II-44 >
D. (1), (2) & (3)
Specification :

26. <HKCE2001Paperll-38> Voltage input 220V

The figure shows a model power line. An a.c. supply and Voltage outputs

{}1Q1{}•,v�w
6V & 12V
two ideal transformers are used to operate a lamp of rating
'6 V, 24 W'. The tot.al resistance of the cables AB and CD
is 10 .Q. If the lamp operates at its rated value, find the The above figure shows a portable transformer. It outputs different voltage by varying the turns ratio between the primary
power loss in the cables. and the secondary coils. AsSUOle that there is no power loss in the transformer and the resistance of the load remains
A. 0.4 W I ; 20 step-up 20 : 1 step-down unchanged, when the output changes from 6 V to 12 V, which of the following statements describ.ing the transformer is/are
B. 3.6W ttansfurmer kansfu�u correct?
C. 160W (1) The number of turns of the primary coil should be doubled and the number of turns of the secondary coil remains
D. 1440W unchanged:
(2) The .input current should be doubled.
27. <HKCEZOOSPaperlI-43> (3) The output power should be 4 times as before.
Vlh.ich of the following statements about long distance power transmission at high alternating voltages are correct? A (2) only
(1) Alternating voltages can be stepped up or down efficiently by transformers. B. (3) only
C. (2) & (3) only
(2) For a given traruimittedpower, the curtent will be reduced if a high voltage is adopted.
D. (1), (2) & (3)
(3) The power loss in the transmission cables will be reduced if a high voltage is adopted.
A (1) & (2) on ly
B. (!) & (3) only 31. <HKCE2008Paperll-41>
C. (2) & (3) on ly
D. (1), (2) & (3) Specification :

Input a.c. 220V


28. < HKCE2006Paper II -38>
Output a.c. 12 V, 100 mA
The photograph shows an adaptor for charging up the battery inside a mobile phone. Which
of the following statements about the adaptor are correct?
(1) It is used to step down the voltage from the mains to a value for the mobile phone. The above figure shows a transfonner. Assume the transfOI:!Ilet is ideal, what is the current drawn from the mains supply if
(2) It can convert the voltage from a.c. to d.c. the transformer delivers currents at the rated value ?
(3) It is black so that it can radiate hea.t more efficiently than those in other colours. A. 5.5mA
A. (1) & (2) only B. 100.lllA
B. (!) & (3) only C. 1200m.A
C. (2) & (3) only D. I830m.A
D. (1), (2) & (3)
I
32.
DSE Physics • Section D : M.C.
EM6 : Alternating Current

<BKCE 2010 PaperII-44 >


PD-EM6-M/09 1
1
1
35.
DSE Physics - Section D : M.C.
EM6 : Alternating Current
< HKCE 2011 Paper Il -43>
PD-EM6-M/101 1

C)1\ § § •••�··
5 A fuse Electrical power is transmitted from a power station to local substations by an altemating current and high voltage. It is
because
(1) the voltage of an alternating current can be stepped up or down using transformers easily.
(2) by using high voltages, electrical power can be transmitted faster along the cables.
(3) by using high voltages, there is less power loss in the transmission cables.
m, A. (1) & (2) only
B. (1) & (3) only
c. (2) & (3) oruy
In the circuit above, the primacy coil of a transformer is connected to the 220 V mains supply with a 5 A fuse. The efficiency D. (1), (2) & (3)
of the transfonner is 90%. What is the max.imum number of identical 40 W light bulbs, operating at their rated values, that
can be connected in parallel to the secondary coil withoutblowing the fuse?
A. 24
B. 25 Part B : HKAL examination questions
C. 27
D. 28 36. <RKAL 1984Paper! - 26>
A power station supplied electrical power to a user. The power generated by the station is 1200 kW. After stepping up, the
voltage transmitted to the cable is 132 kV. If the total resistance of the transmission cable is 550 0, :find th: electrical power
33. < BK.CE 2010 Paper Il- 43 > available to the user.
mcta1 "'" A. 700kW
B. 1155kW
C. 1195kW
D. 1200kW

I •="If----====0
a.c. supply "v coil A coilB a.c. voltmeter
37. <BKAL1995PaperllA-30>

The figure shows a metal core with two coils. When the switch is closed, the a.c. voltmeter shows a reading. Which ofthe
�@m·
following combinations will give the largest voltmeter reading?
A large machine in a factory consumes 10 kW of electrical power at a voltage of 500 V. Ifth.e generntor from the power
station transnrits electrical power to the :fuctozy through cables oftotal resistance 0.2 n, the voltage produced by the generator
""""
material of the metal core no. of turns of coilA no. of turns of coil B
should.be
A. 500 1000 A. 500V
B. copper 1000 500 B. 501V
c. ;ron 500 1000 C. 502 V
D. iron 1000 500 D. 504V

38. < BKAL 2008 Paper IlA-19 >


34. <BKCE 2011 Paperll-42>
When240 V a.c. is applied across PQ ofan ideal transfonner, the voltages
measured across RS and ST are 4V and 8V respectively. Jf6V a.c. is now
applied across ST, what will be the voltages measured across PQ and RS?
s "\
Voltage across PQ Voltage across RS
A. ov ov
B. 180V ov
c. ov 3V
D. 180V 3V
The figure above shows an ideal transfooner. The primary coil of the transformer is connected to the 220V a.c. mains and 39. < HKAL 2013 Paper IIA- 34>
two lamps L1 and.Lz are connected lll series to the secondary coil. Initially, Sis closed. Which ofthe following statements is
correct when Sis opened 'l The electrical power dissipated by a heater when connected to a 10 V d.c. supply is two times of that when the heater is
A. The current in the primary coil of the transformer decreases. connected to a sinusoidal a.c. supply. What is the peak voltage ofthe sinusoidal a.c. supply 'l
A. 5V
B. The voltage across the secondary coil of the transformer increases. B. 5-,/2
C. The brightness ofL1 decreases. C. lOV
D. The brightness ofLz increases. D. 10.fi
I DSE Physics - Section D : M.C.
EM6 : Alternating Current

Part C : HKDSE examination questions


PD-EM6-M/ll
ll
I
44. <BKDSE20l7PaperIA-29>
DSE Physics - Section D : M,C.
EM6 : Alternating Current
PD-EM6-M/12
11

A heater of resistance 100 Q is connected to the mains supply. The r.m.s. voltage of the mains supply is 1 IO V. Which of the
40. <HKDSE Sample Paper IA-33 > following statements are correct?
Power is transmitted over long distances at high alternating voltages. Which statements are cOirect? (1) The peak voltage across the heater is 156 V.
(2) The power dissipated by the heater is 121 W.
(I) Alternating voltages can be stepped up or down efficiently by transformers.
(3) The power dissipated by the heater will be doubled if the r.m.s. voltage of the mains supply doubles.
(2) For a given transmitted power, the current will be reduced if a high voltage is adopted.
A. (1) & (2) oruy
(3) The power loss in the transmission cables will be reduced ifa high voltage is adopted. B. (1) & (3) ooly
A (1) & (2) only C. (2) & (3) only
B. (1)&(3)only D. (1),(2)&(3)
C. (2) & (3) only
D. (1), (2) & (3)
45. <BKDSE2017Paper1A-30>
The input tenninal of a transformer is connected to the 220 V mains supply. Ten identical light bulbs are connected in
41. <HKDSE2014Paper1A-30> parallel to the output terminal ofthe transfonner. All the light bulbs are working at their rated values of '3 V, 1.5 W'. Ifthe
When a heater is connected to a d.c. voltage oflO V, the power dissipated is P. If the heater is connected to a sinusoidal a.c., efficiency of the tronsfonner is 70%, what is the cunent drawn from the mains supply?
½
the power dissipated becomes P. What is the r.m.s. voltage of this a.c. sow-ce? Assume that the resistance of the heater is A. 0.007 A
constant B. 0.048A
C. 0,068 A
A. ✓5V
D. 0.097 A
B. 5✓2 V
C. IOV
46. <BKDSE2018 Paper IA- 30>
D. 10.,/i V
In the circuits below, if a 12 V sinusoidal a.c. is applied across ab and across xy respectively, the voltages across cd and zw
are both 6 V. Now if a 6 V sinusoidal a.c. is applied across cd and across zw r espectively, what would be the voltages across
42. < BKDSE2016 Paper IA-30 > ab and xy respectively ?

A sinusoidal a.c. of a certain frequency delivers a r.m.s, voltage V,.m.,., If its frequency is doubled and its peak voltage is
halved, what would be the r.m.s. voltage ?
A. .!. v.,.,,,_,,
2
. 1
B ✓?.V,�
C. -1-v-- voltage across ab voltage across xy
2./z · A. 12V 12V
D. V,..,,,,. , B. 12V 6V
C. 6V 6V
D. 12V ov
43. < HKDSE 2016 Paper IA - 31 > 47. <HKDSE 2019 Paper IA-29>

In. the above circuit, each light bulb works at its rated value '22 W, 11 V'. The current in the primary coil is 0.25 A. Find the
efficiency of the transform.et.
A. 20%
B. 40%
C. 64%
D. 80%
48 . <HKDSE 2019 Paper IA-30>

48


49. <:HKDSE 2020 Paper IA-29>

12 Va.� ""
□ 6V,0.SA

JV,I.OA

The � shows an ideal transfo<meT with two seeonda,y coils - to two light bulbs matked
'6 V, O.S g and '3 V, J.O N,espectivoly. Whon a 12 V a:c. supply is =ect<d to the primary coil, the
bulbs Workattheir respective rated values. Estimal, the currentin the primary coll.

A. 0.25A
B. 0.S0A
C. 0.7SA
D. 1.0A
DSE Physics - Section D : M.C. Solution PD-EM6-MS I 01 DSE Physics - Section D : M.C. Solution PD-EM6-MS/02
EM6 : Alternating Current EM6 : Alternating Current
HKEAA's Marldng Scheme is prepared for the�• reference. It should not be regatded as a set of model answers.
Students and teachers who are not involved in the marking process are advised to intetpre;t: the Marking Scheme with care. 4.

M.C. Answers :. 80%aas�


(50)
P�"' =40W

1. B 11. B 21. B 31. A 41. B


2. C 12. C 22. B 32. A 42. A
3. D 13. C 23. D 33. C 43_ D
5. B
4. C 14. D 24. B 34. A 44. A P., V,l,
7J = P"", :o :. Ip= 0.2A
5. B 15. D 25. D 35. B 45. D � IP

6. B 16. D 26. A 36. B 46. B 6. B


JO
7. C 17. B 27. D 37. D 47. C
8. A 18. C 28. D 38. D 48. B
,oov9'--=�b==----11�R� },oov
9. B 19. D 29. C 39. C 49. B

10. D 20. B 30. B 40. D 1n


Let the resistance of the machineiy beR.
Since the cables and machinery are in series, they have the same cunent I.

M.C. Solution
2
R = V:,2 = (200) = 4.Q l=�=SOA
P, oox!o'J 2+4
OR
I.
By P"" VI :. (JO X 103) = (200)/ :. J=SOA
Power loss in the cables: PJ,,.. = 12 R = (50)2 (2) = 5000 W = 5 kW
=
Ve !:'.:.
2
7. C

By!i= Ns -5___=� Vs= 20V


2. C VI' Np (200) 100
✓ (I) Ifhigh voltage is used, the cunent through the transmission cables would be small By Vs = IsR I, 0.2A
(20) a I, (100) =
byP=VI :. Vt=> J.J,

✓ (2) ByP=I 2R :. JJ⇒Pi.,..t 8. A


:. less energy is wasted as heat during the transmission ofpower ✓ (!) Transfonner can step-up the voltage ofa.c.
(3) Voltage in power transmission >> voltage in household circuit • (2) Traru.fonner cannot be used to increase the power.
:. voltage need to be stepped down
• (3) Transform.er cannot work on d.c., but battery gives d.c. steady voltage•

3. D
9. B
• (!) The magnitude of voltage in d.c. may not be constant, such as that produced by d.c. generator.

✓ Electron flows in the opposite direction of C\lIICOt.


77 = P,,..
Pm
=�
Vinlw.
(2)
In de., since the direction ofcunent does not change, direction of electron flow does not change.
50% = � I;,, = 0.3A
✓ Current has heating effect on resistor, which does not depend on the direction of current (200)1,
DSE Physics - Section D : M.C. Solution PD-EM6-MS/03
EM6 : Alternating Current
10. D
• (1 ) For 100 % efficiency, (200)Jp = (10)(0.5) :. ]p = 0.025 A

✓ (2) R=K=.!.Q.;,2QQ
I 0.5

✓ Nt>=!'.!.=200=ZO
(3)
N, v,
10

11. B
High voltage is used in electricity transmission, thusP should be a step up transfoz:merto step up the voltage in cable.
After transmission, the voltage has to be stepped down to be used in factory, thus Q is a step down transformer.

12. C
✓ (1) motor : electrical energy ➔ mechanical energy
• (2) lQUdspeaker : electrical energy ➔ sound
✓ (3) transform.et : electrical energy 4 electrical energy

13. C
Jp = 0.5A

Tul1ls ratio in tn:msfonner : !!.:,_ = � = 200 = 20


N, V. 10
It is a step dovm transformer since N1 > Ns.

14. D
.-. (200)(15) - n (40) n = 15

15. D
Tums ratio: Vs = rated voltage = 10 V :. NI' = 5..
= 200 = 20
Ns Vs 10
Al.the voltage cbangesfrom200 V to 10 V, itis a step down transform.er.

Efficiency : = (20x4)
=SO¾
n (200)(0.5)

16. D
• (1) If number of turns of the secondaly coil is increased, secondary voltage is inctealled,
however, power loss would notdecrease, thus the efficiency remains unchanged.
(Although power output :increases, butpowerinput also increases to give the same efficiency.)
✓ (2) Iflaminated iron core is used, eddy ClJifCilt can be reduced, thus power loss is reduced.
✓ (3) If tbicke_r copper wire is used, the resistance of the coils is reduced,
thus beating effect of cunent in the two coils is reduced, sopower loss is reduced.
DSE Pbysics-SectionD: M.C. Solution PD-EM6-MS/05 DSE Physics - Section D : M.C. Solution PD-EM6- MS/ 061
EM6 : Alternating Current EM6 : Alternating Cutrent

24. B 30. B
X (1) Voltage across bulb = 12 V ¢ 6V, thus the bulb cannot work at its rated value. X (!) By Vs: Vp = Ns :N,, ifthe number ofturns oftheprimaiycoil is doubled,
the output voltage would change from 6 V to 3 V.
X (2) Resistanceoftheliglrtbulb: R = Y:_ = (6/ = 6Q
P, ( 6) X (,) Since the output voltage is doubled, by Po,ai = Vi IR, output power Pom becomes 4 times.
Equivalent resistance of the bulb and the left resistor = � = 3 n As there is no powerloss, Pm "" PctJI., thus input power P.m. also becomes 41:hnes.
2 By Pm = Vp /p , as Vp is unchanged, I, should be 4 times.
Thus, voltage across the light bulb is 4 V, the light bulb cannot work at its rated value.
✓ (3) By P = Vi IR, when Vs is doubled, the output power becomes 4 times,
✓ (3) Secondaryvoltage: p; = !OO xI2 = 6V the light bulb work at its rated value.
200
31. A
Since the transformer is ideal, the efficiency is 100%, i.e. powerinput is equal to power output
25. D
Vi•fp = Vsls
✓ (I) The output voltage is 12 V each light bulb shares 6 V work at rated values
(220 VJ I, • (12 VJ (100 mA) :. /p = 5.4SmA"' 55mA.
✓ (2) Two intervals give an output voltage of 6 V work at rated values

✓ (3) Each light bulb is connected across two intervals :. each has an voltage of6 V work at rated values 32. A
Assume the maximum number of light bulbs is n. The maxi:nlum primary CUJ:Telltis 5 A whichis limited by the fuse.

26.
For the lamp to operate at its rated value, the current through the lamp should be 24 = 4 A
6
:. (90%)=� :. n = 24.75
As it is a step down transformer, the current through the wireABCD should be : = 02 A (220)( 5)
2
The maxunutn number oflight bulbs should be 24 so that the fuse would not blow.
Power lossinthecables = I2R = (0.2)2 (10) = 0.4W
33. C
27. D The metal core should use iron so that there is good magnetic flux linkage between the tvro coils.
✓ (I) Power transmission needs transformers for stepping up and down, and transformers work on a.c. only To give larger voltmeter reading, the transf'ormer should be stepped up.
✓ (,) For a give transmitted power, P = VI, thus current I is reduced if voltage Vis increased. Thus, number ofturns in coil A is 500 and number of turns in coil B is 1000 can give the largest voltmeter reading.
✓ (3) By Pi... = I1 R , reduced cunentresulting from high voltage gives smaller power loss.
34. A

28. D Before Sis open,L1 is shorted and thus only I,,, lights up. The resistance isR.

✓ Since the voltage ofa mobile phone is very low, about a few volts, After Sis open, both lamps light up and the equivalent resistanceis 2R, thus the secondacy eumnt decreases.
the adaptor must contain a step-down transformer for stepping down the voltage. ✓ A. Since ls :/p = Np:Ns, with. the same turns ratio, the decrease ofls gives the decrease ofI,.
✓ (,) Since the battery ofa mobile phone is d.c., the adaptor must convert the a.c. voltage of the mains to d.c. X B. The secondary voltage should re.main unchanged as it depends on the turns ratio and V1 only.
✓ (3) Since traru:fotmer would give out heat when it operates, X C. The brightness ofL1 should increase as it is not lit up initially.
black colour can increase the radiation ofheat since black surface is a good emitter of heat.
X D. The brightness of L,. should decrease as the cuttent flowing through it is decreased.

29. C B
35.
(I) Since the secondary voltage of the transformer remains unchanged,
✓ (!) Since 1:ransfonners work on a.c., thus a.c. voltage can be changed easily.
the power dissipated by bulb Xis unchanged, thus the brightness of Xremains unchanged.
X (2) Electrical power istr.msmitted with the speed of light, and is independent of the voltages.
(2) Since secondary current is decreased, by !.£ = Np = constant, primary CUITent should also decrease.

Ip N3 (3) By using high voltage, current in cables is reduced, thus less power is lost in the transmission cables.
DSE Physics - Section D : M.C. Solution PD-EM6-MS /07 DSE Physics - Section D : M.C. Solution PD-EM6-MS/08
EM6 : Alternating Current EM6 : Aliemating Current

36. B 42. A
3
= !_ = 1200xl0 = 9_09 A By .., = v•
I
V 132000 ,,...,,,,_ ✓2
Thus, ifthe peak voltage is halved, the r.m.s. voltage is also halved.

= (1200x1D3) - (9.09)2(550) = 1.155x10 6 W = 1155kW


43. D
- Pou1 P.o1 - (22x2)
37. D = 80%
'ff - Pm - - Vp Ir - (220)(0. 25)
0 03
I=!_= (1 x1 ) = 20A
V (500)
44. A
Voltage across the cables = (20) x (0.2) = 4 V
✓ (!) Peakvoltage:V. = ✓2v..,. = ✓1 x110 = 156V
E.m.f. produced by generator = 500 + 4 = 504 V
✓ (2) Power: P = v,.,,,,,t IR = (110)2 / (100) = 121 W
38. D (3) Since P «: V2, if V doubles, power P should be 4 times.
Since the voltage is directly proportional to the number of turns,
Npq : Ni,.s : Nin = 240 : 4 : 8 = 60 : 1 : 2 45. D
If Vsr = 6 V, then ViQ : Vas : 6 = 60 : 1 : 2 By 7J = � = pout
V,,Q = 180V and VRS = 3V Po, V1n l1n
1.SxlO)
?O¾ = (
39. C (220)Ir
yz )2 Ir = 0.097 A
Ford.c.: Pi= = (10
R R
J7 ' 46. B
For a.c. : P2 = �
For the transformer, if the input voltage 12 V a.c. is applied across ab, the output voltage across ab is 6 V.
Thus, the turns ratio is 12 : 6 = 2 : 1 and it is a step,-down transformer.
!!. = (10): =2 :. V..,. = 7.07V
Pi v..,.
Peakvoltage: Vo= .fi, V""" = ✓2 x7.07 = lOV

40. D
✓ (!) Transformers work on a.c. effic iently. Ifthe input voltage is 6V a.c. a.cross cd, since now the turns ratio is l ; 2,
✓ (2) By P = VI, for the same P, higher V means that smaller transmission current I. the output voltage across ab is 12 V and it becomes a step-up transformer.
✓ (3) By P1oa = 12 R, smaller J means that Pi- can be reduced. For the potential divider, ifthe inputvoltage 12V a.c. is applied across xy, the output voltage across zw is 6 V.

41. B
l2V_::J::z
v'
Ford.c.: P = - .·. (F/ - �O)'
R R -----;,- 6V
y w
Fora.c.: P = V,,.. •.• (1.P) Vm/
2
z = Ifthe inputvoltage 6 Vis applied across zw, since current only flows through the lower part ofthe resistor,
R R
no currentflows to the upper part ofthe resistor, the voltage across the upper part of the resistor is O V,
Combine the two equations : ...!... = (10 )2
Ct ) (V..,. )
2 thus, the voltage across xy is still 6 V.
I DSE Physics -- Section D : Question
EM6 : Alternating Current
PD--EM6-Q/01 I DSE Physics -- Section D : Question
EM6 : Alternating Current
PD-EM6--Q/021

The following list of formulae may be found useful : 2, (b) Find the frequency of the output voltage.

Power in a circuit P=IV=I1 R

Ratio of secondary voltage to primacy voltage in a tr.msfonner

3. < HKCE 1982 Paper I- 9 >


The following pieces of apparatus (as shown in the below .figure) are provided : an a.c. power supply, a C.RO. and a
transformer.
Part A : HKCE examination questions

I. < BKCE 1980 Paper I - 9 >


The figure below is a block diagram showing how electric power is supplied to consumers.

a.c. power supply CRO transformer


(a) One source of energy is from wind. State the conversion of energy in the generators and descnbe how the energy
conversion is possiDle. (3 marks)

In the above diagram, show how you would connect the above pieces of apparatus to display a 10 V a.c. on the CRO.
(6 marks)

(b) Why is the efficiency of a transformer always less than I 00% ? Mention one method of improving transformer
efficiency. (4marks)
4. < BKCE 1983 Paper I - 7 >
(a) 22000 W of electrical power are transmitted through a cable with a resistance of0.5 n at 11000 V.

(i) Find the current passing through the cable.

(c) Explam why a 40 W fluorescent tub e appears to be brighter than a 40 W filament lamp. (2ma,k,)

(ii) Find the power lost in the cable during ttllllSillission.

L_J L__:rta·•
2. < HKCE 1981 Paper I - 7 >

�� (rli) Why should a high voltage be used in power transmission ? (2marks)


200V
50Hz

The primazy coil of a transformer is connected to a 200V, 50 Hz a.c. mains supply. Suppose the primary coil has 2000 turns
and the secondary coil has I 00 turns.
(b) DescnOe briefly how an alternating cunent of 11000V can be stepped down to 200V.
(a) Find the output voltage across the secondary coiL
I
5.
DSE Physics - Section D : Question
EM6 : Alternating Current
< HKCE 1987 Paper I- 8 >
PD -EM6-Q / 0311
I
6.
DSE Physics - Section D : Question
EM6 : Alternating Current
< BKCE 1990 Paper I - 7 >
PD-EM6-Q / 0411
The figure shows an arrangement to study the input and output characteristics of a transformer. z
The below figure shows a 3-pin electrical plug and a transformer. The three wires X, Y, of the transformer are to be
connected to the plug. The plug will be connected to a 200 V a.c. power supply. The output voltage of the 1ransfonner will
be 110 V. Assume the transformer is 100% efficient

X (Green/yellow)

The readings of the ammeters and voltmeters are as follows: Plug


Y(Blue) -�1-1::-­
Reading ofA1 = 2.5 A Z(Brown)

Reading ofAz = 1.8 A (a) (i) To which of the tenninalsA,B, and C of the plug should each of the wires.x; YandZbe connected 7 (2 marks)
Reading of Vi = 12 V
Reading of Vi = 2 V

(a) (i) Calculate the power input of the transfonner. (1 mark)


(ii) Explain briefly why the fuse should be connected to the terminal B. (2marks)

(ii) Calculate the power output offue transformer. (! rrwk)

(iii) Suggest one reason why it is necessary to have the X-wire connection. (2 marks)

(fu) Calculate the efficiency of the transformer. (1 mark)

(iv) Find the turns ratio (primary coil to secondary coil) of the transformer.

(b) Suppose R is now replaced by another resistor ofbigher resistance.

(i) How would the readmg of the ammeter A1 change 7 (! rrwk)


(b) An iron of rated values "11 0 V, 1100 W'' is connected to the output of the transformer and switched on for half an hour.

(i) Calculate the current drawn from the transformer by the iron. (2 marks)
(ii) How would the reading of the ammeter A2. change ? (1 mark)

[ii) Calculate the cost of electricity if one kilowatt-hour of electrical energy costs 80 cents. (2nwks)

(c) Suggest TWO changes in the transfonner which will improve its efficiency. In each case, give ONE reason to support
your suggestion. (4marks)

(iii) If fuses marked 1 A, 3 A and 7 A are available, which one is most appropriate to be used in the plug in the figure 7
Explain your choice. (3 marks)
7,
DSE Physics - Section D : Question
EM6 : Alternating Current

< HKCE 1995 Paper I - S >


PD-EM6-Q/05

I
8.
DSE Physics - Section D : Question
EM6 : Alternating Current
< HKCE 1991 Paper I - 7 >
PD-EM6-Q/06
11

Two students suggest using a 24 V d.c. supply and a 24 V a.c. supply sepatately to operate a l.antpXrated at '6 V, 12 W'.

�� ,11---�
(a) 24V

Two long wires AB and CD of total resistance 4 .Qare used to connect a d.c. power supply to a lamp. The lamp is working at
its rated value '12 V, 24 W'. X
R
(a) Find
(i) the resistance ofthe lamp, (2 marks)
A student connectsXin series with a 24 V d.c. supply and a resistor R (see the Figure above). IfXworks at its rated
value,
(ii) the current flowing through the lamp,
(i) find the current flowing through X,

(rii) the power loss in the wires, (2 marks)

(ii) find the voltage drop across R, (l mark)


(iv) the efficiency ofthe circuit supplying power to the lamp. (2 marks)

(iii) find the resistance of R, (2 marks)


(b) To reduce the power loss in the wires, an a.c. power supply and two transformers are used as shown in the figure below.

(iv) what percentage of the electric power provided by the d.c. supply is dissipated inR 7 (3 marks)

(i) In the figure below, draw wires to connect the terminals of the components according to the figure above.
(3 marks)
{b) The other student suggests thatX can also be made to work by using a 24 V a.c. su.pplytogetherwifu a transformer.

(i) Draw a circuit diagram to show how X, the a.c. supply and the transformer are connected.

�Traosfurm� �

b1Z!l) 1,mp
Power supply

(ii) What is the advantage of using this method over the one shown in the above Figure?

(li) Explain how the ammgement can reduce the power loss in the wires. (3 marks)
(iii) Determine the tum.s ratio (primary to secondary) of the transformer for Xto work at its rated value, and calculate
the primary CIIrrcnt if the transformer is 100% efficient (4 marks)
I
9.
DSE Physics - Section D : Question
EM6 : Alternating Current
< HKCE 2000 Paper 1-10 >
PD-EM6 -Q / 0711

(a) A transform.eris used to operate a '110 V, 1000 W' electric cooker at its rated value from the 220 V a.c. mafus supply in
10.
DSE Physics - Section D : Question
ElYI6 : Alternating Current
< BKCE 2001 Paper I· 5 >
PD-EM6-Q/081

Hong Kong. The primary coil of the transfooner bas 5000 tw::ns and the efficiency of the transfonner is 80%. Find ""'"""'"'
(i) the number ofturns in the secondary coil of the transformer, (2marks)
transmission cables

(u1 the operating resistance of the cooker, (2 marks}


The above figure shows how electrical power generated in a power stationis transmitted over long distances to consumers.

(a) State the function of the transformer T1 .

(lll1 the power input of the transformer,

(b) Explain why a.c. and high voltages are used for long distance power transmission. (3 marks)

(iv) the cunent flowing in the primary coil of the transformer. (2 marks)

(b) Figure 1 shows a travel cooker and the label attached on it. The cooker has a voltage selector switch shown in Figure 2.

Voltage
Model No : EA 2000 11. < HKCE 2004 Paper I-10 >
a.c. 120 V / 240 V
360W
~50-60Hz
120V� 240V
Figure2

(i) A fuse is installed in the cooker. Explain the :function of the fuse. (2 """') ll

el�,,
(ii) Two students make the following remarks about using the cooker in Hong Kong:
John: The voltage selector switch should be set to 120 V and the output ofthe cooker would be 360 W. s

Peter: The voltage selector switch should be set to 240 V and the output ofthe cooker would be less than 360 W.
Explain whether each ofthe above remarks is correct. (4 marks)

The Figure above shows an electric toothbrush. It consists ofa brush unit and a charging unit.
DSE Physics - Section D : Question PD-EM6-Q/09 DSE Physics - Section D : Question PD-EM6-Q/10
EM6 : Alternating Current EM6 : Alternating Current
11. (a) Inside the brush unit, there is a 12 V rechargeable cell for driving a motor. When the toothbrush is in operation, the 12. < HKCE 2005 Paper I -12 >
current flowing through the motor is 1.8 A. Calculate

(i) the power consumed by the motor, and

(ii) the energy consumed bythe motorin.3 minutes. (4marks)


Figure 1
a.c. power supply

Josephine conducts an investigation on tnmsfonneIS. She sets up a circuit as shown in Figure 1.


(b) When the energy stored in the cell has been used up, the brush unit is placed on the charging unit to recharge the cell.
The charging unit is connected to the mains supply and its label is also shown in the above Figure. It takes 16 hours to (a) Josephine varies the input voltage Vi to the tmnsfonnet and records the corresponding output voltage V2- The results arc
rech arge the cell fully. Calculate the energy drawn by the charging unit from the mains supply in 16 hours. (2 tnarks) shown in Table 1. InFigure2,plot agraph of Piagainst Vi.

Hence draw a conclusion for this investigation.

Vi/V Pi/V
(c) The cell inside the brush unit is connected to coilXIocated at the bottom of the unit. Another coil Y is located inside the 1.0 1.7
charging unit with a soft..iron bar fixed inside it When the brush unit is placed on the charging unit, the soft-iron bar 2.0 3.3
lies inside coil X.
3.0 5.1
(i) The brush unit and the charging unit are completely covered by plastic cases and there is no metal contact between 4.0 6.9
them. Explain how a current is produced in the brush unit to recharge the cell (3 marks)
Table 1 Vi/V
0 4

(2marks)

(b) Josephine wants to studythe relationship between the output voltage and the number of turns in the secondary coil of the
(ii) If coil Yhas 11 000 turas, estimate the number of turns of coil X. Assume the output voltage of coilXis 3 V a.c. transfonner. Descn'be how she can conduct the experiment. (2 marks)
(2marks)

(,)

(ili) State the function of the soft-iron bar. (1 mark)

Figure3

f
i
(d) The charging unitis fitted with a two-pin plug as shown in the above Figure.

(i) To which two wires of the mains supply should the pins of the plug be connected? (1 mark) secon�
y
,oil
Josephine adds a bulb to the circuit as shown in Figure 3 above. Suggest a method that Josephine can use to esti mate the
efficiency ofthe transformer. Additional apparatus may be used if necessary. (3 marks)
(ii) Suggest one reason why it is safe for the charging unit to be fitted with a two-pin plug. (I mark)
1 DSE Physics - Section D : Question
EM6 : Alternating Current
PD - EM6 - Q / 11 II DSE Physics - Section D : Question
EM6 : Alternating Current
PD-EM6-Q/12

13. < HKCE 2007 Paper 1-12 > 14. <BKCE2009Paperi-u>


The Figure below shows a setup to generate electricity. A magnet is set into rotation between two fixed solenoids. The The Figure below shows the power transmission of a bell system in a school. A signal generator in the school office produces
output ter:minalsX and Yare connected to a light bulb. a 1O V a.c. signal and is connected to HK as shown in the figure. The signal is then stepped up to 100 V and transmitted to a
classroom through a lo ng transmission cable. Assume all transformers are 100% efficient.

'
axis ofrotation

fixed solenoid

\
�/ fixedsolenoid

cp:,,
,
/
I I

G s0
/

\
, \
,/
/
/
,/
magnet
/

(a) The primary coil of transformer Xhas 500 turns. Find the number of turns in the secondary coil. (1 mark)
X
0

(a) Explain how altemating cunent is generated in the above setup. (3 marks)

(b) Each wire of the transmission cable, MP and NQ, has a resistance of 80 n. Assume the current through the wires
is 0.1 A.

(i) Find the voltage across MP. (2 marks)

(b) The bulb is now removed from the setup. X and fare then connected to the primary coil of a transformer. The
secondary voltage output of the transformer is found to be 12 V. If the toms ratio of the primary coil to the secondary
coil is 1 : 8, find the primary voltage. (2 roarlcs)

(ii) Find the voltage across PQ. (1 mark)

(c) State the advantages of using (iii) Find the power delivered to the loudspeaker. (2 marks)

{i) a.c., and

(ii) highvoltages

for long distance power transmission. (2 marks)


(c) Suggest TWO methods to increase the power delivered to the loudspeaker with the same 10 V a.c. signal. (2 marks)
DSE Physics - Section D : Question
EM6 : Alternating Current
PD-EM6-Q/131

I DSE Physics - Section D : Question


EM6 : Alternating Current
PD-EM6-Q/14 II
15. < HKCE 2011 Paper I -10 > Part B : HKDSE examination questions
(a) A magnet is dropped through a copper coil C1 as sho'Wll in Figure (a). C1 is connected to a resistor.

:y1
16. < HKDSE Sample Paper m -13 >
Josephine conducts an investigation on transformers. Primary and secondary coils are wound on two soft-iron C-cores to
fonn a transformer. She set up a circuit as shown in Figure 1.

Figure(a)
�c,
transformer

L_j,l�tierub, Figure 1

'---��--, :
a.c. power supply
(i) What is the magnetic pole induced at end U of Ci as the magnet approaches it? (1 mark)

1 ':
secondary
coil
(ii) Iftbe resistor is disconnected and the circuit becomes open, it is found that the magnet passes th.rough Ci in shorter
time. Explain this phenomenon. (2 marks)
(a) Josephine varies the input voltage V1 to the transformer and records the corresponding output voltage V2- The results are
shown in Table 1. Figure 2 shows the graph of Vi against Vi. Draw a conclusion for this investigation. (1 mark)

(b) Now, C 1 and another copper coil of smaller number ofturns, Ci, are wound on a soft iron core as shown in Figure (b) to
make a transformer. Ci is connected to an a.c. supply.
p Vi/V Vi!V
a.c. supply,,.....,_ C, 1.5 2.5
3.0 5.1
soft iron core 4.5 1.6
Figure(b) 6.0 10.0
c,
Table 1

(i) What is the use ofthe transformer? {l mark)


Figure2

(li) Suggest two ways to improve the efficiency of the transformer. (2 marks)

(b) Deduce the value of V2 that will be produced when Vi equals 8.0 V. (1mm)
(iii) Another power supply is connected to PQ to replace the a.c. supply. The variation of the current through C2 with
time is shown in Figure (c). A CUIIent flowing from P to Q through Cz is taken as positive.
I/A

(c) Josephine wants to study the relationship between the output voltage and the number of turns in the secondaiy coil of the
Figu.re (c) � transformer. DescnOe how she can conduct the experiment. (2 marks)

0 » t Is
Tom claims that no induced current will be produced in Ci as the current in C2 has no change in direction.
Comment on whether Tom's claim is correct or not (3 marks)
I DSE Physics - Section D : Question
EM6 : Alternating Current
PD - EM6 - Q / 15 II DSE Physics - Section D : Question
E1Vf6 : Alternating Current
PD - EM6 - QI 16

16. (d) 18. <HKDSE Practice Paper IB-9 >


12 V d..c. power supply

Flgure3

'"'"'
joulemeter

Suppose now the heater is connected to a sinusoidal a.c. power supply. The peak value of the voltage of the a.c. power
Josephine adds a bulb to the circuit as shown in Figure 3 above. Suggest how Josephine can estimate the efficiency of supply is 15 V. How would the output power ofthe heater change? (2 marks)
the trans.funner. State the measurement(s) she must take. Additional apparatus may be used if necessary. (3 marks)

19. < l[KJ)SE 2012 Paper IB - 9 >


17. <llKDSE PracticePaperIB-10 >
Read the following description about the 'shaver supply unit' in bathrooms and answer the questions that follow.
Read the following passage about ignition coils and answer the questions that follow.

The danger of electric shock is particularly high in bathrooms. Normal electric socket outlets should not be installed
Ignition coil in bathrooms. As electric shavers and toothbrushes are becoming popular these days, a special unit, called 'shaver
supply unit' is now conunon in bathrooms to provide electricity just fur these low power comu:mption electric
An ignition coil is used to produce sparks from the battery of a car In ignite the fuel in the engine. It is
appliances (see the Figure).
used to produce high-voltage pUIBes from a low-voltage d.c. supply.
The shaver supply unit consists of a transformer in which the secondary is not earthed and is completely isolated
An ignition coil consists of two coils of insulated copper wire that are wound around a common iron core. from the 220 V a.c. mains supply connecting to the primary. It can be used with 220 V or 110 V shavers.
One coil, called the primary coil, is made from relative ly few (tens or hundreds) turns of thick copper wire.
The other coil, called the secondary coil, typically consists of many (thousands) turns of thin copper wire.

When an electric current is passes through the primary coil, a magnetic field is created. The iron core Llvo JSharupEp
vo, !yunlt
guides most of the primary cell's magnetic field to the secondary coil. When the current in the primary coil
is suddenly interrupted, a high voltage pulse of many thousand volts is developed across the secondary
coil. This voltage is then sufficient to cause an electrical discharge to produce a spark 220 V a.c.

(a) Explain why a voltage is developed across the secondary coil when the current in ptim.aty coil is suddenly interrupted. Neutral
(2 marks)

(a) Explain why the chance of electric shock is high in bathrooms.

(b) Suggest one reason why the voltage developed across the secondary coil is v,:;ry large.

(b) Explain what would happen if the human body touches


(c) Assume input power to the primary coil equals to the output power of the secondary coil, explain why thick wire should
be used to construct the primary coil. (2 marks) (i) the live wire of the mains supply in the primary circuit; (2 """"J
I
19.
DSE Physics - Section D : Question
EM6 : Alternating Current

(b) (ii) one of the conducting wires in the shaver circuit outlet
PD-EM6-Q/17
11

(2 marks) 21. <HKDSE2015PaperIB-8>


DSE Physics - Section D : Question
EM6 : Alternating Current
PD-EM6-Q/18

Electricity generated from power plants are transmitted at a high voltage through overhead cables in submban areas.
1

(a) Each overhead cable cOllSists of40 strands ofidentical tl:ll:nsn:li.ssnlinesbundled together.
a strand of
transmission line of
(c) What is the turns ratio of the primary coil to the secondary coil of the transformer so as to provide 110 V? (1 mark) an overhead cable

(i) One single strand of transmission line has a cross-sectional area of 1.3 x 1 o-s m2 and resistivity 2.6 x 1o-8 n m.
Find the resistance n,gJg;g ofa single strand oftransmission line. (2 marks)

20. <HKDSE2012Paperffl-8>
In the circuit shown in the Figure, resistors Ri and R2 represent the heating elements in a heater using ma.ins supply. Both
resistors are immersed in water. (ii) Explain why the resistance per km ofan overhead cable is much smaller than that of a single strand of transmission
----------..'
heater line. Estimate the resistance per km ofan overhead cable. (2 marks)
:
ls ""'v 220 Va.c.
''
(iii) Hence, explain why a bird can stand 'With both feet on a highMvoltage cable without getting an electric shock.
'' (2 marks)

The heater can be operated intwo modes, namely, heating and keeping warm, and it is controlled by the switch S. The power
consumed by the heater in the heating mode is 550 W and in the mode of keeping warm is 88 W. The mains voltage
is220 Va.c.

(a) In which mode is the beater operating when switch Sis open? (1 ma,k) (b) Electrical power of 180 MW is transmitted at a voltage of400 kV through an overhead cable.

(i) Calculate the CUil'CDt canied by the overhead cable. (2nwks)

(b) Find the resistance ofR1.


(ii) Show that less than 0.1% ofthe electrical power is Jost after transmitted through a total ofl 0 km of overhead cable.
(2 marks)

(c) When switch Sis closed, calculate the cumrrt passing through resistor R2. (3 ""'1<s)
(lii) As the voltage drop across this ovedlead cable is negligible, a voltage of 400 kV at the cable's end is stepped down
by an ideal transfonner with turns ratio 12 : I.

(I) Find the secondary voltage from the transfonner. (I mark)

(d) What is the peak value of the sinusoidal current flowing through the heater when switch Sis closed ? (2 ""'1<s) (Il) State ONE factor leading to energy loss in a practical transformer and suggest the COITesponding measure for
improvement. (2 marks)
PD-EM6- QS /01 PD-EM6-QS /02
1
DSE PhYsics - Section D : Question Solution DSE Physics - Section D : Question Solution
EM6 : Alternating Cw:rent EM6 : Alternating Current
HKEAA's Maik:iDa Scheme is pn:pm:cd for the markers' refetence. It should not be� as a set of model llllSWCJ:S.
Students and teachets who are not involved :in the mackine process are advised to :i:meilttet the Markin&: Schenlc with cate. 4. (a) (i) By P = VI [!]
(22000) = (11000)1 l=2A
Question Solution [!]

(ii) Power loss = J R 2


[!]
1. (a) Energy conversion in a generator is from kinetic energy to electrical energy. [2]
= (J,}2 X (Q.5) [l]
It is achieved by the wind driving the coil into rotation inside a magnetic field. [!]
=2W [!]
(b) There is energy loss due to eddy current. [2]
(iii) To reduce the current in the transmission cables [!]
Use latninated core. [2]
and thus reduce energy loss (QR power loss) during power transmission. [!]
OR
There is energy loss due to dissipation of energy by the resistance of the coils. (b) Use a step-down transformer [!]
[2]
Use thicker wires. [2] with twns ratio = ]{p : Ns = 11000 : 200 55: 1 [!]

:1[
(c) In a fluorescent tube, less energy is wasted as heat. [2]

5. (,) (i) Power input = 12 x 2.5 = 30 W [!]


(ii) Power output = 2 x 1.8 = 3.6 W [!]
[!]

1
36
(in) Efficiency = • X 1QQ% = 12% [!)
(200) = (2000) 30
[!)
v, (100)
(b) (i) A 1 decreases [!]
Vs = l0V [!]
(ii) Az decreases [!]
(b) f= SOl!z [!]
(o) (D Use laminated core [!)

"'
to reduce the eddy current induced in the core. [!]

3. Use thicker wires for the coils [!)

1
to reduce the resistance and heating loss in the coils. [!]

i
""""'
:
D l00V a.c. 10001ullls
6. (a) (i) X connected to A Y connected to C Z connected to B
SO<um, < any ONE correct> [!]
5Va.c. < the other TWO correct > [!]

(u) AnyONEofthefollowing: [2]


* The fuse will blow and break the circuit if a fault develops.
* Tenninal.B will be connected to the live wire.

< Use 100 V a.c. > [!] (iii) To prevent electric shock if a fault develops. [2]
< Use 1000 turns as primary> [2 ]
(iv) Tums ratio = 2 00 [!)
<Use 100 turns as secondary> [2] 110

< Correct connection of the CRO > [!] ""1.82 [!]


DSE Physics - Section D : Question Solution PD-EM6-QS/03 DSE Physics - Section D : Question Solution PD-EM6-QS/04
EM6 : Alternating Current EM6 : Alternating Current
6. (b) (i) P = VI [!] 7. (b) (ii) TomsfonnerT,stepsupthevoltage. [!]
(1100) = (110)/ The currentthrough the cable is reduced. [!]
:, I= 10A [!] As powerdissipated in thecables is equal to PR. so the power loss is reduced. [!]

(ii) Energy = 1.1 kW x 0.5 h = 0.55 kWh [!]


cost = $ 0.8 x 0.55 = $ 0.44 [!]
8. (•) (i) CurronttbroughX = !_ = g_ [I]
(iii) Vi 11 = Vd2 [!]
V 6
=2A [I]
(200)11 = (1100)
:. 11 = 5.SA [!] (:ii) VoltagedropacrossR = 24-6 = 18V [!]
The fuse marked 7 A should beused [!]
(iii) Resi.st.anceofR = E.: =
I
.!!2 [!]

= 9!1 [!]
v2
{a) (i) = <11 = 6 n
?
7. R = [2]
P 24 (iv) Percentage = 2(1 8) x 100% [2]
2(24)
2
(ii) / = !_ = < 4) = 2A [2] = 75% [I]
V (12)
OR
(Iii) Powerlossinthecables = I2 R = (2)2 x(4) = 16W [2]
Percentage = 2 (9) x 100%
2
[2]
2(24)
(iv) Efficiency = P°", xl00% [!]
P, = 75% [I]

= -3±._x100% OR
24+16
Percentage = � x 100% [2]
= 60% [!] 2(18) +12
= 75% [!]
(b) (i)
[3]
(b) (i)

< Correct connection of circuit> [!]


< Correct symbols > [!]

(Ii) This method can reduce thepower loss in the circuit. [!]

(lii) Tums ratio = Vp = 24


v, 6
Transfonncr =4
< Two 'Wires connected from power supply to the left side of a transfOlDler > [!]
Primary current = .!.. = E. [!]
< Two wires connected from the transformerto the right side of anothertransformer> [!] V 24

< Two wires. connected from the left side of the other transformer to the lamp> [I] = 0.SA [!]
DSE Physics - Section D : Question Solution PD-EM6-QS/05 DSE Physics - Section D : Question Solution PD-EM6-QS/06
EM6 : Alternating Current EM6 : Alternating Current

Vp = Nr 11. (a) (i) P = VI= (1.2) x (1.8) [1]


9. (a) (i)
Vs Ns = 2.16W [1]
:. 220 = 5000 [1] (ii) E = Pt = (2.16) x {3 x 60) [1]
110 Ns
= 388.8 J (OR 389 J) [1]
:. Ns = 2500 [1]
(.b) E""Pt
V'
Cu) P. R [1] = (3)x(16x60x60) [1]
= 172800) (OR 1730001) (OR 173kJ) [1]
• (llO)'
(1000)
R (c) (i) When the charging unit is connected to the mains supply, an alternating CWTent flows through coil Y. [1]
R = 12.lQ [1]
A changing magnetic field is set up in coil Y and hence in coilX [1]
Output power An induced voltage is setup in coilX to recharge the cell.
(iii) Efficiency = x 100% [1] [1]
Input power

(80%) = lQQQ X 100%


P,
220 11000
Pm =
1250W [1] .. = [!]
3 N,
(iv).P = VI [1] Ns = 150
(1250) • I (220) CoilXhas 150 turns. [1]
:. I= 5.7A [1] (iii) The soft iron bar can increase the strength of the magnetic field. [1]

(b) (i) If an excessive large current flovvs through the cooker [1] (d) (i ) The pins should be connected to the live and neutral wires. [1]
the fuse will melt (blow) and breaks the circuit. [1]
(ii) AnyONEofthefollowing: [1]
(ii) John is not correct. * The charging unit no metal case.
has
The selector switch should not be set to 120 V, as the applied voltage in Hong Kong is 220 V,
* The charging unit has a completely insulated plastic cover.
that is much higher than the rated voltage. [1] * The charging unit is double--insulat.ed.
The fuse of the cooker will blow. [1]

Peter is COll'Cct.
12 (a)
The switch should be set to 240 V, as applied voltage in Hong Kong is 220 V. [1]
Since the applied voltage is slightly less than 240 V, the output power would be less than 360 W. [1]
[1]

10. (a) T1 is used to step up the voltage. [1]

(b) An a.c. voltage is used because it can be stepped up or down by transformers efficiently. [1]

Stepping up the voltage can reduce the cwrent passing through the cables. [1]

This can reduce the power loss in the cables. [1]


[ OR This can increase the efficiency of power transmission.] The output voltage V2 is proportional to the input voltage Vi. [1]
DSE Physics - Section D : Question Solution PD-EM6- QS I 07 DSE Physics - Section D : Question Solution PD-EM6-QS/08
EM6 : Alternating Current EM6 : Alternating Current

12. (b) She should vary the number of turns of the secondary coil nz, and meas1ll'e the corresponding output voltage Vz. [l] 15. (a) (i) South pole [I]
The input voltage Vi and the number of turns of the primary coil n1 should remain unchanged. [!]
(ll) No induced cumnt can flow in the open circuit. [!]
(c) Josephine should use ammeters to measure the primary currentl1 and secondary current lz , [!] The magnet then passes through the solenoid without any resistive magnetic force. [I]
and calculate the input power Vi 11 and the output power Vi Ii [!] (b) (i) To step up the voltage. [IJ
The efficiency ofthe transformer can then be estimated by Vi.12 xl00% [I]
Yi Ii (ii) Use thicker 'Wires in the coils. [!]
Use laminated core. [!]

(iii) As the magnitude of the current is varying, C2 produces a varying magnetic field. [I]
13. (a) The current is induced when the magnetic field between the two s olenoids is changing. [!] C1 will still experience changing magnetic field. [I]
By Lenz's law, the induced CWTeD.t flows in a direction to oppose the change. [l] Current will be induced in C1 and hence, Tom's claim is wrong. [I]
When the magnet rotates towards a solenoid, the induced current flows in one direction,
and when the magnet rotates away :from the solenoid, the induced current flows in the opposite direction. [1]
16. (a) The output voltage V2 is directly proportional to 1he input voltage Vi. < accept Pio:; Vi> [I]
(b) �=Np [!]
Vg Ns
(b) V2 = 8.0x � =- 13.3V <accept 13.2 to 13.6V> [l]
6
v;. a (1) Vr=l.SV [I]
(12) (8)
(c) The input voltage Vi and the number of turns of the primary coil Ni should remain unchanged. [!]
(c) (i) Easyto step up during transmission. [I] She should vary the number oftums ofthe secondary coilN2 of the transformer,
and measure the corresponding output voltage V2. [I]
(ii) Reduce power loss. [I) The relationship can then be studied.

(d) Josephine may use ammeters to measure the primary current 11 and secondaty current 12, [l]
100 She can then calculate the input power Vi 1:i and the ou tput power Pih [I]
14. (a) N8 = 500x
l{)
= 5000 [I]
The efficiency can then be estimated by: efficiency =- outpu t po wer x 100%. [I]
input power
(b) (i) V•IR [!]
a (0.1) (80) a 8 V [!]
17. (a) 'When the primary current is suddenly interrupted, the magnetic field through the secondary coil changes. [!]
(Ii) VrQ = lOO - 8 - 8 = 84V [!]
An e.m.£ is induced across the secondary coil. [!]
(iii) P = VI [!]
= (84) (0.1) = 8.4 W [!] (b) The number of turns of the secondary coil is much larger than that of the primacy' coil. [I]
OR
OR
The rate of change of magnetic flux is very large. [I]
P = VI-I2 R [I]
= {JQQ)(QJ) - (0.1)2 (80 X 2) = 8.4 W [!]
(c) By Vp [p = Vs ls, as secondary voltage is higher, the primacy current is larger. [!]
In order to minimize the beating effect ofthe primary current, thick wire of smaller resistance should be used. [I]
(c) (i) Deliver the power through the cable using a higher voltage.
(OR Increase the number of tums in the secondary coil of transformer X: ) [I]

@ Use thicker wires for the cable.


18. The r.m.s. voltage ofthe a.c. supply =- ..!?..
✓2 = 10.6 V [!]
(OR Decrease the resistance of the cable.) [!] This value is smaller than 12 V, thus the power output ofthe beater decreases. [l]
DSE Physics - Section D : Question Solution PD-EM6-QS/09 DSE Physics - Section D : Question Solution PD -EM6-QS / 10
EM6 : Alternating Current EM6 : Alternating Current

19. (a) In bathroom, humid air and mist contain much water thatis a conductor. [!] 21. (a) {il) The strands are connected in parallel [!]
OR
The water provides a conducting path between the human body and the source of electricity. [!]
The cross-sectional area of cable is greater than each of transmission line. [!]
OR
The water lowers the resistance bet..veen the human bodyand the sourceof electricity. [!] Resistance of cable per km = 2 -0 = 0.05 n km-1 [I]
40
(b) (i) Thehmnan.bodywould getelectric shock [!]
(iii} The resistance oftbe bird's bodyis much larger than that ofthe s hort segment of the overhead cable. [!]
because cummt flows through the body to the Earth and r eturns to the Neutral wire. [!]
OR
(ii) The human body will not getelectric shock [!] The potential difference across the feet is very s mall [!]
because there is no return path for the cutrent (OR there is no complete circuit) [!] Hence, negligi."ble current flows through the bird's body. [!]
(c) Tw:ns ratio = 2 : 1 < accept tums ratio ,.. 2 > (b) (i) By P"' VI [!]
(180 x 106) = (400 x 10 3) l
:. I= 450A [!]
20. (a) keepin g warm [l]
(ii) R = o.o5x10 = o.sn
V' Pi... = 11 R =(450)1 x (0.5) = 101250W [!]
(b) pa -
Percentageofpow erloss = 1012506 x 100% = 0.05625 % < 0.1 % [!]
(220)' 180xl0
_-_ (88) a [!]

(iii) Vs = 400 x_!__ = 33.3 kV [!]


R1 =sson [!] 12

(,) Pow er given out by the resi stor .fu. = 550 - 88 = 462 W [!] (iv) Heating loss due to the beating effectofCUll"ent in the resistance of coils. [!]
P = YI [!] Use thicker wires for the coils. [!]
OR
(4<i2) a ( 22 0) I,
Power loss due to eddy current induced in the s oft iron core. [!]
J2=2.1A [!]
Use laminated soft iron c ore. [!]
OR

Total ctlll'eD.t = !._ "" (SSO) = 2.5A [!]


V (220)
CutrentinR1 = (220) = 0.4A [!]
(550)
Current in&= 2.5 - 0.4 = 2.1 A [!]

(d) Peak current = ✓2 x 2.5 [!]


= 3.54A [!]

21. (a) (i) ConsideralengthoflOOOm


8
00
R = pR, = (2.6xl0- )x(l0 ) ""2_0 n [!]
A (L3x10-s)
Resistance per Ian = 2.0 Q 1cm-1 [I]
Hong Kong Diploma of Secondary Education Examination
Physics - Compulsory part(**�!)-)
Section A -Heat and Gases (#.,;fr,��)
I. Temperature, Heat and Internal energy (;;&. , #.is:fo- Ii�)
2. Transfer Processes (#.,#-,U.;&,tl)
3. Change of State C�.�#.ri!:t�)
4. General Gas Law CtHU\.:tt.�#)
5. Kinetic Theory (�-f,.Ji�,;I;-)
Section B-Force and Motion (:h;ftrii!.:fJJ)
L PositionandMovement (1.tr.fl:fl'if.ifh)
2. Newton's Laws (4-tU:#)
3. Moment ofForce (:/JE)
4. Work,EnergyandPower(fpt.,, lm.½:fl'.:;IJ.!fl.)
5. Momentum (t!l:!i:)
6. ProjectileMotion(4lb.llt.1:U;)
7. Circular Motion (!ID /Jil{il;)
8. Gravitation ( �/ :h )
Section C- Wave Motion (ilt:fn)
1. Wave Propagation (/.t.iY,J.:ffE:i:1!.)
2. WavePhenomena(i&:fh.OC.�)
3. Reflection and Refraction ofLigbt (;,t#.r.&.M .&.#M)
4. Lenses(ilit)
S. WaveNatureofLigbt (,l:.#.r:it.�4f•!i)
6. Sound(�¼)
Section D - Electricity and Magnetism ( it'fl'l.t)
1. Electrostatics(�f-'t$)
2. Electric Circuits ('t.J3.)
3. DomesticElectricity(�J!-Jl!'t)
4. Magnetic Field(.-�)
5. Electromagnet:iclnduction ('t.;Jt�.li)
6. Alternating Current (::C.llii 1!:)
Section E -Radioactivity and Nuclear Energy (1.t.MJJ!.�:ft';/t�)
1. Radiation and Radioactivity C.tiAt:fo'attlJL&t)
2. Atomic Model C.W.7"-ilt1D
3. Nuclear Energy(;$�)

Physics - Elective part (ilH}�!)-)


Elective !-Astronomy and Space Science (*3'.$:tP-h.t*#,!f!,)
I. The universe seen in different scales (;::i:;:p;J �r.,�£r�lf'il.tiitt.)
2. Astronomythroughhistocy(.i't��il(lff-Jil.!l::.)
3. Orbital motions under gravity (:i"JJ'"Fa/J$Jt.Jt.��)
4. Starsandtheuniverse(,lf.JL-.fi,*'ili)
Elective 2 -Atomic World (J,y.7"�.J1.)
1. Rutherford's atomic model Ulli:ril,li"T�'l! )
2. Photoeleclric effect U6 'lll:tt4)
3. Bohr's atomic model ofhyclrogen (JJtjff V:i i.&."Tffi:�)
4. Particles or waves ($!-7" Jlii.illt)
5. Probingintonanoscale(,tl��>r-i!t�)
Elective 3 -Energy and Use ofEnergy (:li5tfafititf,a'J#ti!l)
1. Electricityathome(#l!J1'l'lll:)
2. Energy efficiency in building (lf:�il(l $11;,ij.�lfl.)
3. Energy efficiency in transportation (il#i1.a/J:t.i!!i!lt<fl.)
4. Non-renewable energy sources (:::i- 'if.jlj.!£. :ta !Ill)
5. Renewable energy sources (-r.jlj.!!.�.i&f.)
Elective 4- Medical Physics ( G<W>#Jg�)
L Making sense ofthe eye (B!la/J� 'it)
2. Making sense of the ear (Jf,¥:irel, i')
3. Medical imaging using non-ionizing radiation ( 4f 11!: ��Af ¾;} :!j!-,Ujl�)
4. Medical fmagingusing ionizing radiation (11!:Mt.f!&MG:¥��,lj!:)
I DSE Physics - Section E : M.C.
RAl : Radiation & Radioactivity
Use the following data wherever necessary:
PE - RAl - M / 0111
5.
DSE Physics - Section E : M.C.
RAl : Radiation & Radioactivity
<BKCE 1984 Paper II - 36 >
PE-RAl-M/ 02

Sis a radioactive source which emits radiation as it decays. X X X X X X X X


Charge of electron e = 1.6 x 10- 19 C original direction
�--x---x.....
If all the radiation emitted is bent by a magnetic field in the
direction shown, then the radiation consists of X X X X X
A. a: and y only
The following list of formulae may be found useful : B. � and y only
X X X X ...�........,...::.' X X
',
Law of radioactive decay C. a: only X X X X X X ',x X
D. � only � path afti:r deflection
Half-life and decay constant ln2
t.,_ =­
• k 6. < BKCE 1985 Paper II- 45 >
Activity and the number ofundecayed nuclei A=kN The corrected count rate of a sample of radioactive material was measured on the fust day of each month. The readings on
July 1 and September 1 are 0.8 and 0.2counts per second respectively. \Vhat is the half,.Iife ofthe radioactive material?
A. 7 days
B. 16 days
C. 31 days
Part A : BKCE examination questions D. 46 days

1. < HKCE 1980 Paper Il- 37 >


7. <BKCE 1986 Paper II- 35>
Ina !3 decay, elementl:; haVlllg a half-life of3 days, decays into a stable element Y. If the initial IllaSS ofXis 4 g, what will
be the lDasse$ ofX and Y after 6 days ? The speeds of X-rays, y rays and� rays in air are denoted by vx, v., and v� respectively. Which ofthe following is true?
A. vx>11r>V11
MassofX Mass ofY
B. vx<v.,<v�
A. Og 4g C. vx = v.,=v�
B. 1g 3g D. vx = v-,>v�
C. 2g 2g
D. 3g 1g
8. <HKCE 1987 Paper II- 39 >
2. <BKCE 1981 :Paper Il - 36> Which ofthe following about o: radiation is/are correct?
(1) The mass ofano: particle is about four times that ofa proton.
Ifthe three kinds ofradiations o:, f3 and y are arranged in ascending order of their ionization power, their order is (2) It has a stronger ionizing power than� radiation.
A. 0:, f3, y
B. o:,y,f3 (3) It has a greater penetration power than y radiation.
C. f3,o:,y A. (1) only
D. y, f3, a. B. (2) only
C. (1) &(2)only
D. (2) & (3) only
3. < HKCE 1982 Paper II- 38 >
A radi0$tive substance has a half-life of 10 minutes. Which of the following statements is/are correct? 9. < BKCE 1987 Paper II- 38 >
(1) All the atoms of the radioactive substance will split into 4 equal parts in 5 minutes. Which ofthe following descriptions of the half-life ofa sample ofradioactive isotope is/are correct? The half life is
(2) All the atoms ofthe radioactive substance will decay completely in 20 minutes. (1) the time taken for the mass of the sample to fall to half of its initial value.
(3) All the atoms of the radioactive substance will decay within 10 minutes. (2) the titre taken for the activity of the sample to fall to halfofits initial value.
A. (l}only (3) half ofthe time taken for the sample to decay completely.
B. (2) only A. {l) only
C. (3) only B. (2) only
D. None of them C. (3) only
D. (1) & (2) only

4. < BKCE 1983 Paper ll- 36 >


10. <HKCE 1988 Paper II- 39>
The half-lifo of a radioactive substance is 8 hours. Its initial D:laSS is 3 g. Find the amount of the radioactive substance
.remaimng tlilChanged after 24hours. The activity ofaradioactive source falls to ½ ofits original value in 24 minutes. The half:.life of the source is
A. 0375 g A. 3 min.
B. 0.75 g B. 6min.
C. 1 g C. 8.min.
D. 2g D. 72min.
DSE Physics - Section E: M.C. PE-RAl-M/03 DSE Physics - Section E : M.C. PE-RAl-M/04
RAl : Radiation & Radioactivity RAl : Radiation & Radioactivity
11. < HKCE 1989 Paper n- 40 > 14. <HKCE 1992Paper II- 40>
A radioactive source has a half-life of22 years. After 66 years , what fraction of the source remains undecayed? Which of the following diagrams correctly shows the deflections of a, /3 and y rays in a uniform. magnetic field pointing into
A. 1 the paper?
3 A. B.
B. 1
6
C. 1
8
D. 1
9

12. < HKCE 1990 Paper Il- 40 > C. D.


In an experiment to measure the half-life of a radioactive isotope in a place where the background count rate is 20 counts per
minute, the following results are recorded :

Time/minute 0 2 4 6 8 JO 12 y

Total count rate / counts per minute 116 96 80 69 58 50 44

The half-life is about


A. 4Illlll.. 15. < HKCE 1993 Paper n- 39 >

rn
B. 6 min. Which ofthefollowing signs is used to indicate radioactive material?
C. 8 min. D.

" [i] ·� ' [BJ


D. 10 min,

13, < HKCE 1991 Paper ll- 41 >

Activity/ Bq
40
16. <HKCE1994PaperII-38>
Anange ex, /3 and y radiation in ascending order of their ionizing powers :
30 A o;, /3,r
B. /3,y,a
C. y,a;jl
D. y,jl,a
20

17. <BKCE 1994 Paper II- 41 >


The activity ofa radioactive source drops :frOin 640 Bq to 40 Bq in 2 hours. Find the half-life ofthe source.
A. 7.5:min.
B. 15 min.
C. 24 min..
D. 30min.
20 40 80
18. < HK.CE 1995 Pa per II- 26 >
The activity ofa radioactive source is recorded on a graph as shown above. What is the ha1Hife of the source?
A. 20 min. Which of the following cannot travel throu gh a vacuum?
B. 24 min. A. jl particles
B. Infra-red
C. 28min. C. Microwaves
D. 32 min. D. Ultrasonics
DSE Physics - Section E : M.C. PE-RAl-M/051 DSE Physics - Section E : M.C. PE-RAl -M / 06
RAl : Radiation & Radioactivity RAl : Radiation & Radioactivity
19. <BKCE 1995 Paper ll- 39> 24. <HKCE 1999Paper Il- 38>
Which ofthe following statements about X-rays is/are correct? Which of the following statements about CL particles is incorrect?
(1) X-rays consist of fast moving electrons. A. CL particles can be stopped by a piece ofpaper.
(2) X-rays can blacken photographic films. B. CL particles can blacken photographic films.
(3) X-rays can be used to detect weapons bidden in luggage. C. CL particles have a range ofseveral centimetres in air.
A. (1) only D. ct particles cannot travel through a vacuum.
B. (2) only
c. (1) & (3) only
D. (2) & (3) only 25. < HKCE 1999 Paper II- 26 >
An insulated metal sphere carries positive charges. Which of the following will discharge the sphere ?
20. <HKCE1996Paperll-39> (1) bringing an alpha source near the sphere
Which ofthe following can be deflected by both an electric field and a magnetic field 'l (2) touching the sphere momentarily w.ith a finger
(1) a particles (3) bringing a negatively charged metal rod near the sphere (but without touching it)
A. (1) only
(2) j3 particles
B. (3) only
(3) yrays C. (1) & (2) only
A. (I) only
B. (3) only
D. (2) & (3) only
c. (1) & (2) only
D. (2) & (3) only 26. < HKCE 1999Paper II - 37>
The background count rate recorded by a Geiger-Muller counter is 80 counts per minute. When a radioactive source is
2L < BKCE 1996Paper II- 41 > placed closely in front of the Geiger-Muller tube, the count rate recorded is 560 counts per minute. After 6 hours, the count
The activity of a radioactive isotope falls to -k ofits initial value in one hour. Find the half-life of the isotope. rate drops to 140 counts per minute. Find the half.life of the source.
A. 45 minutes
A. 3.75 minutes B. 1 hour
B. 7.5 minutes C. 1 hour 30 minutes
C. 10 minutes D. 2hours
D. 15 minutes

22. <HKCE 1997Paper Il- 39 > 27. <HKCE 2000 Paper II· 40>
Which of the following statements about j3 particles is incorrect? Which of the following statements about CL particles and y rays is correct?
A. j3 particles can be stopped by a piece of paper. A. Both ofthemaretraosversewaves.
B. j3 particles can be deflected by a magnetic field. B. Both of then;, can be deflected by a magnetic field.
C. j3 particles can blacken photographic films. C. Both ofthem have strong ionizing power.
D. j3 particles can travel through a vacuum. D. Both of them can travel through a vacuum.

23. <HKCE 1998 Paper ll- 40 > 28. < HKCE 2000Paper II - 41 >
Which of the following diagrams conectly shows the directions in which et, P and y radiations are deflected in a uIUform Which one of the following is not a safety precautions for handling radioactive sources?
electric field produced by two ch arged metal plates? A. Users should not eat or drink when handling radioactive sources.

1" ·i
A a B. Users should wear gloves for handling radioactive sources.
C. Radioactive sources should not be held close to the eye for visual examination.
D. Radioactive sources should be stored in wooden boxes after use.

+ + 29. <HKCE2001Paperll-40>
The initial activity of a radioactive isotope is 2000 Bq. After 4 hours, the activity of the isotope drops to 125 Bq.
Find the half-life of the isotope.
A. 15 minutes

"Y
B. 30 minutes
C. 48 minutes
C.
' D. D. 60 minutes

< HKCE2002Paper n- 41>

�·
30.
+ + Which of the following particles cannot be deflected by a magnetic field?
A. a-particles
B. !>-particles
C. neutrons
D. protons
DSE Physics - Section E : M.C. PE-RAl-M/07 DSE Physics - Section E : M.C. PE-RAl-M/08
RAl : Radiation & Radioactivity RAl : Radiation & Radioactivity
31. <HKCE2003Paper1I-40> 34. < BKCE 2005 Paper Il- 24 >

Which ofthe following statements about ex particles and y rays is/are correct? X X X X X X
(1) They can both be deflected by a magnetic field.
r
(2) ct particles have a stronger ionizing power than rays.
(3) They are emitted with ahnost the same speed in radioactive decay.
source �---;--;--;--;--;--;�
_,.,,,--- � x x x x x x
r=ffi
� Geiget-Mullertube
leadcastle
A. (1) only X X X X X X Q� /
B. (2) only
C. (!) & (3) only
A radioactive source is placed in front of a uniform magnetic :field pointing into the paper as shown above. If a high count
D. (2) & (3) only
rate is recorded at positions P and Q, what kinds of radiation have been detected ?
p Q
32. < HK.CE 2003 Paper II- 41 > A. y "
B. y �
Activity of PI Bq Activity of Q I Bq C. � "
D. � y
800 800
35. < HKCE 2006 Paper n- 42>
A radioisotopeXhas a half-life of2 days while another radioisotope Yhas a half-life of 1 day. Initially there are Nundecayed
atoms ofX and 8 Nundecayed atoms ofY. After how many days will Xand Yhave the same number ofundecayed atoms ?
400
A. 3 days
200 B. 4 days

, L-+-.::::==-­
10
C.
D.
6days
8 days
Tune/min Timelmin

The figures above show the variation of the activities of two radioactive sources P and Q with time. Find the ratio of the
36. < HKCE 2007 Paper n - 24 >
half-life ofP to that of Q. Count rate / counts per minute
A. 1: 1
B. 1: 2
C. 2: l
D. 4: l

33. <HKCE2004Paperll-41>
Different absorbers are placed in turn between a radioactive source and a Geiger--Muller tube. Three readings are taken for
each absorber. The following data are obtained:

Absorber Count rate/ 5'1

- 200 205 198

Papo,c 197 202 206

5 mm aluminium 112 108 111

25mmlead 60 62 58

SO mm.lead 34 36 34 Time / minute


0
What type{s) of radiation does the source emit? Susan performs an experimetlt in which a radioactive source is placed closely in front of a GM counter. The graph shows the
A /3 only variation of count rate with time. What is the half-life of the radioactive substance?
B. yonly A. 4mimltes
B. 5 minutes
C. /3 and y only
C. 8 minutes
D. a, /3 andy D. I0minutes
PE-RAl-M/09 PE-RAl-M/10
1
DSE Physics - Section E : M.C. DSE Physics - Section E : M.C.
RAl : Radiation & Radioactivity RAl : Radiation & Radioactivity
37. <HKCE2007PaperJI-26> 43. < HKCE 2010 Paper Il-24 >
Some dangerous substances are stored in a metal container inside a wooden box as
shown in the figure. What metal should be used for the container and what type of
substance is stored?
Metal used Type of substance stored
A. hon Radioactive p Q
B. I,on F'lamnlable 1n the figure above, two charged metal balls P and Q are hung by insulating threads. P is positively charged while Q is
C. Lerul Radioactive negatively charged. An et source is put near the balls without touching them. Which of the following :figures shows the
D. L""1 Flammable situation after a period of time?
°""'i'-"""""'-'"°""'-'�"-'
A. B.

38. < HK.CE 2008 Paper II• 24 >


Which of the following descriptions about the half-life ofa radioactive substance in a s�le is correct?
A. !t is equal to half of the time for all the radioactive nuclei ofthe substance to decay.
B. It is equal to half ofthe time for a radioactive nucleus of the substance to decay. p Q
C. Itis equal to the time for the sample to reduce its mass by half. p Q
D. It is equal to the time for half of the radioactive nuclei of the substance to decay. 'if 'if
o, source o, source

39. <HKCE2008PaperII-25> C. D.
Which of the following actions will maximise a person's exposure to radiation?
A. Using a GM tube and counter to measure the background radiation in laboratory.
B. Eating food that has been sterilised by exposure to gamma radiation.
C. Listening to radio.
D. Going for a flight to a distant place in a high-flying aeroplane. p Q
p Q
'if 'if
40. < BKCE 2008 Paper n - 27 > o, source o, source
Which of the following statements about p particles is correct ?
A. f3 particles car:ry positive charge. 44. < BKCE 2010Paper Il- 23 >
B. fJ particles can be deflected by a magnetic field.
C. fJ particles cannot be deflected by an electric field. The initial activity of a sample of radioisotope is 960 Bq. Its activity drops to 240 Bq in 2 minutes. How much more time
D. /J particles can be stopped by a sheet of paper. would be required :fur its activity to become 30 Bq?
A. 2minutes
B. 3 minutes
41. <HKCE 2009 Paper Il -26 > C. 4minutes
D. 5 nrinutes
The half-life ofa radioactive sample is 15 hours. The initial count rate recorded is 1000 counts per minute. After 15 hours,
the count rate :recorded becomes 528 counts per minute. What is the background count rate '! (.Measured in counts per
minute.) < BKCE 2011 Paper II- 45 >
A. 25
B. 28
c. 50
D. 56
radioactive source
ina lead box
42. <HKCE 2010 Paper Il-4S >

lov
In the figure, a /3 particle enters a region with a magnetic field magnetic field
Y
pointing into paper and an electric field ofllilknown direction. X X X X X X
The /3 particle has no deflection. What is the direction of the

...
,Bparticle X X X X X X
electric :field? The figure shows a radioactive source placed near two parallel metal plates X and Y that are connected to a power supply.
A. X X X X X X When a GM tube is moved along the dotted line (- · - · -), the count rate shows a significant increase atP and Q respectively.
'Which of the following statements is correct when a magnetic :field pointing out ofpaper is applied betweenX and Y?
B. ➔ X X X X X X A. The count rate at P decreases and the count rate at Q remains the same.
C. r B. The count rates at P and Q remain the same.
C. The count rate at P decreases and the count rates at Q and R increase .
D. .J. D. The count rates atP, Q andR are equal.
PE-RAl-M/11 PE-RAl-M/12
1
DSE Physics - Section E : M.C. DSE Physics - Section E : M.C.
RAl : Radiation & Radioactivity RAl : Radiation & Radioactivity

46. <RK.CE2011Paperll-22> 51. < BKAL 1985 Paper I - 34 >


Which of the following statements about a, /J and y:radiations is incorrect?
A. Only yradiation can travel through a vacuum.
B. a radiation can be stopped by an aluminium plate of 5 mm thick.
C. pparticles are fast moving electrons.
D. yradiation can blacken a photographic film.

47. < HKCE2011 Paper II-23 > A dish containing an alpha-source is placed inside a gold leaf electroscope. If the gold-leaf is originallypositively charged,
A radioactive source is put in front of a GM tube. The initial count rate is 1050 counts per minute. It is known that the what will happen to it after a few minutes ?
half.life of the source is 4 hours and the background count rate is 50 counts per minute. What is the most likely count rate A. It will increase in divergence.
after 8 hours ? B. It will increase in divergence and then decrease.
C. It will collapse.
A. 50 counts per minute
D. It will collapse and then re-diverge.
B. 125 counts per minute
C. 250 counts per minute
52. < HKAL 1988 PaperI- 44 >
D. 300 countsperminute
An alpha-source originally consisted entirely of the element polonium. After the emission of an a-particle, each polonium
nucleus becomes a lead nucleus. At the end of two years, the source was found to contain 98%lead and 2% polonium. 'What
is the composition of the sample at the end of one year ?
Part B : HKAL examination questions A. 25% lead, 75% polonium..
B. 50% lead, 50% polonium..
C. 75% lead, 25% polonium.
48. < HKAL 1980 Paper I- 32 > D. 86%lead, 14%polonium.
Which of the graphs below correctly shows the variation of the activity A of a radioactive sample with the number N of the
undecayed nuclei in the sample? 53. < 1IKAL 1990 Paper I - 48 >
A. R C �
A radioactive source is placed in front of a GM counter. Various absorbers are placed between the source and the
GM counter and the count-rate recorded. The following results were obtained:
Absorber Counts per minute
no absorber 712
a sheet ofpaper 504
5 mm thick aluminium sheet 496
25 mm thick lead block 218
From the above result, the radiation(s) emitted by the source is/are
49. <HKAL 1984 Paper I -33 >
A. a: and y rays only
An alpha particle (a:) makes a collision with a helium nucleus (He) in a cloud chamber. Which of the following diagrams best B. /3 and y rays only
represents the probable set of tracks ? C. a rays only
A. B. D. /3 rays only

54. <BKAL 1992Paperl-45>


A radioactive source consists ofa mixture of two radioisotopes P and Q. The half-life ofPis 1 hour and that of Q is 2 hours.
Botb.Pand Q have stable daughter nuclei. The initial corrected count rate due to the mixture is 600 counts per minute. After
C. D. 4 hours, the corrected count rate drops to 60 counts per minute. What was the initial count rate due to P only?
A. 200 counts per minute.
B. 360 countsperm.inute.
C. 400 counts per minute.
D. 480 counts per minute.

55. <BKAL 1994Paper IlA - 44 >


50. <HKAL 1985 PaperI-31 > A GM counter is placed in front of an a-source and a count rate of 120 counts per minute is recorded. After a time equal to
the half-life of the a-source, the count mte drops to 64 counts per minute. If a 5 mm thick lead sheet is inserted between the
Proactinium decays with a half-life of72 s. The value of the decay constant is
a-source and the detector, the count rnte would probably be
A. 9.6 X 10-3 S.
A. O counts per minute.
B. 9.6 X lQ-3 g-l .
B. 4 counts pertninute.
C. 0.014S-1 . C. 8 counts per minute.
D. 49.9 s. D. 16countsperminute.
I
56.
DSE Physics - Section E: M.C.
RAl : Radiation & Radioactivity

< BKAL 1996 Paper IlA - 44 >


PE-RAl -Ml 1311
60.
DSE Physics - Section E : M.C.
RAl : Radiation & Radioactivity

< BKAL 2001 Paper IIA- 42 >


PE-RAl-M/14

A counter is placed near a radioactive source that has a halHife of 1 hour. The counter registers 100 counts per minute The activity of a radioactive sample was 70 Bq at time t = 5 minutes and 49 Bq at t = 10 minutes. 'What is its activity at
at noon and 80 counts per min at 1 p.m. The expected count rate at 3 p.m. on the same day should be time t=O?
A. 50 c.p.m. A. 112 Bq
B. 55 c.p.m. B. 100 Bq
C. 60c.p.m. C. 95Bq
D. 65 c.p.m. D. 91 Bq

57. < BKAL 1998 Paper IIA- 41 > 61. < BKAL 2003 Paper IIA- 44 >
The activity of a sample of radioisotopes decreases to ½ of its initial value in 12 s. How much more time is needed for the A nuclide in a radioactive sample has a probability of 10--6 to decay in one second. What is the approximate half-life of
activity to decrease to t of its initial value 7 the sample?
A. 1 day
A. ., B. 1 week
B. 8 s C. 1 month
C. 12s D. 1 year
D. 16s

58. < BKAL 2000 Paper IIA- 44 > 62. < HKAL 2004 Paper ITA- 42 >
A radioactive source emits both a. and y radiation. A GM counter placing close to and i:n front of the source records a count The activity ofa radioactive sample is 1.0 x 106Bq. The half-life of the sample is 5.3 yems. Estimate the number of nuclei
rate of 500 counts per minute. The background count rate is 50 counts per IIllll.ute. Three different materials are placed in lll the sample that decay in the first day.
tum between the source and the counter. The following results are obtained. A. 5.2 X 102
B. 3.2 x 108
Material Recorded count rate / counts per minute C. 8.6 X }0lO
D. It cannot be estimated as the initial number ofnuclei in the sample is not given.
(Nil) 500
Cardboard X
l mm ofaluminium 63. < HKAL 2005 Paper IlA- 24 >
y
Smmoflead z The activity of a radioactive source depends on
(1) the number of active nuclei in the source
Which ofthe following is a sui1able set ofvalues for x, y and z ? (2) the half-life of the source
(3) the nature of the nuclear radiation emitted by the source
X y z A (1) only
A. 350 350 150 B. (3) only
B. 350 150 50 C. (1)&(2)only
c. 350 150 0 D. (2) & (3) only

...
D. 150 150 50
< BKAL 2006 Paper IIA- 24 >
59. <HKAL2001 PaperIIA-45 >
Some typical radiation doses are given as follows :
The table gives the corrected count rate (in counts per minute) from three Slllllples of radioisotopes at three different times.
Radiation dose
Isotopes omm 20 min 40min
Watcbinl!' television 0.005 mSv / hr for watc1..;.._ ev=• dav in a vear
X 480 243 119
fl,,;.,,. in an am:raft 0.001 mSv / hr
y 135 32 9
X-mvcheck 0.020 mSv each time
z 168 118 93
Arrange the following lll ascending order of total radiation dose lll one year.
From the above result, it can be concluded that (1) Watching television for 4 hours every day
(1) X produces the most penetrating radiation. (2) Travelling on an allcraft for 10 hours every month
(2) Yhas the largest decay constant. (3) Taking X-ray check every 6 months
(3) Zhas the longest half-life. A (1), (2), (3)
A (1) only
B. (3) only B. (2), (1), (3)
C. ( 1) &(2) only C. (1), (3), (2)
D. (2) &(3) only D. (3), (1), (2)
DSE Physics - Section E : M.C. PE-RAI-M/15 DSE Physics - Section E : M.C. PE-RAI-M/16
RAl : Radiation & Radioactivity RAl : Radiation & Radioactivity

65. < HKAL 2006 Paper IIA - 23 > Part C : HKDSE examination questions
Which of the following gives the correct inteipretation of the decay constant of a radioactive substance?
A. It is the rate ofdisintegrations of the substance. 71. < HKDSE SamplePaper IA - 35 >
B. It is the number of disintegrations of the substance occurring on one half-life of the substance.
C. ltis the fraction of the active nuclei that undergoing decay in one second. On which ofthe following does the activity of a radioactive source depend ?
D. It is equal to the reciprocal of the half-life of the substance. (I) the nature of the nuclear radiation emitted by the source
(2) the half-life of the source
66. < HK.AL 2007Paper IlA -24 > (3) the number of active nuclei in the source
Radioactive source P consists of 64 x 10 11 active nuclei. Another source Q consists of 8 x 10 12 active nuclei. The half-lives A. (1) only
ofP and Qare 2 days and 3 days respectively. After how long will the number of active nuclei in the two sources be equal?
B. (2) only
(Assume that the daughter nuclides ofbothP and Qare stable.)
A. 6 days C. (1) & (2) only
B. 9days D. (2) & (3) only
C. 12days
D. 18 days
72. < HKDSE SamplePaper IA-36 >
67. < BKAL 2011 Paper IIA-43 > Different absorbers are placed in turn between a radioactive source and a Geiger-Muller tube. Three readings are taken for
Radioactive nuclides X and Y have half-lives 2 hours and 4 hOUIS respectively. The decay of both nuclides gives stable each absorber. The following data are obtained:
daughter nuclides. Initially samples Pand Q contain equal number of atoms of nuclide X and nuclide Y respectively. Which
of the following statements are correct? Absorber Count rate I s- 1
(I) The iilitial activity of sample Pis higher than that of sample Q.
(2) After 8 hours, sample Pcontains more active nuclei than sample Q. - 200 205 198
(3) After 8 hours, the chance of a nucleus ofXin sample Pdecaying :in the next second is greater than that ofa nucleus Paper 197 202 206
of Yin sample Q.
S mm alwninium 112 108 111
A. (1) & (2) only
B. (1) & (3) only 25mmlead 60 62 58
c. (2) & (3) only
SOmmlead 34 36 34
D . (1), (2) & (3)
'What type(s) of radiation does the source emit?
68. < HKAL 2012 Paper IlA- 44 > A J3 onfy
The activity ofa radioisotope is 250Bq at time t= 0 and S4 Bq at t=< 30 min. What is its activity at t = 10 min? B. r only
A. I30Bq C. J3 and y only
B. lS0Bq D. et,)3andy
C. l 8SBq
D. It cannot be found as its half-life is not given.
73. < HKDSEPractice Paper IA 35 > M

69. <HKAL2013PaperIIA-45>
Arrange the following lengths in ascending order of magnitudes.
(1) range ofcx-particles in air
(2) grating spacing ofa typical diffraction grating used in a school laboratory
(3) wavelength of ultra-violet radiation
A. (!), (2), (3)
B. (!), (3), (2)
C. (3), (1), (2)
D. (3), (2), (1)
A radioactive source is placed in front of a uniform magnetic field pointing into the paper as shown above. The count rates
70. < HKAL 2013 Paper IIA- 43 > recorded by the GM tubes at X and Y are 101 counts per minute and 400 cotmts per minute respectively. Which of the
following deductions must be correct?
The initial activity of two radioactive sources, X and Y, are the same. Both X and Y decay to give stable daughter nuclei.
The ratio of the activity ofXto that of Y after 12 hours is 4: L If the half-life ofXis 6 hours, what is the half-life of Y? A. The source does not emit a radiations.
A. 1.5 hours B. The source emits ,Bradiations.
B: 2 hours C. The source emits yradwtions.
C. 3 hours
D. 12hours D. The background count rate is about 100 counts per minute.
DSE Physics - Section E : M.C. PE-RAl-M/17 DSE Physics - Section E : M.C. PE-RAl-M/ 18
RAl : Radiation & Radioactivity RAl : Radiation & Radioactivity

74. < HKDSE Practice Paper IA - 34 > 79. < HKDSE2015 Paper IA -32>
Which of the following statements about Ct and ,8 particles is/are correct ? Some factories make use of radioactive source for manufacturing. Workers are required to wear clothes with film badges to
(1) The mass of an a particle is greater than that ofa ,Bparticle. measure the dosage of radiation received over a period oftime. Which type of radiation below CANNOT be monitored by
(2) aparticles have a monger penetrating power than /J particles. the film badges 7
(3) .An asource can discharge a positively charged metal sphere nearby. A. o;-radiation
A. (1) only B. )}-radiation
B. (2) only C. y-radiation
C. (1) & (3) only D. X-rays
D. (2) & (3) only

75. <BKDSE2012PaperIA-35> 80. < BKDSE 2016 Paper IA- 32>


Which of the following statements about ionizing radiations is/are correct 7
A certain radioactive isotope X has a half-life of 20 hours. After a time interval of 10 hours, what is the approximate
fraction (f) ofa sample of the radioactive isotope Xremaining ? (1) The ionizing power of a--particles is mu.ch stronger than that of ,8-particles.
(2) ;<-radiation can be completely shielded by a 10cm thick concrete wall.
A. ¼ :SJS: ½ (3) Ionizing radiations a, ft and y all undexgo deflection in an electric field .
B. f = t A (1) only
c. ¼>J> ½ B. (1) & (2) only
C. (1) & (3) only
D. f> ¼ D. (2) & (3) only

76. < HKDSE2012 Paper IA- 34 >


The figure shows a schema.tic diagram of an X-ray tube in which the
filamentF and the metal target Tare connected to terminals P and Q
---- p
E.H.T.
Q
�----,
81. < HKDSE 2016 Paper IA- 33 >
Two radionuclides X and Y are of half-lives 3 hours and 4 hours respectively and initially there are Nx and NY undecayed
nuclei respectively. After 24 hours, the number ofundecayed nuclei ofboth nuclides becomes the same. Find Nx: Ny.
of an E.H.T. Which statement is correct ? A. 8: 1
A . P is the positive terminal and X-rays are emitted from T. B. 4:3
B. Pis the positive terminal and X-rays are emitted fromF. C. 4:1
C. Q is the positive terminal and X-rays are emitted from T. D. 2: 1
D. Q is the positive temunal and X-rays are emitted fromF.

82. <BKDSE2017Paper IA-32 >


77. < HKDSE2013 Paper IA· 35 >
'Which ofthe following statements about particles Pandy rays is correct 7
Polonium-210 is a pure et-emitter with a half-life of 140 days and it will decay into lead, which is stable. Initially there is a A. Only p particles can ionize air particles.
sample containing 420 mg ofpure polonium-210. Estimate the mass ofpolonium-210 left after 70days. B. Only y rays can travel through vacuum.
A. 315 mg
C. Bothofthemcan be detected by a photographic film
B. 297mg D. Both ofthem cmycharge.
C. 210mg
D. 105mg
83. <HKDSE2017PaperlA-31>
78. < HKDSE2014 Paper IA- 32 > Which of the following diagrams best shows the deflection of a and /J particles in an umform electric field in vacuum 7
A GM cmmter is placed close to and in front of a radioactive souroe which emits both o; and y radiations. The count rate A. + B. +
recorded is 450 counts per minute while the background count rate is 50 counts per minute. Three different materials are
placed in tum between the source and the counter. The following results are obtained.

Material Recorded count rate/ counts per minute

(Nil) 450 �
cardboard X

1 mm of aluminium y
2mmoflead z c. + D. +
Which ofthe following is the most suitable set of values for x,y and z?
X y z
A 300 300 100
B. 300 100 50
C. 100 100 0
D. 100 50 50
84. <HKDSE 2019 Paper IA�31 >

85. <HKDSE 2020 Paper IA-30>

The background count iatc in an experiment is determined using a GM counter. Four readings of the
minute
eounttlllC in each me takm. Which setof,eadings below is the most probable?

t•minute l"' minute 3"" minute 4111 minute


A. s 62 8 69
B. 40 40 40
C. 60 so 30 20
D. 29 26 31 35

p-.....:.p--Q
�R -z_ T
T-..s�
DSE Physics - Saction E : M.C. Solution PE-RAl -MS/ 01 DSE Physics - Section E : M.C. Solution PE-RAl-MS/02
RAl : Radiation & Radioactivity RAl : Radiation & Radioactivity
HKEAA's Matking Scbcme is prepat'ed for the mamm• refe:cnce. It &hould not be re&aJ;ded as a set ofmodelanswcra.
Students and teachers who are not involved in the nw:king proceas are advised to interpret the Mai:kini: Scheme with care. 3. D
(!) Decay does notmean splitting of the atoms.
M.C. Answers In 20 minutes, that is, 2 half.Jives, there is still 25% of radioactive atomsleft.
(2)
I. B 11. C 21. D 31. B 41. D (3) Half-life is the time taken for half oftbe number ofradioactive atoms to decay.

2. D 12. B 22. A 32. C 42. D


4. A
3. D 13. C 23. A 33. C 43. C
3 � 1.5 � 0.75 � 0.375
4. A 14. B 24. D 34. B 44. B
After 24 hours, the massremaining unchanged is 0.375 g
5. D 15. B 25. C 35. C 45. A

6. C 16. D 26. D 36. A 46. A 5. D

7. D 17. D 27. D 37. C By using Left-hand rule :


47. D
Direction of magnetic field is into paper and direction of magnetic force is downward ⇒ Direction of[ is towards the left
8. C 18. D 28. D 38. D 48. C
As direction of] is opposite to velocity ⇒ the radiation carries(-) charge ⇒ j3 radiation
9. B 19. D 29. D 39. D 49. D
10. C 20. C 30. C 40. B 50. B
6. C
Cerrectedcountrate: 0.8--+ 0.4-----+ 0.2
51. C 61. B 71. D 81. C As in between July 1 and Sep tember 1, there is 62 days. Half-life = � = 31 days
2
52. D 62. C 72. C 82. C
53. A 63. C 73. B 83. A
7. D
54. D 64. C 74. C 84. D X-ray and y-ray are both electromagnetic waves .·. they travel with the speed of light
55. C 65. C 75. C 85. D �-particles are fast moving electrons, but not electromagnetic waves.
�-particles travel with a speed less than that oflight.
56. D 66. D 76. C
57. C 67. B 77. B 8. C
58. A 68. B 78. A ✓ (!) Ana.-parti.cle consistsof2 protons and 2 neutrons, and mass of proton and neutron is nearly the same.
✓ (2) ionizing power; a.>�> y
59. D 69. D 79. A
�) a.-particles have shortest range in air:::::, weakest penetrating power
60. B 70. C 80. A

9. B

M.C. Solution (!) The product of decay also carries mass, thus the total mass of the sample should remain unchanged.
✓ (2) Half-life isthe time taken for the activity to drop to half of the mitial value.
1. B It takes 1 half-life for half of the number of undecayed nuclei to decay,
�)
MassofX: 4g�2g� lg but it does not mean another half is to be decayed in the next half-life.
MassofY: 4-1=3g
10. C
2. D l--4.!_ ---,---), .!_ --)- ,!_
Ionization power : o: > j3 > y 2 . 4 8

In ascending order : y, j3 , o: 24
Half-life = = 8 min.
3
DSE Physics • Section E : M.C. Solution PE-RA1 -MS/03 DSE Physics · Section E : M.C. Solution PE-RAl-MS/04
RAl : Radiation & Radioactivity RAl : Radiation & Radioactivity

C 19. D

l � .!.. � .!. � l (1) X-rays is a transverse wave, they do not consist of any particles.
2 4 8
✓ (2) X-rays can affect films, and be detected by films.
After 66 years, the fraction of the source remains undecayed is ..!_ ✓
8 Cl) X-rays are used in airport to detect weapons in luggage.

12. B
20. C

Time/minute 0 2 4 6 8 10 12 Charged particles can be deflected by both a magnetic field and an electric field.
a is (+)-charged and J3 is (-)-charged, they can be deflected; y is neutral, it cannot be deflected.
Corrected count rate/ cpm 96 76 60 49 38 30 24

As the initial corrected count rate (96 cpm) reduces to abouthalf(48 cpm) in 6minutes 21. D
half-life is about 6 minutes. .x! 1 x2. 1 ,!. 1 ,!_ l
1 ------1...-). - � - ---4 - � -
2 4 8 16
13. C 60 "" 15
:. Half-life = min.
As activity drops from 40 Bq to 20 Bq in 28 minutes :. halMife = 28 min 4

14. B 22. A
By using Left-hand rule : • A. J3 particles can penetrate through paper but stopped by a thln sheet of aluminium..
ill magnetic force on ct which is positive is towards the left ✓ B. J3 particles are (-)-charged particles:::::, deflected by B-field
@ magnetic force on J3 which is negative is towards the right ✓ C. J3 particles are radiation, they can blacken films and be detected.
Thus, a is deflected to the left while J3 is deflected to the right ✓ D. /3 are particles, thus they can travel in vacuum.
As o: is much heavier, the degree of deflection of o: is much smaller than that of ]3.
23. A
15. B a is (+) charged, it is attracted towards the negative plates and thus deflected towards the right
It is a symbol for all types ofradioactive substances. J3 is(-) charged, it is attracted towards the positive plate and thus deflected towards the left

Since J3 is lighter than a, thus the deflection of J3 is greater.


16. D
Ioniring power : y < J3 < a.
24. D
✓ A. a-particles have a very low penetrating power in matter, they are stopped by a piece of paper.
17. D
✓ B. a-particles are radiation that can blacken films.
,,.!. ,,!. x.!. ,,.!.
640----.....+ 320 --4 160 -4 80 � 40 ✓ C. a,.particles have a very short range in air, about several centimeters.
D. a are particles, they can travel in vacuum.
:. Half-life=�= 0.5homs = 30min.

25. C
18. D ✓ a-source emits a-particles that can iocize airmolecules to give ion-pairs.
(1)
✓ A. J3-particles are particles that can travel in vacuum The ion-pairs can discharge sphere.
✓ B. Infra-red is a type ofelectromagnetic waves that can travel in vacuum ✓ Touching the sphere with a finger js an Earthing process that can discharge the sphere.
(2)
✓ C. Microwave is a type of electromagnetic waves that can travel in vacuum
(3) Since the rod does not touch the sphere,
D. Ultrasonics are sound waves with frequency> 20000 Hz, sound waves cannot travel in vacuum. there is no flow of charge and does the charge in the sphere remains the same
DSE Physics - Section E : M.C. Solution PE-RAl-MS/05 DSE Physics - Section E : M.C. Solution PE-RA1 -MS /06
RAl : Radiation & Radioactivity RAl : Radiation & Radioactivity
26. D 34. B
Corrected count rate initially = 560 - 80 = 480 counts per minute Pdetects y radiation sincey does not deflect in magnetic field
Corrected count rate after 6 hours = 140 - 80 = 60 counts per minute Q detects 13 radiation since magnetic force acts downwards on negative charged particles by using Left hand rule.
Change of corrected count rate after each half..life: 480 --). 240 --+ 120 � 60
35. C
Half-life = � = 2 hours
X: N � ½ N � ¼N � N t
27. D Y: 8N �4N�2N�N�½N�¼N�tN
A a:notawave After 6 days, both X and Yhave the same number of undecayed atoms of ¼N
B. y: do not have charge => cannot be deflected by B-field
c. y : weak ionization power 36. A
✓ D. ct :particles can travel in vacuum; y: electromagnetic waves can also travel in vacuum. From the graph, the backgrolllld radiation is 50.
The initial total count rate is 350, thus the initial corrected count rate is 350- 50 "" 300.
28. D After one half-life, the corrected count rate should drop to 150, thus the total count rate is 150 + 50 = 200.

Radioactive sources should be stored in lead castles but not a wooden box only The total count rate drops to 200 after 4 minutes, thus the half-life is 4 minutes.

since lead is the most effective material to stop the radiations


37. C
Lead is the suitable metal to be used in the container since most radiation can be blocked by lead.
29. D
The symbol represents RADIOACTIVE substance.
Activity: 2000 ---➔ 1000 500 250 125
Number ofhalf-lives = 4 38. D
• A Time for all the radioactive nuclei to decay is infinite, half ofthis time is also infinite.
Half-life = 4 hours x ¾ = 1 hour = 60 minutes
• B Time for a radioactive nucleus to decay is random.
• c Mass ofthe sample remains unchanged since the sample includes the mother nuclei and daughter nuclei..
30. C
✓ D. Time for half ofthe radioactive nuclei to decay is the definition ofbalf-life.
Ne utrons are neutral particles. They would not be deflected bylllilgt1etic field or electric field.
39. D
31. B • A. The person still receives the background radiation only, no extra radiation is received.
(1) o: particles carry positive charges, they can be deflected by a magnetic field. • B Food that bas been sterilized by exposure to gamma radiation does not have radiation remain.
y rays are neutral, they cannot be deflected by a magnetic field. • C Listening to radio does not receive any radiation.
✓ (2) The ionizing power in descending order is o:. > j3 > y ✓ Passengers in high-flying aeroplane receive greater amount of cosmic radiation.
D.
(3) The speed of ct particles is less than the speed oflight in air but y i:ays have the same speed as light in air
40. B
32. C • A pparticles carry negative charge since they are electrons.
The half-life ofPis 10 minutes. ✓ B. f3 particles can be deflected by a magnetic field, direction of deflection is found by Left hand rule.
The half-life of Q is 5 minutes, thus after 10 lllmutes, the activity of Q drops to one quarter. • c. pparticles can be deflected by an electric field, towards the positive plate.
Ratio of the half-life ofPtothat ofQ = 10: 5 = 2: 1 • D pparticles can penetrate through sheets of paper, they can be stopped by aluminium.

33. C 41. D
After inserting the paper, the count rate is approximately unchanged, thus the source does not emit ct. Assume that the background count rate is b counts per lllmute.
After inserting the 5 mm Al. the count rate drops significantly, thus the source emits l3. After one half-life, the corrected count rate is reduced to bali
After inserting the lead, the count rate drops significantly, thus the source emits y.
:. (1000 - b) x ½ = (528 - b) :. b = 56
DSE Physics - Section E : M.C. Solution PE-RAl-MS/07 DSE Physics - Section E : M.C. Solution PE-RAl-MS/08
RAl : Radiation & Radioactivity RAl : Radiation & Radioactivity
42. D 49. D
By Left hand rule, the magnetic force is pointing downwards. After colliding with a helium nucleus, it must be aright-angledfork track.
In order to balance the magnetic force, the electric force should be pointing upwards. Option B is not correct since the angle of separation is not 90°.
Since the electric force is opposite to the electric :field for a negative charge, the ,Bparticle,
thus the electric field is pointing downwards. 50. B

k = 1n2 = ln2 = 9.6xto~3 s-'


43. C t (72)
½

After a period oftime, both the ballsP and Qare dischargedby the ions produced by the o; particles.
Thus, the two neutral balls would not exert forces on each other. 51. C
As the alpha particles would ionize the air, the ions then discharge the gold-leaf;
44. B thus the gold-leaf would collapse.
(j) 11�
(240) = (960) (½) •• t1,2 = 1 min.
211

(30) = (240) (±) t=3min. 52. D


111
(l)

OR At the end of2 years, there is 2 % Polonium remaining, thus

(2%) = (100%) e-k(2) :. k = 1.96 yeat"1


(J) (240) = (960) e-k(2) k = 0.693 mm-1
OJ (30) = (240)e"" OM31 t=3min At the end ofl year: N = No e-CU6JCI) = 0.14No

;. There is 14% of Polonium and 86% of Lead.


45. A
AtP, the radiation is ,8 since it is attracted upwards towards the positive side of the electric field. 53. A
At Q, the radiation is ysince it is not deflected by the electric field. Presence of paper: shows a significant drop in counts per minute ⇒ the source emits o:-rays
After applying the magnetic field, the magnetic force acting on /Jis upwards, thus the count rate atP decreases. Presence of Al : shows a slight change in counts per minute ⇒ the source does not emit /3-rays
As the radiation at Q is ywhich is not affected by magnetic field, the count rate at Q is the same.
Presence of Pb block: shows a significant drop in counts per minute ⇒ the source emits y-rays

46. A
54. D
A. All the three types of nuclear radiations can travel through a vacuum.
Letx be the initial count rate ofP, then (600-x) is the initialcount rate of Q.
✓ B. a radiation can be stopped by a piece of paper, and also by a thicker piece of aluminium.


C.
D.
/J particles are electrons moving with high speed.
All the three types of nuclear radiations, including_r, can blacken a photographic fihn.
Q, (600-x)
, ... 600-x
2
,- 600-x
4

47. D After 4 hours X 600-x


60 X = 480 cpm
16 4
Initialcorrectedcountrate = 1050 - 50"" lOOOcountsperminute
Number of half-life period = 8/ 4 = 2
55. C
Correctedcountrate after 8hour.; = 1000 x (½ 'f = 250 counts per minute Let b be the background radiation.
Countrateafter8hours = 250+50 = 300countsperminute After one half-life, the corrected count rate is reduced to half.

(120-b)x ½ = (64-b)
48. C
:. b = 8
By A= k N :. A cc N, activity is rurectlyproportional to the number ofundecayed nuclei.
After inserting the lead sheet, all the a particles would be absorbed.
The graph is a straight line passing through the origin. Thus, the detector can then only measure the background radiation, 'Which is 8 counts per minute.
DSE Physics - Section E : M.C. Solution PE-RAl -MS/ 09 DSE Physics - Section E : M.C. Solution PE-RAl-MS/10
RAl : Radiation & Radioactivity RAl : Radiation & Radioactivity
56. D 62. C
Let b be the background radiation. Since the time ofl day is much less than the half-life of5.3 years, activity remains constant in 1 day.

(100-b)x ½ = (80-b} :. b = 60cpm l!,J,l =AM= (I.Ox I06)x (1 x24x3600) = 8.6x 10 10

Activity of the source at noon = 100- 60 = 40 cpm OR


1n2
.After3hours, activity: 40 � 20 � 10 � 5 k= 4.147x10-9s-1
5.3x365x 24x3600
: . Expected count rate = 60 + 5 = 65 cpm
By Ao = kNo :. No = 2.41138 x 1014

57. C

When the activity drops to t ofits initial value : (½A}: A· e-k(l1) k = 0.0916s-1 l!,J,l = 2.41138 X 10 14 - 2.41052X 1014 = 8.6X }0IO

When the activit y drops to ¼ of its initial value: (¼A)= A. e-(M9I6)(t+i1) :. t = 12s 63. C
By A = k N :. Activity A depends on
CD decay constant k or half-life t112
58. A
@ number of undecayed nuclei Nin the source
Cardboard: Due to the abs01ption ofa:--radiation, the count rate should dr op .
2
By t = ln ,decayconstantkisrelatedtotb.ehalf-life.
lmmofA l: Since there is no (3-radiation. the count rate should remain the same as x. I k
SmmofPb: The count rate should drop due to the partjal absorption ofy'"Ill.diation.
However, the lead would not absorb all they-radiation, thus the count rate cannot drop to 50 cpm. 64. C
Thus, the value ofzshould be 150. Radiation dose of watching television for 4 hours evecy day = 0.005 mSv/hr x 4 hr = 0.02 m.Sv
(1)
(2) Radiation dose of flying inan aitcraft = 0.001 mSv/hr x 10 h/month x 12months = 0.12mSv
59. D
(3) RadiationdoseofX--raycheck = 0.020mSvx 2 = 0.04mSv
(1) Type ofradiation cannot be known from the count rate at different times
Ascending order of total radiation dose : (1), (3), (2)
✓ (2) Yhas the shortest half-life since after 20 minutes it drops to about ¼
By decay constant k = 1n 2 :. Yhas the largest decay constant 65. C
tin
Decay constant is the probability of decay per unit time,
✓ (3) Zhas the longesthalf-life since after 20 minutes it drops only to about 70 %
it� the fraction of the active nuclei present that decay in one second.

60. B
66. D
By A = Ao e-kt
Fornu clideP: 64 ➔ 32 ➔ 16 ➔ 8 ➔ 4 ➔ 2 ➔ l ➔ ½ ➔ ¼ ➔ ¼ :timetaken = 2x9 = 18days
(JJ (70) = Ao e-k(5)
FornuclideQ: 8 ➔ 4 ➔ 2 ➔ 1 ➔ ± ➔ ¾ ➔ f :timetaken = 3x6 = 18days
Cl) (49) = Ao e-k{10)

70
49
= "' k = 0.0713 A0 = lOOBq
By (64
OR

x 10 2) x(½)'n = {8x 102) x(½J'


1 1
3

61. B
The decay constant k is the chance of decay per unit time.
:. k = I(rs-1
3-t/2=-t/3
Half-life = ln2 = l n2 = 6.93xl0 5 s = 8days "" 1 week
k 10 ➔ t = 18days
DSE Physics - Section E : M.C. Solution PE -RAl-MS/11 DSE Physics - Section E : M.C. Solution PE-RAl-MS/12
RAI : Radiation & Radioactivity RAI : Radiation & Radioactivity
67. B 73. B
✓ (1) By k = In 2 I t112 ,Xhas shorter half-.Jife, thusXhas greater decay constantk. A. Since there is no count rate recorded at positions aboveX, the source may or may not emit ara.diations.
By A = k N,Xhas greater decay constant k, thus the activity ofX in sampleP is higher. ✓ B. Since the count rate at fis greater thanX, there must be some radiation deflected downwards to reach Y.
(2) After 8 hours, the number ofXin P drops to III6 and the number of Yin Q drops to 1/4. By Lefthand rule, the doV1Dward magnetic force should act on negative particles, that is, ,Bradiations.
Thus the number ofXin sampleP is less.
C. The source may or may not emit yradiation,
✓ (3) The chance of decay in unit time is the decay constant. as the count rate atX may consist of yand background or background only
As. the decay constant ofXis greater, the chance is also greater.
D. The source may emit yradiation, thus the count rate of 101 cpm atXmay be due to rand background.

68. B
74. C
By A= Aoe-"'
✓ (!) The mass ofan aparticle is the mass of a helimn nucleus but the mass of a ,Bparticle is nearly zero.
(54) = (250) e -k(30)
:. k = 0.051I min- 1 (2 ) The penetrating power of ais weaker than ,8.
At t=lOmi n.: A= ( 250)e -(0.05ll)(l0) = 15 0B q ✓ (3) aparticles can ionize the air, the ions then discharge the charged metal sphere.

69. D 75. C
(3) wavelength of ultra-violet radiation is of the o rder of 10---s m By N= N.,(½)1°120 = 0.70 7Nc
(2) grating spacing is of the order ofI0 m -5
f= .!!_ = 0.707
(1) range of o: in air is ofthe order of 10-2m ( a few cm) N.
0.75 > J> 0.5
70. C
ForX:12hoursistwohalf-lives :. Ax =Aox(±'f = ¼Ao 76. C

After 12 hours, Cathode rays (beam of electrons) is emitted fromFwhich is the negative terminal, thus Q is the positive terminal.

Ax:Ay=4:1 When electrons hit the metal target at T, X-rays are emitted from T.

;. Ay =¼Ax = ¼ X (¼Ao) = kAo


77. B
ForY: A= Ao(.!_)'''"'
2 M M (.!_.),1,l/l
• 2
(16Ao) = Ao (.!_)C'2J1,.,, tia = 3
2 (420) c.!.) 1on40
2
half-life of Y = 3 hours
297mg

71. D
78. A
(!) The nature or type of radiation (a, /3, y) emitted would not affect or relate to the activity of the source.
Note thaty radiation can never be totally absorbed.
✓ (2) The activity A is proportional to the decay constant k, which is related by the half-life.
Thus, zmust be greater than the background radiation of SO counts per minute.
✓ (3) The activity A is p roportional to the number of active (undecayed) nuclei N.
The only option is A thatz is 100 counts per minute.
72. C
After inserting the paper, the count rate is approximately unchanged, thus the source does not emit er. 79. A
After inserting the 5 mm Al, the count rate drops significantly, thus the source emits j3. a-radiation cannot pass through the plastic bag to reach the film, thus a-radiation cannot be detected by film badge.
After inserting the lead, the count rate drops significantly, thus the source emits y. The other 3 types of radiation can pass the plastic bag to reach the film inside the badge to be detected.
DSE Physics - Section E : M.C. Solution PE-RAl-MS/13 DSE Physics - Section E : Question PE-RAl-Q / 01
RAl : Radiation & Radioactivity RAl : Radiation and Radioactivity

80. A The following list of formulae may be found useful:


✓ (1) Ionizing power ofionizing radiation in descending order is a.> � > y > X-ray
Law of radioactive decay
X (2) y-radiation can never be completely shielded by concrete wall, no matter how thick it is.
X Only a. and � will be deflected in an electric field as they carry charges. Half-life and decay constant ln2
(3) 11' =­
y radiation is an electromagnetic wave, it cannot be deflected in an electric field. k

Activity and the number ofundecayed nuclei A=kN


81. C

By N = No (½)'''�'
Part A : BKCE examination questions
Nx (½)2413 = Nv (½):!414
1. < HKCE 1982 Paper I- 8 >
6
Nx (½)' = Ny (½)
N
Nx N,
,,,,�
Ny I

82. C
X A. Both /3 and r can ionize air particles.
X B. Both /3 and r can travel through vacuum.
✓ C. Both of them can be detected by a film.
X D. Only /J carries charge, r is neutral.

83. A
In an electric field. ,Bthat carries negative charge will deflect towardsthe(+) plate, thus it deflects upwards.
a that carries positive charge will deflect towardsthe (-)plate, 1hus it deflects downwards.
As mass of/J is much smaller than that of a, the deflection of /J should be much great er than that of a.

t I nrin

The above figure show the decay curves oftwo radioactive elements X and Yboth emitting �-particles. No is the number of
radioactive atoms present at time t""' 0 and N is the number at the end oft nrinutes.

(a) VJhatarethehalf-lives ofXand Y?

(b) A mixture of X and Y is placed in front of a Geiger counter. Initially, they have the same number of radioactive atoms.
Which of ht e two, X or Y, will be mainly responsible for the reading shown on the Geiger counter during the first four
minutes? Estimate the fraction of the total number of counts due to that element. (5 marks)
DSE Physics - Section E : Question PE-RAl -Q /02 DSE Physics - Section E : Question PE-RAl -QI 03
RAl : Radiation and Radioactivity RAl : Radiation and Radioactivity

2. < HKCE 1983 Paper I - 9 > 3. <HKCE 1983Paperl-9>


The activity from a sample of Radium is measured at two-day intervals. The readings are tabulated below: A Geiger-Muller counter is placed ona bench.

(a) Explain why the counter registers a reading even when no radioactive source is placed nearby. (1 mark)
Time/ days 0 2 4 6 8 10

Activity I Bq 100 68 47 32 22 15

(b) When a radioactive source is placed near the counter, the counter registers 520,510 and 514 counts per minute in the
(a) Plot the decay graph below to show the activity against time. first three consecutive minutes. Explain why the three readings differ from each other ? {2 marks)

(c) 'When a piece of paper is placed between the source and the counter, the counter registers 540, SlO and 512 counts per
minute in the first three consecutive minutes. However, when the paper is replaced by an alummium sheet, the counter
gives readm.g of7, 9 and 8 counts per minute in three consecutive minutes.
Whattype(s) ofradiation (ex,� ory) is/are being emitted by the source? Giveareasonforyouranswer. (4marks)

4. < HK.CE 1984 Paper I- 9 >


Two paraDel metal plates A and B are placed in a vacuum chamber as shown in the figure below. They are connected to a
d.c. supply. A hole Hls drilled in plate A. A particleP passes through hole Hand accelerates towards plate B.

�-=��-B--1
� ...-...,.'•--�
'''
: HI ':
'
' ' ''
''
'--------------'
(a) What is the sign ofthe charge carried by P?

(b) The particle Pis emitted from a radioactive source which undergoes a-, �- and y-decay simultaneously.

(i} What kind of particle (a,� ory) shouldPbe? (I"""')


(b) From the graph, find

(i) the activity of the sample after 5 days, and (ii) Draw a diagram to show how to prevent the other two kinds of particles from reaching H. Show the tracks of the

(ii) the half-life of the sample.


particles in your diagram.
Al B
(4 ma rks)

I
DSE Physics - Section E : Question PE-RAl -Q / 04 DSE PhYsics - Section E : Question PE-RAl-Q/05
RAl : Radiation and Raclioacti.vity RAl : Radiation and Radioactivity

5. < HKCE 1986 Paper I- 9 > 7. <BKCE1989Paperl-9>


A student measures the count rate of a source of half-life of25 min using a GM counter in a room with very little background
radiation. Initially the reading of the GM counter shows 560 counts per second.

(a) What should be the count rate of the source after 25 minutes?

(b) Plot a graph to show the theoretical count rate recorded by the GM counter for the :first 100 minutes.

Time/min.
0 IO 20 30 40 so 60
The figure above shows the variation of count rate of a radioactive source measured by a GM counter 'With time.

(a) Find from the figure, the backgrmmd count rate of the room.

(b) Find the count rate due only to the radioactive source at time 0.

(c) Determine the half-life of the radioactive source.

6. < HKCE 1987 Paper I - 9 >


The below figure shows a pair of positively charged aluminium foils.

aluminium foil (c) The actual readings of the GM counter are as follows:

(a) When the aluminium foils are placed near an ct source, the foils are found to gradually collapse. Briefly explain why. 50 75 100
Time/min. 0
(2marlcs)
Count rate / counts per second 560 154 70 31

Do you think that the GM counter is working properly? Explain briefly.


(b) Would the foils collapse faster or slower if the foils were placed near a r source instead ofan a. source? Explain briefly.
(2mmks)
I DSE Physics - Section E : Question
RAl : Radiation and Radioactlvity
PE-RAl - Q /06 II DSE Physics - Section E : Question
RAl : Radiation and Radioactlvity
PE-RAl-Q/07

8. < HK.CE 1989 Paper I - 9 > 9. (b) Exp!run bri,fly


(a) What is the major source ofbackground radiation
(i) why the experiment is carried out in an evacuated chamber;
(i) at an altitude of 10000 m above sea-level;
{ii) inside the Lion Rock Tunnel ;
(iii) in an underground coalmine ? (3mm)

(ii) the use ofthe lead block in the set*up. (2marks)


(b) It is reported that the background radiation in a concrete building is higher than that in a wooden hut A person thus
decides to move to a wooden hut Do you think that his decision is wise? Explain briefly. (3 marks)

(c) Ifa piece of cardboard is placed between the source and the lead block, what type(s) of radiation would be recorded on
the photographic film? (2 marks)

9. < HK.CE 1990 Paper I- 9 >

==""'="===
PQ
Photographic film

X X X X

X X X X (d) Suggest an alternative detector to replace the photographic film in the experiment. (2mm)
X X X X

X X X X

Lead block

I SourceX

The above figure shows the set-up of an experiment carried out in an evacuated chamber to study the radiation from a 10. < HKCE 1993 Paper I 7 >
radioactive sourceX X emits ct, 1,3 and y radiation. A magnetic field (pointing into the paper) is applied. The photographic
M

film is developed and marks in the positions P and Qare observed. A cloud chamber is used to observe the tracks ofaMparticles.

(a) In the figure below, sketch and label the paths of the tx, j3 and 'I radiation emitted from the sourceX (5mm) (a) Descn'be the tracks of erparticles in the cloud chamber. (2mm)

=="'="'==== Photographic film


PQ

X X X X
X X X X
X X X X (h) An a*particle collides with a helium nucleus to form a fork track. \Vhat is the angle of the fork track and what does
this angle indicate? (2 marks)
X X X X

I SourceX
DSE Physics - Section E : Question
RAl : Radiation and Radioactivity
PE-RAl-Q / 08

I DSE Physics - Section E : Question


RAl : Radiation and Radioactivity
PE -RAl-Q I 09 i
lL <HKCE1995Paperl-7> 11. {c) If the experiment in (b) is repeated with another source Q which emits both a. and y radiation, a different set ofreadings
In a school laboratory, the background count rate recorded by a GM counter is 100 counts per minute. would be obtamed, as shown in the below table.

(a) The counter is placed close in front ofaradioactive source P. The following results are obtained:
Mareri,l Recorded count rate / counts per minute
Time t/b.our 0 20 40 60 so 100 120
- 750
Recorded count rate / counts er minute 620 400 270 199 157 133 118
Paper X
(i) Find the corrected count rate at t = O. (1 marl<)
5 mm.Aluminium y
(ii) Plot the graph of the corrected count rate against time on the graph below. (5 =ks) SmmLead z

From the following list, choose suitable values for x,y andz _:
0. 100,195,540, 750
{Note: A reading may be used more than once.)

12. < BKCE 1999 Paper I - 6 >


To investigate the kin.d(s) of radiation emitted by a radioactive source, a Geiger-Muller counter is placed close in front of the
source and sheets of different absorbers are placed in tum between the source and the counter. Three readings w:e taken at
one-minute intervals for each absorber. The following results are obtained:

Recorded count rate / counts per minute


Absorber
1st readin11: 2nd readine: 3rd readine:
- 700 710 693
paner 702 703 701
(iii) Hence :find the half-life ofthe source. (1 marl<) 1 mm.Aluminium 313 320 317
5mm.Lead 98 101 100

(b) To find out the kind(s) ofradiation emitted by P, sheets of difference materials are placed in tum between P and the The background count rate recorded by the counter is 100 counts per minute.
counter. The following results ere obtained:
(a) Explain why the three readings for each absorber are not identical. (1 mark)
Marerutl Recorded count rate / counts per minute
- 620
Pancr 623
5 mmAluminiutn 98 (b) Explain how the above results show that the source emits � :radiation only and it does not emit a: andr radiation.
(4=ks)
5mmL,rui 101

Explain howthe result shows that P emits � radiation only and it does not emit ex. or y radiation. (3 -
DSE Physics - Section E : Question PE-RAI-Q/10 DSE Physics - Section E : Question PE-RAI-Q/11
1
RAl : Radiation and Radioactivity RAI : Radiation and Radioactivity

13. <HKCE2001Paperl-11> 14. <HKCE 2005 Paper I - 8 >

Recorded count rate atP I counts per min Carol performs an experiment to measure the half-life ofa radioactive source. She places a Geiger-Muller tube in front of the
Vcrticalliue source and the following results are obtained:
s
d Battery(
Lea
block

R,=w{3j�
w ----�- _____fb=
l

/GMtube
Time t/hour

Cotmt rate / counts per minute


0

400
10

225
20

154
30

119
40

107
50

105
60

100
70
102

(a) Plot a graph of the coimt rate against time in the Figure below.
Qi
ij-2ocm-i
Figure2

Figure 1 shows a set-up used to study the radiation from a radioactive source. A GM tube is placed at position P, which is at
"""'1

20 cm from the source. Two coils A and B connected to a battery and a switch Sare placed between the source and the GM
tube as shown. Initially, Sis open and the variation of the count rate recorded by the GM tube with time is shown in Figure 2.

(a) Explain why the count rate shown in Figure 2 is not due too: particles, no matter what kinds of radiation are emitted
by the source. (2 marks)

(b) Now switch S is closed. The GM tube is placed at positions P and Q in turn (see Figure l) and the count rates recorded
are shown in Figure 3 and 4 respectively. When the GM tube is placed at any paint vertically above P, an average count
rate of 100 counts per minute is recorded at each point.

�b._ �t=:.,�
Recorded count rate atP / counts per min Recorded count rate at QI counts per min

Figure3 Flgurc4

(i) State the direction ofthe magnetic field formed between coils A and B. (1 mark)

(ii) What kind of radiation is recorded when the GM tube is held at any point vertically above P ? Explain your
""""· (3 """")

(iii) What conclusion about the radiation emitted by the source can you draw from Figure 3 and Figure 4? Explain your
answer. (4marks)

(iv) Explain why the sum of the average count rates recorded in Figure 3 and Figure 4 is greater than that recorded in
Figure 2. (2 marks)
(b) Estimate the background count rate.

(c) The above experiment cannot determine whether o: particles are emitted by the source. Suggest a method for :finding
out the :mswer. (2 m arks) (c) Estimate the corrected count rate at t = 0. Hence, or othern'Ule, estimate the half-life of the source.
DSE Physics - Section E : Question PE-RAl-Q/12 . DSE Physics - Section E : Question
1
PE-RAl-Q/13
RAl : Radiation and Radioactivity RAl : Radiation and Radioactivity

15, < HKCE 2006 Paper I- 8 > 16. < BK.CE 2007 Paper I - 8 >
Workers of nuclear plants are required to wear film badges (see Figure 1) to monitor their exposure to radiation. Inside the In a physics lesson, a teacher uses the apparatus shown in Figure 13 to find the range of a particles in the air. Describe the
film badge, an opaque plastic bag is wrapped around a sheet of photographic film. Aluminium and lead sheets are also procedures ofthe experiment. (4marlcs)
placed inside the badge (see Figure 2) so that the types of incoming radiation can be distinguished.
Cl source with
1 mm aluminium sheet a bolder
3 mm aluminium sheet
5 mm lead sheet

photographic film
v,,rapped inside an
opaque plastic bag

area of the open windo w


01/01/2006 ,!�\�
counter (for measuring count rate)
-114�� . GM tube

Figure 1 Figure2 metre rule

(a) Whattype(s) of radiation cannot be detected by the badge 'I (1 mark)

(b) 'Why is an opaque plastic bag used to wrap the photographic film? (1 mark)

(c) The films of three workers John, Mary and Ken were developed. The Table below shows the de gre es of blackening on
different regions of the films inside the badges which they wore.
17. <BK.CE 2008 Paperl-12 >
Degree ofblackening ( 0 - 5 ) {a) A teacher places a radioactive source 1 cm in front of a Geiger-Muller tube (GM tube) and measures the count rate.
Regions on the film ( 0 = not blackened: 5 =most blackened) When be inserts a piece ofpaper between the radioactive source and the GM tube, be finds that there is no significant
John M•-· Ken change in the measured count rate. State the conclusion about the type of radiation emitted from the radioactive source.
(1 marl<)
Benea1h the onen window 5 5 5
Beneath the 1 mm aluminium sheet 5 3 4
Beneath the 3 mm allUllinium sheet 5 1 2
Beneath the 5 mm lead sheet 4 0 0 The teacher then conducts another experiment to investigate the deflection of radiations inside a magnetic field as shown in
Figure 1. The GM tube can be rotated from 0° to 180° around the magnetic field. Figure 2 shows the count rate recorded
(i) Based on the results in the above Table, explain which type(s) of radiation John and Mary are definitely being at different angles with or without the magnetic field.
exposed to respectively. (3 marks) 180 °

/
/
/,,;-;-;: -�-�:;z��
xx xx
xxxx
x x x �..
xxxx
region with magnetic field
pointing into the paper

[s :
radioactive source / xx xx :; x \
in aleadbox f ���� xxxx \
(u) Give one reason why different degrees ofblackening were recorded on the films of Mary and Ken. (1 marl<) xxxx xxxx '
Figure 1 x
..;-:--�x�x�ix��H"7-�xi�t�it---,',- 90
°

' xxxx xx
\ XX XX X 1
'
'
xx xx xx
xxxx xxxx
xxxx x xxx ,
,' j
(d) Suggest one hazard of exposure to ionizing radiations. (1 ma,k) '.... ����� ����,/
.... �
.... .. _
.. --- _..... 7'-......,.. GM tube
� connected to co1.mter
DSE Physics - Section E : Question PE-RAI-Q/14 DSE Physics - Section E : Question PE-RAl-Q/15
RAl : Radiation and Radioactivity RAl : Radiation and Radioactivity

17. Part B : BKAL examination questions


Recorded count rate/ counts per minute

18. < BKAL 1983 Paper JIB� 6 >


250 f---------,,0,-
, -----------
, '
J : � without magnetic :field

2oof------------+'·-.�,�-------------- E
, ··,�
V 1,
fr
I• .1

!50 >-----------�,!,!, . 'i


Flgure2 ,. '
,. .,h
ti

1oof--....::===<F =='-----i'-
withmagneticfi.eld
--
:_t
, ,-,------
I \
, ------

.,,.· .,,.. -· ..... ' .."� \\


"-,.

50 �-�•-•L--•�
-�----- ·<�-: ::,-.: ... -::-.-:;,��'
60 120 180 240

�----�'----'�-------�'---,.0/' A radioactive source emits � particles. The corrected count rates are recorded and plotted in the above figure.
0 50 90 180
(a) A student comments that the readings have been taken carelessly because the points plotted in the above Figure do not
(b) Estimate the count rate due to the background radiation. (1 mark) fall on a smooth curve. Do you agree? Explain your reasoning. (l mark)

(c) Using the result in Figure 2, explain why it can be concluded that the radioactive source emits fl and r rays. (4 marks)

(b) Use the above F.igure to estimate the half life of the radioactive source.

(c) Estimate the decay constant ofprotactinium-234. (1 mark)

(d) Estimate the count rate due to each type of radiation at 6 =: 90° without the magnetic .field. Write the answer in the Table
below.
(d) Express in words the relationship between the decay constant and the probability of an atom decaying. (!mm<)
Type of radiation Count rate/ counts per minute

a 0

/3

r
(2 msooJ
PE-RAl-Q/ 16 PE-RAl-Q/17
1
DSE Physics - Section E : Question DSE Physics - Section E : Question
RAl : Radiation and Radioactivity RAl : Radiation and Radioactivity

Part C : HKDSE examination questions 20. When Voyager I was launched, the number ofplutonium-238 atoms in the source was 3.2 x 10:S.
Given : half-life ofplutonium--238 = 87.74 years.
19. <HKDSE Practice Paper m � 11 >
Take 1 year= 3.16 x 107 s.
The decay of radioactive isotope protactinium-238 (Z38Pa) has a half-life of approximately 136 s. A sample ofZ3 8pa is put in
front of a GM tube and the initial count rate is 1000 counts per minute. The background count rate is 50 counts per minute. (b) (i) Find the activity, in Bq, of the plutonium source at the time of launch.

(a) It is known that the decay of 238Pa does not emit yradiation. Suggest a simple test to verify the radiation from 238Pa is
,8 radiation but not a radiation. (3 marks)

(ii) When a plutonium-238 atom decays, it releases 5.5 MeV of energy. Estimate the power, in kW, delivered by the
source at the time oflauncb. (2 marks)

(b) Estimate the decay constant of l.38 Pa.

(c) Hence, or othorwi.se, estimate the time taken for the count rate to drop to 250 counts per minute. (3 marks)
(1ii) The RTG of Voyage I is still in operation as Voyage I just left the solar system in September 2013 after it
'MIS launched 36 years ago. Estimate the corresponding power delivered by the plutonium source, expressed in
percentage ofthe power delivered at the time oflaunch. (2 marks)

20, < HKDSE 2014 Paper m - 10 >


Voyager I is a space probe designed by NASA to operate for over ten years in space. It was equipped with a radioisotope
them1oelectric generator (RTG) which can convert the energy released from the decay of a :radioactive source into electrical
power. Voyage/ operates with a plutonium-238 radioactive source that undergoes o:--decay.

(a) The plutonium-238 source is sealed inside a thin metallic casing of the RTG. The photo shows a NASA staff handling
the RTG with his bare bands. Explain why it is fine for it to be handled by the staff in this way. (1 mark)
DSE Physics - Section E : Question Solution PE-RAl-QS/01 DSE Physics - Section E : Question Solution PE-RAl- QS /02
RA! : Radiation and Radioactivity RAl : Radiation and Radioactivity
HKEAA's Marlcing Scheme :is prepared for the :markers' reference. It should not be zegarded as a set of model answers.
Students and teachers who are not involved in the marking :PIOCeSS are advised to inteqmet the Marklng Scheme with care. 3. (•) There is background radiation. [ll

Question Solution (b) It is due to the random nature ofradiation. [2J

I. (a) Half-life ofX = 4 min (o) /3 radiation only [ll


[ll
Half-life of Y = 32 min
There is no ct since a; is stopped by paper. [l]
[lJ
/3 is present since /3 is absorbed by aluminium and the count rate is reduced. [l]
(b) Xwill be mainly be responsible for the reading [ll There is no y since the count rate left is very small which should be due to background radiation. [ll
Ratio ofthenumber of counts due toXtothat of Y = 0.5: 0.08 [2J

0.5
Fraction of total number of counts due to X = = 0.862 [2J 4.
0.5+0.08 (•) The charge is positive. [ll

(b) (i) a. particle [ll

2. (ii) Use magnetic field [l]

0
�-
0
< correct deflection of a. to the slit>
I
[ll
< correct indication of the direction of the magnetic :field > [1]
< /3 and y correctly deflected > [l]
OR
Use electric :field [ll

·yYl· �-
< cOJrect deflection of a: to the slit> [l]
< correct indication of the direction of the dectric :field or polarity> [l]
< j3 and y correctly deflected > [ll
< Two axes correctly labeled > [ll
< Scales properly marked> [lJ
< Points correctly plotted> [l] 5. (,) Background count rate = 40 counts per minute [2]
< Curve correctly draVID > [l]
(b) CoWlt rate at time 0 200-40 (ll
(b) {i) activity after 5 days = 39 Bq < accept 38 Bq to 40 Bq >
. [lJ 160 counts per minute (l]
(il') half-life = 3.7 days < accept 3.6 to 3.8 days> [ll (o) HalfMlife = 7 minutes < accept 6.5 minutes to 7.5 minutes> [2]
DSE Physics - Section E : Question Solution PE-RAl-QS/03 DSE Physics - Section E : Question Solution PE-RAl -QS /04
RAl : Racliation and Radioactivity RAl : Radiation and Radioactivity

6. (a) The a. particles ionize air. [1] 8. (b) Both Yes or No are acceptable but the reasons should be consistent. [l]
The ions then. discharge the aluminium foils. [1] Reason for Yes : [2]

(b) The foils would collapse slower [1] * The cumulative effect ofradiation is hannfu1
since y has weaker ionizing power [1] Reason for No : {any ONE) [2]
* The background radiation in a concrete building is weak and not hazardous
7. (a) 280countspersecond [2]
* The chance ofbeing harmed by other factors such as fire in a wooden hut is increased
< No mark is awarded if OIJly answer Yes or No>
[4]

9. (a) p Q
===o=>F====Photogrnphic film

r < 2 rays reaching P, Q> [l]


'
< o; radiation - towards the left> [1]
< y radiation - no deflection> [l]
...,.._
" .,..,r ,
, r
,­ < 13 radiation - towards the right> [l]
r+r,
< 13 radiation not reaching the :film> [l]

TT

(b) (i) o:-particles have short range in air. [2]

(ii) To produce a fine beam ofradiation [2]

(c) yradiation only [2]

(d) GMtube [2]


time/min.
75

(c) Yes, the GM counter is working properly. 10. (a) Thetracksare(ANYTWO): [2]
[1]
The readings do not match the theoretical curve exactly due to the random nature of radiation. [2] * straight
* thick
* short
8. (a) (i) cosmic radiation from the outer space [1] * of about the same length
(u) from the rock [l]
(b) The angle is 90a [l]
(ili) from the coal (or carbon) [l] The masses of an o; particle and a helium nucleus are the same. [l]
DSE Physics · Section E : Question Solution PE-RAJ -QS I 05 DSE Physics - Section E : Question Solution PE-RAJ-QS/06
RAl : Radiation and Radioactivity RAJ : Radiation and Radioactivity

11. (a) (i) Corrected count rate= 520 counts per minute [1] IL (b) OR
The source does not emit a radiation
(ii) Trme t/hour 0 20 40 60 80 100 120 [l]
since the recorded count rat e almost remains tmchanged when a sheet of paper is inserted.

Corrected count rate / counts per minute 520 300 170 99 57 33 18 The count rate drops significantly when aluminium is inserted, this illustrates that it emits ,B radiation.

The source does not emit-y radiation


Corrected count rate/ counts per ininute because the count rate already drops to background rate when aluminium is inserted.

(C) X = 540 [lJ


y = 540 [1]
z "" 195 [1]

12. (a) This is due to the random nature ofradiation. [l]

(b) As the count rates remain unchanged when a sheet of paper is inserted,
the source does not emit a radiation. [l]
Ju the count rates drop significantly'Mlen 1 mm aluminium sheet is inserted,

the source emits f3 radiation. [l]


As the count rates drop to background radiation when 5 mmlead is inserted,

the source does not em.it y radiation.

13. (a) The range of a particles in air [lJ


is only a few centimeters. [lJ

(b) (i) The magnetic field is directed from B to A. [lJ


0 20 6-0 80
(ii) The count rate is due to background radiation only. [1]
< Correct label oftl).e two axes with units> [1]
p or y cannot be deflected upward. [2]
<An appropriate scale (not less than 1 cmto 50 c.p.m. and I cmto 10 hours)> [1]
(ill) As the count rate at P and Q are greater th.an the background radiation, some radiations are detected. [1]
< Correct points plotted > [1]
As the radiation detected at P is not deflected by the magnetic field, it must be y radiation. [lJ
< Smooth curve drawn > [1]
As the radiation detected at Q is deflected downwards by the magnetic field, it must be f3 radiation, [1]
(iii) From the graph, half-life "" 25 hours < from 23 to 27 hours is acceptable> [1] So the source em.its f3 and y radiation. [1]
(b) The source does not emit a radiation
(iv) The background radiation is recorded at both Figure 3 and Figure 4, [!]
since the recorded cotmt rate almost remains unchanged when a sheet of paper is inserted [!] so the background radiation is coWJ.ted twice and thus the sum is greater. [1]
The count rate drops significantly when aluminium is inserted, this illustrates that it emits /3 radiation. [1]
{c) Place the GM tube close in front of the source. [IJ
The source does not emit y radiation
Insert a piece of paper in between.
because the count rate is approximately unchanged when 5 mm lead is inserted. [IJ Ifthe count rate drops significantly, a particles are emitted. [l]
DSE Physics - Section E : Question Solution PE-RAl-QS / 07 DSE Physics - Section E : Question Solution PE-RAl-QS/08
RAl : Radiation and Radioactivity RAl : Radiation and Radioactivity

14. (a) 15. (c) (i) Johnis exposedtoyradiation [I]


Count rate / counts per minute
sinceycan pass through the 5 mm lead sheet andb lacken the :film. [I]
Mary is exposed to � radiation since the film under the aluminium sheets is blackened
but the film under the 5 mm lead sheet is not blackened. [I]

(ii) The radiation dose received byKen is higher than that ofMary. [I]

(d) AnyONEofthefollowing: [I]


* It can destroy living cells.
* It can cause cancer.
* It can cause the genetic change.

16. Place the ct source close to and facing the GM tube. [I]
Increase their separation gradually and observe the count rate reading. [I]
Mark the point for the rapid drop in count rate. [I]
Measure the distance between et source and the GM tube with the metre rule to give the range. [I]

17. (a) No et radiation from the source. [I]

(b) SOcountsperminute (SOcpm) <accept SO to 60cpm> [l]

(c) Withmagnetic field, a peak of coimtrate appears at 50 ° . [I]


As j3-particles are negatively charged, they deflect inside the magnetic field. [I]
With magnetic field, a peak of current still exists at 90 °. [I]
As y-ray does not have charge, it does not deflect inside the magnetic field. [I]

(d) j3 so < accept SO to 60 > [I]


< Two axes correctly labelled with units> [I]
< An appropriate scale> 150 < accept 140 to 160 > [l]
[I] y

< Correct points (at least 7 points correct)> [I]


< A curve through the points > [I]
18. (a) Nol It is due to the random nature ofradiation. [I]
(b) Thebackground count rate is 100 counts per minute. < from 95 to 105 is acceptable> [I]
(b) Halflife = 72 s < from 60 to 84 s is acceptable> [I]
(c) The correctedcmmtrate at t=O is 300 counts per minute. <from.295 to 305 is acceptable> [I]
The half-life is 8 hours. < from 7 to 9 hours is acceptable> [I] (c) k == lnZ = 0.00963 s- 1 <from 0.00825 s- 1 to O.Dl16 s- 1 are acceptable> [I]
72

(d) The decay constant is equal to the probability of decay of an atom per unit time. [I]
15. (a) o: radiation cmmotbe detected. [I] OR
(b) To prevent light rays from entering the bag and blackening the film. [I] The decay constant is equal to the probability of decay ofan atom in 1 s. [I]
PE-RAl-QS I 09 Hong Kong Diploma of Secondary Education Examination
DSE Physics - Section E : Question Solution
RAl : Radiation and Radioactivity Physics - Compulsory part ( ,1.,JHJ! ;;-)
Section A - Heat and Gases (�:fu JilUHI:)
1. Temperature. Heat and Internal energy (ill.£ , M::fo' r"J ils)
19. (a) Insert a piece ofpaper between the sample and the GM tube. 2. Transfer Processes (�#�:i&>U.)
The count rate will reniainapproximately unchanged, showing that n o a radiation is emitted [1] 3. Change of State ('m/$/rott.�)
4. General Gas Law (f,;&Jlllt�#)
Insert a S mm aluminium plate between the sample and the GM tube. [1] 5. IGneticTheory (7/"-f;J!fh'tit-)
The count rate will drop to the background count rate level, showing that p radiation is emitted. [1] Section B - Force and Motion (:h :fl;1�1t;)
1. PositionandMovement (1!!.It.,fr,;f$'f!J)
2 3 1
(b) k = 1n = 5.10 x 10- s- [1] 2. Newton'sLaW'S(4'-'i.li�.f.1t)
136 3. Moment ofForce (h �)
4. Work,EnergyandPower(fh'1, JJsi!:,fr,-')7�)
(c) Initial corrected count rate = 1000- 50 = 950 cpm 5. Momentum(tli:11:)
Final corrected count rate = 250 - 50 = 200 cpm 6. Projectile Motion (4:l!tt�ti;)
[1]
7. drcularMotion ( !lll fti �11,)
By C = Coe-"'
eo(½J''"' 8. Gravitation (�! :h)
By C=

(½f"
OR [1]
Section C -Wave Motion (1£1½)
e-5.lO><!O'"' I I. WavePropagation(i.t�-ffU!!)
{2QQ) = (950) (200) a (950) 2. WavePhenomena(ii.W,,.;Jt.�)
t = 306s t = 306s 3. Reflection and Refraction of Light (-'Ciro &..M .&.-11/"Af)
[1]
4. Lenses (:i!:11:)
5. Wave NatureofLight (:16i¥Jut�!l¼·!i.)
6. Sound C *¾)
(a) The penetrating power of a is so weak that it cannot pass through the thin metallic casing of the RTG. Section D-Electricity and Magnetism ('ili:;:fi,.fl't)
20 . [1]
1. Electrostatics (m-1i:4':)
2. Electric Circuits (1!:4-)
(b) ( i) k = lnZ = InZ = 2.50 X 10-10 S-1 [1] 3. Domestic Electricity ( �.§-J!l 'l!r)
1u2 (87.74x3.16x10 7 ) 4. Magnetic Field (Zii:1§-)
'i
5. Electromagnetic Induction (1t:Zii<l?.4)
Ao = kN = (2.50 X !0-1°) (3.2 X 1025) [1] 6. Alternating Current (�1J!i't)
= 8.00 x 1015 Bq [1] Section E-Radioactivity and Nuclear Energy (�Af"Yl.$.:fu��)
1. Radiation and Radioactivity ($ii#'f!':a:.M a�)
(ii) P = EA 2. AtomicModel (�-1-�zll!.)
3. Nuclear Energy (#;ils)
= (5.5 X 10 5 X 1.6 X 1Q-l9} (8 X 10 15) [ 1]
= 7040W"" 7.04kW [1] Physics - Elective part (ll!1/HI' ;;-)

.(iii) By P = EA Elective 1-Astronomy and Space Science (�X.,¥,ft,;iJ;t�#-¥)


:. Po:: A I. The universe seen in different scales (;i:::r.;i �r.,#£'ffl(l':f'ifii!ii-tt.)
2. Astronomy through history (�X4'>i¥1-ft-Alo1!:.)
[2] 3. Orbital motions under gravity(�i]"f(il).fvr..il.i!.1»)
4. Stars and the universe (-i!.£'fl'*'ili)
OR Elective 2 -Atomic World (,W. Tilt$)

f. 1 (½)f.u. (½?f7.
1. Rutherford's atomicmodel (1.t£:t&./f.T,nl;�)
74 2. Photoelectric effect (;It. 'QtU)
= = = x100% = 75.2% [2] 3. Bohr's atomicmodelofhydrogen (;,liilifl/J i.�+.fit�)
4. Particlesorwaves($t.-1-�£t)
< accept using the ratio of number of nuclei N> < accept 75% > 5. Probing into nano scale (a��;/Hf·-l'f.)
Elective 3-Energy and Use of Energy (�:i;:fi,J/Gaf.��ffl)
1. Electricity at home ( �� ffl -t)
2. Energy efficiency in building (;it.1/!-fl/Jils�tt.*)
3. Energy efficiencyin transportation (�-�{il)iig!!li.it$)
/ 4. Non-renewable energy sources (;;r:�*1:..@.il-)
5. Renewable energy sources ("tif*1:.�U!t)
Elective4- Medical Physics (-§�Jihl!$')
1. Making sense ofthe eye (Elli(:)/! '5)
2. Makingsenseoftheear(.lfl(:),tl.'g')
3. Medical imaging using non-ionizing radiation (�f- 1!l;Nf.ti.M¥-$-�-/!,$)
I 4. Medicalimagingusingion.izingradiation ('19:,.♦iAf§�i§�.!j.',)
I DSE Physics - Section E : M.C.
RA2 : Atomic Model

The following list of formulae may be found useful:


PE-RA2-M/ 0111
Law ofradioactive decay

Half-life and decay constant

Activity and the number ofundecayed nuclei

Part A : HKCE examination questions


1. <BKCE 1980 Paper ll- 40 >

A radioactive nuclide 1 X undergoes radioactive decay inside a magnetic field


cloud chamber. Tb.e radiations emitted are subjected to a

000
acting out

/0,,
lilagnetic field and the resulting tracks are as shown in the figure.
ofthepaper
What are the atomic number and the mass number of the
remaining nuclide?
Atomic Number Mass Number
0 0
0r00
A. Z-2 A-4
B. Z+l A-4
radioactive
C. Z+! A nuclide
D. Z-1 A-4

2. < HKCE1980 Paper II- 39 >

The two isotopes ii Cl and :� Cl of chlorine have different


(1) numbers ofprotons
(2) number ofneutrons
(3) chemical properties
A (1) only
B. (2) only
C. (3) only
D. (I) & (2) only

3. <HKCE1981Paperll-31>
Which of the following statements concerning isotopes of an element is/are correct ?
(1) They have the same number of neutrons.
(2) They have the same chemical andphysical properties.
(3) They have the same atomic number but different mass numbers.
A. (1) only
B. (3) only
C. (!) & (2) only
D. (2) & (3) only

4. < HK.CE 1981 Paper Il- 35 >


Which of the following represents an alpha decay?
(1) �U ➔ 23�tTh
(2) 2�At ➔ 2 �Bi

(3) z��Tl ➔ 2:Pb


A. (1) only
B. (3) only
c. (1) & (2) only
D. (2) & (3) only
I
5. < HKCE 1983 Paper II- 24 >
DSE Physics • Section E : M.C.
RA2 : Atomic Model
PE-RA2-M/0211

I
11.
DSE Physics - Section E : M.C.
RA2 : Atomic Model

<HKCE 1990 Paper II-41 >


PE-RA2
-M
/03
I
The atomic structure of isotopes of the same element differ from each other by having different numbers of
A. electrons.
1�
Ifthe nucleus of an atom is represented by the symbol 2 X. it means that this atom bas
B. neutrons. (1 )
131 protons in its nucleus.
C. electrons and protons. (2) 83 electrons outside its nucleus.
D. electrons and neutrons. (3) 214 neutro ns in its nucleus.
A. (1) only
6. < BKCE 1984Paper II- 34> B. (2)only
An ancient piece of wood was tested for its age by carbon-14 dating method. The normal emission rate from 2 g of carbon C. (3) only
from a living plant is 20 cotmts per minute. If the rate from 2 g of carbon from the wood is 5 counts per minute, and the half D. (!) & (2) only
life of carbon 14 is 5700 years, 'Mlat is the approxnnate age of the wood in years ? (Background radiation may be neglected.)
A. S700x4
B. 5700x2 12. <HKCE1992Paperll-3.9>
C. 5700/2
:l.l�U decays by emitting two o; particles and two /3 particles. Which of the follO'Ning represents the resulting nuclide?
D. 5700/4
A 2:Th
7. <HKCE 1985 Paper n - 43 >
B. 2: u
During radioactive decay, 2;g X becomes ��y. Which of the following statements would be correct ?
(1) The change would involve a. decay only.
C. �;Ra
(2) One a. particle and two /3 particles would be emitted. D. 2;�Th
(3) Xand Yare two isotopes of the same element.
A (1) only
B. (2) only 13. < HKCE 1994 Paper Il- 39 >
C. (I) & (3) only Which of the following symbols represents a neutron?
D. (2) & (3) only
A. �n
8. < HKCE 1988 Paper II- 40 > B.
A U-235 nucleus would change to Ac-227 through a series of decay: C. �n
�u � 2;1Th � 2;JPa � �Ac D. \n
'What kind ofparticles are emitted at stages%, YandZin the radioactive decay chain sho'Nn above ?
X Y Z 14. < HKCE 1995 Paper II - 40 >
A a a � A radioactive nuclide W decays to a nuclide Zby emitting one ct-particle and two /3-pa..-ticles as shown below.
B. � a �
C. � � a W -1!.._.,, X � Y -1!.._.,, Z
D. a � a
Which of the following statements aboutnuclides W, X, Y andZis/are correct?
9. < HKCE 1988 Paper II- 38 > (1) WandZareisotopes.
The atomic number of Tin is 50 and its mass number is 112. Which of the following is an isotope of Tin? (2) Xhas the greatest atomic number.
A. l��X (3) Yhas the greatest mass number.
B. 1!! X A {I) only
C.
1
!;x B.
C.
(3) only
(!)& (2) only
D. 1�X
D. (2) & (3) only

10. <HKCE 1989 Paper II-39 >


15. <HKCE1997Paperil-40>
� U eventually decays to 2: Pb. What is the number of o; particles and /3 particles emitted during the decay? Which ofthe following is not an application ofradioactivity?
" p A. Carbon-14 dating
A 7 4 B. Examination of foetuses (babies not yet born)
B. 7 10
C. 14 10 C. Killing cancer cells in human bodies
D. 28 4 D. Steriliz.ation of food
Ii DSE Physics - Section E : M.C.
RA2 : Atomic Model

16. < BK.CE 1998 Paper II- 41 >


PE-RA2
-M
/041
1
Ii
19.
DSE Physics - Section E : M.C.

RA2 : Atomic Model

< HKCE 1999 Paper II- 40 >


P E-
RA2-
M/ 0
5 I
Radioactive
source Which of the following applications of radioactivity makes use ofthe fact that a radioactive nuclide has a constant half-life ?
A. Carbon-14 dating
Alunrinimnshoot - - - - - -�0�--lJ--➔ffi,..._- - B. Preservation of food

D C. Smoke detectors
D. Thickness gauge
Detector
In a factory producing aluminium sheets of 1 mm thickness, a thickness gauge is used to monitor the thickness of aluminium 20. < HKCE 2001 Paper II- 41 >
sheets. Which of the following states the comet radioactive source to be used in the thickness gauge and the reason behind ?
The below shows part of a radioactive series.
Sour« Reason
A. a The amount of a particles passing through aluminium depends on its thickness.
B. fJ The amount of/3 particles passing through alurcinium depends on its thickness.
Which of the following nuclei are isotopes of the same element?
C. fJ /3 particles are less hannfu1 to human beings.
A. Pand Q
D. r r radiation has the greatest penetrating power.
B. PandR
C. PandS
17. < HK.CE 1998 Paper II - 39 > D. QandS
A nucleus Xemits a beta particle to form a daughter nucleus Y. Which of the following statements is/are correct ?
(1) Xand Yhave the same number of neutrons. 21. < HKCE 2001 Paper II- 39 >
(2) The number ofprotons in Xis greater than that in Yby 1.
Radium (�Ra) decays by emitting an a. particle to form a product nucleus X. Which of the following shows the correct
(3) The total numbers of neutrons and protons inX and Y are equal.
equation for this decay ?
A. (!)only
B. (3) only A. 2;:Ra + a. ---+ 2;� X
C. (!) & (2) o nly
B. 2;!Ra ---+ �X + a.
D. (2) & (3) only
C. �Ra ---+ �X + a.

18. < BKCE 1999 Paper Il- 39 > D. �R a ---+ 2:X + a.

Mass number A
22. < HKCE 2002 Paper II - 42 >
Np Which of the following is/are application(s) of radioactivity?
237
(1) to estimate the age of ancient remains
236 (2) to kill bacteria in food
p (3) to transmit signals over long distances
235
A. (2) only
Q B. (3) only
234
C. (!) & (2) ooly
233
R s D. (1)&(3)only

Atomic number Z
90 91 92 93 23. < HKCE 2002 Paper II - 40 >
2
The above diagram shows the mass number A and atomic number Z of a few nuclides. The isotope ofneptunium (Np) shown A radioactive isotope 2;� Th undergoes a series of decay processes to fonn a daughter : Pb. How many a-particles and
decays by emitting an a particle and then a ft particle.. �-particles have been emitted in this decay process ?
Which of the following represents the resulting nuclide ? No. of «-particles No. of P-particles
A. p A. 6 7
B. Q B. 7 6
C. R c. 7 8
D. S D. 8 7
I
24,
DSE Physics - Section E : MC.
RAZ : Atomic Model

< BKCE 2003 Paper Il- 42 >


PE-RAl-M/ 61
0 1
I
29. < BKCE 2007 Paper
DSE Physics - Section E : M.C.
RA2 : Atomic Model

n - 25 >
PE-RAl-
M/0 1
71

Which of the following are essential criteria in choosing radioactive sources as medical tracers "inhuman bodies ? In the upper atmosphere, neutrons are produced by the action of cosmic rays. These neutrons interact with nitrogen nuclei as
{1) The sources should have a short half.life. shown in the following reaction:
(2) The radiation emitted should have a weak ionizing power. 1iN
(3) The radiation emitted should not be deflected by an electric :field. �n+ ➔ X+ iH
A (1) & (2) only
B. (1)& �) <mly ElementXwill then emit a ftparticle.
c. (2)&mon1y The nuclear reaction is as follows : X ➔ Y + -� 13 .
D. (1), (2) & (3)
What is the final product Y?

n- 42 > A.
1
!c
i:c
25. < HKCE 2004 Paper
In order to detect cracks in an undergrotmd oil pipe, an engineer proposes adding a radioactive source to the oil Which of B.
the following sources is most suitable ?
A. a rsource with a half-life of a few hours
c. i;
N
B. a rsource with a half-life of several years D. �N
C. an a source with ahalf-life of a few hours
D. an a source with a half-life of several years
30. < HKCE 2009 Paper Il- 25 >
26. < HKCE 2005 Paper n- 25 >
c2;i Th) decays by emitting a P particle to form a daughter nucleus X Which of the following equations X --�"----> y z

-
A thorium nucleus
represents this decay ? The above shows part ofa decay series. Which of the following deductions is/are correct?

-
A 2:x+ P (1) X and Z are isotopes ofth e same element
��Th

-
(2) Xhas two more neutrons than Z,
B. ';:Th 2;!x + fJ

-
(3) Zhas one more proton than Y.
c. �Th 2;�x + P A. (I)only
B. (3) <mly
D. 2;� Th ��x+p C. (1) & (2) on ly
D. (2) & (3)only

27. < HK.CE 2006 Paper Il-43 >


31. < BKCE 2010 Paper II-25 >
X--"=---. Mass number
The diagram shows the mass number and atomic number ofa
radioactive nuclide K, After undergoing the following decays,
P--�-> s it becomes Z.
235
K
234
In the above two decay series, P and Yare two isotopes of the same element. Which of the following pairs ofnuclides may
be isotopes ? 233
(1) XandR Which of the follow.ingnuclides represents Z? 232
(2) YandS Q
A. p 231
(3) ZandQ
230 s R p Atomic
A. (1) & ( 2) <mly B. Q number
80 81 82 83 84 85
B. (1) & �) on!Y C. R
C. (2) & 0) only
D. (1), (2) & (3) D. S

28. < HKCE 2006 Paper Il-27 > 32. <HKCE2011Paperll-24>

r
Some fresh foods are exposed to radiations from radioactive isotopes for a short time so that the micro-organisms in the
2
A � U nuclide undergoes a certain number of a and P decays and becomes � Pb. F'md the number of ,8 particles emitted.
foods can be killed Why are the irradiated foods not harmful to people '\W.o eat them'?
A. 2
A. r radiation is an electromagnetic wave.
B. r radiation has a high penetrating power. B. 3
C. r radiation does not have a high ionizing power. c. 4
D. r radiation does not make the foods radioactive. D. S
I DSE Physics - Section E : M.C.
RA2 : Atomic Model
PE-RA2-M/08 II I DSE Physics - Section E : M.C.
RA2 : Atomic Model
PE- RA2- Ml 09 II
Part B : HKAL examination questions 38. < BKAL 2010 Paper IIA- 42 >
A radioactive source having a half-life of 5.3 years has an initial activity of 2500 Bq. A cancer treatment requires 10 seconds
33. < BKAL 1981 Paper I- 33 > of irradiation ofthis source to give a certain number of radiation particles on a cancer site. If the same treatment is required
after 2 years by this radioactive source. what should be the time of irradiation to give the same number of radiation particles?
A stationary radioactive nucleus of mass number N emits an alpha particle, leaving a daughter nucleus of mass number N-4.
The ratio of the kinetic energy of the alpha particle to the kinetic energy of the daughter nucleus is A. 13 s
A. (N-4)/4 B. 15 s
B. N2/(N-4)l c. 18 s
C. (N-4"f!N D. 21 s.
D. (N-4)2/42

34. < HKAL 1994 Paper IIA. - 45 > Part C : BKDSE examination questions
A stationaxy U-238 nucleus undergoes a.-decay. What is the ratio of the kinetic energy of the daughter nucleus to that ofthe
o:-particle? 39. < HKDSE 2012 Paper IA - 36 >
A. 238:4
Isotopes of an element have different mass number A and neutron number N. Which of the following N - A plots correctly
B. 4:238 shows the relationship ofN and A for any given element ?
C. 234:4 A. B.
D. 4:234

35. < HKAL 1995 Paper IJA - 44 >


Z::Ra decays to � Rn with a half-life of1600 years. 'Which ofthe following statements is/are correct?
(1) Alpha particle is given out in the decay.
(2) All �Ra has decayed after 3200 years. c. D.
(3) The half-life of 2;:Ra can be shortened by heating.
A. (1) only
B. (3) only
C. (1) & (2) only
D. (2) & (3) only

36. < BKAL 1997 Paper IlA- 43 >


40. < HKDSE 2013 Paper IA- 34 >
� Ra undergoes a series of decay to become the stable end-product 20J Pb. What is the number of !3-particles emitted in this
�!u undergoes a - /3 - /3 - a decay and becomes a nuclide X What are the atomic number and mass number ofX?
series?
A. 4 atomic: number mass number
B. 6 A. 90 230
B. 90 234
c. 10 c. 88 230
D. 14 D. 88 234

37. <HKAL2009PaperIIA-44> 41. < HKDSE 2014 Paper IA- 31 >


Inp..decay, a neutron inside the nucleus changes into a proton and an electron, which is emitted as a p-particle. Radioactive Nucleus Wdecays to nucleus Z as shov;n:
nuclide plutonium 2;:Pu becomes lead 20JPb after a series of alpha and beta decays. Throughout the whole process, how Which of the following statements is/are correct ?
many neutrons inside a 2:!Pu nucleus have undergone such change ? (I) NucleusXhas 1 more proton than nucleus Y.
(2) Nucleus Whas 2 more neutrons than nucleus X
A. 3 (3) Wand Z are isotopes of the same element.
B. 6 A. (1) only
B. (2) only
C. 9 C. (1) & (3) only
D. 12 D. (2) & (3) only
I DSE Physics - Section E : M.C.

RA2 : Atomic Model


PE-RA2-M/ 10 II DSE Physics - Section E : M.C. Solution
RA2 : Atomic Model
PE-RA2-MS/Ol

HKEAA's Marking Scheme is prepared for the mm::km.' I'Cfetence. It should not be regazded as a set of model answers.
42. <HKDSE 2015 Paper IA -33 > Stttdents and teachers who are not involved in the marking process are advised to intt:ipret the Marlcing Scheme wi1h care.
A piece of ancient wood is dated using caxbon-.14 dating method. It registers a corrected count rate of lLO counts per minute
while a fresh wood sample cut from the same kind of trees gives a corrected count rate of 15.6 counts per minute. What is the M.C. Answers
approximate age of the wood found in the archaeological site? Given: half-life of carbon-14 is 5730 years.
A. 890 years I. A II. B 21. C 31. A 41. D
B. 1300 years
C. 2000 years 2. B 12. D 22. C 32. C 42. D
D. 2900 years 3. B 13, B 23. B 33. A 43. B
4. C 14. C 24. A 34. D 44.B
43. < HKDSE 2018 Paper IA- 32 >
X and Y are two radioactive nuclides. The ratio of the mass of an atom ofX to that of an atom of Yis 1 : 2. The half-lives of 5. B 15. B 25. A 35. A 45. 8
X and Y are T and 2T respectively. If two samples consisting of purely X and Y respectively have the same initial mass, :find
the ratio of the number ofundecayed nuclei ofXto that of Yafter a period of 4T.
A. 1 :4 6. B 16. B 26. D 36. A 46. D
B. 1 :2 D 27. D 47. C
7. 17. B 37. B
C. I: 1
D. 2: 1 8. D 18. D 28. D 38. A
9. B 19. A 29. C 39. D
44. <HKDSE 2019 Paper IA--32>
10. A 20. C 30. B 40. A

M.C. Solution
1. A
The magnetic field is directed out of paper.
The magnetic force is towards the right
By using Left-hand rule, the direction of current is upwards,
that is same as the direction of motion.
Thus the radiation carries (+) charge, it must be a-particle.

:. 1X ➔ 1'=.W + 1He

2. B
45. <HKDSE 2020 Paper IA-31>

p �R
P -�-Q ---;-.._ S �
-----z__ T
X


(1)

(2)
Both have 17 protons.
No. ofneutrons in i�Cl = 35-17 = 18
No. ofneutrons in f;ct =37-17 =20

X
(3) Isotopes have identical chemical pr operties.
NncJide P can d�y ioto .imclide Tthrough ehheT process P- Q- R - Tor process P- Q- 8- T a.s
$hmm. Which deductions below are com:ct?
3. B
(1) P and Tare isotopes.ofthe same clem=it.
QaodSbavctheumcnumbcrofprotoos.
X (1) Isotopes have different number of neutrons but have same number of protons.
(2)
(3) S has one more neutron than R. Different mass number represents different physical properties.
X (2)
A. (l}m,d(2)omy ✓ (3) This is the definition ofisotopes.
B. (l) and (3) Cllly
C. (2) and (3) only There is question in next page
D. rn. fZ1 and m
46. <HKDSE 2020 Paper IA-32>

The decay constant ofa radioisotope of an element

A. is random.
B. depends on pressure ,nd temperature.
c. is directly proportional to the number ofnucleons. in 1he isotope.
D. is an identifying cbaracterimc ofthat isotope.

47. <HKDSE 2020 Paper IA-33>

Two radioactm: samples P and Qare freshly prepared. It is found that when � ofall the nuclei ofP
16
63 half-life of p ,
I,a-,odecayed, of all the nuclei of Qbave also d..,.yed. Find the ratio
64 half-life of Q

A. I 4
B. 2 3
C. 3 2
D. 4 I
DSE Physics - Section E : M.C. Solution PE-RA2-MS/02 DSE Physics - Section E : M.C. Solution PE-RA2-MS/03
RA2 : Atomic Model RA2 : Atomic Model

4. C 12. D
✓ (!) �U ➔ �0Th +�He
✓ (2) 2i!At ➔ �Bi + 1He
2
Balancing the mass number :
• ( 3) 21:in ➔ l�Pb + fX
238=A+2x4 :. A=230
Balancing the atomic number:

5. B 92 = 2 + 2 X 2 + 2 X (-}) Z = 90
Isotopes have different number of neutrons but have same number ofprotorui, so as the electrons.
13. B
6. B Upper number = Mass number of neutron = 1
20� 10 S700- l 5
Lower number = Charge ofneutron = 0
:. Age ofthe wood = 5700 x 2
14. C
7. D Assume the mass number of Wis 100 and the atomic number of Wis 50. (OR any two arbitrary values)
(1) �X ➔ jHe + 2;:A; since the atomicnumberofYis 90, there must be �-decay in the series.

✓ (2) ✓ (!) Wand Zhave the same atomic number, thus they are isotopes.
✓ (3) BothX and Yhave the same atomic number 90.
✓ (2) The atomic number ofXis 51, which is the greatest among the above 4 nuclides.

(3) The mass number of Yis 96, which is less than that of WandX, not the greatest.
8.
Balancing the equation: ziJu ---1!!4 �1Th � 2!:Pa � �;Ac
15. B
X=a
✓ A_ Carbon-14 dating makes use of the detection of/3-radiation emitted by caibon for archaeological study
B. Examination offetuses makes use ofultrasoimd
✓ C. Killing cancer cells makes use ofy-rays
9. B ✓ D. Sterilb.ation offood makes use ofy-rays
Isotopes have different mass number but same atomic number, ie. 50.
:_ ;!
1 X and T'm have the same atomic number and thus they are the same element. 16. B
p particles are only partly absorbed by thin sheet ofalumi:nimn
10. A amount of 13 particles passing through depends on its thickness
p can be used to check thickness of alumillium sheets
Balancing the mass, 235 = 207+4x •• X =7
Balancing the charge, 92 = 82 + 2(7) + y(-1) y =
4 17. B

11. B Number ofneutron inX = A-Z


(!)
(1) number ofprotons = atomic number = 83 Number ofneutron in Y = A -(Z + 1)
✓ (2) numberofelectrons = numberofprotons = 83 (2) The number ofprotons in Yis greater than that inXby 1

(3) numberofneutrons = massnumber-atomicnumber = 214-83 = 131 ✓ (3) Same mass number ⇒ same total numbers ofneutrons and protons
DSE Physics - Section E : M.C. Solution PE-RA2-MS/04 I DSE Physics - Section E : M.C. Solution PE -RA2-MS / 05
RA2 : Atomic Model RA2 : Atomic Model

18. 26. D
"•' Np � 233
" X � 233
nS The symbol of/J is -� �-

Thus the mass number ofX is unchanged and the atomic number ofX should be 9I.
19. A
✓ A Age of ancient :findings can be found by C-14 that emit 13 radiation with a constant half-life
B. Use y-rays to kill bacteria and germs 27. D
C. Use cM·a.diation to ionize the air Since P and Y are two isotopes, they must have the same atomic number but different mass number.
D. Use fl-radiation to check thickness of aluminium sheets
Assume the atomic number ofP and Yare both equal to n.

20. C
Assume the atomic number ofP is Z
Atomic number of Q = Z - 2
np � n+IQ � n+2R � ns
Atomic number of R = Z - 2 + 1=Z -1
Atomic number of S = Z - 2 + 1 + l = Z ✓ (1) BothX andR have the same atomic number of n + 2.
Thus, P and Shave the same atomic number, they are isotopes of the same element ✓ (2) Both Y and Shave the same atomic number of n.
✓ (3) Both Zand Q have the same atomic number of n + L
21. C
22�Ra
28. D
After the foods have been exposed to y radiations,
22. C
✓ carbon 14 dating is used to estimate the age of ancient remains the foods will not become radioactive,
(1)
✓ 0) gamma rays are used to kill bacteria in food since there is no radioactive source in the foods.
X
(3) microwaves are used to transmit signals over long distances
29. C
23. B The neutron interacts with nitrogen:
Balancing the equation : 2;6 Th ----
2
�� Pb + x i a. + y -� 13 1
�n + iN ➔ :X+ :H
1

234=206+4x x = 7 The equation for the nuclear reaction :


90 = 82+2( 7)+(-l)y .. y=6 i:x ➔ i;y+ _to

The final product Y is 1; N


24. A
✓ (1) The sources should have a short half-life so as to reduce the hann:fu1 effect to human bodies
✓ 30. B
(2) The radiation should have a weak ionizing power so that it can cause less harmful effect to human bodies
Assume the atomic number ofX is 50 and the mass number ofXis 100 :
(3) /)-radiation, which can be deflected by an electric field, can be used as medical tracers.
Since human bodies do not have electric field, this is not a criterion in choosing medical tracers.

(I) Since the atomic numbers ofXand Z are not equal, they are not isotopes of the same element
25. A
(2) The number of neutrons ofX is 50 and the number of neutrons ofZ is 96 - 49 = 47
r source should be used Thus Xshould have 3 more neutrons than Z.
since it bas great penetrating power to pass through the pipe wall and the ground to be detected. ✓ (3) The number of protons of Y is 48 and that ofZ is 49,
The balflife should be short in order to reduce the harmful effect to the enviromnent thus Z bas one more proton than Y.
DSE Physics - Section E : M.C. Solution PE-RA2-MS/06 DSE Physics - Section E : MC. Solution PE-RA2-MS/07
RA2 : Atomic Model RA2 : Atomic Model

31. A 37. B
Massnumber ofK is 234 and atomic number ofK i s 84. Consider the mass number : 244 = 208 + a (4) a = 9

�:K � �ix � n�y ---1.._,,. �z There are 9 alpha particles emitted in the series.
Considertheatomicnumber: 94, = 82 + 9x(2) + b(-1) b= 6
The final product isP which has the mass number of230 and atomic number of83.
There are 6 beta particles emitted in the series.
As each emission of beta particl e involves a decay ofneutron, there are6 neutrons having such change.
32. C
238 = 210+4a • • a, = 7
38. A
92=82+2x7-!3 Theinitialactivity:Ao = 2500Bq.

33. A
A:ftertwoyears, theactivityAbecomes: A= (250o,tJ l )"" = 1925Bq

2 By M= A Mand same trea1ment needs th e same number ofradiation particles M


KE= .!.mv2 = (mv)
2m :. /iN = (2500) X 10 "" (1925) X t :. t = 13 s
2
sinc e the daughter nucleus and the o; particle must have thesame m agnitude of momentum after theexplosion

:. KE cc 39. D
LetZbe the atomic number, which is equal to the number ofprotons.
A=Z+N :. N=A-Z
Compared with y = m x + c, thegraph is a straight linewith slope +l and with a negative y•intercept

34. D 40. A
i
KE= .!.mvi = (mv AtomicnumberofX= 92-2+1+1-2 = 90
2 2m
MassnumberofX= 238-4-0-0-4 = 230
since th e daughter nucleus and the o: particle must have thesame magni tude of momentum after the explosion

:. KE o:: KE,ru,1.... = � = _4_ ,,, _±_ 41. D


KEa m..,,1""' 238-4 234 1
�gw�:x�!!Y� ;;z
Assume that the atomic mass and atomicnumber of Wis 100 and 50 respectively.
35. A • (1) Nucleus X should have I less proton than nucleus Y.
✓ (1) Balancing the equation: �:Ra ➔ �Rn+ ;He. ✓ (2) Nucleus Whas 2 more neutrons and 2 more protons than nucleus X.
• (2) It takes infinite time for all �Ra to decay. ✓ (3 ) Since Wand Zhave the same number ofprotons, they ar e isotopes of the sam e element
• (3) Nuclear change cannot be changed by the surrounding temperature.
42. D
36. '
A
Method©: C= C0 (1) l'z "'"
Balancing the mass number, .•. (11.0) a (15.6) (l)''"" :. lo g (ll.O) =l og(!.) x -'- :. t = 2888 "" 2900 years
226 = 206+4x x=S 2 15.6 l25730

Balancing the atomic number, Method@: k = Jnl"' �= 1.21 x lo-4year-1


88 = 82+2(5)+y(-1) tJ/i 5730
y=4

:. 4 P-particles are emitted. By C = Co e~kt t "" 2887 "" 2900 years


I DSE Physics - Section E : MC. Solution
RA2 : Atomic Model
PE-RAZ-MS I 08
I I DSE Physics - Section E : Question
RA2 : Atomic Model
PE-RA2-Q I 0111
=""'=''-­
43. B Use the following data wherever necessary:
Toi ·1=
Initial number of atoms (nuclei) = -==·tm Avogadro constant
Mass ofone atom

Since 1he initial mass ofX andY are equal, initial number of undecayed nuclei ec �--,'--- The following list of formulae may be found useful:
Mass of one atom
Since mass ofone atom ofX : mass of one atom ofY = 1 : 2 Law ofradioactive decay

:. initial. number ofnuclei ofX : :initial number ofnuclei of Y = 2 : 1 In2


Half-life and decay constant t�:a:­
Let the initial number of nuclei ofX and Ybe 2No andNo respectively. ' k
Afteraperiodof4T: Activity and the number of undecayed nuclei

2t =
N0 x ( ½J r i ¼No Part A : HKCE examination questions
/fy =

:. Nx :Ny = 1:2 1. < HKCE 1980 Paper I - 9 >


(a) A factory aims at producing aluminium sheets of 1 mm thickness, A radioactive source and a detector is used to monitor
the thickness of the aluminium sheet manufactured as shO'Wll in the figure below.

Radioactive
source

Ainmininm shoot - - - - -
--r:'0,------"5'--?ffi<r-- -
D
Detector

(i) State what type ofsource (ix, f3 ory) should be used. (1 mark)

(u) Explain briefly why the other two types ofsource are not used. (2 marks)

(b) Give TWO other applications of radioactivity. (2=ks)


I
2.
DSE Physics - Section E : Question
RA2 : Atomic Model

< HKCE 1981 Paper I - 9 >


PE-
RA2-
Q/02 I I 3. < HKCE 1982 Paper I- 8 >
DSE Physics - Section E : Question
RA2 : Atomic Model
PE-
RA2-Q
/
03 I
(a) (a) What are the mass numbers of

m- -
radioactive
source
(i) Cl-particles,
Figure l 20om -----•A
(ii) 13-particles, and

(iii) neutrons ? (3 marks)


radioactive
M
source
FigDTe2

20cm (b) The parent Cl source is � Ra. Ifthe daughter nucleus of Ra after Cl decay is X, write down the equation ofthe Cl-decay-.
N
(3 nwk,)

�� U is a radioactive source giving a., /3 and y radiations.

(i) If 2:U decays by emitting four ct-particles and two /:I-particles, what will be the atomic number and mass number
(c) If 7;1X decays by emitting one Cl particle and one 13 particle to form a stable product nucleus Y, what will be the atomic
of the resulting nucleus? (6 marks)
number and mass number ofY? (2 marks)

(ii) A GM counter is placed at A as shown in tp.e Figure I about 20 cm from the source. What types of radiation can (d)
144
be received by the counter at A ? (2 marks) / A

B ,/
142
N-o,
of C
(iii') An electric field is applied across the metal pl.ires Mand N as shown in the Figure 2 so that Mis connected to the
Neutrons 140
/
positive tenninal andNis connected to the negative terminal. ofa voltage supply. The GM counter is now moved to
B about 20 cm from the source. DescnOe and explain what h appens to the count-rate. (2 marks) ✓
/ D
138
,/

136 Atomic Number


88 90 92 94
(b) A volume of solution containing a radioactive isotope with an activity of 4400 Bq is now injected into the blood stream
of a patient. After 20 hours the activity of 10 cm• of blood becomes 2 Bq. If the half-life of the isotope is 10 hours,
estimate the volume ofblood inside the person. The above figure shows a radioactive decay series : A ➔ B ➔ C ➔ D ➔ E
(3 mar.ks)
(i) State what particles are emitted at each stage.

(c) If an Cl-particle is emitted from an atom of � Ra during the decay process, what will be the mass number and the
atomic number of the daughter atom? (2marks) (ii) What is the mass number of C? (I nwk)
I
4.
DSE Physics - Section E : Question
RA2 : Atomic Model

< HKCE 1991 Paper I- 7 >


PE
-RA2-
Q/0
4 i 1
5.
DSE Physics - Section E : Question
RA2 : Atomic Model

< HKCE 1993 Paper I- 7 >


PE -
RA2-
Q / 05
I
(a) Two metal platesX and Y are connected to a sensitive ammeter and an extta high tension supply (E.H.T.). A lead boxP In an experiment to measure the half-life of a radioactive isotope of sodium in a place where the background count rate is
is placed near the metal plates as shown in the below figure. 100 counts per minute, the following result is obtained :

=
Time/hour 0 20 40 60 80 100 120

--�--x Totalcount rate/counts per min. 1100 498 259 161 125 110 104

LeadboxP A
(a) Suggest TWO major sources of background radiation. (2 marks)

E.H.T.

(i) Sketch the electric field pattern betweenX and Y. The direction of the field should be shown.
(b) Plot the graph of the CORRECTED count rate against time on graph paper. Hence find the half-life of the isotope.
(it) If a radioactive source emitting o:; particles is placed in P, the ammeter shows that a current is flowing. ExplaID. (6nwks)
whythereisacurrent. (2marks)

(iii) Explain what happens to the ammeter reading ifthe source in (ii) is replaced by one emitting y rays? (2 m arks)

(iv) Suppose now a radioactive source 2;1Pa is placed inP. 2fiPa decays by emitting a� particle and yrays to form
a daughter nucleus U.
(1) Write down an equation for the decay.

(2) On the below figure, sketch and labelthe paths ofthe radiation emitted by the source.

--'---X

LeadboxPH A

E.H.T.
(b) Leaks in underground oil pipes can be detected by adding a small amount of radioactive source into the oil being
pumped. Oil flows out from the leaks and radioactivity is detected on the ground around the leaks.

(i) Which type of source (a., 13 ory) is suitable? Explam briefly. (2marks)

(c) By considering its half-life, state whether the isotope is suitable to be used for injecting into a patient's vein so as to
investigate his blood circulation. Give your reason. (3 marks)

(ii) Two sources emitting the suitable type of radiation of half-lives 50 years and 10 hours are available. Which one
should be used? Explain briefly. (3 marks)
I DSE Physics - Section E : Question
RA2 : Atomic Model

< HKCE 1994 Paper I - 6 >


P E-
RA2 -Q
/06 I I
7.
DSE Physics - Section E : Question
RA2 : Atomic Model

< HKCE 1996 Paper I - 6 >


PE -RA2-Q I 0711
6.
The below figure shows part of a decay series. Radioactive
142
""""
/A
s✓ �
14-0
Number '-c rrbolt
of 138
Neutrons D
Detector
136
E A factory produces detergent contained in plastic bottles. The following method is used to monitor the amount of detergent
I/ contained in each bottle : a radioactive source is placed on one side of the conveyor belt at the level to which the detergent is
134 Atomic Number expected to fill and a detector is placed at the same level on the oilier side as shown in the figure above.
86 88 90 92
(a) Which type of radioactive source (cc, f3 ory) should be used? Explain briefly why the other two types are not suitable.
(a) From the figure, name the particle which is emitted in each of the following changes:
(3 marks)
(i) A➔B
(ii) B➔C (2 marks)

(b) State two nuclides in the series which are isotopes of each other. (1 mark)
(b) Suggest one suitable detector for the above system. (lmarlc)

(c) The final stable nuclide of the series isx; whose atomic number is 82 and the number ofneutrons is 126.
(c) Explain how the monitoring system can detect bottles of detergent that have not been filled up to the required level.
(i) Fmd the mass numbers ofA andX
(3 marks)

(ii) Fmd the total number of ct particles emitted from A toX

(d) Some of the nuclides in the figure also emit y-radiation when they decay. However, it is impossible to identify these
nuclides from the figure. Explam briefly. (2 marks)
(d) Two sources emitting the suitable type of radiation of half-lives 10 minutes and 5 years are available.

(i) Expl3lll what is meant by the half.life of a radioactive source.


(e) A GM counter is placed 20 cm from a radioactive source which undergoes the decay as shown in the above figure. The
corrected count rates obtained in three consecutive minutes are 1027, 1011 and lO I 8 counts per minute respectively.

(i) What type(s) of radiation emitted by the source can reach the counter? Explain briefly. (2 marks) (ii) Which source should be used? Explain briefly.

(ii) Explain what is meant by a CORRECTED count rate. (2 marks)

(e) State two safety precautions that factory workers should take when handling radioactive sources. (2 marks)

(iii) Explain briefly why the three readings differ from each other. (2marks)
I DSE Physics - Section E : Question
RA2 : Atomic Model
PE - RA2 - QI 8
0
I I DSE Physics - Section E : Question
RA2 : Atomic Model
PE-RA2
-Q I 0
9 I
8. < HKCE 1997 Paper I - 6 > 8. (d) Xis placed very near the metal plates and the galvanometer reading is recorded every 30 seconds. The results obtained
are shown below :
High voltage
d.c. suPPly
�----� o-----� Time/ s 0 30 60 90 120 150
Current/ 72 48 32 22 15 10
(i) Plot a graph of current against time on graph paper.
Metal plates
I\ t -
" . ' ' ".
• •....+4+�-- i'-
: :t1.#j:
J: 1

-!- 4 1-t

Two metal plates are COllD.eclcd to a high voltage d.c. supply and a galvano.o:ieter as shown in the Figure above. When a
+
radioactive source X emitting a particles is placed very near the metal plates, the galvanometer shows that a current is
flowing. WhenX is moved a small distance away from the two plates, the galvanometer reading quickly drops to zero.

(a) Explain why there is a current and why jtis present onlywhen Xis verynoar the metal plates.

(b) �X decays by emitting an a: particle to fonn a stable nucleus Y. Write down an equation for the decay. What is the
neutron number ofY? (3 marks)

(i1) Hence find the half�Jife ofX


(c) How would the galvanometer reading be affected ifX is replaced by a � source ? Explain briefly. (2marks)
(Note : You may assume that the activity of the source is directly proportional to the current.)

(e) Explain whyXis not suitable for use as tracers. (1 mark)


1 DSE Physics - Section E : Question
RA2 : Atomic Model
PE-RA2-Q I 10
I I DSE Physics - Section E : Question
RA2 : Atomic Model
PE-RA2-Q/U I
9. < HKCE 1998 Paper I - 6 > 10. < HKCE 2000 Paper I - 11 >
The radioactive isotope of sodium, �Na , decays by emitting a ,8 particle to form a stable isotope of magnesium (Mg). (a) X and Y are two radioactive nuclides with half lives of 12 hours and 2.6 years respectively. Both two nuclides decay by
emitting a /1 particle to fom1 stable product nuclides.
(a) Write down an equation for the decay. (2nwks)
(i) After emitting a j]particle, how would the atomic number and mass number of nuclide Xbe changed ? (2 marks)

(b) Suppose you are given the following apparatus :


(it) Descnbe the changes in activity (in disintegrations per second) of a specimen of nuclide X and a specimen of Y
a GM counter, a sheet ofpaper and a 5 mm thick aluminium sheet. after one day. (2 marks)
Descn'be how you can demonstrate that �iNa emits ,Bparticles and does not emit aparticles.

(ill) Comment on the following statement:


The mass of the specimen containing nuclide X will be reduced by approximately half in 12 hours. (2nwks)

,�
i:
(b) A factory produces alumilrium sheets 1 mm in thickness. The thickness of the sheets is monitored by a gauge as shown
in the figure below. A jJ source is used in the gauge.
(c) The half-life of �Na is 15 hours. A sample of �Na withanactivityof32 x 103 disintegrations per second is injected
into the blood stream of a patient. After 45 hours, 6 cm.3 of blood is taken out from the patient's body and its activity is
found to be 5 disintegrations per second.

�i Na will have elapsed after 45 hours ? (1""'1<) Alumi:nimn.sheet---@�:,-------------;,;�,- -


LT
(i) How many half-lives of

Radioactive source

(i) Explain why a and r sources are not used in the gauge. (2nwks)

(ii) Estimate the volume of blood in the patient's body. (3 =ks)

(ll) Which ofthe nuclides (X or 1') is more suitable to use as the radioactive source? Explain your answer. (2 marks)

(ill.) Suggest two reasons for USUlg �Na in this dilution test (2nwks) (iii) The count rate recorded should be around 90 counts per second when the thickness of the aluminium sheet is 1 mm.
On a certain day when the gauge is operating properly, the following: data are recorded :

Time/s 0 10 20 30 40 50 60 70 80 90 100

Recorded count rate / counts per s 90 89 91 90 90 88 66 64 90 89 89

(d) State an application of radioactive isotopes, other than tracers, in each of the following fields ; Describe and explain the variation in the readings in the above table.

(i) Medicine (1 mark)

(ii) Industry (I mark)


1 DSE Physics - Section E : Question
RA2 : Atomic Model
PE -RA2 - Q / 1211
I DSE Physics - Section E : Question
RA2 : Atomic Model
PE-RA2-Q / 13

i
ytc-�-r-�:1/
11. < HKCE 2002 Paper I- 10 > 12. < HKCE 2004 Paper I- 9 > to an alarm cireuit

r---<'-=:::::J---------1111---�
7£#<r2,ow,
Iodine-131 ( 1�1) is a radioisotope which decays by emitting a !3-particle and y rays. It is used in hospitals to test the kidneys

Lcium
ofpatients. During the test, an iodine-131 solution is injected into the bloodstream of a patient As the blood passes through
the kidney, iodine-131 will be absorbed by the kidney and eventually excreted out of the body with urine. Ifthe kidney is not
functioning properly, both the absorption and excretion rates of iodine-131 will decrease. A y-dctector is placed near the ,Jot
electrode ..,,...,-
kidneys of the patient to detect the activity ofthe :radiation cotnlllg from the kidneys as shown in the above figure. SOlll'CC

(a) UsingXto denote the daughter nucleus, write down an equation for the decay ofan iodine-131 nucleus.
Figure 1 m.,...,
Figure 1 shows a smoke detector. The circuit inside the detector is shown in Figure 2. A small amount of1he radioisotope
americium-241 ( 2:�Am) is placed between two electrodes. The two electrodes are cOJlD.ected to a battery and an alann
(b) Explain why the !3-particles emitted by iodine-131 fail to reach the detector. (1 mark) circuit The detector has slots in it to allow airflow.
(a) An americium-241 nucleus decays by emitting an a-particle to form a daughter nucleus neptunium (Np), with a half-life
of 432 years.
(c) The half-life ofiodine-131 is 8 days. (i) Write down an equation for the decay ofan americium-241 nucleus. (2 =ks)
(i) State the meaning of 'half-life'. (2 rrwks)

(ii) Find the number of neutrons in the daughter nucleus.

(ii} For safety purposes, the activity of iodine-131 solution in the test should not exceed 1.5 x 108 disintegrations per
second. When an iodine-131 solution is prepared, its activity is 6 x 108 disintegrations per second. How many (b) Under normal conditions, a small current flows in the circuit inside the detector. However, when smoke particles enter
days after preparation would the solution be suitable for the test? (2marks) the detector, the current drops significantly. This triggers the alm:m to sound.
(i) Explain why a current flows between the electrodes under normal conditions. (3 marks)

(fu) Activity/ disintegrations per second

(ii) Suggest one possible reason why the current drops when smoke particles enter the detector. (2 marks)
Left kidney

(c) Explain why it is preferable for the radioactive source used in smoke detectors to have a long half-life. (2marks)
"--------------..Time/min
The above graph shows the variation of the activities of the radiation detected from the right and left kidneys of a
patient wi1h time. Which kidney do you think is not functioning properly ? Explain your answer. (3 marks)
(d) Carbon-14 ( 1!C ) is a radioisotope which decays by emitting fJ particles and has a half-life of 5700 years. Explain
whether this source is suitable for use in smoke detectors or not (2 marks)

(iv) Besides iodine-131, technetium-99m is another radioisotope that can be used in the kidney test. Technetium-99m
emits y radiation only and its half-life is 6 hours. Which of these two sources do you think is more preferable for
use in the kidney test 7 Explain your answer. (4 marks) (e) People are conce.med about the biological hazards of radiation. If you are the manufucturer of the above descnbed
smoke detector, how would you explam to the public that using the detector will not pose any health hazard ? (2 marks)
II
13.
DSE Physics - Section E : Question
RA2 : Atomic Model

< HKCE 2005 Paper I - 7 >


PE -RA2-Q /1411
I
14, <HKCE2009PaperI�7>
DSE Physics - Section E : Question
RA2 : Atomic Model
P E-
RA2-
Q/ 1511
Read the following passage aboutlodine-131 therapy and answer the questions that follow. Radon-222 (Rn-222) has a half-life of 3.8 days and undergoes a radioactive decay series as shown in the Figure below to
become a stable nuclide LeruH06 (Pb-206).
Iodine-131 is a radioisotope which emits.Bandy radiation. It can be used for thyroid cancer treatment.
Mass number
A patient suffering from thyroid cancer will first undergo an operation to have the thyroid gland removed.
However, some thyroid tissue may remain in the neck of the patient or may be carried in the blood stream to
other parts of the body. Iodine-131 if:I then used to trace and get rid of the remaining thyroid tissue in the
body.

Iodine-131 therapy consists of two stages. In Stage 1, the patient will take a low dose of Iodine-131 to trace
the remaining thyroid tissue. A detector is placed near the patient to monitor the activity of the radiation
coming from the patient.

In case any remaining thyroid tissue is spotted in Stage 1, the patient will then take a higher dose of Iodine-
131 in Stage 2. The iodine will be absorbed by the thyroid tissue and the radiation emitted can kill the
cancer cells.

Special hospital rooms are designed for patients who receive Stage 2 of the therapy. The rooms have metallic
shielding in the doors and reinforced walls. Inside the rooms, there are plastic covers on the furniture, doors,
handles and switches.

Source: Iodine-131 Therapy, The Ohio State University Medical Center, 2003.

(a) Explain why, in Stage 1, 13 radiation from the patient cannot be detected by the detector. (1 mark)
20
.:/:t.:�-� ·,---, ,- .-,-.- ,--- ,· -.., '
LL.W..U..U..U..U.'-'..U..U..W..U..U..U..U..U..LL.W..U..U..W..U..W..W..W..W..U..W..U.•Atomic number
82 83 84 85 86

(a) Estimate the mass ofundecayed Rn-222 after 15.2 days if its initial mass is I x Io-5 g.

(b) 1n Stage 2, which kind of radiation is more effective in killing cancer cells ? Explain your answer.

(b) State the nuclear radiation emitted in process (I) indicated in the above Figure. (I nmk)

(c) Write down a nuclear equation for process (2) indicated in the above Figure, (2 marks)
(c) State one special feature of the hospital rooms designed for patients receiving Stage 2 of the therapy and explain its
function. (2 marks)

(d) Determine the total number of a particles and the total number ofP particles emitted in the radioactive decay series from
Rn-222toPb-206. (4marks)
1
1S. <HKCE2010Paperl-8>
DSE Physics - Section E : Question
RA2 : Atomic Model
PE-RA2-Q /16

I I16.
DSE Physics - Section E : Question
RA2 : Atomic Model

< BK.CE 2010 Paper I- 7 >


PE-RA2-Q / 17

I
Read the following passage about low-level radioactive waste and answer the questions that follow. Carbon-14 dating can be used to identify the age of some objects. Living organisms contain a constant proportion of
carbon-14. After an organism dies, the amount ofcarbon-14 in it decreases due to decays. We can estimate the age of an
object by measuring the activity of carbon it contains.
Low-level Radioactive Waste
(a) Carpon-14 undergoes decay as shown in the following nuclear equation, where D denotes the daughter nucleus.
Industrial, medical and educational institutions in Hong Kong generate small amounts of low-level radioactive waste.
Such waste produces no detectable heat output and is of low radioactive level. Weakened radiation sources from '!C ➔�+�
hospitals and educational institutions are examples oflow-level radioactive waste.
Find the values ofx and y. (2,n,,ks)
For many yean;, most of the waste had been stored in disused tunnels and hospitals. The Government considers that in
the long nm the low-level radioactive waste should be stored in a pmpose-built facility. After about two years of
construction, the Low-level Radioactive Waste Storage Facility (the Fac.ilit:Y) (see the Figure below) at Siu A Chau, an
uninhabited island to the southwest of Lanta.u Island, was successfully commissioned and began its operation in July
2005. It comprises a shielded waste storage vault, a fully equipped laboratory. an automatic control room, an advanced (b) In a piece ofwood found, the activity of 10 g ofcarbon is 35 disintegrations per minute. It is known that the activity due
wastewater treatment plant and specially designed waste reception and processing area. The radiation levels inside and to 10 g of carbon in a living plant is 140 disintegrations per minute. Estimate the age of this piece of wood. Given that
outside the Facility are continuously monitored to ensure safe operation. the half-life ofcarbon-14 is 5700 years. (3 marks)

The Low-level Radioactive Waste Storage


Facility at Siu A Chau - Storage Vault

17. <HKCE2011Paperl-7>
It is known that plutonium-238 ( 2;!Pu} decays by emitting one ci. particle.

(a} Write a nuclear equation for the decay of plutonium-238. Use the symbol Y as the daughter nucleus. (2 marks}

(b) A sample ofplutonium-238 is put in a cloud chamber. Some tracks are seen.

(a) State one characteristic oflow-level radioactive waste. (1 mark) (i) Describe the tracks that are seen.

(b) Explain why Siu A Chau is suitable for the storage oflow-level radioactive waste. (1 mark) (ii) No tracks can be seen when the sample is covered by a piece ofpaper. Explain. (2nmks)

(c) Suggest an instrument to monitor the radiation levels inside and outside the Facility. (c) Plutonium-238 can be used in heater units in spacecrafts for outer space missions. It is known that the power of the
heater unit is directly proportional to the activity of plutonium-238 contained. Each heater unit has a power of 2 W
when it is newly manufuctured. How long can a newly manufuctured heater unit last if the minimum power ou1put
required is 0.25 W?
Given : half-life of plutonium-238 = 87.7 years (3 marks)
(d) In hospitals, rndioactive sources are used as tracers. The radioactive source is injected into a patient's body and the
radiation level is monitored with detectors outside the body. Explain whyy source is suitable for using as tracers.
(2nmks)
I DSE Physics - Section E : Question
RA2 : Atomic Model

Part B : .HKAL examination questions


PE-RA2
-Q/1 1
81
I
19.
DSE Physics - Section E : Question
RA2 : Atomic Model

(b) Fmd the age of the rock sample.


PE-RA2-Q/ 91
1 1

18. < HKAL 1993 Paper IIB-12 >

The :figure below shows the decay series for an isotope of uranium: 2;1U.

144

142 (c) Give two factors that determine the activit;y of a radioactive source.

"'
140
N,m,on P,
number

136
Atomic
134
88 89 90 9l 92 number
20. < HKAL 2009 Paper I- 8 >
(a) Name the particles emitted when
Carbon-14 dating is used in archaeological study to detennine the age of an ancient sample.
(i) Uranium (U) decays to Thorium (Th); and (1 nwk)
(Given : mass of one mole of carbon-12 = 12.0 g and half-life of carbon-14 = 5730 years )

(a) (i) Calculate the decay constantk, in s-1, ofcarbon-14.

(ri) Thorium (Th) decays to Protactinium (Pa). (I mark)

(ii) The relative abundance of carbon-14 in living things is only one wbon-14 atom for every 7.2 x 10 11 atoms of
carbon-12. Calculate the activity for 1 g of carbon in living things. (3 marks)
(b) Given that the half-life of �U is 7.1 x 10 8 yeani, what would be the percentage of �U left ina sample after a period
ofl 08 years ? (3 marks)

(b) (i) Explain the origin of carbon-14 in the atmosphere and why the abundance of carbon-14 in living things, such as
plants, remains more or less constant (3 marks)

19. < HKAL 1995 Paper I -10 >


The age of a sample ofrock containmg potassium-40 can be estimated by observing its activity. Potassium-40 decays to give
the stable isotope of Argon. The activity ofa sample is found to be 1.6 Bq We the original activity ofa similar rock having
the same mass is 4.8 Bq. The half-life ofpotassium-40 is 1.3 x 1(}9 years.

(a) (i) Find the decay constant ofpotassium-40. (2 marks) (ii) An archaeologist measured anactivity of2 Bq from 60 g of carbon in a piece ofancient bone. Use the result in (a),
estimate the age of the bone. (3 marks)

(ii) Give the physical meaning ofthe decay constant of a radioactive isotope.
I DSE Physics - Section E : Question
RA2 : Atomic Model

Part C : HKDSE examination questions


PE-RA2 -Q /20

I I22. <HKDSE2016Paper1B-9>
DSE Physics - Section E : Question
RA2 : Atomic Model
PE -
RA2-
Q /2111
Part of the decay series ofuranium-238 (U-238) is shown below. The end product lead-206 (Pb-206) is stable.
21. <HKDSE2013Paper1B-9>
Carbon-14 dating can be used to identify the age of some objects which have the 14C isotope, as it is radioactive and decays nJu ----"---�
9

by emitting a ft-particle. A piec e of wood sample is exammed using carbon-14 dating and its activity is 0.2 Bq. The half-life 4.5 x 10 years
of 14Cis 5730years. Given: I year = 3.16 x 107 s
(a) When a U-238 nucleus decays to a Pb-206 nucleus, how many a-particle(s) and ,B-particle(s) are emitted? (2 mark s)
(a) Calculate the decay constant of 14C in s·1• Hence find the number of 14C nuclei in this wood sample. (3 marks)

(b) As the first decay in the above chain from U to Th has a half-life much longer than those of subsequent decays, the
decay from U-238 to Pb--206 can be simplified to a single decay with half-life4.5 x 109years :

��U -----,- ---• 2��Pb


4.Sx 109 years
Suppose that a uranium-bearing rock contains only U-238 and no Pb-206 at the time when it was formed long ago by
solidification of molten material. In a particular sample of the rock, it is found that atpresent, the ratio :

number of Pb - 206 atoms 2


Assume that liVlllg organisms contain a constant proportion of carbon-14 in the ratio of14C J 12C = 1.3 x 10-12 during its life
time via intake ofcarbon dioxide (CGi) from the atmosphere. numberofU-238atoms 3

(b) The carbon content ofthis wood sample is found to contain a total of 1 x 1023 carbon nuclei. Estimate the number of 14 C (1) Estimate the age of the rock. Assume that all Pb-206 atoms come from the decay ofU-238 originally present in the
nuclei in the sample originally when it died. sample and ignore the small number ofU-238 atoms which have decayed but have not yet become Pb-206.
(1 mark)
(2 marlcs)

(ll') State, with a reason, whether the answer in (b} (i) is an overestimate or underestimate of the age ofthe rock if some
Pb-206 atoms have actually been lost (2 marks)

(iil) The graph lll. the Figure below shows how the number ofU-238 atoms in the sample varies with time t subsequently
(c) Estimate the age ofthis wood sample in years using the results found in (a) and (b). while t = 0 denotes the present time. On the same Figure, sketch a graph to show the variation of the number
ofPb-206 atoms in the sample with time. (2 marks)
no. ofatoms

'
' U- 8
0
PE-RA2-Q/22 PE-RA2-Q/23
I
DSE Physics - Section E : Question DSE Physics - Section E : Question
RA2 : Atomic Model RA2 : Atomic Model

23. <HKDSE2017Paperm-to> 24. < HKDSE 2018 Paper m -10 >


Dust may adhere to the surfaces of photos and films due to electrostatic attraction. To remove the dust effectively, a special (a) Part of the decay series ofradium-226 (Ra-226) is shown below. Ra-226 decays to radon (Rn) by emitting an ct particle
brush with a thin slice of polonium-210 c 2: Po) fixed near the brush hair as shovm. in the below Figure may be used. with half-life 1600 yean;. The end product in the series is lead (Pb), which is stable.
Polonium-210 undergoes a decay and the daughter nucleus lead (Pb) is stable.
_____, Pb

OJ �� Pb, 2� Pb and 20�Pb are three stable isotopes of lead. State, with a reason, which isotope can be the end
product in this series. (2 marks)

ti$•Blic.e:'Of.:POiom'um;.2J'.0
(ti) In a certain laboratory, aRa-226 source has been used for 50 years. Estimate the percentage ofundecayed Ra-226
left after this period (2 marks)
'��
(a) Write a nuclear equation for the decayofpolonium-210.

(b) Spark counter can show the ionizing power of radiations. The Figure indicated the main features of a type of spark
counter in school laboratories.

(b) Briefly explain how the a particles help clean the charged dust (2 marks)

A spark counter consists of afine metal wire mounted a few mm beneath an earthed metal gauze. The wire is connected
to the positive terminal ofan E.H.T. (Extra High Tension) supply so that a very intense electric field is set up between
the wire and the metal gauze. When a Ra-226 source is brought near the gauze, sparks giving out flashes of light and
crackling sound are produced at irregular intervals.

(c) Briefly explain why the polonium-210 slice must be fixed near to the brush hair. (l mark} (i) Explain why the sparks occur at irregular intervals. (1 """<)

A Ra-226 source used in school laboratories is usually said to emit ct, (:J, as well as y radiations.
(ii) Explain why (:J, radiation is also emitted even though the source is primarily an a-emitter. (1 nwk)
(d) The manufacturer recommends that the brush should be returned to the factory for replacement of the polonium-210
slice every year. Taking the activity of a newly replaced polonium-210 slice as 1 tmit, find its activity after one year
(365 days). Given: half-life ofpolonium-210 is 138 days. (2 marks)

(iii) Why is the sparking mainly caused by a radiation rather than p or r radiation ? Suggest a simple way to verify this.
(2 """1<s)
DSE Physics - Section E : Question Solution PE-RA2- QS I 01 DSE Physics - Section E : Question Solution PE-RA2-QS/ 02
RA2 : Atomic Model RA2 : Atomic Model
HKEAA's Matlcing Scheme is prepared for the markers' tefcrence. It should not be regarded as a set of model answers.
Students and teachers who are not involved in the marking process are advised to u,.t:c:n,ret the Madcing Scheme with care. 3. ca) (i) mass number of a: = 4 [!]

Question Solution (a) mass number of !3 = 0 [!]

(iii) mass number ofneutron = 1 [!]


L {a) (i) � should be used. [l]

(ill a: is not used because it is totally absorbed by the aluminium sheet [l ] (b) �: Ra ----)- � X + i a: [3]
OR
et is not used because its penetrating power is too weak that it cannot pass through alumiruum sheet. [l] (c) atomicnumber = 91-2+1=90 (!]
mass number =234 - 4 = 230 [!]
y is notused because the count rate would not be affected significantly by the aluminium sheet. [!]
OR (d) (i) A � B : a particle [!]
y is not used because its penetrating power i s too strong that almost all y will pass through the sheet. [I]
B � C : !3 particle [!]
(b) Any TWO ofthe following: [2] c� D : a: particle [!]
* radiotherapy D-----+ E : a: particle [!]
* estimate the age of archaeological samples (ii) MassnumberofC = 141 +92 =233 [!]
*

=r=r i==r
r :
medical tracer

* sterilization
* leakage test of underground oil pipes
4. ( a) (i)

* Sllloke detection

[Note : Thickness gauge is NOT acceptable since itis the application in part (a).)

< Direction of electric field lines is downwards> [!]


2. (a) (i) Atomicnumberofrcsultingnucleus"' 92-4x2+2 < The electric field lines are parallel and evenly spaced > [!]
[2]
= 86 (!] (ii) Air molecules are ionized by a particles. [!]
Massnumber of resulting nucleus = 238 - 4 x 4 [2] The ions then move to the metal plates to conduct a cmrent [!]
= 222 [I]
(iii) The ammeterreading decreases; (OR becomes zero) (!]
(ii) �andy [2] since the ionizing power of y radiation is very weak. [!]
(iii) Count-rate decreases [!]
[!]
since only background. radiation can be detected at point B [!]
(2)
(b) After20hours: A= 440ox(½r = 1100 [!] -----:S,-t-
--- --,,,,--x
B
r V
10 = 2
1100
[!] □f---===-------, y
------,-----Y
Volume ofblood = 5500 cm3 [!]

(c) Massnumber = 224-4 = 220 [!] < !3 is deflected upwards> (!]


Atomic number = 88 -2 = 86 [!] < y is not deflected > [l]
DSE Physics - Section E : Question Solution PE-RA2-QS/03 DSE Physics - Section E : Question Solution PE-RA2-QS/ 04
RA2 : Atomic Model RA2 : Atomic Model

4. (b) (i} Arsourceshouldbeused. [!] 5. (b) Half-life "" 15hours <I4-16hoursisacceptable> [!]
Since the penetrating power ofy is high enough to reach the ground. [!]
(c) Yes, it is suitable [!]
(ii) The source with halflife 10 hours should be used. [!]
The half-life is long enough for the doctor to d iagnose the patient. [1]
Reason: (Any ONE of the following) [2]
* It gives less pollution to the environment as its activity disappears quickly
The half-life is short enough to cause less harmful effect on the patient [1]

* It causes less hannful effect to the environment as its activity disappears quickly OR
The half-life is not too short [1]
and not too long. [!]
s. (,) Any TWO of the following: [2]
* Cosmic radiation from the space
* Radiation from the rocks
* Radiation from air
6. (a) (i) o: particle [!]

* Radiation from food (Ii) /3 particle

(b) (b) AandDareisotopes ofeachother. {OR BandE) [!]


Time/hour 0 20 40 60 80 100 120

Corrected count rate / cpm 1000 398 159 61 25 10 4 (c) {i) MassnumberofA = 142+90 = 232
[1]
Corrected count rate/ cpm Mass number ofX = 126 + 82 = 208
1000
(h') Total number of o: particles emitted = 232-208 [!]
4
• 6 [1]

(d) y emission does not change the atomic number and mass number ofthe nuclide. [2]

. :.+' - ,- {e) (i) /3 and y radiation can reach the counter because their ranges in air are longerthan 20 cm. [2]
' ""'-+ '
_,
; (ii) A corrected count rate is equal to the recorded count rate minus the background count rate. [2]

'.
.-1-1+• :·-i
(fu') The readings differ d ue to therandom nature of radiation. [2 ]

400

7. (a) A /3 source should be used. [!]


An a: source is not suitable because a: particles cannot pass through the container. [!]
A y source is not suitable because y rad iation is too penetrating. [!]
(OR cannot be absorbed by the container)

Time/hour (b) AGM tube {OR Geiger Muller tube) (OR GM counter) can be used.
0 20 120
< Two axes labelled correctly> [!] (c) If a bottlenot filled up to the required level passes the source,
< Suitable scales chosen> [!] the counter will record a much higher reading than that when an acceptable bottle passes th e source, [2]
< At least 5 points plotted correctly> [!]
since the /3 radiation does not pass through the detergent and hence isnot absorbed. [!]
< Smooth curve drawn > [!]
DSE Physics - Section E : Question Solution PE-RA2- QS I 05 DSE Physics - Section E : Question Solution PE-RA2-QS/06
RA2 : Atomic Model RA2 : Atomic Model

7. (d) (i) The half-life is the time taken for half ofthe number ofundecayed nuclei in the source to decay. [2] 8. (d) (i)
Current/ µA
OR
The half-life is the time taken fur the activity ofthe source to fall to halfof its initial value. [2]
OR

The half-life is the time taken for the mass of the undecayed nuclei in the source to decay. [2]

(u) The source with half-life 5 years should be used. [!]

Reason: (ANY ONE) [2]

* The source will decay slowly and can be used for a long time.

* The activity ofthe source will be very stable to be used for a long time.

* The source vvith half-life 10 minutes will decay rapidly


and the registered count rate is unstable even when no bottles are present.

(e) AeyTWOofthcfollowings: [2]


* Wearing film badges or other detecting devices

* Working behind lead-glass windows

* Handling :radioactive sources using special forceps (OR remote controlled robots)

* Wcaring protective coveralls

* Radioactive sources should not be pointed towards human bodies

* Radioactive sources should be stored in lead castles and returned to the storage box after use

* Workers should wear disposable gloves to handle the radioactive sources


10

8. (a) Some air molecules are ionized by the a-particles. [!]


++,
- -·H
..,. -
·,·-·,··,-, - , ,., ,-, , . ,-.-,, ., ,,
The ions then move to the two respective plates to form a current. [!] 0 �
As a-particles have a very short range in air, the source must be placed very close to the plates. [!] < Labelled axes with units > [!]
< An appropriate scale> [!]
< Correct points plotted > [!]
< Smooth curve> [!]
< Mass number and atomic number ofY are correct> [!]
< Equation is correct> [!] (ii) From the graph, the halfwlife of the source is 52 s. [!]
< from 50 s to 54 s is acceptable >
The neutron number ofY is 132. [!]
(e) AnyONEofthefollowing: [!]
(c) The galvanometer reading decreases {OR becomes very small) [!] * The penetrating power of a-particles is too low to be used as tracers.
because the ionizing power of,8-particles is weaker. [!] * The half-life ofXis too short to be used as tracers.
9. (a) �Na
DSE Physics - Section E : Question Solution
RA2 : Atomic Model
PE-RA2-QS/07

[2]
1
10.
DSE Physics - Section E : Question Solution
RA2 : Atomic Model

(b) (i) a: source is not used because the penetrating power ofa particles is too low.
PE-RA2-QS/08

[1]
l1

y source is not used because the penetrating power ofy radiation is too high. [I]
(b) The GM tube is held close from the source and its reading is noted. [l]
Insert a piece ofpaper between the GM tube and the source. [1] (ll) Nuclide Yis more suitable. [I]
The count rate would remain unaffected. This shows that the source does not emit ct particles. [l] As nuclide Yhas a longer half-life, its activity remains stable over a longer period oftime. [I]
Insert the aluminiumsheet between the tube and the source. The count rate would drop significantly. [!]
(ri.i) The reading remains steady from t = 0 to 50 sand from t = 80 to 100 s. [!]
This shows that the source emits 13 particles.
The small variation within this period is due to the random nature of radioactive decay. [I]
(c) (i} Numberofha!f-liveselapsed = � = 3 [1] The reading drops significantly from t= 60 to 70 s.
15 [I]

(ii) Total activity in the blood ofthepatientafter45 hours = 32 x 103 x (½J -" 4000 [!]
The aluminium. sheet in this period is thicker than the normal value. [I]

V 4000
[l]
6 5
OR [2]
V = 4800 cm. 3 [l]
(b) The P particles fail to pass through the human body. [I]
(c) (iii) luiyTWOofthefollowing: [2]
* The half-life is long enough for medical diagnosis. OR

* The half-life is short enough to reduce the hanniiil effect to the human body. The p particles are absorbed by the human body. [1]
* Thedaugh.tetnuclei Mg isstableand has noharmfuleffect
(c) (i) The half-life is the time taken for the activity ofthe source to drop to half of its initial value. [ 2]
(OR Sodium and magnesium have no harmful chemical effects on human body,)
(it) No. ofhalf-life = 2 [I]
(d) {i) Any ONE ofthe followings: [1]
* Radiotherapy
Thesolution is suitable after 2 x 8 = 16 days [!]

* Medical tracer (fu) The left kidney is not functioning properly


*
[!]
Sterilization of medical equipment since the activity in the left kidney increases at a lower rate. [2]
(ii) Any ONE of the followings: [l] (Iv) Tecbnetium-99m is more preferable than iodine-131 for use in the test.
* Thickness gauge
[I]

*
Since tecbnetium-99m has a shorter half-life [I]
Food preservation (Steriliz.ation ofbeef)
and does not emit P particles, [I]
*Le,bg,-
* Radioactive lightning conductor so it causes less harmful effect to the patient. [I]

* Smoke detector
[2]
10. (a) (i) The atomicnumberincreases byone. [!]
The mass number remains unchanged. [1] (it) Numberofneutrons = 237-93 = 144 [1]

(ii) The activity of spccimenXwill fall to a quarter ofits original value. [1] (b) (i) The ex-particles will ionize the !Ill' to give ions. [2]
The activity of specimen Y will remain approximately unchanged.. [1] The ions then move to the electrodes to give a CUirellt [I]
(iii) As the mass of p particles emitted is very small, [1] (li) The smoke particles block the movement ofthe charged particles. [!]
the mass of the specimen would almost remain unchanged after 12 hours. [!] As a result, fewer ions reach the electrodes, so the current drops. [I]
DSE Physics - Section E : Question Solution PE-RA2- QS /09 DSE Physics - Section E : Question Solution PE-RA2- QS / 10
RA2 : Atomic Model RA2 : Atomic Model
I
12. (c) The activity of the source will remain stable for a long period of time. (OR decay very slowly) [!] 15. (a) It produces no detectable heat output [!]
So the detector can be used for a longer timer. (OR The source needs not be replaced frequently.) [!] OR
It has a low radioactive level [!]
(d) As ,8 particles have a weaker ionizing power, [!]
the current flowing between the electrodes will b e extremely small. [!]
(b) It is because it is an uninhabited place. [!]
So Carbon-14 is not suitable.
(c) GM counter ( OR GM tube) [!]
(e) Any ONEofthefollowing: [2]
OR
* The radiation dose from the smoke detector is very small
photographic fihn [!]

* The radiation from the smoke detector is much less than the background radiation. (d) Ithasweakionizingpower [!]

* The source used in the smoke detector is a very weak source.


and causes less hatmful effect to the human body. [!]

* ex-particles have a very short range in air. OR


It has strong penetrating power [!]
and canpass through the body to be detected outside the body. [!]
13. (a) The penetratingpower of !3 radiation is too low. [!]
OR
!3 radiation cannot penetrate through human body. [!] 16. (a) x = 7 [!]
y = 14 [1]
(b) !3 radiation is more effective in killing cancer cells.

(½J
[!]
Since the ionizing power of!3 is higher than �t ofy radiation. [!] OR
(b) 35 = 140 140➔70➔35 [1]

(c) The rooms have metallic shielding in the doors and walls. [!]
:. n = 2 [1]
They can prevent radiation from leaking out of the rooms. [!]
OR The age of the wood = 2 x 5700 "' 11400 years [1]
Inside the rooms, there are plastic covers on the furniture, doors, handles and switches. [!]
This prevents otherpersons using the room from being contaminated. [!]
(OR ;o:) [2]

(b) (i) The tracks are straight [1]


14. (a) Number ofhalWves = lS.2 = 4 [!]
3.8 OR
The tracks are thick.
Mass ofRn-222 left = 1 x 10-5 x (½ )4 = 6.25 x 10-1 g [!]
(ii") As o: radiation has weak penetratingpower, [1]
(b) " [!] they are stopped by thepaper. [1]

[2] (c) 2W ➔ I W ➔ 0.5W ➔ 0.25W [!]


OR
(½)" .-. n = 3
(d) Let a and b be the number of a particles and ,8 particles respectively.
222-206=4a [!] o�s =
[1]
a = 4 [!]
86-82=4x2-b [!] Hence, the heater can last 3 half-lives. [1]
b =4 [!] Time = 3 x 87.7 = 263.1 years < accept 263 years > [1]
18. (a) (i) o:particle
DSE Physics - Section E : Question Solution
RA2 : Atomic Model
PE-RA2-QS/11

[!]
I
20.
DSE Physics - Section E : Question Solution
RA2 : Atomic Model

(b) (i) Carbon-14 is formed when neutrons produced by cosmic rays collide with nitrogen.
1�N+tn➔
PE-RA2-QS / 1211
(ii') /3 particle [!]
'!c+:,P [!]

Carbon-14 forms radioactive carbon dioxide and is taken up by plants for photosynthesis.
(b) k =�
7.lxl08
= 9.76x10-10 year-1 [!] ( OR Carbon-14 is taken up by animals through eating.) [!]
This exchange maintains the same abundance inside a living thing until it dies. [I]
[I]
(li) Activity of the bone per gram: A = ,.3_ = 0.0333 Bq [!]
:. perccntageleft = 90.7% [!] 60

OR By A= A0 e-kt :. (0.0333) = (0.268) C -(3.S4xl0-ll)I [I]


t = 5.43 IQ ll S = 17200 years [!]
-' " '4
X

_J{_ a ( 2) [!]
N, OR
o')l{7,WO')
= (½J = 0.907 [I]
By A•A,,
(!)"'
2
• (0.0333) a (0268)[½)
,mao
[!]
.-. percentage left = 90.7% [!]
t = 17200 years [!]

(a) (i) k ln2 OR ln2


19. [I]
l.3x10 9 1.3xl09 x365x24x3600 21. �Z
(a) k "" =cc-'in '-c-= [!]
5730x3.16xl0
7

= 5.33 X 10-10 yearl OR 1.69 X 10- s-1


11
[!]
By A= kN
(ii) The decay constant ofa radioactive isotope is the probability of decay ofthe nuclei present [!] :. {0.2) = (3.83 x 10-12) N [!]
per unit time. [!]
:. N = 5.22 x IQIO [!]
(b) (b) No = (1 X 1023) X (1.3 X 10-� = 1.3 X 10 11 [!]
OR (1.6) = (4.8) e-t.6!lx!O-", [!]
(c) N = No (f"'kt
= 2.06 x 109 years OR = 1016 s [!] 3 3 "')t
(5.22 x 101°) = (1.3 x 1011) e-( .s xW
t t 6.50x
[I]
(c) © Thenumberofundecayednuclei present [!] :. t = 2.38 x IO" s = 7540 years < accept 7500 to 7600 years> [!]
@ The decay constant of the radioactive source < OR The half-life of the radioactive source> [!]

22. {a) 238 = 206 + 4 n« :. n« = 8 [!]


ln2 = ln2
20. (a) (i) k = [!]
/112 (5730x365x24x3600) 92 = 82 + 8X(2) + n�( -1) :. n�=6 [!]
= 3.84 X IQ-12. s~l [!]

(:h') Number of carbon-14 atoms in 1 g of carbon:


I I
[OR N = N0 e-k1 and k
'"
= ln2 J [!]

= - X 6.02xl0z:i X --- = 6.97xl010 [!] 3


[J)l1(4,S><lo'J

7.2xlOll c,) a 2
N
12
A = kN = (3.84 x 10-12) x (6.97 x 101°) [!]
t = 3.32 x 109 years < accept 3.3 x 109 years> OJ
= 0.268 Bq < accept 0267 Bq> [I]
DSE Physics - Section E : Question Solution PE-RA2-QS I 13 DSE Physics - Section E : Question Solution PE- RA2 -QS / 141
RA2 : Atomic Model RA2 : Atomic Model

22. (b) (it") Answer in part (i) is underestimate. [!] 24. (a) (i) 226-206 = 20 whichisamulti.pleof4(foro:) [!]
The origilialnmnber ofU-238 should be greater. 2
:. � Pb is the end product [!]
present number of U -235 at oms
The ratio is in fact smaller than � [!]
originalnumberofU-238atoms 5
[!]
thus, longer time should have been elapsed.

OR = 97.9% <accept 98%> [!]


OR
Answer in part (i) is underestimate. [!]
Since more U-235 should have been decayed, [!] k= �= 4.33xl0--4
1600
thus longer time should have been elapsed.
!!__ = e
-kl = e-(4.33xl0-< )x(SO)
[!]
(iii) N,
no. of atoms
= 97.9% <accept 98%> [!]

Pb- , o, (b) {i) Due to random nature ofradiation [!]

(ii) Some of the daughter products ofRa-226 may emit ,8particles [!]

(fuJ Since the ionizing power of f3 and rare weaker than that of a [!]
U-' 8
Any ONE of the following: [!]
0
* Raise the source to a distance greater than the range of a, sparks will cease.
* Insert a paper between the source and the gauze, sparks will cease.

< Initial no. ofPb-206 is at 2 units as U: Pb= 3 : 2 initially> [!]


<Final no. ofPb is at4.5 units since U + Pb = 5 and finally Uhas 0.5 unit> [!]

23. (a)
2 2
:Po ➔ :Pb + iHe < accept i o; > [2]

(b) The a; particles ionize the air molecules. [!]


The ions neutralize the charges on the dust (OR film surface). [!]

(c) This is because a. has a short range ofa few centimetre in air. [!]

36S/138
j
(d) A = (I) X (-) [!]
2
= 0.160 unit [!]

OR

k = lnZ = 0.005023 [!]


(138)
A = (1) e-(O.ll05!W)(36SJ = 0.160 unit [!]
Hong Kong Diploma of Secondary Education Examination
Physics - Compulsory part (M,l-;is,i-)
Section A- Heat and Gases C�:f"t.:tt.)
1. Tcmperature,Heatan dintemalenergy(;l!.� '#..'fl'Pl:it.)
2. TransferProcesses(#..#$.ilUI)
I DSE Physics - Section E : MC.
RA3 : Nuclear Energy

Use the following data wherever necessary :


PE-RA3
- M/ O l
ll

3. Ch angeofState(*h!Mli;t.l!i!:)
4. General Gas Law (f-:iUi.ilt:$::4:t) Atomic mas s unit U = 1.661 X 1Q·2'7 kg (1 u is equivalent to 931 MeV)
5. Kinetic Theory (�"TiU(rtll,)
Section B- Force and Motion (:/J;fi,;r(�) Speed oflightin vacuum
1. PositionandMovement(,utJ::ft,;f,;t,i)
2. Newton's1aws(4'Mit.f.lt)
Chargeofelectron e= L6xlo·19c
3. Moment ofForce (:h�)
Avogadro constant
4. Work.EnergyandPower(fp/.1, �:li:if�:W.'.fl.)
5. M omentum(l!lii:) Molar gas constant
6. Projec ti le Motion (:&lttl�tiJ)
7. CircularMotion(!HJ;llliltlJ)
8. Gt-avitation(�!fl)
The following list of formulae may be found useful:
Section C- Wave Motion (iltt/J)
1. Wave Propagation (iti¥.141Ht) Law of radioa ctive decay
2. Wave Phenomena (it�J.Jl�)
3. Reflectionand RefractionofLlght (76>¥.1.&.Af.&.# Af)
ln2
4. L enses (.i!it) Half.life anddecay constant t1.• = -
5. WaveNatureofLight(,;'c.��:th#-ht) k
6. Sound(�¼)
Activity and the numberofundec ayed nuclei A=kN
Section D - Electricity and Magnetism ( 11!:::fi>.i:i)
1. Electr ostatics C*1!!:4!-) Mass--energy relationsh ip M=!:i.mc2
2. Ele ctricCircuits(1!!:J!S.)
3. Domestic Electricity { *� Jff 'Ii!:)
4. MagneticField(tii::J¼)
5. Electromagn etic Induction (1!!:-filiUU
Part A : HK.CE examination questions
6. Alternating Current (3(.$1,i_ 1!:)
Section E- Radioactivity and Nuclear Energy (�MJJtt.'fl';Jt�I;)
1. < HKCE 1983 Paper II - 37 >
1. Radiation and Radioactivity (�Af'f";!;x.M;,U'l)
2. Atomic Model (,W-T��) In the following nuclear reaction:
3. NuclearEnergy(#int)
�;Al + iHe ➔ ��p + X
Physics - Elective part (i!1ffF 1t) what is the mass nwriberand atomic number ofX?
Elective I -Astronomy and Space Science (1-.�.!¥,,fi,fl/t.1-.#<$-) Mass number Atomic number
1. The universe seen in different scales (:f: !llJ :!!M;!l.i.r 1,1/ � 'iii ilii tie.)
A. 0
2. Astronomythrough history (��*llllff-Jil.Jt)
3. Orbital motions under gravity (�hTllll'-M..it.iltlJ) B. 0 -1
4. Stars and the universe (•/1.!l.'f" q:'jjj) c. 4 2
D. 0 0
Elective 2-Atomic World (4 "'T-tJI:�)
1. Rutherford's atomic model C.tl&#i&.T.ilt�)
2. Photoelectric effect (;It, 1t£t..1.)
2. < HKCE 1986 Paper II- 40 >
3. Bohr's atomic modelofhydrogen(;ti 1,1/ itli T-fiti!)
4. Particlesorwaves(Jl'.tt.T.;lilt) Which of the following equationsrepresent(s) possible nuclear reac tions?
5. Probinginto nanoscale(1it�.tfi;f.'!lt.J})
(1) 1�B + neutron --,,. �Li+ a.particle
Elective 3 - Energy and Use of Energy (.tl;t�fisi'N.�11t.lfl')
1. Electricity at home (��/fl it) 2
�Bi --> �Po + f3 particle
(2)
2. Ener gy efficien cyin building (��tro�i/ii.a!fi)
3. Energy efficiency in transportation (.;t#'.tfrott.ij.a!fi)
4. Non•renewable energy sources (;f:";;J".jlj-:itt,i!f.) i
(3) 1 N + a particle --> '! 0 + proton
5. Renewable energy-sources ('if.j?}.:i:fisi:l!i.)
A. (l)on.J,y
Elective 4 -Medical Physics C -1-!¥:#Jl£ �) B. (2) onl,y
1. Making sense of the eye C !llH{I S; 1") C. (1) &(3) o nly
2. Makingsenseoftheear(�'9"<i\'i) D. (2) & (3)only
3. Medical imaging usingnon•ionizingradiation (.'/f 'lltNE.t&M M*i.HU�)
4. Medical imaging using ionizing radiation (1tN!:.t&Af-U-,f,:J:Hf.4'e)
! l.l=====D
...
3.
RA3 YSlC .;;;:s - S ecllEnergy
;;;:SE;;;::PhNuclear
;;;:
= ;;;: ;;;: = = ;;;: = ;;;:
< HKCE 1991 Paper II - 39 >
E -
·on E : M .c . =====P - RA =3 M /02 11
_ _;;;: - = - :::.1._ I
7. < HKCE 2005 Paper II- 26 >
DSE Physics - Section E : M.C.
RA3 : Nuclear Energy
P-
E RA
-3 M/03 l
:Kr+
"Which of the following equations represents(s) possible nuclear reaction(s) ? Which of the following nuclear reactions is a nuclear fusion?

(1) ii AI+ iHe --4 �p + proton A. �;u n -......----+ 1�Ba +


+ 2n
1
B. iN + n � :c + iH
1
(2) �H+ !H -4 iHe+neutron
C. ;H + fH ---+ iHe + n
(3) �� Th � �Pa + � particle D. ��U --,. �Th+ a
A. (1) only
,.
--
B. (2) only < HKCE 2008 Paper II- 26 >
C. (1) & (3) only Which of the following nuclear reactions is a fission ?
D. (2) & (3) only 1
A. :z:iJu + , n .:BA + �!Kr+ 3 �n
1

4. < BKCE 1993 Paper II - 41 > B. ��u + , n


1 23
iPu + 2 -�e
c. ��u+ l�N ------'J, �!Es+ 4 �n
�Al+ iHe ----+
D. �H + �H ------'), iHe + ,n
In the above nuclear reaction, what are the atomic number and mass number ofX?
Atomic number Mass number
•• < HKCE 2009 Paper II - 27 >
A. -1 0 Which ofthe following conditions is/are necessary to sustain the chain reaction in the nuclear fission ofurani.um-235 ?
B. -1 1 (1) Each fission produces a large amount of energy.
c. 0 1 (2) At least one neutron is released in each fission.
D. l 0 (3) Each :fission produces two smaller nuclei
A. (1) only
B. (2) ooly
S. <HKCE1996Paperil-37> C. (1) & (3) only
In the below nuclear reactions, what do x; Y and Z represent ? D. (2) & (3) only
2 2:Pb + X
(1) �Po 10. < HKCE 200.9 Paper Il- 24 >

(2) ; H + \H iHe + y ;H+�H ➔ ;He+n

(3) �Pa + Z Which of the following descriptions about the nuclear reaction above is correct ?
��Th
A. It is a nuclear fission.
X y z B. It is a nuclear fusion.
C. It is a chain reaction.
A. an ct particle a proton a !3 particle D. It is a radioactive decay.
B. an ct particle a neutron a !3 particle
C. an a. particle a neutron yrays
D. a !3 particle ,n,utron yray s
Part B : BKAL examination questions
6. < HKCE 2004 Paper n- 40 > 11. < HKAL 1.980 Paper I- 49 >

�H + � ➔ ;z + aneutron Which ofthe following equations represent possible nuclear reactions?


1
(1) �B + neutron ..............- �Li+ tm:

i
Find the values ofx and y in the above nuclear reaction. 2
(2)
2
��Bi � �Po .+ beta particle
X
A. 2
y
1
(3)
1
i 1
N + He ---?> ; 0 + proton
A. (1) ooly
B. 2 2
B. (3) on ly
C. 3 1
C. (1) & (2 ) only
D. 3 2
D. (2) & (3} ooly
I12.
DSE Physics - Section E : M.C.
RA3 : Nuclear Energy

< � 1992 Paper I- 48 >


P E-
RA3-
M/0
411
I
17. <HKAL2012PaperlIA-4S>
DSE Physics - Section E : M,C.
RA3 : Nuclear Energy
PE-
RA3-
M/ 0511

1� N + alpha particle ----+ proton + X Which ofthe following nuclear reactions are accompanied with a mass defect?
In the above nuclear reaction, X represents (l) the union oflzydrogen isotopes to form helium
A . i;o
�H + �H --+ �He+ �n
B. 1;F
(2) the natural radioactive decay ofradium-226
C. 1;N
1� 1;:Ra � �Rn+ iHe
D. C
(3) the emission of a ,B-particle from a nucleus
13. < HKAL 1992 Paper I - 44 > �n � iH + -� e
The main reason why a chain reaction can occur in a nuclear reactor using uranium-235 is that A. (1)&(2)only
A. a large quantity of energy is evolved in each fission. B. (i) &(3) on ly
B. the products ofnuclear fission are highly radioactive. C. (2) & (3) only
C. plutonium is produced and it undergoes further fission. D. (1),( 2)&(3)
D. more than l neutron is produced when a nucleus undergoes fission.

14. < HKAL 2003 Paper IIA- 43 > 18. <HKAL2013 Paper IIA -44>
The sun and stars give out their power mainly by The sun radiates energy at a constant rate of4.0 x 1025 Wby a nuclear fusion process. The mass ofthe sun is 2.0 x 1030 kg.
(1) radioactive decay. Estimate the lifetime ofthe sun if0.07% ofits mass is converted into radiation energy during the sun's lifetime.
(2) nuclearfission. Given; 1 year= 3.15 X 101 S
(3) nuclear fusion.
A. (I) only A. 1.0 X 106 years
B. (3) onl y B. 1.0 X 10 10 years
C. (!) & (2) only C. 1.0 X 1012 years
D. (2) & (3) only
D. LO x 10 17 years

15. ◄ HKAL 2004 PaperIIA - 44 >


i
Two deuterons, �H, combine together form a helium isotope, He, with the release ofenergy as shown below.
Part C : Supplemental exercise
2�H ➔ �He+ X + energy
Which of the following statements are correct ? 19. In which type of nuclear reaction are the daughter nuclei heavier than the mother nuclei ?
(1) This is an example of nuclear fusion. A. a-decay
(2) The total mass of !He and Xis greater than that of the two �H. B. ,B-decay
(3) Xis a neutron. C. r-emission
) '""'-A. (!) & (2) only
!ll. (!) & (3) only D. nuclear fusion
C. (2) & (3) onl y
D. (!), (2) & (3)
20. A worker at a nuclear plant walks into a room and is accidentally exposed to a small amount of radiation. The worker will
A. lose consciousness.
16. < HKAL 2005 P ap er 1IA - 22 >
B. feel very hot.
The following equation represents a nuclear fission reaction ofU-235, producing q neutrons. C. feel painful. \
D. feel no effect.
What are the values of the atomic number Zand the number q?
z q
21, In the Sun, energy is released when hydrogen nuclei collide and form heaviernuclei. This process is called
A. diffusion.
A. 37 2
B. 36 2 B. fission.
C. 36 3 c. fusion.
D. 34 3 D. ionization.
DSE Physics - Section E : M.C.
RA3 : Nuclear Energy
PE-
RA3-M
/ 06 I I DSE Physics - Section E: MC.
RA3 : Nuclear Energy
PE-RA3-M/0711

22. In a particular chain reaction, a neutron collides with a heavy nucleus. The nucleus then splits to give two lighter nuclei, 27. The main reason why a chain reaction can occur in a nuclear reactor using uranium is that
energy and
A. a large amount of energy is released in each fission.
A. alpha particles.
B. the products ofnuclear fission are bigblyradioactive.
B. - beta particles.
C. uranium splits into two smaller fragments.
C. �protons.
D. fission neutrons are produced
D. ' neutrons.

28. Ifthere were accident in a nearby nuclear power plant, which ofthe following is NOT the way that the radioactive substances
23. Vlhich of the following show(s) nuclear fission?
released in the accident can spread to the neighbouring lands?
23
(1) �U + �n A. Bywind
B. Byrain water
(2) z:!Pu � 23
� U + � He
c. By -
D. Byplants

A. (1) oilly
ii. ·(3) only 29. Which ofthe following is NOT the disadvantage of using nuclear energy?
C. . (1) & (2) only
A. The capital investment of a nuclear power plant is very large.
D. (2) & (3) o nly
B. There must be leakage ofradiation i n a nuclear power plant
C. Once accident occurs, it would be veryserious.
24. A U-235 nucleus 'Will split when it captures
D. The disposal. ofradioactive waste is a difficult problem.
A. an alpha particle.
B. a beta particle.
C. a neutron. 30. Which ofthe following do(es)NOT make use ofnuclear fusion?
D. aproton. (1) A nuclear bomb
(2) A hydrogen bomb
25. The Stm. releases its energy mainly by (3) Emission oflightby a star
A. (1) only
(1) radioactive'decay.
B. (3) only
(2) nuclear fission. C. (1) &(2) only
(3) nuclear fusion. D. (2) & (3) only
A. (!)only
B. (3)only
31. Which ofthe followings are the advantages ofusing nuclear energy?
C. (1) & (2) only
D. (2)&(3)on ly (1) Nuclear energy causes less pollution to our environment
(2) The running cost ofpower plant using nuclear energy is lower.
26. The following equations represent some typical nuclear reactions : (3) Nuclear energy is the only choice other than the use of fossil fuel.
(I) !Be + :H -,), �Li + ;He A. (l) &(2) only
B. (1) &(3) only
(II) ;H + �H ---+ 1He + �n C. (2) &(3) only
D. (!), (2) &(3)
(III) :i;;u + tn ---+
1
i�La + �!Br + 3 �n
,. Which of the following descriptions ofthese reactions is/are correct? 32. Which ofthe following are the advantages ofusing nuclear fusion to generate electricity?
(1) Reaction (I) represents an o:-decay. (1) The fuel for nuclear fusion is hyd rogen which has unlimited supply in oceans.
(2} Reaction (II) represep.ts a nuclear fusion. (2) The waste products in nuclear fusion are not radioactive.
(3) Reaction (Ill) represents a nuclear fission. (3) The nuclear fusion takes place at a very high temperature.
A. (l)only A. (1) &(2} only
B. · (1) & (2) only B. (1) &(3) only
C, (2) & (3)only C. (2) & (3) only
D. (1), (2) &(3) D. (1), (2)&(3)
I
33.
DSE Physics - Section E : M.C.
RA3 : Nuclear Energy

Which of the following are the difficulties to use nuclear fusion for generating electricity?
(1) Nuclear fusion can only take place at a very high temperature.
PE
-RA3-M
/0811
I DSE Physics - Section E : M.C.
RA3 : Nuclear Energy

Part D : HKDSE examination questions


PE-
RA3-
M/0
911

(2) No physical container can withstand the high temperature that fusion occurs. 38. < HKDSE Sample Paper IA-34 >
(3) It is difficult to dispose the waste products of the fusion. Which ofthese is a nuclear fusion reaction?
A. {1) & (2) only
B, (!) & (3) only
C. (2) & {3) only
D. (!), (2) & (3)

�:Ra -
34. Which of the following nuclear reactions is/are an example of fusion?
(1) 2;!N + �He+ energy

(2) '!c ---i>


1
iN + -�e + energy
39. < BKDSE Practice Paper IA-36 >
· {3) ;H + ;H --), ;He+ �n + energy For the following nuclear reaction, state the type of reaction and determine the energy released.
A. (1) only
�H+ �H � jHe+ �n
B. (3) only
C. (Ij & (2) only
Given: mass of� H = 2.014 u
D. (2) & (3) only
mass of�H = 3.016u
35. Which ofthefollowing nuclear reactions is an example of fusion? mass of j He = 4.003 u
A. Carbon-14 decaying to nitrogen and an electron mass of �n = 1.009 u
B. Two heavy hydrogen nuclei becoming helium and a neutron
Type of reaction Energy released
C. Radium-226 decaying to radon-222 and an alpha particle
A. fusion 0.018MeV
D. Sodium-24 decaying to magnesium..Z4 and a beta particle B. fu,ion 16.76MeV
C. fission 0.018MeV
36. In the following nuclear decay: D. fission 16.76MeV
;tNa --4 ::Mg + -� jl
Given the data below: 40. < BKDSE 2013 Paper IA-36 >
mass of ';tNa "' 23.99096u The sun releases huge amount of energy through thermonuclear fusion while at the same time its mass decreases. The
average power released by the sun is about 3.8 x 1026 W. Estimate the decrease in mass ofthe sun in one second.
mass of ;;Mg "" 23.98504 u
A 4.2 x 10 6 kg
energy releas� in the decay "' 5.00216 MeV B. 4.2 X 109 kg
Calculate the rest mass of the beta particle released C. 1.3 x 10 15 kg
A. 0.00025 u
D. 13 x 10 18 kg
B. 0.00055 u
C. 0.00085 u
D. 0.00952u
41. < HKDSE 2014 Paper IA- 33 >
A radium nucleus decays to a radon nucleus by emitting an a-particle. The energy released in the process is 4.9 Me',:.
37. A star releases energy by nuclear fusion continuously. The mass efthe star decreases by 2 x 10 16 kg in one year. Calculate
Compared to the mass of a radium nucleus, the total mass of a radon nucleus and an a-particle is
the average power released by the st.ar.
A. s.7 x 1025 w A. 5.4 x 10-11 kg less.
B. 6.9 x 10 25 W B. 5.4 x 10-11 kg more.
C. 7.2xl025W C. 8.7x10-30kg less.
D. 8.6 x 10 25 W D. 8.7x10-30kg more.
I DSE Physics - Section E : M.C.
RA3 : Nuclear Energy
PE-
RA3-M 10
/ I DSE Physics - Section E : M.C. Solution
RA3 : Nuclear Energy
PE-RA3 -MS/ 01

HKEAA's Marking Scheme is prepared for the markets' refexence. n should not be regarded as a set of model answers.
42. < HKDSE 2015 Paper IA - 31 > Studemli; and teachers who are not invoivcd in the marking process are advised to interpret the Marlring scheme with care.
Which ofthe following nuclear reactions is/are spontaneous reaction(s) ?
(1) ft
Na ➔ �Mg+ -� e M.C. Answers
1 B 1. A 11. D 21. C 31. A 41. C
(2) � + �n ➔ ;Li+ !He

(3) ;a+iH ➔ iHe+�n


2. D 12. A 22. D 32. A 42. A
A. (1) only 3. D 13. D 23. A 33. A 43. B
B. (3) only
c. (1) & (2) oruy 4. C 14. B 24. C 34. B 44. D
D. (2) & (3) only
5. B 15. B 25. B 35. B 45. A
43. <HKDSE2017 PaperlA-33 >
6. D 16. C 26. c· 36. B 46.C
The following shows the decay ofuranium-238 (2:U ).
7. C 17. D 27. D 37. A
8. A 18. B 28. D 38. B
Given that: mass of �U = 238.05079 u 9. B 19. D 29. B 39. B
mass of�Th"" 234.04363u
10. B 20. D 30. A 40. B
mass of iHe = 4.00260u
Which ofihe following statements is/are correct?
(1) The temperature required to start the decay is about 107 K. M.C. Solution
(2) The energy released in the decay of one uranium-238 nucleus is 425 MeV.
i
(3) All the energy released in the decay becomes the kinetic energy of He. '7 1. A
A (1) only i;Al +�He ➔ �p + JX
B. (2) only
C. (1) & (3) only Mass number = 1
D. (2) & (3) only Atomic number = 0

44. < HKDSE 2018 Paper IA -31 > 2. D


Which of the following nuclear reactions is/are possible for a chain reaction to occur? • (1)
1
�B + Jn ➔ !Li + 1H e Mass number is not balanced.
(1) �H + iH ----+ in + iHe
:Kr ✓ (2) :Bi ➔ �Po + -�e Both mass number and atomic number are balanced.
2 21
1
(2) 2;�u + �n ---> :!aa + + 3�n
(3) �Pu + �n ---> 1:ce +
23
!!Kr+ 3�n ✓ (3) iN +�He ➔ 1iO + :P
1
Both mass number and atomic number are balanced.
A (l)only
B. C2l oruy 3. D
c. (1) & (3) only
D. (2) & (3) o ,tly • (1) ��Al + !He ➔ �p + !P Atomic number is not balanced.
✓ (2) �H + �H ➔ ;He+ ;n Both mass number and atomic number are balanced.
45. < HKD SE 2018 Paper IA �33 >
. ✓
Given: mass ofproton = 1.007276 u (3)
23
�Th ➔ �/Pa+ -�e Both mass number and atomic number are balanced.
. mass ofneutron = 1.008665 u
mass of iHe nucleus=3.01603 0u 4. C
When a � He nucleus is formed from 2protons and 1 neutron,
Balancing the equation, fr AI + 1He ➔ i�P + 01X
A. 6. 7 MeV energy is released.
B. 6.1 MeV energy is required. Atomic number = 0
C 8.0 MeV energy is released. Mass number = 1
D. 8.0MeV energy is required.
There is question in next page
46. <HKDSE 2019 Paper IA-33>
Ii DSE Physics - Section E : M.C. Solution
RA3 : Nuclear Energy
PE-RA3
-MS/0
2 I DSE Physics · Section E : M.C. Solution
RA3 : Nuclear Energy
PE-RA3-MS/03

I
,. B 13. D
( 1)
2
:Po ➔ m�Pb + iHe x, a. particle Neutrons can trigger the further fissions ofthe remaining u.235 nuclei, thus maintain the chain reaction

(2) iH + �H ➔ iHe + 01 n Y, noutron


14. B
z, !3 particle
(3) �Th ➔�Pa+ -�e
:!:l9
• (1) The energy released in decay is neglig:tl>le compared with that in nuclear fission or fusion.

6. D
• (}.) Nuclear fission occurs for element ofvery high atomic number.
The sun and stars do not contain these elements.
The symbolofneutronis tn
✓ (3) The sun and stars contain mainly hydrogenand helium for nuclear fusion to take place.
Balance the mass number : 2 +x=4 + 1 :. X =3
Balance the atomic number : 1 + 1 = y +0 :. y = 2
15. B

7. C ✓ (1) i
Small nuclei � H combining to form large nuclei He is a fusion process.
A. This is an example of nuclear fission. • (2) Since energy is released, there must be mass defect Thus the total mass of the product should be smaller.
B. This is an example ofbombardment ofparticle into a nucleus. ✓ (3) 2�H➔ �He+ �x .. X is a neutron.
✓ C. Fusion is the combination oftwo smaller nuclei : H-2 and H-3 to form a larger nucleus : He-4.
D. This is an exam.pie of-alpha decay.
16. C
(D 235 + 1 = 141 + 92+ q
8. A
✓ A. It is a typical fission ofU-235, triggered by a neutron. 92+0=56+:z+0

B. It is a bombardment of particle reaction. :. :z = 36 and q=3


c. It is a bombardment ofparticle reaction.
D. It is a fusion. 17. D
✓ (1) Since energy is released in this nuclear fusion, thus there is mass defect
9. B ✓ (2) Since energy is released and becomes the kinetic energy of a-particle, there is mass defect
( 1) Fission would produce a large amount ofenergy, but the energy cannot sustain the chain reaction. ✓ (3) Since energy is released and becomes the kinetic energy of �·particle, there is mass defect.
✓ (}.) In each fission, neutrons &e produced and these neutrons can trigger the further :fission ofU-235,
thus the chain reaction can be sustained. B
(3) Fission produces two smaller nuclei, but these smaller nuclei cannot sustain the further fission. E= mc z = (2.0xl030x0.07¾)x(3x108}2 = 1.26x104"J
E = Pt
B :. (1.26x10"")=(4.0xl{r".)t t=3.15x1017 s=lxl010 years
Ai; the reaction involves the combination ofH-2 and H-3 to become He-4, it is a fusion reaction.
19. D
11. D A. After a-decay, the danghternucleus has its mass number decreased by 4.
• (1) 1
�B + in ➔ 1Li + iHe Mass number is not conserved. B. After j3-decay, the dangbter nucleus has its mass slightly decreased due to mass defect.
c. A:ftery•emission, the mass of the nucleus is slightly decreased due to mass defect
✓ (2)
2
�Bi ➔ :uiPo +-�e Both mass number and charge are conserved.
✓ D. After nuclear fusion, the nucleus is heavier since it consists ofsnialler nuclei combining together.
✓ (3)
1
;N + ;ae ➔ 1!0 + fp Both mass number and charge are conserved.
20. D
12. A
There is no immediate effect on a worker who is exposed to smallamount of radiation.
Balancingthemass numberandatomicnumber: 1;N + 1He ➔ fp+ 1;0. However, the radiation is accumulative in the human body and thus increases the chance of cancer in the future.
DSE Physics - Section E: M.C. Solution PE-RA3-MS/04 DSE Physics - Section E : M.C. Solution PE-RA3-MS/05
RA3 : Nuclear Energy RA3 : Nuclear Energy
I
21. C 30. A
The Stmmakes use of nuclear fusion to give out solar energy. ✓ Q) A nuclear bomb makes use of nuclear :fission, NOT nuclear fusion.
X
(2) A hydrogen bomb indeed makes use of nuclear fusion ofhydrogen to give out energy.
22. D
X (3) A star, likes the Sun, makes use of nuclear fusion to give out energy.
After nuclear fission, 2, 3 or 4 fission neutrons may be produced to cause further fission, thus give the chain reaction.

23. A 31. A
✓ (1) It is a typical fission ofU-235. ✓ (!) Nuclear energy makes use of fission does not produce air pollution.
X (2) It is an a-decay. ✓ (2) Nuclear energy is cheaper once the chain reaction starts.
X
(3) It is fusion. X
(3) Other than the fossil fuels, there are renewable energy resources
such as solar energy, wind energy and hydroelectric energy.
24. C
"When a neutron is captured by a U-235 nucleus, the neutron will trigger the fission ofthe uranium nucleus. 3 2. A
✓ (!) In ocean, there is unlimited supply ofwater that consists of hydrogen.
25. B
✓ (2) The water products in nuclear fusion are helium which are noble gas and not radioactive.
X (1) The Sun does not contain radioactive nuclei to give radioactive decay.
X (3) Fusion takes place at hlgh temperature is a disadvantage, not advantage.
X (2) The Sun does not contain large nuclei to give fission.
✓ (3) The Sun contains mainly hydrogen that undergoes fusion to give out solar energy.
33. A
✓ (!) Nuclear fusion takes place at very lngh temperature; it is not easy to produce such a high temperature.
26. C ✓ (2) Even the high temperature (about IO 000 000°C) is aclneved, all containers will change to gases.
X (!) Reaction (I) is the bombardment of proton on the Be-9 nucleus to give two other nuclei. The waste products of fusion is clean and not radioactive, and thus no disposal problem.
X
(3)
✓ (2) The combination of two smaller nuclei to give a large nucleus is called fusion.
✓ (3) The split up ofa large nucleus to give two smaller nuclei is called fission. 34. B
X (1) lt is an example ofa.-decay.
27. D X (2) It is an example of /3-decay.
The fission neutrons can trigger further fissions of the remaining uranium nuclei to give the chain reaction.
✓ (3) It is an example ofnuclear fusion.

28. D
35. B
✓ A. Wind can carry the radioactive waste from one place to another place.
Fusion is combining two smaller nuclei (hydrogen nuclei) to fozm a large nucleus (the helium).
✓ B. Rain water can carry the radioactive waste :from one place to flow to another place.
✓ C. Alrimals can bring the radioactive waste and move to another place.
X D. Since plants cannot move, they cannot canyradioactive waste from one place to another place. 36. B
As I u is equivalent to 931 MeV,
29. B mass equivalent of the energy released = 5-00216 "" 0.00537 u
✓ A. Building a nuclear power plant is very expensive to ensure every safety measure. 931
X B. A good design ofnuclear power plant e nsures no leakage ofradiation to the environment By conservation of mass and energy,
✓ C. If explosion occurs in a nuclear power plant, it would cause disastrous effect to the environment 23.99096 "" 23.98504 + m� + 0.005373
✓ D. Since the wastes of fission product are radioactive, their disposal causes a series problem. :. m� = 0.00055 u
I DSE Physics - Section E : M.C. Solution
RA3 : Nuclear Energy
PE-RA3 -MS /06 II DSE Physics - Section E : M.C. Solution
RA3 : Nuclear Energy
PE-RA3-MS/07

37. A 44. D
By !::.E = twJ cz (1) Chain reaction only occurs in nuclear fission. This is a nuclear fusion, chafu reaction does not occur.
:. AE = (2x 10 16)x(3 x 108)2 = 1.8 x 10'3J ✓ (2) There are 3 fission neutrons that can trigger the remaining u.235 to give chain reaction.
Assume 365 days in 1 year. ✓ (3) There are 3 fission neutrons that can trigger the remaining Pu•239 to give chain reaction.

Average power: P = !!.. __l_.S_x__l O'_'_ = 5.7xl025 W


t 36Sx24x3600 45. A
Mass defect = 1.007276 x 2 + 1.008665 - 3.016030 = 0.007187 u
38. B Energyreleased = 0.007187x931 = 6.7MeV
X A. This is an example ofnuclear fission.
✓ B. Fusion is the combination oftwo smaller nuclei: H-2 and H-3 to form a larger nucleus : He-4.
X C. This is an example ofbombardment ofparticle into a nucleus.
X D. This is an example ofalpha decay.

39. B
Since the reaction is to combine two hydrogen nuclei into a helium nucleus, it is a fusion reaction.
Mass defect = 2.014 + 3.016 - 4.003 - 1.009 = 0.018 u
Enezgyrcleased = 0018 x 931 = 16.76 MeV

40. B
In l second, energy released is 3.8 x 1026 J.
By /!,,E = twJc Z
(3.8xlOU)=6»1(3x108)1 :. twJ=4.2xI09 kg

41. C
By Einstein's equation:
/::.E=t:.mc1
:. (4.9 X 106 X 1.6 X 10-1� = l::,m X (3 X 108)2 :. t:.m = 8.7 x 10-30kg
Since energy is released, there is mass defect
Thus, the total mass ofthe daughter nucleus Yanda-particle is less than the mother nucleus.

42. A
✓ (1) This is a beta-decay reaction, which is spontaneous.
X (2 ) This is a bombardment reaction, triggered bythe hitting of neutron onto the nucleus B� 1 0.
X (3) This is a fusion reaction, which occurs when the temperature is hlgh enough.

43. B
X (1) The decay is spontaneous, can take place in any temperature.
✓ (2) Mass defect: t:,m = 238.05079 - 234.04363 - 4.00260 = 0.00456 u
Energyreleased: E = 0.00456 x 931 = 4.25 MeV
• (3) The energy released in the decay will become the kinetic energy of both Th and He.
I DSE Physics - Section E : Question
RA3 : Nuclear Energy
PE - RA3
-Q /
01 I I
1.
DSE Physics - Section E : Question
RA3 : Nuclear Energy
PE-RA3-Q/02

(d) The development of nuclear energy is a controversial issue. Do you support the development of nuclear energy? State
I
Use the following data wherever necessary :
the reasoning to support your point ofview. (4 marks)
Speed of light in vacuum

Charge of an electron e = 1.60 x 10- 19 C

Avogadro constant NA = 6.02 x 1023 moJ~l

Atomic mass unit U = 1.661 X 10-27 kg (1 u is equivalent to 931 MeV)


Part B : HKAL examination questions

The following list of formulae may be found useful: 2. < HKAL 1983 Paper IlB - 7 >

Half-life and decay constant 0.18 unit lost 0.007 unit lost
to atmosphere to atmosphere
Activity and the number ofundecayednuclei

Mass-energy relationship M=l::.mc2 0.323 unit


of electricity

reactor 0.49unit
Part A : HKCE examination questions to coolingwater
In a nuclear reactor using U-235 as fuel, 1 unit of fission energy produced would undergo the changes shown in the above
1. < HKCE 2003 Paper I - 9 > figure, that finally 0.323 unit of electrical energy is obtained.
In 1986, a disastrous nuclear accident happened at the Chernobyl Nuclear Station. A large amount of radioactive substance (a) Suppose the electrical power output ofthisplantis I066MW.
was released and spread to neighbouring countries. The radiation levels recorded in these countries were much higher than
the nonnal background count rate. (i) Calculate the total power generated by the reactor. (! nruk)

(a) State two sources ofbackground radiation. (2mru-ks)

(ii) Find the total power lost to the atmosphere. (1 mark)

(b) State one way by which the radioactive substances released in the accident were spread to neighbouring countries.
(! mruk)

(b) The turbine is cooled by circulating water through it at the rate of 48 m3 s- 1• Calculate the rise in temperature of the
cooling water. (Density of water = 103 kgm-3 , specific heat capacity ofwater= 4200 J :icg-1 K-1) (3 marks)

(c) One of the radioactive isotopes released in the accident was caesium-13 7 (Cs-137). The following equation shows how
Cs-137 is produced:
(c) In the reactor, energy is produced by the fission ofuranium-235 atoms.
mu+ n 141 Ba + 92Kr + 3n
(i) If z = 4, find the values ofx and y and state their physical meanings. (4marks) Given: z.su,,,235.0409 u; 141 Ba= 140.914lu; 92Kr= 91.9250 u; n= l.0086u.
Calculate the number ofuranium atoms which undergo fission in I s.

(ii) The half-life ofCs-137 is 30 years. Suppose that a soil sample contaminated by Cs-137 is 30 years was found to
have an initial activity of 1.2 x 1()6 Bq (disintegrations per second). A physicist comments that the contaminated
sample will affect the envirorunent for more than 300 years. Justify the physicist's claim 'With calculations. You
mayasmime that the activity of a non-contaminated sample of similar nature is 200 Bq. (3 marks) (d) The nuclear plant is designed to produce power continuously for 10 years without refuelling. Estimate the mass of
uranium-235 required. Given that the molar mass ofU�235 is 235 g. (2 marks)
Ii
3.
DSE Physics - Section E : Question
RA3 : Nuclear Energy

< BKAL 1993 Paper IIB - 12 >


PE-RA3 -Q / 03

i Ii
S.
DSE Physics - Section E : Question
RA3 : Nuclear Energy

< BKAL 2002 Paper I - 7 >


PE-RA3 -Q / 0411
The follovvin g equation represents one of the nuclear r eaction in a fission reactor : The energy released by the Sun is the result of nuclear fusion in its core, where hydrogen are fused together into helium
nuclei through a complicated process. The overall reaction can be simplified by the following equation:
�U + �n � l�Kr + 1:Ba + 3�n
iHe + other particles ofnegligiDle mass + energy
Given: the mass of one nucleus of �1 U = 235.0439 u
(a) Why is the above process of formin g he lium nuclei from protons very difficult to achieve on Earth, but easily achieved
!�Kr = 90.9234u
at the Sun's core? (2 marks )
1:Ba = 141.9164u

�n = l.0087u

(a) According to the above equation, find the mass defec t between the reactants and products when one �; U nucleus (b) Given: mass ofhydro gen = 1.00728 u
undergoes fission. (2 marks) mass ofhelium = 4.00150 u
Calculate the energy r eleased in each fusion by the Sun. Express your answer in joule.

23
(b) If4.00 x 10-5 kgof �U undergoes fission in one second, calculate the rate of energy production. Take the mass of
one moleof ��U as235 g. (3ID"ks) (c) Calculatethe total energy r eleased by the Sun for every kilogram of hydrogen fu sed to form helium nuclei. Take the
mass of one mole ofhydrogen (� H)be I g. (2 marks)

4. < HKAL 1998 Paper llB - 5 >


A reaction which takes place in a nuclear reactor is shown by the following equation :
6. < BKAL 2010 Paper I- 4 >
23�
U+�n ➔ 1!!Ba+:.;Kr+3 �n+173.8 MeV
A nucleus of radon ( � Rn)decays to an isotope ofpolonium (Po) by emitting an a-particle.
Massofonenuclideof �U = 235.0439u
Given: massofaradonnucleus = 222.0176u
Massofonenuclideof ':Ba = 141.9164u massofapoloniumnucleus = 218.0090u
Massofonenuclideof �Kr= 90.9234u mass ofan a-particle = 4.0026u

(a) Calculate the massofa neutron, express the answer in atomic mass unit. (3 marks) (a) Write an equation for thedecay and find the energy released, in MeV, in the d ecay. (3 marks)

(b) The fuel in the reactor contain 1.0 x 104 kg ofU-235. calculate the total energy released when all the U-235 nuclei in
the fuel haveundergone fission. Take the mass ofone mole ofU-235 as 235 g. (3 marks) (b) The energy released in the deca y becomes the kinetic energy of the deca y products. Explain quantitatively why the
a-particle takes most of the decay energy. Assume that the parent nucleus is at rest initially. (2 marks)

(c) If the average power output of the react or is 500 MW and the efficiency of conversion of nuclear energy to electrical
energy is 40%, estimate the time for whic h the fuel can be used. (2 marks) (c) Hence cal culate the speed v of the a-particle, assume all the decay energy is transferred to the a-particle. (2 mmi<.s)
I
7. <HKAL2012PaperI-6>
DSE Physics - Section E : Question

RA3 : Nuclear Energy


PE-RA3-Q/05

I I DSE Physics - Section E : Question

RA3 : Nuclear Energy

Part C : Supplemental exercise


PE-RA3 -QI 06

I
Iodine-131 ('�I) is a common radioactive nuclide found in radioactive waste from nuclear power plants. It undergoes
P decay and becomes a stable nuclide Xenon-131 (' 31Xe) with a half-life of8.02 days. 8. (a) When an alpha particle strikes a beryllium( :Be) nucleus, one caxbon ( 1� c) nucleus and one particle Q are formed.
Write down the nuclear equation. What is the particle Q? (3 marks)
(a) Write down the decay equation oflodine-131. (1 mark)

(b) (i) Estimate the initial activity of 1 kg oflodine-131 in Bq. (3 marks)


Given: mass of one mole oflodine-131 = 131 g (b) A nuclear power plant makes use of nuclear fission of uranium to generate electrical power at a rate of 500 MW. The
internal energy that can be extracted from l kg ofuranium fuel in the fission reactor is about 5.6 x 10 12 J. The efficiency
of energy conversion to electrical form in the nnclear reactor is only 30%. (1 MW = 106 W)

(i) What is the electrical energy snpplied in one day ? (2marks)

(li) Assuming that all the decay energy oflodme-131 becomes heat, find the initial heating power of 1 kg oflodine-131
intheunitW. (4marks) (ii) If electrical energy costs $0.9 for 1 kWh, how much does it cost for the electrical energy generated in one day?
(4nwk,)
Given: mass ofaniodine-131 nucleus = 130.90612 u

mass of a Xenon-131 nucleus = 130.90508 u


mass of an electron = 0.00054 u

(ill) Calculate the electrical energy that can be produced by 1 kg of uranium in the fuel rod. (2marks)

(c) Even after a reactor is shut down and nuclear fission is completely stopped, fission products like Iodine-131 keep on
producing heat. Explain why we cannot stop the Iodin.e-131 from producing heat. (2 marks) (iv) Fmd the mass of uranium fuel used in one day. (2nwk,)

(c) Some people propose that nuclear energy should eventually replace oil and coal as sources of energy supply. Do you
agree with this ? List 3 reasons to support your argument (4 marks)

(d) Iodine-123 is another radioactive isotope ofiodine. It emits y rays and has a half-life of 13 hours. As thyroid in the
human body readily absorbs iodine, Iodine-123 is commonly Ulled as a medical tracer for diagnosis of thyroid diseases.
Give ONE reason why Iodine-123 is more suitable to be used as medical tracer than Iodine-13 I. (1 mark)
DSE Physics - Section E : Question PE-RA3-Q/07 I DSE Physics - Section E : Question PE-RA3-Q/08
RA3 : Nuclear Energy RA3 : Nuclear Energy

9. In a nuclear reactor for generating electricity, Uranium�235 undergoes :fimion 10 generate energy. Part D : HKDSE examination questions

< HKDSE Sample Paper m � 14 >


(a) Descnlle the process ofnuclear fission ofUraniuim-235. (3 marl<,) 10.
In April 1986, a disastrous nuclear accident happened at the Chernobyl Nuclear Power Station. A large quantity of various
radioactive substances was released and spread to neighbouring cmmtries. The radiation levels recorded in these countries
were much higher than the normal background radiation count rate.

(a) State ONE source ofbackground radiation.

(b) The waste products from a nuclear reactor contain isotopes which are radioactive and emit 13 radiation. They are stored
in sealed metal cans for 200 years until the activity decreases to 400 Bq that can be disposed of.
(b) One ofthe radioactive isotopes released in the accident was caesium•137 (Cs•137). The following equation shows how
(i) Explain how these isotopes are produced. (2 marks) Cs· 137 is produced

Given: mass ofonenuclide of


23
iu = 235.0439u
1
� Cs"" 136.907lu
(ii) What is meant by the term radioactive ? (2 marl<,)
:�Rb"" 94.9399u
tn = 1.0087 u.

(i) What is the value ofx ? (1 mark}

(lll) State the reasons why metal cans are used to store the waste products. (1 mmk)

(ii) Find the energy release in the fission of one U•23S nuclide in MeV. (2 marks)

(iv) It is knovm. that the half life of the radioactive isotope in the metal cans is 25 years. What is the initial activity of
the waste products in the cans? (3 marks)

(iii) The half.life of Cs..137 is 30 years. A soil sample contaminated by Cs•l37 has an activity of 1.2 x 106 Bq. A
physicist comments that the co ntaminated sample will affect the environment for more than 350 years. Justify the
physicist's claim with calculations. It is known that the activity ofan uncontaminated soil sample is 200 Bq.
(2 marks)
(v) Calculate the initial number ofatoms ofthe radioactive isotope in the metal cans. (3 ma rks)

(c) David wonders why nuclear fusion is not used to generate electricity. Suggest two reasons to explam this. (2 marks)
11. < HKDSE 2012 Paper m • 11 >

Radium.•226 ( � Ra) undergoes o:-decay into radon (Rn).

(a) Write a nuclear equation for the decay. (2marlcs)


I
11-
DSE Physics • Section E : Question
RA3 : Nuclear Energy

(b) Given: massofaradiumnucleus=226.0254u


mass of a radon nucleus= 222.0176 u
PE· RA3 • Q / 0911 DSE Physics • Section E : Question Solution
RA3 : Nuclear Energy
PE-RA3-QS/ 01

BKEAA's Marking Scheme is prepal'Cd for the matkers' reference. It should not be regarded as a set of model answers.
Students and teachers who are not involved in the marking process arc advised to iDtcipret the Marking Schc:me with care.

mass ofan a-particle = 4.0026 u Question Solution


Calculate the energy released in the decay in MeV.
1. (a) Any TWO of the following :
*
[2]
Cosmic radiation from the space
* Radiation from rocks
* Radiation in air
(c) 1 curie (Ci) is defined as the activity of 1 g of radium. The activity of a radium source used in laboratories is * Radiation from food
about 5 µCi. Estimate the number of radium atoms in this source and hence find its activity expressed in disintegratiOllS
per second. The half-life ofradium-226 is 1600 years and take the mass ofone mole of radium as 226 g.
* Rawation from our bod y
(lµCi =lx10-6 Ci) (3marks)
(b) Any ONE of the following:
*
[!]
bywind
* by rain
* by water in river
* by imported food

12. < HKDSE 2015 Paper IB - 10 > (c) (i) x = 92-37=55 [!]
Scientists had been experimenting controlled fusion in a nuclear reactor in which deuterium { � H) and tritium (� H) undergo y = 235+1-137-4 = 95 [!]
the following nuclear fusion : x is the atomic number of Cs [!]
y is the mass number of Rb [!]
�H + �H ➔ iHe + �n + energyreleased
(ii) Number ofhalf-life in 300 years = 10 [!]
Given : mass ofa deuterium nucleus 2.014102 u
Activity after 10 half-lives = (12 x 106) x (½ ) 1° = 1172 Bq [!]
mass of a tritium nucleus 3.016049 u
mass of a helium nucleus 4.002602 u After 300 years, the activity is still higher than that ofnon-contaminated sample, thus his claim is correct. [!]
mass of a neutron = 1.00866Su OR
(a) Calculate the energy released, in MeV, in the above nuclear fusion. (2 marks) ByA=Ao- (!)"'"'
.. (200) = (l.2x106)x(.!..)1/3o [!]
2
t = 377 years [!]
{b) In the nuclear reactor, deuterium and tritium exist as plasma, which is a mixture of ions at a very high temperature. A time longer than 300 years is required for the activity to drop to safe level, thus his claim is correct. [!]
To start the fusion reaction, the average kinetic energy of the ions in the plasma has to reach the minimum value
of0.2MeV.
(d) I support the development of nuclear power since
(i) Explain why a very high temperature is needed for nuclear fusion to occur. (2 marks)
it is cheaper as the running cost is lower, and [2]
itis clean since it does not produce air pollution and acid rain. [2]
OR
I do not support the development ofnuclear power since
(ii) Estimate the order ofmagnitude of the :minimum temperature at which fusion of deuterium and tritium nuclei would
be possible if the plasma can be regarded as an ideal gas. (2 marks) itis dangerous as once accident occurs, it would be very serious, and [2]
itis expensive as the capital investment is very high. [2]
< accept other reasonable answers >
There is question in next page
13. <HKDSE 2019 Ppaper-IB-10> 14. <HKDSE 2020 Paper 1B -10>

(a) Toe equation below represents "11Clear fission ofuranium-235 (IJ-235). Given: mass of proton = 1.0073 u
massofapai.1icle =4.0015u
mass of 1jN nucleus = 13.9993 u
235 l u1 · 92 1
92U+ 0n ➔ �6Ba+ 36Kt+x0u+200MeV
mass of 1JO nucleus = 16,9947 u
(f} What is the value ofx? (1 marl<)
When a stationary 1� nucleus is bombarded by an a particle., the following nuclear reaction can be triggered
with products 1JO andXfly off:
(Ji) State anecessaey condition for chain reaction offission to occur� (] JllBik)

(a) What is X? (I mark)

Scientists fi>und evidence in Oklo, Amea that� nuclear fission oeeurred two billion (2 x 10� years ago.
The unmiom mineral .ore mined from OJdo at prE.Sent is found to ha� 0,6% concentration by mass of U�23 S *(b)Based on energy consideration,. estimate the minimum kinetic energy� in MeV, of the a particle required
(see the 1able below), m,foh is much lowor 1han usual. for such a nuclear reaction to occur. (2 marks)

(b} Toe table gives the infonnlllion ofU-235 and U-233 in a,amploofuraninm mineral ore follnd in OJdo.
G!ven:lialf-life ofU-235 =7.04,: IO'yeai,; (c) However, when conservation of momentum is also taken into account, the a particle must possess a
kinetic energy greater than that found in {b) to bring about such a reaction. Explain. (2 marks)
2 x 109 years :ago at present
U-235 m,kg 0.060 kg (Le. 0.6¾ conceottation by mass)
U-:2$3 13..S56kg 9.940 kg (i.e. 99.4% conci;ntrati.on by mass)

•(i) Estimll1ethe amoimt mo(m kg) ofU-235 in 11,esample 2 x lo' ye= ogo.

(ii) Hace determine whether natural nuclear fusion of U-235 was posnole 2 x IO' years ,go. For
fission of U·235 to imppon. its ooru;emmion by mass fu the uranium mineral ore has lo be at
least3%. (I mm:k)

� must be underground water in the vicinity of this uraniUm-ricb. mineral deposit for natural nllelear
.fission to Ile pOSSI"ble. Since water can 9low down the fust neutrons from .fission, these neutrons can easily be
captured by U-235.
(c) In fact the chain reaction stopped even befure the coru:entration by mass of U-235 dropped to 3%.
Explain why this occwred. (2 mm)
DSE Physics - Section E : Question Solution PE-RA3-QS /02 DSE Physics - Section E : Question Solution PE-RA3-QS I 031
RA3 : Nuclear Energy RA3 : Nuclear Energy

2. (a) (i) As 1 unit ofnuclearpower can only give 0.323 unit of electricity, the efficiency is 32.3% 3. (a) Mass defect= (235.0439u) - (90.9234u + 141.9164u + 2x1.0087u) [1]
Power generated by the reactor 1066 +32.3% 0.1867 u [l]

3300MW [l] (b) Method © :

(ii) Power lost to the atmosphere = 3300 x (0.18 + 0.007) !!.. = 4.ooxrn-s x 6.02 x 1023 = 1.025 x 1020 s-1 [1]
t 0.235
= 617MW [1]
p= !!..e = (1.025 X 10� X [0.1867 X 1.661 X }Q-27 X (3 X J08)Z} [1]
(b) Power delivered to cooling water = 3300 x 0.49 t
= 1617MW = 2.86xlOg W [1]
[l]
In 1 s, volume ofwater circu.lating is 48 m1 Method@:

:. mass = volume x density = 48 x 103 kg !!_ = 4.00xlO-s x 6.02 x 1023 = 1.025 x t02°
s-1 [1]
t 0.235
By E = m c f:..T and consider the time of1 s.
p = !!...E = (J.025 X 102°) X (0.1867 X 931 X 106 X 1.6 X 10- 1') [l]
(1617 x W) = (48 x 1000) x (4200) x t::r [l] t

•• l::,.T= 8K = 2.85 x lcY'W [1]


[l]
Method@:
(c) Mass defect :
p = (4 X 10-5) X
0•1867 )( (3 )( 108:}2
1:,m = (235.0409 + J.QQ86) - (}40.9141 + 91. 9250 + 3 X 1.QQ86) [2]
235.0439
= 0.1846u [l] = 2.86 x 109 W [l]
Energy released in each fission :
E = 0.1846 X 931 X 106 X 1.6 X 1Q-:t9
= 2.75 X IO-l1 J 4. (a) massdefect:b.nl = 173·8 = 0.1867u [1]
[l] 931
OR 235.0439 = 141.9164 + 90.9234 + 2 x mn + 0.1867 [l]

E = mc2 mn = l.0087u [1]

= {Q.1846 X 1.661 X 10-27) X (3 X 108)2 l.O·xl0 4


(b) NumberofU-235nuclei = x6.02xl023 = 2.56x1028 [l]
= 2.76x 10-11 1 0.235
[l]
Energy released = 2.56 x tots x173.8 MeV [l]
Number of uranium atoms undergoing fission in 1 s :
=4.45xl030 MeV <accept 7.12xl017 J> [l]
By P =!!...E
t
{c) Total electrical energy released by the fuelrods = 4.45x 1()30 x 106 x 1.6 x10-1g x 40%
.• (33QQ X 106) = !!..x (2.75 X JQ-ll) = 2.848 X 1017 J
t [l]
By E = Pt
!:!._ = 1.20 X 1020 g-1 [l]
(2.848 X 1017) = (500 X 106) t
:. t = 5.70xl08 s <accept 1.58xl05 hours or 6590days or 18.lyears> [l]
(d) Mass ofU-235 needed in 1 s = l. 20xl020
x0.235
6.02xl0 23
5
= 4.684 x 10- kg [l]
Mass ofU-235 needed in IO years = 4.684 x 10-s x 10 x 365 x 24 x 3600 5. (a) The temperature in the Sun's core is so high 1hat the hydrogen nuclei have sufficient large kinetic energy [I]
to overcome the strong electrostatic repulsion between them. [l]
= 1.48 x 104 kg [l]
I
5.
DSE Physics - Section E : Question Solution
RA3 : Nuclear Energy

(b) t.m = (4)(1.00728) - 4.00150 = 0.02762 u


PE-
RA3 -
QS /
04

[I]
i 7.
DSE Physics - Section E : Question Solution
RA3 : Nuclear Energy

(b) (ii) 6m = 130.90612 - 130.90508 - 0.00054 = 5 x lo-4u


PE-RA3-QS/05

[I]

/J.E = l:,m c2 "' (0.02762) (1.661 X 10-27) (3 X 1Q'1')2 = 4.13 X :IO-IZ J [I] E = ($ X 1Q ) X (931 X
-4
106 X 1 .6 X 10- 9)
1
[I]

OR = 7.45 X 1Q~l4 J (I]

!JE =0.02762 X 931 X 10 X 1.6 X lQ-19=4.11 X 1Q-ll J


6
[I]
OR

E = 6m c2 = (5 x I0-4 x 1.661 x 10-27) x (3 x 108) 2 [I]


(c) Numberofhydrogenatoms in l kg ofhydrogen = _QL_x(6.02 x 1023)
3
(lxl0- ) =7.47 X 1Q-l4 J [I]

Number of fusion by 1 kgofhydrogen= _O_)_x (6.02 x HP) x .!. "" 1.505 x 1025 [I] P = EA=(7.45 xI0-14)(4.60x10'8)
3
{lxl0- ) 4
3.43 x 10s w <accept 3.4x 105 W to 3.Sx I05W> (I]
Energy released by 1 kgofhydrogen = I.SOS x 1026 x 4.13 x 10-12
(c) The decayofaradioisotope is determined by the half.life (OR decay constant). [I]
= 6.22 x 10 14 J <accept 6.14 x 1014 J to 6.28 x 10 14 J> [I]
It cannot be changed by human factors or surrounding factors. [I]

(d) Any ONE ofthe following: [I]


6. (a) �Rn -->
1
!!Po + ;He * Iodine-123 emits y rays that give less harmful effect to human body.
*
[I]
The half-lifeoflodine-123 is shorter, thus give less harmful effect to human body.
Mass defect= 222.0176u - {218.0090u + 4.0026 u) = 0.006u [I]
* Iodine-123 emits y rays that have greater penetrating power to be detected outside the human body.
Energyreleased = 0.006x931 =5.586MeV <accept 5.59MeV> (I]

(b) Themassratiooftheproductsis 1np0 :m,.,.=218:4


[I]
Byconsezvation of momentum, the speed ratio si V,o : v«=4: 218 [I]
A = 4+9-12=1 and Z=2+4-6=0 (I]
The kinetic energyofthe products is KEP. :KE"= ½ (218)(4)2 : ½(4)(218)2 = 4: 218 [I]
Q si a neutron [I]
Thus, most of the energy released is given to the o:.
(b) (i) E =Pt
(c) By E = .!.mv 2 = (500 X 106) (1 X 24 X 60 X 60) [I]
2
= 432 X 1013 J (I]
(5.586 X 106 X 1.6 X 10-19) = .!.x (4.0026 X 1.661 X JQ-27} v2 [I]
2
(:it) Unitofelectricalenergy = 432X1013 (I]
.. v = l.64xl07 ms-1 (I] 3600000
= 1.2 x 107 kWh [I]

Cost l.2x107 x$0.9 (I]


< accept -� /3 > [I] = $10800 000 (I]

(b) (i) Numberofatoms ofH31 in I kg = __I_ x 6.02 x 10 23=4.60 x 10 24 (iii) Electrical energy produced by 1 kg ofunmium = 5.6 x 10 12 x 30% [I]
(I]
131x10-3
=l.68xI0l:zJ [I]
Decay constantofl-131: k= lnZ= ln2 = l.00xl0-6s-1 [I]
ti,2 8.02x24x3600 432xl0 13
(iv) Massofmaniumfuel used inone day [I]
l.68x1012
lnitialactivity:A 0 = kN0 =(l.00 x J0 )(4.60 x 1024) = 4.60 x 10 18 Bq
-6
[I] =25.7kg [I]
DSE Physics - Section E : Question Solution PE-RA3-QS/06 DSE Physics - Section E : Question Solution PE-RA3 -QS I 07
RA3 : Nuclear Energy RA3 : Nuclear Energy

8. (c) Agree [!] 10. (a) Any ONE of the following: [!]
Reasons : (any THREE of the following) < accept other reasonable answers> * cosmic radiation from space

* *
[3]
Reserves of oil and coal are limited. radiation from rocks

* Nuclear energy is cheaper. * radiation from air

* Nuclear energy causes less pollution. * radiation from food

* Nuclear energy does not produce greenhouse gases. * radiationfrom human bodies

OR [!]
Disagree [!]
(ii) Mass defect = 235.0439 - (136.9071 + 94.9399 + 3 x1.0087) = 0.1708 u [!]
Reasons : (any THREE of the following ) < accept other reasonable answers> [3 ]
* The capital investment ofa nuclear plant is high. Energy released = 0.1708x 931 MeV = 159 MeV [!]
* The disposal of radioactive waste causes a serious problem.
(iii) Activity ofthe sample after 350 years :
* If there is accident, the damage to public is large.
A = (1.2 x 106) x (..!.) 3so/ao
* Some other resources of energy maybe used, e.g. solar energy. 2
••. A=369Bq [1]

Since the activity is larger than 200 Bq, the claim is cmecf. [!]

OR

9. (,) When a neutron is captured by a Uranium-235 nucleus, the nucleus undergoes fission. [!] (200) = (1.2 x 106) x cly/30 .-. t = 377 years [!]

It then splits into two smaller nuclei and together 'With some fission neutrons. [!] Since the time for the activity to drop to 200 Bq is longer than 250 years, the claim is correct. [!]

During the fission , large amount of energy is released. [!]

(b) (i) These isotopes are produced as the by-products [!]

of the fission of the Uranium-235. [!]


< atomic number and mass number of a correct> [!]
(ii) Radioactive is used to descnOe an unstable nucleus [!] < atomic number and mass number of Rn correct> [1]
that may emit a.:, !3 or y radiation to form a more stable nucleus. [!]
(b) Mass defect :
(iit) Metal cans are used since !3 radiation cannot pass through the metal [!] t.m = 226.0254 - 222.0176 - 4.0026 = 0.0052 u [!]

200 = Energy released :


(iv) Number ofhalf-liv es = g [!]
25 E = 0.0052x931 = 4.84MeV [!]
Initial activity = 400 x 28 [!]
(c) Mass of radium = 5 µg
= 102400 Bq [!]
Number of radium atoms in this source :

1n2 ln2
N = 5xIO x 6.02 x 1023 = 1.332 x 1016
"'"'
(v) Decay constant: k = = = 8.79xl0-l0 5-l [!] [!]
tu, 25x365x24x3600 226

By A0 = kN0 [!] Decay constant :

(102400) = (8.79 x 10-1")N0 k = 1n2 = � ln_2 = 1.374 x 10-u s- 1 [!]


�� � =�=
t112 1600x365x24x3600
No = 1.16 X 1014 [!]
Activity:
(,) (D Nuclear fusion can only occur under very high temperature th.at is not �asy to achieve. [!] A = k N = (1.374 x 10-11) x (1.332 x 1016) 1.83 x 105 disintegrations per second
"'
"' [!]
No physical container can withstand the high temperature th.at fusion occurs. [!] < accept 1.83 x 10 Bq> < accept 1.82 x 10 Bq>
5 5
I DSE Physics - Section E : Question Solution
RA3 : Nuclear Energy
PE-RA3
- QS /0
8 I Hong Kong Diploma of Secondary Education Examination
Physics - Compulsory part ( !.ib-f}� 1t-)
Section A -Heat and Gases (#,� .$..fflt)
L Temperature,Heatandlntemalenergy(�li., �'f"Rif.)
12. (a) tw = 2.014102 + 3.016049 - 4.002602 - 1.008665 = 0.018884u [!] 2. Transfer Processes (#.#SU!:U.)
3. Change ofState (%����)
E = 0.018884x931 = 17.58MeV <acceptl7.6MeV> [!] 4. GeneraIGasLaw (¼:l&.ffi.:tt,t-tf)
OR 5. Kinetic Theory (1}--f-:iliQ1t)
E = 0.018884 x 1.661 x Io-27 x (3 x 108}1 = 2.823 x 10 -11 J [l] SectionB -Force and Motion (iJ:f,:��)
I. Position and Movement (1:!! .lf.¥ :§ii;)
(b) (i) To overcome the electrostatic repulsion between the two (positive) nuclei, 2. Newton's Laws ("rffilt.1:jt")
[l]
3. Moment ofForce (:ME)
the temperature must be very high so that the ions has sufficient kinetic energy to come close to each other. [1] 4. Work,EnergyandPower(fp/J • if.:fi:¥:,IJ�)
5. Momentum (:!\l.,'ffi:)
6. Projectile Motion (�ilt�WJ)
(ii) Ek= !_....!.__T
2 N, 7. CircularMotion([lllffl>IWJ)
8. Gravitation ( 5'1 h)
:. (0.2x1Q6xI.6x10-19)= 3 (8.31) ·T Section C- Wave Motion (i&::f»)
[IJ
2 (6.02xlo") I. Wave Propagation (i.to¥.l#:ii!.)
:. T= l.55X 109K 2. Wav e Phenomena (i.t:!¥/.JJl.�)
3. Reflection and Refraction ofLlght ( J6�&.I# ll-bi"M)
Order of magnitude oftemperature = 109 K < exact answer not accepted > (1] 4. Lenses (;£$!)
5. WaveNatureofLight(J6fBiit.ilJ#-11.)
6. Sound(�¼)

m
Section D - Electricity and Magnetism ( 'il!:;fu-Zl't)
13. (a) r-3 IA 1. Electrostatics (M-11!:.!Jb)
I 2. Electric Circuits C 11!:Jl3.)
3. Domestic Electricity(,f..% }lj :t)
(li) More """"1>m are produced la eoch fissicn lbr trigg<ring IA 4. MagneticField(.tlli)
furtherfissions,.ie-.:r> 1. 5. Electromagnetic Induction (11!;.@llx,M,)
6. Alternating Current (�5/ii.11!:)

m m-lfl(Je-« Section E - Radioactivity and Nuclear Energy (tt.M m �lf"'-'ft �)


(b) L Radiation and Radioactivity ($illt�t«.MJJ!.at.)
1o 2. Atomic Model (,W.'T"��)

....,.[=
k=!!.!.{- 9-.3%xlo- yr-1)

,.,.. ]
3. NuclearEnergy(;!a:�5)

m.
0.()6 • e
7.Gf.,,:10. IM
m Physics - Elective part (;J!if<I' ii-)
Elective 1-Astronomy and Space Science (�5C.�lf"'Alt.�#�)
m, - 0.429J82332 (ig) • 0.430 (l<g} IA
2
*
1. The universe seen in different scales (.:f:.13.1 J!f.:J;/lli.To¥.i 'iii ifigfe,)
2. Astronomy through history (1'.:SC.${8.ft-Al.31:.)
3. Orbital motions widergravity(frhT�.fh:i!.,!.:t'.1)
(ii) 0.430 •0.0307U91 .,3.1 %>3% 4. Starsandtheuniverse(•/j.J.�*'ili)
13556+0.430
Thus ruibnl noclet.r fission was
possible. 1),1/lA Elective 2 -Atomic World (,l,y. Ti!t-Jf-)
I. Rutherford's atomic model (lit£'f,U.'T"�lt-M)
I
2. Photoelectric effect(�'ttt.li)
(<) =···. IA
3. Bohr's atomic modelofhydrogen (�lff,l#.1 A't.&.-'f-;fJt�)
4. Particles or waves ($.'T"illc.iK)
nnc Energ,vreleased pY fissim drys 1JJ) dl.e � Vr'atl,r. IA) 5. Probing into nano scale(nl.�#J;r.i!t.ff,.)

� fission m.igbt stop wilbout .slow ueatrons. IA Elective 3 -Energy and Use of Energy ( � itifr:�ti.l!i.�itffl)
2 1. Electricity at home ( �.§ ffl 11!:)
2. Energy efficiency in building (;t_��.@;l,Mt�)
3. Energy efficiency in transportation(;{#,-1:"#.l �5/lf.!Jt�)
4. Non�renewable energy sources (.:f:. -;i--.jl};t_ /is.if.)
5. Renewable energy sources (�.pj.!£H6�)
Elective 4 -Medical Physics (i}$-Jj6;1.!$-)
1. Making sense ofthe eye (Bll#.1� 1?)
2. Makingsenseoftheear(.IF-#.l�'it)
3. Medical imaging using non�ionizing radiation Ula 'Ill: 14/t.iitM Y$£1t. ,!J!,)
4. Medical imaging using ionizing radiation ( 1lt$� M -U- :!f!, �{t$)

You might also like